Anda di halaman 1dari 202

DIKTAT

PEMBINAAN OLIMPIADE MATEMATIKA


MATERI DASAR

DISUSUN OLEH :

EDDY HERMANTO, ST

SMA Negeri 5 Bengkulu


Jalan Cendana Nomor 20
Bengkulu
SINGKATAN

AHSME : American High School Math Exam


AIME : American Invitational Mathematics Examination
APMO : Asian Pasific Mathematical Olympiad
ARML : American Regions Mathematics League
Alabama MC : Alabama State-Wide Mathematics Contest
COMC : Canadian Open Mathematics Challenge
Hongkong PSC : Hongkong Preliminary Selection Contest
India RMO : India Regional Mathematical Olympiad
MATNC : Mu Alpha Theta National Convention
ME VXNY : Mathematical Excalibur Volume X Nomor Y
NHAC : Niels Henrik Abel Contest
OMITS : Olimpiade Matematika ITS
OSK : Olimpiade Sains Indonesia SMA/MA Tingkat Kabupaten/Kota
OSK SMP/MTs : Olimpiade Sains Indonesia SMP/MTs Tingkat Kabupaten/Kota
OSN : Olimpiade Sains Indonesia SMA/MA Tingkat Nasional
OSN SMP/MTs : Olimpiade Sains Indonesia SMP/MTs Tingkat Nasional
OSP : Olimpiade Sains Indonesia SMA/MA Tingkat Provinsi
OSP SMP/MTs : Olimpiade Sains Indonesia SMP/MTs Tingkat Provinsi
South Carolina MC : South Carolina Mathematics Contest
QAMT : Queensland Association of Mathematics Teacher
USAMTS : USA Mathematical Talent Search

ii
KATA PENGANTAR

Alhamdulillah Penulis ucapkan kepada Allah, SWT karena dengan karunia-Nya Penulis
dapat menyelesaikan penulisan diktat versi 5.2 ini. Diktat ini Penulis tulis dalam rangka
mempermudah tugas dalam mempersiapkan siswa-siswa SMA menghadapi olimpiade matematika
pada tahap-tahap awal.

Menurut pengamatan Penulis, masih ada jurang pemisah yang cukup jauh antara siswa-
siswa dari pulau Jawa dengan dari luar pulau Jawa. Masih sangat banyak siswa-siswa di luar pulau
Jawa yang belum memahami persoalan-persoalan dasar di bidang Olimpiade Matematika sehingga
mengalami kesulitan besar ketika akan menghadapi OSN. Buku ini berusaha menjawab tentang
persoalan-persoalan mendasar di bidang Olimpiade Matematika tersebut.

Penulis menyarankan sebelum menyampaikan isi materi buku ini, Pembina Olimpiade di
sekolah dapat menyampaikan materi pendahuluan tentang pembuktian langsung maupun tidak
langsung serta Strategi Penyelesaian Masalah (Problem Solving Strategies) yang dapat dipelajari dari
beberapa buku yang ada seperti Langkah Awal Menuju ke Olimpiade Matematika karya Wono Setya
Budhi, Problem-Solving Strategies karya Arthur Engel maupun Problem-Solving Through Problems karya
Loren C. Larson.

Penulis juga mengharapkan Para guru pembina olimpiade dalam membina siswa-siswa
juga memberikan soal-soal pada tingkatan OSN pada kegiatan umpan balik yang dapat dilaksanakan
setelah guru menyelesaikan pembinaan pada setiap babnya ataupun setelah menyelesaikan seluruh
bab pada buku ini.

Ucapan terima kasih kepada semua pihak yang telah membantu dalam penyelesaian
diktat ini, khususnya kepada isteri tercinta Penulis, Rosya Hastaryta, S. Si, yang telah memberi
dukungan yang besar kepada Penulis serta juga telah melahirkan puteri pertama kami, Kayyisah
Hajidah, pada tanggal 2 Desember 2009.

Penulis merasa bahwa diktat ini masih jauh dari sempurna. Untuk itu Penulis
mengharapkan saran dan kritik dari Pembaca yang budiman sebagai bahan perbaikan diktat ini.

Akhir kata semoga buku ini dapat bermanfaat yang sebesar-besarnya bagi Pembaca
sekalian.

Bengkulu, September 2011

EDDY HERMANTO, ST
Email : eddyhbkl@yahoo.com
http://baktiolimpiade.wordpress.com

iii
DAFTAR ISI

HALAMAN JUDUL …………………………………………………………………… i


SINGKATAN …………………………………………………………………… ii
KATA PENGANTAR …………………………………………………………………… iii
DAFTAR ISI …………………………………………………………………… iv

BAB I ALJABAR
1. Pemfaktoran dan Penguraian …………………… 1
2. Barisan dan Deret …………………… 7
3. Fungsi …………………… 18
4. Suku Banyak …………………… 23
5. Persamaan …………………… 29
6. Sistem Persamaan …………………… 44
7. Ketaksamaan …………………… 50
8. Statistik Sederhana …………………… 57

BAB II TEORI BILANGAN


1. Sifat-Sifat Penjumlahan Dan Perkalian Dua Bilangan …………………… 59
2. Sifat-sifat Keterbagian …………………… 61
3. Uji Habis dibagi …………………… 65
4. Faktor Persekutuan Terbesar (FPB) Dan Persekutuan Terkecil (KPK) …………………… 67
5. Banyaknya Faktor Positif …………………… 70
6. Kekongruenan …………………… 72
7. Bilangan Bulat, Rasional dan Prima …………………… 77
8. Bilangan Kuadrat Sempurna …………………… 82
9. Fungsi Tangga dan Ceiling …………………… 84

BAB III GEOMETRI


1. Trigonometri …………………… 87
2. Garis …………………… 92
3. Segitiga …………………… 100
4. Segi Empat …………………… 120
5. Segi-n Beraturan …………………… 125
6 Lingkaran …………………… 126
7. Garis Lanjutan …………………… 134

BAB IV KOMBINATORIK
1. Kaidah Pencacahan Dan Penjabaran Binom Newton …………………… 142
2. Kejadian dan Peluang Suatu Kejadian, Pengambilan Contoh …………………… 169
Dengan dan Tanpa Pengembalian
3. Prinsip Inklusi Eksklusi, Peluang Kejadian Majemuk …………………… 179
4. Pigeon Hole Principle (Prinsip Lubang Merpati) …………………… 186

KUNCI JAWABAN LATIHAN …………………… 191

iv
Ebook ini gratis dan dapat diprint maupun dikopi serta dapat disebarkan melalui
softcopy atau hardcopy, namun dengan syarat :
1. tidak untuk dikomersilkan.
2. tidak mengubah sebagian baik sedikit maupun banyak dari isi buku sehingga
hak cipta maupun identitas Penulis seolah-olah hilang.
3. tidak menghilangkan halaman ini.

v
Pembinaan Olimpiade Matematika

BAB I
ALJABAR

1. PEMFAKTORAN DAN PENGURAIAN

x2 − y2 = (x + y)(x − y)
Beberapa bent uk pemfakt oran maupun penguraian yang harus diket ahui adalah :

(ii) x3 − y3 = (x − y)(x2 + xy + y2)


(i)

(iii) x3 + y3 = (x + y)(x2 − xy + y2)


(iv) x3 + y3 + z3 − 3xyz = (x + y + z)(x2 + y2 + z2 − xy − xz − yz)
(x + y)(x − y) 2 = x3 − x2y − xy2 + y3
(vi) (an − bn) = (a − b)(an-1 + an-2b + an-3b2 + ⋅⋅⋅ + abn-2 + bn-1) dengan n ∈ bilangan asli
(v)

(vii) (an + bn) = (a + b)(an-1 − an-2b + an-3b2 − ⋅⋅⋅ − abn-2 + bn-1) dengan n ∈ bilangan ganj il

(ix) x4 + 4y4 = (x2 + 2y2 + 2xy)(x2 + 2y2 − 2xy)


(viii) (x + 1)(y + 1)(z + 1) = xyz + xy + xz + yz + x + y + z + 1

(x) (x + y) 2 = x2 + 2xy + y2
(xi) (x + y + z) 2 = x2 + y2 + z2 + 2xy + 2xz + 2yz
(xii) (x − y) 2 = x2 − 2xy + y2
(xiii) (x + y) 3 = x3 + y3 + 3xy(x + y)
(xiv) (x − y) 3 = x3 − y3 − 3xy(x − y)
(xv) (x + y) 4 = x4 + 4x3y + 6x2y2 + 4xy3 + y3
(xvi) (x − y) 4 = x4 − 4x3y + 6x2y2 − 4xy3 + y3
Penguraian bent uk (x + y)n unt uk n > 4 dapat menggunakan binomial Newt on yang akan dit erangkan

Berdasarkan bent uk (vi) dan (vii) didapat fakt a bahwa (a − b) membagi (an − bn) unt uk n asli dan (a + b)
dalam bagian lain.

membagi (an + bn) unt uk n ganj il yang t erkadang digunakan unt uk menyelesaikan soal pada t eori
bilangan.

Cont oh 1 :
(OSK 2004 SMP/ MTs) Nilai dari 5050 2 − 4950 2 = LL

Perhat ikan bahwa a2 − b2 = (a + b)(a − b) maka


Solusi :

5050 2 − 4950 2 = (5050 + 4950 )(5050 − 4950 )


5050 2 − 4950 2 = (10000 )(100 ) = 1000000
5050 2 − 4950 2 = 1.000

(OSK 2005 SMP/ MTs) Salah sat u fakt or dari 173 − 53 adalah ⋅⋅⋅⋅⋅⋅⋅
Cont oh 2 :

A. 5 B. 17 C. 13 D. 273 E. 399

Perhat ikan bahwa a3 − b3 = (a − b)(a2 + ab + b2) maka


Solusi :

173 − 53 = (17 − 5)(172 + 17 ⋅ 5 + 52)


173 − 53 = 12 ⋅ 399 (Jawaban : E)

Eddy Hermanto, ST 1 Aljabar


Pembinaan Olimpiade Matematika

(OSK 2011 Tipe 1) Jika A = 5x + 5−x dan B = 5x − 5−x maka A2 − B2 adalah ⋅⋅⋅⋅⋅⋅⋅
Cont oh 3 :

Solusi :
A = 5x + 5−x dan B = 5x – 5−x
A + B = (5x + 5−x) + (5x − 5−x) = 2 ⋅ 5x
A − B = (5x + 5−x) − (5x − 5−x) = 2 ⋅ 5−x
A2 − B2 = (A + B)(A − B) = 2 ⋅ 5x ⋅ 2 ⋅ 5−x = 4
Jadi, A2 − B2 = 4.

Cont oh 4 :
7− 2
2
Rasionalkan penyebut dari .

Solusi :

Mengingat bahwa (a + b)(a − b) = a2 − b2 maka j ika penyebut t ersebut dikalikan dengan lawannya akan
Kuadrat dari suat u bent uk akar menghasilkan bilangan t ersebut .

( 7+ 2 )
didapat bent uk penyebut yang rasional.

7− 2
2
= 7− 2
2
⋅ 7+ 2
7+ 2
= 2
5
Akhirnya didapat penyebut yang merupakan bilangan rasional, yait u 5. Cont oh di at as adalah cont oh
dengan penyebut nya merupakan penj umlahan dua bilangan. Jika penyebut nya merupakan penj umlahan
lebih dari dua bilangan, maka perkalian dengan lawanya dilakukan lebih dari sat u kali.

Cont oh 5 :
2+ 3 − 5
1
Rasionalkan penyebut dari

(2+ 3 )+ 5
Solusi :

2+ 3 − 5
1
= 2+ 3 − 5
1

(2+ 3 )+ 5
((2 + 3) − 5 ) ((2 + 3 ) + 5 ) = (2 + 3 )2 − ( 5 )2 = 2 + 4 3
2+ 3 − 5
1
= 2+ 3 + 5
2+ 4 3
⋅ 2 3 −1
2 3 −1
= 3 3 + 4 + 2 15 − 5
22
3 3 + 4 + 2 15 − 5
2+ 3 − 5
1
Jadi, = 22

Cont oh 6 :
(OSP 2010 SMP/ MTs) Jika p = 14 − 13
1
dan q = 14 + 13
1
, maka nilai dari p2 + pq + q2 adalah ⋅⋅⋅⋅⋅⋅⋅⋅

Solusi :
pq = 14 1− 13
⋅ 14 + 13
1
= 14 −13
1
=1
14 + 13 + 14 − 13
14 − 13 14 + 13 14−13
1 1
p+q= + = =2 14
2

p2 + pq + q2 = (p + q) 2 − pq
(p + q) = 56

p2 + pq + q2 = 56 − 1 = 55
Jadi, nilai dari p2 + pq + q2 adalah 55.

Eddy Hermanto, ST 2 Aljabar


Pembinaan Olimpiade Matematika

Cont oh 7 :
(OSP 2008) Jika 0 < b < a dan a2 + b2 = 6ab, maka a +b
a −b = ⋅⋅⋅⋅⋅

Solusi :
a +b

( )
Misalkan x = a −b maka
a +b 2
x2 = a −b
2 a 2 + b 2 + 2 ab
a 2 + b 2 − 2 ab
x =
Karena a2 + b2 = 6ab maka x2 = 8 ab
4 ab =2
a +b
a −b
Jadi, = 2

Cont oh 8 :
Hit unglah 2+ 3 − 2− 3 .

Solusi :
Misalkan X = 2+ 3 − 2− 3
X2 = (2 + 3 ) + (2 − 3 ) − 2 22 − 3
X2 = 4 − 2
X2 = 2
Maka 2+ 3 − 2− 3 = 2

1 + 28 ⋅ 29 ⋅ 30 ⋅ 31 adalah ⋅⋅⋅⋅⋅⋅
Cont oh 9 :
(AIME 1989) Nilai dari

Mengingat bahwa 1 + (n − 1)(n)(n + 1)(n + 2) = (n(n + 1) − 1)2 maka


Solusi :

1 + 28 ⋅ 29 ⋅ 30 ⋅ 31 = 29 ⋅ 30 − 1
1 + 28 ⋅ 29 ⋅ 30 ⋅ 31 = 869.

2x + 3 − 7 − x = 1.
Cont oh 10 :
Tent ukan nilai x yang memenuhi persamaan

2x + 3 − 7 − x = 1
Solusi :

Karena bilangan dalam akar harus t ak negat if maka penyelesaian persamaan t ersebut harus memenuhi
syarat − 32 ≤ x ≤ 7
2x + 3 = 1 + 7−x
2x + 3 = 1 + 7 − x + 2 7 − x
3x − 5 = 2 7 − x

Eddy Hermanto, ST 3 Aljabar


Pembinaan Olimpiade Matematika
Kuadrat kan kedua ruas dengan syarat 3x − 5 ≥ 0 sebab 7−x ≥0
9x2 − 30x + 25 = 4(7 − x)
9x2 − 26x − 3 = 0
(9x + 1)(x − 3) = 0
x = 3 at au x = − 19 (t idak memenuhi syarat x ≥ 53 )
Unt uk x = 3 maka 2 x + 3 − 7 − x = 3 − 2 = 1.
Jadi, nilai x yang memenuhi adalah x = 3.

Cont oh 11 :
Jika a2 = 7b + 1945 dan b2 = 7a + 1945 dengan a dan b adalah bilangan real berbeda, maka nilai dari ab
adalah ⋅⋅⋅⋅⋅

Solusi :
a2 = 7b + 1945 dan b2 = 7a + 1945
a2 − b2 = 7(b − a)
Karena a ≠ b maka a + b = −7
a2 + b2 = 7(b + a) + 3890
(a + b) 2 − 2ab = 7(a + b) + 3890
49 − 2ab = −49 + 3890
ab = −1896

Cont oh 12 :
a, b, c dan d adalah bilangan real t ak nol yang memenuhi
a2 + b2 = 1
c2 + d2 = 1
ac + bd = 0
Bukt ikan bahwa ab + cd = 0.

Solusi :
Karena ac + bd = 0 maka
b =− c ⋅⋅⋅⋅⋅⋅⋅⋅⋅⋅⋅⋅⋅⋅⋅⋅⋅⋅⋅⋅⋅⋅⋅⋅⋅⋅⋅ (1)
a d

Misalkan a
= − dc = k maka
a = bk dan d = −ck
b

a2 + b2 = 1
b2(k2 + 1) = 1
k2 + 1 = b12 ⋅⋅⋅⋅⋅⋅⋅⋅⋅⋅⋅⋅⋅⋅⋅⋅⋅⋅ (2)
c2 + d2 = 1
c2(k2 + 1) = 1
Subt it usikan persamaan (2).
b2 = c2
ab + cd = bk ⋅ b + c (−ck)
ab + cd = b2k − c2k
ab + cd = (b2 − c2)k
Karena b2 = c2 maka ab + cd = (b2 − c2)k = 0
Jadi, t erbukt i bahwa ab + cd = 0.

Eddy Hermanto, ST 4 Aljabar


Pembinaan Olimpiade Matematika

LATIHAN 1 :

3a + 4b a 2 + 2b 2
1. Jika 2 a − 2b = 5 maka t ent ukan nilai dari ab

(1945+ 2011)2 + ( 2011−1945 )2


adalah ⋅⋅⋅⋅⋅⋅⋅⋅

( )( )( )( )
19452 + 20112
2. Nilai dari

3. (AIME 1986) Tent ukan nilai dari 5+ 6+ 7 5+ 6− 7 5− 6+ 7 − 5+ 6+ 7 .

4. Jika x + y + 3 x + y = 18 dan x − y − 2 x − y = 15 , maka x⋅y = ⋅⋅⋅⋅⋅

5. Tent ukan nilai X yang memenuhi X = 3 − ( ⎝


) ⎠ ⎝
(
5 ⎛⎜ 3 + 5 ⎞⎟ + 3 + 5 ⎛⎜ 3 − 5 ⎞⎟

)
6. Jika diket ahui bahwa 14 y 2 − 20 y + 48 + 14 y 2 − 20 y − 15 = 9, maka t ent ukan nilai dari
14 y 2 − 20 y + 48 − 14 y 2 − 20 y − 15 .

7. (OSP 2006) Himpunan semua x yang memenuhi (x − 1) 3 + (x − 2)2 = 1 adalah ⋅⋅⋅⋅⋅

( x +1)2
x2
8. (Canadian MO 1992) Selesaikan persamaan x2 + = 3.

9. (OSP 2007) Tent ukan semua bilangan real x yang memenuhi x4 − 4x3 + 5x2 − 4x + 1 = 0

10. (AIME 1983) w dan z adalah bilangan kompleks yang memenuhi w2 + z2 = 7 dan w3 + z3 = 10. Apakah
nilai t erbesar yang mungkin dari w + z ?

11. (Balt ic Way 1999) Tent ukan semua bilangan real a, b, c dan d yang memenuhi sist em persamaan
berikut :
abc + ab + bc + ca + a + b + c = 1
bcd + bc + cd + db + b + c + d = 9
cda + cd + da + ac + c + d + a = 9
dab + da + ab + bd + d + b + a = 9

12. Jika x = 3 4 + 3 2 + 1 , maka nilai dari (1 + 1 3


X ) adalah ⋅⋅⋅⋅⋅⋅

13. (AIME 2000) Tent ukan t epat kedua akar real persamaan 2000x6 + 100x5 + 10x3 + x − 2 = 0.

(10 +324 )(22 +324 )(34 +324 )(46 +324 )(58 +324 ) .
(4 +324 )(16 +324 )(28 +324 )(40 +324 )(52 +324 )
4 4 4 4 4
14. (AIME 1987) Tent ukan nilai dari 4 4 4 4 4

15. (Balt ic Way 1993 Mat hemat ical Team Cont est ) Tent ukan semua bilangan bulat n yang memenuhi
25
2 + 625
4 −n + 25
2 − 625
4 − n adalah bilangan bulat

x − 1x + 1− 1x
16. (Canadian MO 1998) Tent ukan penyelesaian x real yang memenuhi persamaan :
x=

Eddy Hermanto, ST 5 Aljabar


Pembinaan Olimpiade Matematika
17. (AIME 1990) Bilangan real a, b, x dan y memenuhi ax + by = 3, ax2 + by2 = 7, ax3 + by3 = 16 dan
ax4 + by4 = 42. Tent ukan nilai dari ax5 + by5.

18. (OSN 2003 SMP/ MTs) Diket ahui a + b + c = 0. Tunj ukkan bahwa a3 + b3 + c3 = 3abc

19. (OSN 2009 SMP/ MTs) Set iap sisi suat u kubus dit uliskan sebuah bilangan asli. Set iap t it ik sudut nya
diberi nilai yang merupakan hasil kali dari t iga bilangan pada t iga sisi yang berpot ongan di t it ik
sudut t ersebut . Jika j umlah semua bilangan pada t it ik-t it ik sudut t ersebut sama dengan 1001,
t ent ukan j umlah semua bilangan yang dit uliskan pada sisi-sisi kubus t ersebut .

Eddy Hermanto, ST 6 Aljabar


Pembinaan Olimpiade Matematika

1, 2, 3, 4, 5, ⋅⋅⋅ dikat akan sebagai barisan karena mempunyai suat u pola t ert ent u dengan rumus suku ke-
2. BARISAN DAN DERET

1 + 2 + 3 + 4 + ⋅⋅⋅⋅ disebut sebagai deret .


n adalah n.

Ada beberapa barisan dan deret yang akan dibahas.

A. Barisan dan Deret Arit mat ika


1. Pengert ian, rumus suku ke-n dan rumus Jumlah n suku pert ama

a, a + b, a + 2b, a + 3b ⋅⋅⋅ adalah barisan arit mat ika dengan suku pert ama = a dan beda = b.
Barisan arit mat ika adalah barisan yang set iap dua suku berurut an memiliki selisih yang konst an.

Un = a + (n − 1)b
Suku ke-n, Un, dirumuskan dengan :

Jumlah n bilangan pert ama, Sn, dirumuskan dengan


Sn = n2 (2a + (n − 1)b) = n2 (a + Un)

Sn = (a) + (a + b) + (a + 2b) + ⋅⋅⋅ + (a + (n − 1)b)


Bukt i :

Sn = (a + (n − 1)b) + (a + (n − 2)b) + (a + (n − 3)b) + ⋅⋅⋅ + (a)

2 Sn = n ⋅ (2a + (n − 1)b)
Jumlahkan kedua persamaan di at as didapat

Sn = n2 (2a + (n − 1)b) = n2 (a + Un) (t erbukt i)

Diket ahui barisan 2, 5, 8, 11, ⋅⋅⋅. Tent ukan suku ke-10 dan j umlah 4 suku pert ama.
Cont oh 13 :

2, 5, 8, 11, ⋅⋅⋅ adalah barisan arit mat ika dengan suku pert ama 2 dan beda 3.
Solusi :

Suku ke-10, U10 = 2 + (10 − 1) ⋅ 3 = 29


Jumlah 4 suku pert ama = 42 ⋅ (2 ⋅ 2 + (4 − 1) ⋅ 3) = 26

Sebuah barisan j umlah n buah suku pert ama dirumuskan dengan Sn = 3n2 − 15n, maka U3 = ⋅⋅⋅⋅
Cont oh 14 :

Perhat ikan bahwa j ika kit a mengurangkan j umlah n suku pert ama, Sn dengan j umlah n − 1 suku
Solusi :

Jadi, Un = Sn − Sn-1.
pert ama, Sn-1, maka akan didapat kan suku ke-n, Un.

Un = (3n2 − 15n) − (3(n − 1)2 − 15(n − 1))


Un = 3n2 − 15n − 3n2 + 6n − 3 + 15n − 15
Un = 6n − 18
Maka U3 = 6(3) − 18 = 0
Cara lain adalah dengan langsung menghit ung U3 = S3 − S2.
S3 = 3(3)2 − 15(3) = 27 − 45 = −18
S2 = 3(2)2 − 15(2) = 12 − 30 = −18
U3 = S3 − S2
U3 = −18 − (−18)
U3 = 0

Eddy Hermanto, ST 7 Aljabar


Pembinaan Olimpiade Matematika

∑ (2k + 3) = LL
Cont oh 15 :
n
Nilai dari
k =1

Solusi :

∑ (2k + 3) = 5 + 7 + 9 + ⋅⋅⋅ + (2n + 3) merupakan deret arit mat ika dengan a = 5 sert a b = 2.
Jika nilai k dari 1 sampai n dij alankan didapat
n

(2(5) + (n − 1)2)
k =1
n

∑ (2k + 3) = n
Sn = 2
n
2
Jadi, + 4n
k =1

Cont oh 16 :

bernilai dua kali nilai suku pert ama adalah suku ke ⋅⋅⋅⋅⋅⋅
(OSK 2006) Pada sebuah barisan arit mat ika, nilai suku ke-25 t iga kali nilai suku ke-5. Suku yang

Solusi :

un = a + (n − 1)b = 2u1 = 2a
u25 = 3(u5), maka a + 24b = 3(a + 4b) sehingga a = 6b

6b + (n − 1)b = 2(6b)
n=7
Suku t ersebut adalah suku ke-7

Cont oh 17 :

dengan 20, maka keliling segit iga t ersebut adalah ⋅⋅⋅⋅


Sisi-sisi sebuah segit iga siku-siku membent uk barisan arit mat ika. Jika sisi hipot enusa sama

Solusi :

segit iga t ersebut dapat dimisalkan dengan 20, 20 − x dan 20 − 2x dengan x adalah bilangan
Karena sisi t erpanj ang segit iga sama dengan 20 dan membent uk barisan arit mat ika maka sisi-sisi

posit if.

(20 − 2x)2 + (20 − x)2 = 202


Karena ket iga sisi membent uk segit iga siku-siku maka

400 − 80x + 4x2 + 400 − 40x + x2 = 400


5x2 − 120x + 400 = 0
(x − 4)(x − 20) = 0

Jika x = 20 maka sisi-sisi segit iga t ersebut adalah 20, 0 dan −20 yang t idak mungkin merupakan
x = 4 at au x = 20

sisi-sisi segit iga.


Jika x = 4 maka sisi-sisi segit iga t ersebut adalah 20, 16 dan 12 yang membent uk sisi-sisi segit iga
siku-siku.
Jadi, keliling segit iga t ersebut = 20 + 16 + 12 = 48.

2. Suku Tengah

U t = U1 +2U n
Misalkan Ut menyat akan suku t engah dari suat u barisan arit mat ika maka :

dengan n merupakan bilangan ganj il

Eddy Hermanto, ST 8 Aljabar


Pembinaan Olimpiade Matematika

Diket ahui 3, ⋅⋅⋅, 13, 15, ⋅⋅⋅ adalah barisan arit mat ika. Tent ukan suku t engah barisan t ersebut .
Cont oh 18 :

3, ⋅⋅⋅, 13, 15 adalah barisan arit mat ika. Maka U1 = a = 3 dan Un = 15.
Solusi :

Maka suku t engah, Ut = 12 (3 + 15) = 9

3. Sisipan
Misalkan set iap dua bilangan berurut an pada barisan arit mat ika disisipi k buah bilangan namun
t et ap membent uk barisan arit mat ika. Maka beda barisan t ersebut akan memiliki perubahan
dengan suku pert ama t et ap.
Misalkan bB = beda barisan yang baru dan bL = beda barisan yang lama. Hubungan keduanya
adalah

k +1
bL
bB =

Pada set iap dua bilangan berurut an dari barisan 2, 12, 22, 32, 42. ⋅⋅⋅⋅ disisipi sebanyak 4
Cont oh 19 :

bilangan. Tent ukan suku ke-100 dari barisan yang baru.

Beda barisan yang baru adalah bB = k +L1 = 410+1 = 2


Solusi :
b

U100 = a + 99bB = 2 + 99 ⋅ 2 = 200


Suku pert ama, a = 2.

Suku ke-100 = 200.


Jadi, suku ke-100 barisan t ersebut adalah 200.

Misalkan ada barisan u1, u2, u3, ⋅⋅⋅, un yang bukan merupakan barisan ait mat ika sebab un − un-1
4. Barisan Arit mat ika Bert ingkat

t idak konst an. Tet api apabila diambil D1(n) = un − un-1 lalu D2(n) = D1(n) − D1(n − 1) dan
set erusnya sampai pada suat u saat Dk(n) − Dk(n − 1) bernilai konst an. Maka kit a dapat
mengambil kesimpulan bahwa rumus j umlah n suku pert ama, Sn, barisan t ersebut merupakan
polinomial pangkat k.

Diket ahui barisan 1, 3, 6, 10, 15, 21, ⋅⋅⋅. Tent ukan rumus j umlah n suku pert ama, Sn.
Cont oh 20 :

Kalau diperhat ikan, barisan 1, 3, 6, 10, 15, 21, ⋅⋅⋅ bukanlah barisan arit mat ika. Tet api rumus
Solusi :

3, ⋅⋅⋅, n yang merupakan barisan arit mat ika.


suku ke-n barisan t ersebut t ernyat a merupakan rumus j umlah n suku pert ama dari barisan 1, 2,

Maka kit a dapat menyelesaikan soal t ersebut dengan menganggapnya merupakan barisan
arit mat ika bert ingkat .

n S(n) D1(n) = S(n) – S(n − 1) D2(n) = D1(n) − D1(n − 1) D3(n) = D2(n) − D2(n − 1)
1 1
2 4 3
3 10 6 3
4 20 10 4 1
5 35 15 5 1

Eddy Hermanto, ST 9 Aljabar


Pembinaan Olimpiade Matematika
Karena D3(n) konst an maka dapat diambil kesimpulan bahwa rumus Sn merupakan polinomial
pangkat 3.
Misalkan S(n) = an3 + bn2 + cn + d.

n S(n) D1(n) = S(n) – S(n − 1) D2(n) = D1(n) − D1(n − 1) D3(n) = D2(n) − D2(n − 1)
1 a+b+c+d
2 8a+4b+2c+d 7a+3b+c
3 27a+9b+3c+d 19a+5b+c 12a+2b
4 64a+16b+4c+d 37a+7b+c 18a+2b 6a
5 125a+25b+5c+d 61a+9b+c 24a+2b 6a

6a = 1 ⋅⋅⋅⋅⋅⋅⋅⋅⋅⋅⋅⋅⋅⋅⋅⋅⋅⋅⋅⋅⋅⋅⋅⋅⋅⋅⋅⋅⋅⋅⋅ (1)
Dari kedua t abel didapat bahwa :

12a + 2b = 3 ⋅⋅⋅⋅⋅⋅⋅⋅⋅⋅⋅⋅⋅⋅⋅⋅⋅⋅⋅⋅⋅⋅⋅⋅⋅⋅⋅⋅⋅⋅⋅⋅⋅⋅⋅⋅ (2)


7a + 3b + c = 3 ⋅⋅⋅⋅⋅⋅⋅⋅⋅⋅⋅⋅⋅⋅⋅⋅⋅⋅⋅⋅⋅⋅⋅⋅⋅⋅⋅⋅⋅⋅⋅⋅⋅⋅⋅⋅⋅⋅⋅⋅ (3)
a + b + c + d = 1 ⋅⋅⋅⋅⋅⋅⋅⋅⋅⋅⋅⋅⋅⋅⋅⋅⋅⋅⋅⋅⋅⋅⋅⋅⋅⋅⋅⋅⋅⋅⋅⋅⋅⋅⋅⋅⋅⋅⋅⋅⋅⋅⋅⋅ (4)
Dari pers (1) didapat a = 16
Dari pers (2) didapat b = = 12 3− 2

Dari pers (3) didapat c = 3 − 7( 16 ) − 3( 12 ) = 18−67 −9 = 13


2

Dari pers (4) didapat d = 1 − 16 − 12 − 13 = 6−1−63− 2 = 0


n ( n +1)( n + 2 )
Maka rumus j umlah n suku pert ama, S(n) = 1
6 n3 + 1
2 n2 + 1
3 n= 6

B. Barisan dan Deret Geomet ri


1. Pengert ian, rumus suku ke-n dan rumus Jumlah n suku pert ama

konst an. Misalkan a, ar, ar 2, ⋅⋅⋅ adalah barisan geomet ri dengan suku pert ama = a dan rasio = r
Barisan geomet ri adalah barisan yang set iap dua suku berurut an memiliki perbandingan yang

maka :

Un = a ⋅ r n-1
Suku ke-n, Un, dirumuskan dengan :

(
a r n −1 )
Jumlah n bilangan pert ama, Sn, dirumuskan dengan

r −1
Sn =

Diket ahui barisan 2, 6, 18, 54, ⋅⋅⋅ . Tent ukan suku ke-5 dan j umlah 4 suku pert ama barisan
Cont oh 21 :

t ersebut .

2, 6, 18, 54, ⋅⋅⋅ adalah cont oh barisan geomet ri dengan suku pert ama 2 dan rasio 3.
Solusi :

Suku ke-5, U5 = 2 ⋅ 35-1 = 162


(
2⋅ 34 −1 )
Jumlah 4 suku pert ama = 3−1 = 80

Pada barisan geomet ri diket ahui U8 = 36 dan S7 = 52, maka S8 = ⋅⋅⋅⋅⋅


Cont oh 22 :

Solusi :
U8 = 36 dan S7 = 52

Eddy Hermanto, ST 10 Aljabar


Pembinaan Olimpiade Matematika
Pada barisan arit mat ika maupun geomet ri berlaku Sn − Sn−1 = Un.
S8 − S7 = U8
S8 = 52 + 36 = 88.

Cont oh 23 :

t erbent uklah deret arit mat ika, maka rasio barisan t ersebut adalah ⋅⋅⋅⋅
Tiga bilangan membent uk deret geomet ri dengan j umlah 65. Jika suku ke-3 dikurangi 20

Solusi :
Misalkan bilangan-bilangan t ersebut adalah a, ar dan ar 2. Maka
a, ar dan ar 2 − 20 membent uk barisan arit mat ika sehingga
2ar = a + ar 2 − 20
2ar + 20 = a + ar 2 ⋅⋅⋅⋅⋅⋅⋅⋅⋅⋅⋅⋅⋅⋅⋅⋅⋅ (1)
a + ar + ar 2 = 65 ⋅⋅⋅⋅⋅⋅⋅⋅⋅⋅⋅⋅⋅⋅⋅⋅⋅⋅ (2)

ar = 15 ⋅⋅⋅⋅⋅⋅⋅⋅⋅⋅⋅⋅⋅⋅⋅⋅⋅⋅⋅⋅⋅ (3)
2ar + 20 + ar = 65

50 = a + 15r ⋅⋅⋅⋅⋅⋅⋅⋅⋅⋅⋅⋅⋅⋅⋅⋅⋅⋅ (4)


Subt it usikan persamaan (3) ke (1) didapat

(50 − 15r)r = 15
Subt it usikan persamaan (4) ke (3).

10r − 3r 2 = 3
(3r − 1)(r − 3) = 0
r = 13 at au r = 3
1
Jika r = maka ket iga bilangan t ersebut adalah 45, 15 dan 5 yang membent uk barisan geomet ri
dan 45, 15, −15 yang membent uk barisan arit mat ika.
3

Jika r = 3 maka ket iga bilangan t ersebut adalah 5, 15 dan 45 yang membent uk barisan geomet ri
dan 5, 15, 25 yang membent uk barisan arit mat ika.
Jadi, rasio barisan t ersebut adalah 13 at au 3.

2. Suku Tengah

U t2 = U 1 ⋅ U n
Misalkan Ut menyat akan suku t engah dari suat u barisan geomet ri maka :

dengan n merupakan bilangan ganj il

Diket ahui 2, 6, 18, 54, 162, ⋅⋅⋅⋅ adalah barisan geomet ri. Tent ukan suku t engah dari barisan
Cont oh 24 :

t ersebut .

Solusi :

U t = 2 ⋅ 162 = 18
2, 6, 18, 54, 162 adalah barisan geomet ri. Maka U1 = a = 2 dan Un = 162.
Maka suku t engah,

3. Sisipan
Misalkan set iap dua bilangan berurut an pada barisan geomet ri disisipi k buah bilangan namun
t et ap membent uk barisan geomet ri. Maka rasio barisan t ersebut akan memiliki perubahan
dengan suku pert ama t et ap.
Misalkan r B = rasio barisan yang baru dan r L = rasio barisan yang lama. Hubungan keduanya

rB = k +1 rL
adalah

Eddy Hermanto, ST 11 Aljabar


Pembinaan Olimpiade Matematika

Pada set iap dua bilangan berurut an dari barisan 2, 32, 512, 8192, ⋅⋅⋅⋅ disisipi sebanyak 3
Cont oh 25 :

bilangan. Tent ukan suku ke-7 dari barisan yang baru.

Rasio yang baru, rB = k +1 rL = 4 16 = 2 .


Solusi :

Suku pert ama, a = 2.


U7 = ar 6 = (2)(26) = 128
Suku ke-7 = 128.

( )
4. Barisan geomet ri t ak hingga
Dari persamaan Sn = r −1 j ika n å ∞ maka S∞ = 1−ar dengan syarat −1 < r < 1.
a r n −1

Rumus t ersebut merupakan rumus j umlah dari suat u barisan t ak hingga dengan suat u syarat
t ert ent u.

Cont oh 26 :
Tent ukan nilai dari 2 + 1 + 1
2 + 1
4 + ⋅⋅⋅

Solusi :
Persoalan di at as t ermasuk barisan geomet ri t ak hingga dengan a = 2 dan r = ½
2 + 1 + ½ + ¼ + ⋅⋅⋅ = S∞ = 1−ar .
2 + 1 + ½ + ¼ + ⋅⋅⋅ = 1− 12
2
= 4.

Maka nilai dari 2 + 1 + 1


2 + 1
4 + ⋅⋅⋅ = 4.

Cont oh 27 :

ganj il adalah 4. Suku ke-6 barisan t ersebut adalah ⋅⋅⋅⋅


Jumlah suat u deret geomet ri t ak hingga adalah 6 dan j umlah dari suku-suku yang bernomor

Solusi :
Misalkan suku pert ama barisan geomet ri t ak hingga t ersebut adalah a dan rasio r.
a + ar + ar 2 + ⋅⋅⋅ = 1−ar = 6 ⋅⋅⋅⋅⋅⋅⋅⋅⋅⋅⋅⋅⋅⋅⋅⋅⋅⋅⋅⋅⋅⋅ (1)
Suku-suku yang ganj il adalah a, ar 2, ar 4, ⋅⋅⋅ yang membent uk barisan t ak hingga dengan suku
pert ama a dan rasio r 2.
a + ar 2 + ar 4 + ar 6 + ⋅⋅⋅ = 1−ar 2 = 4 ⋅⋅⋅⋅⋅⋅⋅⋅⋅⋅⋅⋅⋅⋅⋅⋅⋅⋅⋅⋅⋅⋅ (2)
Subt it usikan persamaan (1) ke (1−ar )(1+1r ) = 4 sehingga
1+ r
1
= 23
3 = 2 + 2r
r = 12
Subt it usikan persamaan di at as ke persamaan (1) didapat
a=3
Maka suku ke-6 = U6 = ar 5 = 32
3
.

C. Barisan dan Deret Lainnya sert a Bent uk Tak Hingga

barisan yang berbent uk 1, 1, 2, 3, 5, 8, 13, 21, ⋅⋅⋅ yang merupakan penj umlahan dari dua bilangan
Suat u barisan t idak harus masuk ke dalam salah sat u dari dua bent uk di at as. Sebagai cont oh adalah

Eddy Hermanto, ST 12 Aljabar


Pembinaan Olimpiade Matematika
sebelumnya. Unt uk menyelesaikan persoalan yang dit anyakan memerlukan penget ahuan t erhadap
pola dari barisan t ersebut .

n ( n +1)( 2 n +1)
Beberapa cont oh rumus deret lainnya :
12 + 22 + 32 + ⋅⋅⋅ + n2 =

( ( ))
6

13 + 23 + 33 + ⋅⋅⋅ + n3 =
n n +1 2
2

Berikut ini dibahas bent uk-bent uk t ak hingga yang dapat diselesaikan dengan memisalkan dengan
suat u variabel.

Cont oh 28 :

Hit unglah nilai dari 2 2 2 2 L

Solusi :

Misalkan 2 2 2 2 L = X.
Kuadrat kan kedua ruas. Pada ruas kiri akan didapat kan bent uk t ak hingga semula. Maka
2X = X2
X(X − 2) = 0

Karena 2 2 2 2 L t idak mungkin sama dengan 0 maka 2 2 2 2 L = 2.

Jadi, 2 2 2 2 L =2

Cont oh 29 :

Hit unglah bent uk t ak hingga berikut , 2 +


3
2+
.
3
2+
3
M

Solusi :

Misalkan 2 +
3
2+
= X maka
3
2+
3
M

2+
3
=X

X2 − 2X − 3 = 0
X

(X − 3)(X + 1) = 0

Karena 2 + t idak mungkin sama dengan −1 maka 2 +


3 3
2+ 2+
= 3.
3 3
2+ 2+
3 3
M M

Jadi, 2 +
3
2+
=3
3
2+
3
M

Eddy Hermanto, ST 13 Aljabar


Pembinaan Olimpiade Matematika

D. Prinsip Teleskopik
Prinsip t eleskopik banyak digunakan unt uk menyederhanakan suat u deret . Ada dua bent uk umum

∑ (a − ai ) =(a 2 − a1 ) + (a3 − a 2 ) + (a 4 − a3 ) + L + (a n − a n −1 ) + (a n +1 − a n ) = a n +1 − a1
yang dikenal, yait u penj umlahan dan perkalian sebagai berikut :
n
a. i +1
i =1

∏ = ⋅ ⋅ ⋅ L ⋅ n ⋅ n +1 = n +1
n
ai +1 a 2 a3 a 4 a a a
b.
i =1 ai a1 a 2 a3 a n −1 a n a1

(1 − 13 )(1 − 15 )(1 − 17 )L(1 − 2003 )(1 − 2005 )(1 + 12 )(1 + 14 )(1 + 16 )L(1 + 2004 )(1 + 2006 )=L
Cont oh 30 :
1 1 1 1

Misal S = 1 − ( )(1 − 15 )(1 − 17 )L(1 − 2003 )(1 − 2005 )(1 + 12 )(1 + 14 )(1 + 16 )L(1 + 2004 )(1 + 2006 )
Solusi :
1 1 1 1 1

⋅ ⋅ 76 ⋅ L ⋅ 2004
2005 ⋅ 2 ⋅ 4 ⋅ 6 ⋅ L ⋅ 2006
3
2 4 3 5 7 2007
S=
S = ⋅ ⋅ 54 ⋅ 54 ⋅ 76 ⋅ 76 ⋅ L ⋅ 2004
2005 ⋅ 2004 ⋅ 2006
3 5
2 3 2005 2007
3 2
2007
S= 2006

Cont oh 31 :
Tent ukan nilai 1⋅2
1
+ 2⋅3
1
+ 3⋅4
1
+ 4⋅5
1
+ ⋅⋅⋅ + 2005⋅2006
1
.

Solusi :

1⋅2 = 1 − 2 ; 21⋅3 = 12 − 13 ; 31⋅4 = 13 − 14 ; ⋅⋅⋅⋅⋅⋅⋅⋅⋅⋅ ; 20051⋅2006 = − 2006


Soal di at as merupakan cont oh penerapan prinsip t eleskopik.

(11 − 12 ) + ( 12 − 13 ) + (13 − 14 ) + ( 14 − 15 ) + L + ( 2005 )


1 1 1 1 1

+ ⋅⋅⋅ + − 2006
2005

1⋅2 2⋅3 3⋅4 4⋅5 2005⋅2006


1 1 1 1 1 1 1
+ + + =

1⋅2
1
+ 2⋅3
1
+ 3⋅4
1
+ 4⋅5
1
+ ⋅⋅⋅ + 2005⋅2006
1
=1− 1

+ ⋅⋅⋅ +
2006

1⋅2 2⋅3 3⋅4 4⋅5 2005⋅2006


1 1 1 1 1 2005
Jadi, + + + = 2006

⋅ 34 ⋅ 56 ⋅ L ⋅ 1997
Cont oh 32 :
1 1 1
(Canadian MO 1997) Bukt ikan bahwa 1999 < 2 1998 < 44 .

⋅ 34 ⋅ 56 ⋅ L ⋅ 1997
1998 dan Q = 3 ⋅ 5 ⋅ 7 ⋅ L ⋅ 1999
Solusi :
1 2 4 6 1998
Misal P =
PQ = 12 ⋅ 34 ⋅ 56 ⋅ L ⋅ 1997
1998 ⋅ 3 ⋅ 5 ⋅ 7 ⋅ L ⋅ 1999 = 1999
2
2 4 6 1998 1

Jelas bahwa P < Q


P2 < PQ sehingga P2 < 1999 1
< 441 2 . Maka P < 441

P> 1
3 ⋅ 53 ⋅ 75 ⋅ L ⋅ 1997
1
1999 =
1999
1 1
Maka didapat 1999 < P < 44
1 1 3 5
1999 < 2 4 6 ⋅ ⋅ ⋅L⋅
1997
1998 < 44
1
(t erbukt i)

Eddy Hermanto, ST 14 Aljabar


Pembinaan Olimpiade Matematika

LATIHAN 2 :

t engahnya 20, sert a beda deret t ersebut adalah 3. Maka U6 = ⋅⋅⋅⋅


1. Sebuah deret arit mat ika t erdiri dari n suku (ganj il). Jumlah semua sukunya 260, besar suku

2. Perhat ikan barisan bilangan 500, 465, 430, 395, ⋅⋅⋅. Suku negat ifnya yang pert ama adalah ⋅⋅⋅⋅⋅⋅

∑ (4 + 2k i ) = 77 ∑ k i = 14 , maka nilai ∑ (4 + 2k ) = LL
11 7 11
3. Diket ahui dan
i =1 i =1 i =8
i

4. Pada suat u deret arit mat ika berlaku u2 + u5 + u6 + u9 = 40. Maka S10 = ⋅⋅⋅⋅⋅

5. (OSK 2009) Jika xk+1 = xk + 1


2 unt uk k = 1, 2, ⋅⋅⋅ dan x1 = 1 maka x1 + x2 + ⋅⋅⋅ + x400 = ⋅⋅⋅⋅⋅⋅⋅

6. (OSP 2006) Hasil penj umlahan semua bilangan bulat di ant ara 3
2006 dan 2006 adalah ⋅⋅

7. (OSK 2006) Diket ahui a + (a + 1) + (a + 2) + ⋅⋅⋅ + 50 = 1139. Jika a bilangan posit if, maka a = ⋅⋅⋅⋅⋅

kuadrat sempurna adalah ⋅⋅⋅⋅⋅⋅⋅⋅


8. (OSK 2011 Tipe 3) Bilangan bulat posit if t erkecil a sehingga 2a + 4a + 6a + ... + 200a merupakan

9. (AIME 1984) Barisan a1, a2, a3, ⋅⋅⋅, a98 memenuhi an+1 = an + 1 unt uk n = 1, 2, 3, ⋅⋅⋅, 97 dan mempunyai
j umlah sama dengan 137. Tent ukan nilai dari a2 + a4 + a6 + ⋅⋅⋅ + a98.

10. Misalkan un adalah suku ke-n dari suat u barisan arit mat ika. Jika uk = t dan ut = k maka t ent ukan nilai
dari suku ke-(k + t ).

11. (OSK 2004) Agar bilangan 20 + 21 + 22 + ⋅⋅⋅ + 2n sedekat mungkin kepada 2004, haruslah n = ⋅

12. Jika 9 − 7x, 5x − 6 dan x − 1 adalah t iga suku pert ama deret geomet ri t ak hingga, maka j umlah
suku-sukunya adalah ⋅⋅⋅

yang bernomor genap berj umlah 3/ 4 , maka suku pert amanya adalah ⋅⋅⋅⋅⋅
13. Pada suat u deret t ak hingga, suku-suku yang bernomor ganj il berj umlah 9/ 4 sedangkan suku-suku

j umlah 2 adalah ⋅⋅⋅⋅⋅


14. Bat as-bat as nilai a supaya deret geomet ri t ak berhingga dengan suku pert ama a konvergen dengan

15. (OSP 2006) Afkar memilih suku-suku barisan geomet ri t akhingga 1, 1


, 1
, 1
, ⋅⋅⋅ unt uk membuat
. Tiga suku pert ama pilihan Afkar adalah ⋅⋅⋅⋅⋅
2 4 8
1
barisan geomet ri t akhingga baru yang j umlahnya 7

16. Tent ukan j umlah dari 2


3 − 4 + 94 − 74 + 8
27 − 494 + LL

suat u barisan geomet ri dengan rasio sama dengan 2. Jumlah barisan arit mat ika it u adalah ⋅⋅⋅⋅⋅ ⋅⋅
17. Tiga buah bilangan merupakan barisan arit mat ika. Bila suku t engahnya dikurangi 5, maka t erbent uk

18. Tent ukan rumus j umlah n suku pert ama dari barisan 4, 10, 20, 35, 56, ⋅⋅⋅

Eddy Hermanto, ST 15 Aljabar


Pembinaan Olimpiade Matematika
19. (AIME 1992) Misalkan A adalah barisan a1, a2, a3, ⋅⋅⋅ dengan a19 = a92 = 0 dan ∆A didenisikan dengan
barisan a2 − a1, a3 − a2, a4 − a3, ⋅⋅⋅. Jika semua suku-suku barisan ∆(∆A) sama dengan 1, maka nilai a1
adalah ⋅⋅⋅⋅⋅⋅

20. (MATNC 2001) Tent ukan j umlah 100 bilangan asli pert ama yang bukan bilangan kuadrat sempurna.

membent uk barisan arit mat ika sedangkan b, c, d membent uk barisan geomet ri. Jika d − a = 30
21. (AIME 2003 Bagian Pert ama) Diket ahui 0 < a < b < c < d adalah bilangan bulat yang memenuhi a, b, c

maka t ent ukan nilai dari a + b + c + d.

22. (OSK 2009) Bilangan bulat posit if t erkecil n dengan n > 2009 sehingga
13 + 2 3 + 33 + L + n 3

merupakan bilangan bulat adalah ⋅⋅⋅⋅⋅⋅⋅⋅⋅


n

23. (AIME 1985) Barisan bilangan bulat a1, a2, a3, ⋅⋅⋅ memenuhi an+2 = an+1 − an unt uk n > 0. Jumlah 1492
bilangan pert ama adalah 1985 dan j umlah 1985 bilangan pert ama adalah 1492. Tent ukan j umlah
2001 bilangan pert ama.

24. Nilai x yang memenuhi persamaan :

x x x..... = 4 x + 4 x + 4 x + ...
adalah ⋅⋅⋅⋅⋅⋅⋅

25. Hit unglah nilai dari

6−
5
3+
4
3+
4
3+
4
M
26. (OSK 2006/ AIME 1990) Barisan 2, 3, 5, 6, 7, 10, 11, ⋅⋅⋅ t erdiri dari semua bilangan asli yang bukan
kuadrat at au pangkat t iga bilangan bulat . Suku ke-250 barisan t ersebut adalah ⋅⋅⋅⋅⋅

27. (AHSME 1996) Barisan 1, 2, 1, 2, 2, 1, 2, 2, 2, 1, 2, 2, 2, 2, 1, 2, 2, 2, 2, 2, 1, 2, ⋅⋅⋅ memiliki blok


angka 1 yang berisi n buah angka 2 pada blok ke-n. Tent ukan j umlah 1234 bilangan pert ama.

28. Misalkan f adalah adalah fungsi yang memenuhi f(n) = f(n − 1) + n


2007
unt uk set iap n bilangan asli.
Jika f(0) = 1945 maka t ent ukan f(2007).

29. (NHAC 1997-1998 Second Round) Tent ukan nilai dari 1


1x 2 x 3 + 1
2 x3x 4 + L + 1996 x1997
1
x1998 .

(1 − )(1 − )(1 − )L(1 − )(1 − )


30. (OSK 2003) Berapakah hasil perkalian
1 1 1 1 1
22 32 42 20022 20032

+ + +L+
31. Tent ukan j umlah dari :
1+ 2 2+ 3 3+ 4 99 + 100
1 1 1 1

Eddy Hermanto, ST 16 Aljabar


Pembinaan Olimpiade Matematika

∑ (k + 1)(k + 2) .
2012
k 2 + 3k + 1
32. Hit unglah nilai
k =1

33. (AIME 2002) Barisan x1, x2, x3, ⋅⋅⋅ memenuhi xk = k 2 +k


1
. Jika t erdapat bilangan berurut an sehingga
xm + xm+1 + ⋅⋅⋅ + xn = 1
29 , maka t ent ukan semua pasangan (m, n) yang memenuhi.

34. (AIME 2001) Barisan a1, a2, a3, a4, ⋅⋅⋅ memenuhi a1 = 211, a2 = 375, a3 = 420 dan a4 = 523 sert a
an = an−1 − an−2 + an−3 − an−4. t ent ukan nilai dari a531 + a753 + a975.

35. (AIME 1998) Barisan 1000, n, 1000 − n, n − (1000 − n), (1000 − n) − (n − (1000 − n), ⋅⋅⋅ dengan n

maka barisan t ersebut adalah 1000, 100, 900, −800. Suku ke-4 barisan t ersebut negat if. Jadi, unt uk
bilangan bulat berakhir ket ika bilangan negat if pert ama muncul. Sebagai cont oh unt uk n = 100

n = 100 maka barisan t ersebut memiliki panj ang 3. Tent ukan n sehingga panj ang barisan t ersebut
maksimal.

36. (OSN 2008 SMP/ MTs) Suat u barisan bilangan real mempunyai suku-suku didefinisikan sebagai

un = ar n-1 j ika n = 4m − 3 at au n = 4m − 2
berikut .

un = −ar n-1 j ika n = 4m − 1 at au n = 4m


dengan a > 0, r > 0, dan m bilangan bulat posit if.

( )
Bukt ikan bahwa j umlah semua suku ke-1 sampai dengan suku ke-2009 adalah
a 1+ r − r 2009 + r 2010
1+ r 2

1 + 112 + 212 + 1 + 212 + 312 + ⋅⋅⋅ + 1 + 1999 2 +


37. (USAMTS 1999-2000 Round 4) Tent ukan nilai dari
1 1
S=
20002

k 3 −1
, k = 2, 3, 4, ⋅⋅⋅, 100 lebih dari
k 3 +1
2
38. (Balt ic Way 1992) Bukt ikan bahwa hasil kali 99 bilangan 3 .

Eddy Hermanto, ST 17 Aljabar


Pembinaan Olimpiade Matematika

3. FUNGSI
A. Pengert ian
Misalkan diket ahui fungsi y = f(x) = 3x−+21x .

( 2 )+1
Unt uk mencari nilai dari f(2) maka cukup menggant i x di ruas kanan dengan 2.
Jadi, f(2) = 3− 2 ( 2 ) = −3
Salah sat u fungsi yang dibahas di dalam kelas adalah fungsi kuadrat , yait u fungsi yang berbent uk y =

Nilai x yang menyebabkan y maksimum adalah xp = − 2ba


f(x) = ax2 + bx + c

(b
Nilai y maksimum = ymaks = a(xp) 2 + bxp + c at au ymaks = − 4 a
2
− 4 ac )
Terkadang suat u fungsi t idak hanya memiliki sat u variabel, t et api dapat lebih dari sat u variabel.
Sebagai cont oh adalah f(x,y) = xy + x2y + y3. Unt uk mencari f(1, 2) cukup menggant i x = 1 dan y = 2
dari persamaan t ersebut didapat f(1, 2) = 2 + 2 + 8 = 12.

Cont oh 33 :
Misal f adalah suat u fungsi yang memet akan dari bilangan bulat posit if ke bilangan bulat posit if dan
didefinisikan dengan : f(ab) = b⋅f(a) + a⋅f(b). Jika f(10) = 19 ; f(12) = 52 dan f(15) = 26. Tent ukan
nilai dari f(8).

f(120) = f(10 ⋅ 12) = 12f(10) + 10f(12) = 12 ⋅ 19 + 10 ⋅ 52 = 748 ⋅⋅⋅⋅⋅⋅⋅⋅⋅⋅⋅⋅ (1)


Solusi :

f(120) = f(8 ⋅ 15) = 8f(15) + 15f(8) = 8 ⋅ 26 + 15f(8) = 208 + 15f(8) ⋅⋅⋅⋅⋅⋅⋅⋅⋅⋅⋅ (2)
748 = 208 + 15f(8)
Jadi, f(8) = 36

Cont oh 34 :
(AHSME 1998) Misalkan f(x) adalah fungsi yang memenuhi
(a) unt uk set iap bilangan real x dan y maka f(x + y) = x + f(y) dan

Nilai dari f(1998) adalah ⋅⋅⋅⋅⋅


(b) f(0) = 2

Solusi :
f(x + y) = x + f(y) dan f(0) = 2
Alt ernat if 1 :
f(x) = f(x + 0) = x + f(0)
f(x) = x + 2
Maka f(1998) = 2000

2 = f(0) = f(−1998 + 1998) = −1998 + f(1998)


Alt ernat if 2 :

Maka f(1998) = 2000

Cont oh 35 :
1
Jika f(x) adalah fungsi yang t idak t erdefinisi unt uk x = 0 dengan f(x) + 2f( x ) = 3x. Tent ukan f(x).

Solusi :
f(x) + 2f( 1
x ) = 3x ⋅⋅⋅⋅⋅⋅⋅⋅⋅⋅⋅⋅⋅⋅⋅⋅⋅⋅⋅ (1)
1
Jika y = x maka

Eddy Hermanto, ST 18 Aljabar


Pembinaan Olimpiade Matematika
1 3
f( y
) + 2f(y) = y
sehingga
3
f( 1x ) + 2f(x) =
2f( 1x ) + 4f(x) = 6x ⋅⋅⋅⋅⋅⋅⋅⋅⋅⋅⋅⋅⋅⋅⋅⋅⋅⋅⋅⋅⋅⋅⋅⋅⋅ (2)
x

Kurangkan persamaan (2) dengan (1) didapat


3f(x) = 6x − 3x
2− x 2
Jadi, f(x) = x

B. Fungsi Komposisi
Fungsi komposisi merupakan gabungan lebih dari sat u fungsi.
Misalkan diket ahui fungsi f(x) dan g(x). Jika ingin mencari pemet aan suat u nilai t erhadap fungsi f(x)
yang hasilnya dilanj ut kan t erhadap fungsi g(x), maka akan digunakan fungsi komposisi.
Pemet aan t erhadap fungsi f(x) yang dilanj ut kan oleh fungsi g(x) dit ulis sebagai (g(x) o f(x)).
Didefinisikan (g(x) o f(x)) = g(f(x)).

Diket ahui f(x) = 3x + 5 dan g(x) = 7 − 3x. Tent ukan pemet aan x = 2 oleh fungsi f(x) dilanj ut kan g(x).
Cont oh 36 :

Solusi :

g(11) = 7 − 3(11) = −26


f(2) = 3(2) + 5 = 11

Jadi, pemet aan x = 2 oleh fungsi f(x) dilanj ut kan oleh g(x) menghasilkan nilai −26.

(g(x)of(x)) = g(f(x)) = g(3x + 5) = 7 − 3(3x + 5) = −8 − 9x


Cara lain adalah dengan memanfaat kan definisi fungsi komposisi.

Unt uk x = 2 maka nilai g(f(2)) = −8 − 9(2) = −26.


Jadi, pemet aan x = 2 oleh fungsi f(x) dilanj ut kan oleh g(x) adalah −26.

Cont oh 37 :
Diket ahui f(x) = x + 7 dan (fog)(x) = 5x + 3. Tent ukan g(x).

Solusi :
f(g(x)) = g(x) + 7

g(x) = 5x − 4
5x + 3 = g(x) + 7

Jika g(x − 5) = 7x + 3 maka t ent ukan g(x).


Cont oh 38 :

Solusi :

Pada alt ernat if 1 ini kit a ubah variabel x pada ruas kanan ke dalam (x − 5).
Alt ernat if 1 :

g(x − 5) = 7x + 3 = 7(x − 5) + 38
Maka g(x) = 7x + 38

Kit a misalkan x − 5 = y sehingga x = y + 5.


Alt ernat if 2 :

Maka g(y) = 7(y + 5) + 3


g(y) = 7y + 38
Maka g(x) = 7x + 38

Eddy Hermanto, ST 19 Aljabar


Pembinaan Olimpiade Matematika

Diket ahui (gof)(x) = 2x2 + 5x − 5 dan f(x) = x − 1. Maka g(x) = ⋅⋅⋅⋅⋅⋅


Cont oh 39 :

(gof)(x) = 2x2 + 5x − 5
Solusi :

g(f(x)) = 2x2 + 5x − 5
g(x − 1) = 2x2 + 5x − 5

g(x − 1) = 2(x − 1)2 + 4x − 2 + 5x − 5


Alt ernat if 1 :

g(x − 1) = 2(x − 1)2 + 9x − 7


g(x − 1) = 2(x − 1)2 + 9(x − 1) + 2
Maka g(x) = 2x2 + 9x + 2

Misalkan y = x − 1 maka x = y + 1
Alt ernat if 2 :

g(y) = 2(y + 1) 2 + 5(y + 1) − 5


g(y) = 2y2 + 9y + 2
Maka g(x) = 2x2 + 9x + 2

Cont oh 40 :
Diket ahui (gof)(x) = 4x2 + 4x dan g(x) = x2 − 1 dan berlaku f(x) > 0 unt uk x > − 12 , maka f(x − 2)
adalah ⋅⋅⋅⋅

Solusi :
g(f(x)) = 4x2 + 4x
(f(x)) 2 − 1 = 4x2 + 4x
(f(x)) 2 = 4x2 + 4x + 1 = (2x + 1) 2
Karena f(x) > 0 unt uk x > − 12 maka

f(x − 2) = 2(x − 2) + 1
f(x) = 2x + 1

f(x − 2) = 2x − 3
Jadi, f(x − 2) = 2x − 3

C. Fungsi Invers dari y = f(x)


Berdasarkan fungsi y = f(x) = 3x−+21x dari ket erangan sebelumnya j ika diket ahui nilai x kit a dengan
mudah mencari nilai y. Bagaimana caranya bila yang diket ahui adalah nilai y dan kit a dimint a
mencari nilai x unt uk nilai y t ersebut ? Persoalan ini dapat diselesaikan apabila kit a bisa
mendapat kan fungsi inversnya yait u x = f(y).

Tent ukan invers dari fungsi y = f(x) = 3x − 8


Cont oh 41 :

y = 3x − 8
Solusi :

y +8
x= 3
x +8
Didapat fungsi inversnya adalah f -1(x) = 3

Eddy Hermanto, ST 20 Aljabar


Pembinaan Olimpiade Matematika
Cont oh 42 :
x +1
Tent ukan invers dari fungsi y = f(x) = 3− 2 x .

Solusi :
Dari y = x +1
3− 2 x didapat 3y − 2yx = x + 1 sehingga
x(2y + 1) = 3y − 1
3 y −1
x= 2 y +1
3 x −1
Didapat fungsi inversnya adalah f -1(x) = 2 x +1

D. Hubungan fungsi invers dengan fungsi komposisi.


Misalkan f −1(x) dan g−1(x) bert urut -t urut menyat akan fungsi invers dari f(x) dan g(x). Maka
(f o g) −1(x) = (g−1 o f −1)(x)
(g o f) −1(x) = (f −1 o g−1)(x)

Cont oh 43 :
2 x +3
Jika f(x) = 5x + 3 dan g(x) = 5− x maka t ent ukan (f o g) −1(x).

Solusi :
Alt ernat if 1 :
Berdasarkan ket erangan dalam pembahasan mengenai fungsi komposisi akan didapat
(f o g)(x) = 75x−+x30 .
Maka invers dari (f o g)(x) t ersebut adalah
(f o g) −1(x) = 5 xx+−30
7

Alt ernat if 2 :
Berdasarkan apa yang dipelaj ari sebelumnya akan didapat
f −1(x) = x 5−3 dan g−1(x) = 5xx+−23 sehingga
5 x −30
(g−1 o f −1)(x) = x+7
Jadi, didapat (f o g) −1(x) = (g−1 o f −1)(x).

LATIHAN 3 :

1. Jika f(x) = −x + 3, maka f(x2) + (f(x)) 2 − 2f(x) = ⋅⋅⋅⋅⋅⋅⋅

2. Diket ahui g(x) = x + 1 dan (gof)(x) = 3x2 + 4. Maka f(x) = ⋅⋅⋅⋅⋅⋅⋅

3. (OSK 2007) Misalkan f(x) = 2x - 1, dan g(x) = x . Jika f(g(x)) = 3, maka x = ⋅⋅⋅⋅⋅⋅⋅⋅⋅

4. Diket ahui (fog)(x) = 5x. Jika g(x) = 5 x −1


1
, maka f(x) = ⋅⋅⋅⋅⋅⋅⋅⋅

5. Fungsi g(x) = x2 + 2x + 5 dan (f(g(x)) = 3x2 + 6x − 8, maka f(x) = ⋅⋅⋅⋅⋅⋅⋅⋅

6. Jika f(x) = 2x + 1 ; g(x) = 5x2 + 3 dan h(x) = 7x, maka (fogoh)(x) = ⋅⋅⋅⋅

Eddy Hermanto, ST 21 Aljabar


Pembinaan Olimpiade Matematika
7. Dit ent ukan f ( x ) = ax +1
2− x
. Jika f −1(4) = 1, maka f(3) = ⋅⋅⋅⋅⋅⋅⋅

8. Jika f −1
( x) = x +1
x
dan g −1 ( x ) = 2 x − 1 , maka ( g o f )−1 (x ) = ⋅⋅⋅⋅⋅⋅

9. Jika f(x) = x 2 + 1 dan (fog)(x) = x 2 − 4 x +5


x−2 dan berlaku g(x) ≥ 0 unt uk x > 2, maka g(x − 3) = ⋅⋅⋅⋅⋅

f ( x)
10. (OSK 2011 Tipe 3) Misalkan f suat u fungsi yang memenuhi f(xy) = unt uk semua bilangan real
posit if x dan y. Jika f(100) = 3 maka f(10) adalah ⋅⋅⋅⋅⋅
y

f ( 1x ) + 1x f (− x ) = 2 x
11. (OSK 2003) Misalkan f suat u fungsi yang memenuhi

unt uk set iap bilangan real x ≠ 0. Berapakah nilai f(2) ?

12. (AHSME 1996) Sebuah fungsi f : Z å Z dan memenuhi


n + 3 j ika n ganj il
f(n) =
n
2 j ika n genap
Misalkan k adalah bilangan ganj il dan f(f(f(k))) = 27. Tent ukan penj umlahan digit -digit dari k.

13. (OSP 2004) Misalkan f sebuah fungsi yang memenuhi f(x) f(y) − f(xy) = x + y, unt uk set iap bilangan
bulat x dan y. Berapakah nilai f(2004) ?

14. (OSK 2006) Jika f(xy) = f(x + y) dan f(7) = 7, maka f(49) = ⋅⋅⋅⋅

memenuhi f(x + y) = f(x) + f(y) + 6xy + 1 dan f(−x) = f(x). Nilai dari f(3) sama dengan ⋅⋅⋅⋅⋅⋅
15. (NHAC 1998-1999 Second Round) Misalkan f adalah fungsi unt uk semua bilangan bulat x dan y yang

16. (OSP 2009) Suat u fungsi f : Z å Q mempunyai sifat f ( x + 1) = 11+− ff (( xx )) unt uk set iap x ∈ Z. Jika f(2) =
2, maka nilai fungsi f(2009) adalah ⋅⋅⋅⋅⋅

17. (AIME 1988) Misalkan f(n) adalah kuadrat dari j umlah angka-angka n. Misalkan j uga f 2(n)
didefiniskan sebagai f(f(n)), f 3(n) sebagai f(f(f(n))) dan set erusnya. Tent ukan nilai dari f 1998(11).

Eddy Hermanto, ST 22 Aljabar


Pembinaan Olimpiade Matematika

4. SUKU BANYAK
A. Pengert ian Suku Banyak

x2 − 5x + 9
Perhat ikan bent uk-bent uk alj abar berikut :

(ii) 4x3 + 6x − 2x
(i)

(iii) 2x4 − 7x3 + 8x2 + x − 5


(iv) −2x5 + x4 + 7x3 − 8x2 + 3x − 4
Bent uk-bent uk alj abar di at as disebut j uga dengan suku banyak at au polinom dalam peubah
(variabel) x. Yang dimaksud deraj at suat u sukubanyak dalam peubah x adalah pangkat t ert inggi dari
peubah x yang t ermuat dalam suku banyak t ersebut .
Suku banyak pada (i) memiliki deraj at 2 sedangkan suku banyak pada (ii), (iii) dan (iv) bert urut -
t urut berderaj at 3, 4 dan 5.

B. Kesamaan Suku Banyak


Misalkan t erdapat dua buah suku banyak f(x) dan g(x) yang dinyat akan dalam bent uk umum sebagai

f(x) = anxn + an-1xn-1 + an-2xn-2 + ⋅⋅⋅ + a1x + ao dan


berikut :

g(x) = bnxn + bn-1xn-1 + bn-2xn-2 + ⋅⋅⋅ + b1x + bo


Kalau f(x) sama dengan g(x) (dapat dit ulis f(x) ≡ g(x)) maka harus memenuhi
an = bn ; an-1 = bn-1 ; an-2 = bn-2 ; ⋅⋅⋅ ; a1 = b1 ; ao = bo

Diket ahui kesamaan dua buah suku banyak p(x + 1) + q(x − 1) ≡ 7x − 3. Nilai p + 2q = ⋅⋅⋅⋅⋅
Cont oh 44 :

p(x + 1) + q(x − 1) ≡ 7x − 3
Solusi :

px + p + qx − q ≡ 7x − 3
(p + q)x + p − q ≡ 7x − 3

p + q = 7 dan p − q = −3 yang menghasilkan p = 2 dan q = 5


Berdasarkan kesamaan dua suku banyak maka

Jadi, p + 2q = (2) + 2(5) = 12

C. Pembagian Suku Banyak


Sebagaimana pembagian dalam bilangan, pembagian suku banyak pun memiliki kemiripan dengan

f(x) = p(x) ⋅ g(x) + s(x)


pembagian pada bilangan t ersebut . Pembagian f(x) oleh p(x) dapat dit ulis sebagai berikut :

dengan
f(x) adalah suku banyak yang akan dibagi
p(x) adalah pembagi
g(x) adalah hasil bagi
s(x) adalah sisa pembagian
Sebagaimana dalam pembagian bilangan, persyarat an s(x) adalah bahwa pangkat t ert inggi (deraj at )
dari s(x) harus kurang dari p(x).
Cara pembagian dalam suku banyak pun mengikut i pembagian dalam bilangan.

Tent ukan hasil bagi dan sisanya j ika 4x4 + 3x3 − 2x2 + x − 7 dibagi x2 + 4x − 2
Cont oh 45 :

f(x) = 4x4 + 3x3 − 2x2 + x − 7 = (x2 + 4x − 2) ⋅ q(x) + s(x)


Solusi :

Karena f(x) berderaj at 4 maka q(x) akan berderaj at 2 sehingga q(x) = ax2 + bx + c

Eddy Hermanto, ST 23 Aljabar


Pembinaan Olimpiade Matematika
Karena koefisen x4 dari f(x) sama dengan 4 maka koefisien x2 dari q(x) j uga 4 sehingga a = 4.
Kalikan 4x2 dengan (x2 + 4x − 2) didapat 4x4 + 16x3 − 8x2. Kurangkan 4x4 + 3x3 − 2x2 + x − 7 dengan
4x4 + 16x3 − 8x2 didapat −13x3 + 6x2 + x − 7. Karena koefisien x3 sama dengan −13 maka koefisien x
dari q(x) sama dengan −13 sehingga b = −13.
Kalikan −13x dengan (x2 + 4x − 2) didapat −13x3 − 52x2 + 26x. Kurangkan −13x3 + 6x2 + x − 7 dengan
−13x4 − 52x2 + 26x didapat 58x2 − 25x − 7. Karena koefisien x2 sama dengan 58 maka konst ant a dari

Kalikan 58 dengan (x2 + 4x − 2) didapat 58x2 + 232x − 116. Kurangkan 58x2 − 25x − 7 dengan 58x2 +
q(x) sama dengan 58 sehingga c = 58.

232x − 116 didapat −257x + 109.


Jadi, 4x4 + 3x3 − 2x2 + x − 7 = (x2 + 4x − 2) ⋅ (4x2 − 13x + 58) − 257x + 109.
Maka pembagi dan sisa j ika suku banyak 4x4 + 3x3 − 2x2 + x − 7 dibagi x2 + 4x − 2 bert urut -t urut
adalah 4x2 − 13x + 58 dan − 257x + 109.

Cont oh 45 merupakan pembagian suku banyak dengan cara pembagian bersusun. Apabila
pembaginya linier maka pembagian j uga dapat dilakukan dengan cara horner.

Tent ukan hasil bagi dan sisanya j ika f(x) = x3 + 2x2 + 3x − 5 dibagi x − 2
Cont oh 46 :

−5
Solusi :
1 2 3
2
2 8 22
1 4 11 17 +

Maka pembagian f(x) = x3 + 2x2 + 3x − 5 oleh x − 2 akan menghasilkan x2 + 4x + 11 dengan sisa 17.

D. Teorema Sisa

f(x) = p(x) ⋅ g(x) + s(x)


Dari penj elasan sebelumnya t elah kit a dapat kan bahwa

Jika diambil p(x) = x − k maka akan didapat f(x) = (x − k) ⋅ g(x) + s

Jadi, didapat suat u t eorema bahwa j ika suku banyak f (x) dibagi oleh x − k maka sisanya
Jika diambil x = k maka didapat f(k) = s

adalah f (k).

( )
Teorema di at as dikenal dengan nama t eorema sisa at au dalil sisa.

f − ba .
Lebih lanj ut dengan cara yang sama didapat bahwa j ika f(x) dibagi (ax + b) maka sisanya adalah

Tent ukan sisanya j ika f(x) = x4 − 6x3 − 6x2 + 8x + 6 dibagi x − 2


Cont oh 47 :

Dengan t eorema sisa akan didapat sisa j ika f(x) dibagi x − 2 adalah f(2).
Solusi :

Sisa = f(2) = 24 − 6 ⋅ 23 − 6 ⋅ 22 + 8 ⋅ 2 + 6 = −34.


Jadi, sisa j ika f(x) = x4 − 6x3 − 6x2 + 8x + 6 dibagi x − 2 adalah −34.
Coba kerj akan soal di at as dengan cara Horner.

Diket ahui bahwa f(x) j ika dibagi x − 1 bersisa 5 sedangkan j ika dibagi x + 1 akan bersisa 3. Maka j ika
Cont oh 48 :

f(x) dibagi x2 − 1 akan memiliki sisa ⋅⋅⋅⋅⋅⋅⋅

Eddy Hermanto, ST 24 Aljabar


Pembinaan Olimpiade Matematika

Persoalan ini sama saj a dengan menent ukan s(x) j ika f(x) = (x − 1)(x + 1) g(x) + s(x) dengan f(1) = 5
Solusi :

dan f(−1) = 3.
Telah dij elaskan sebelumnya bahwa deraj at sisa selalu kurang dari pembagi. Karena deraj at
pembagi sama dengan 2 maka kit a dapat memisalkan bahwa deraj at sisa sama dengan 1.
Jadi, dapat dimisalkan bahwa s(x) = ax + b.

Maka f(x) = (x − 1)(x + 1) g(x) + ax + b


Jika t ernyat a deraj at s(x) sama dengan 0 maka akan didapat bahwa a = 0.

Karena f(1) = 5 maka f(1) = a + b = 5 ⋅⋅⋅⋅⋅⋅⋅⋅⋅⋅⋅⋅⋅⋅⋅⋅⋅⋅⋅⋅⋅ (1)


Karena f(−1) = 3 maka f(−1) = −a + b = 3 ⋅⋅⋅⋅⋅⋅⋅⋅⋅⋅⋅⋅⋅⋅⋅⋅⋅⋅⋅⋅⋅ (2)

Jadi, f(x) dibagi x2 − 1 akan memiliki sisa x + 4


Dari persamaan (1) dan (2) didapat a = 1 dan b = 4

E. Teorema Fakt or
Set elah mempelaj ari t eorema sisa, maka selanj ut nya akan dipelaj ari pengert ian fakt or dalam suku
banyak. Pengert ian fakt or dalam suku banyak dapat dinyat akan dalam bent uk t eorema fakt or

Misalkan f (x) adalah suku banyak. (x − k) merupakan f akt or dari f (x) j ika dan hanya j ika
berikut :

f (k) = 0
Perhat ikan bahwa pernyat aan di at as merupakan biimplikasi. Sehingga pernyat aan di at as memiliki

(1) Jika (x − k) merupakan fakt or dari f(x) maka f(k) = 0


art i :

(2) Jika f(k) = 0 maka (x − k) merupakan fakt or dari f(x)

Pada cont oh di at as memiliki art i j uga bahwa k adalah merupakan akar-akar persamaan f(x) = 0.
Jika f(x) merupakan suku banyak dalam deraj at n maka ada paling banyak n buah akar real
persamaan f(x) = 0.

Cont oh 49 :
Tunj ukkan bahwa (x + 2) merupakan fakt or dari f(x) = x4 + 3x3 + 4x2 + 8x + 8.

Solusi :
f(−2) = (−2)4 + 3(−2)3 + 4(−2) 2 + 8(−2) + 8 = 0
Karena f(−2) = 0 maka sesuai t eorema fakt or maka (x + 2) merupakan fakt or dari f(x). Terbukt i.

Tent ukan semua fakt or linier dari x4 − 2x3 − 13x2 + 14x + 24 = 0


Cont oh 50 :

Solusi :

Fakt or bulat dari 24 adalah ±1, ±2, ±3, ±4, ±6, ±8, ±12 dan ±24. Kit a akan mencoba akar-akar
Kit a akan mencoba menyelesaikannya dengan horner.

−2 − 13
t ersebut j ika ada.
1 14 24
−1 − 14
1
1 0
1 −1 −14 0 24 +

Karena sisanya t idak sama dengan 0 maka (x − 1) bukan fakt or.


1 −2 − 13 14 24
− 26 − 24
2
2 0
1 0 −13 −12 0 +

Eddy Hermanto, ST 25 Aljabar


Pembinaan Olimpiade Matematika

Karena sisanya sama dengan 0 maka (x − 2) adalah fakt or linier.


x4 − 2x3 − 13x2 + 14x + 24 = (x − 2)(x3 − 13x − 12)

− 13 − 12
−1
1 0
−1 1 12
1 −1 −12 0 +

x4 − 2x3 − 13x2 + 14x + 24 = (x − 2)(x + 1)(x2 − x − 12)


Karena sisanya sama dengan 0 maka (x + 1) adalah fakt or linier.

x4 − 2x3 − 13x2 + 14x + 24 = (x − 2)(x + 1)(x − 4)(x + 3)


Maka fakt or-fakt or linier dari x4 − 2x3 − 13x2 + 14x + 24 adalah (x + 1), (x − 2), (x + 3) dan (x − 4).

Jika p ( x ) = a n x + a n −1 x + a n − 2 x n − 2 + L + a1 x 1 + a 0 adalah polinomial dengan pembuat nol :


F. Teorema Viet a
n n −1

x1, x2, x3, ⋅⋅⋅, xn, (dengan kat a lain x1, x2, x3, ⋅⋅⋅, xn adalah akar-akar p(x) = 0) maka hubungan-

x1 + x 2 + x3 + L + x n −1 + x n = − aann−1
hubungan berikut berlaku :

∑x x = x1 x 2 + x1 x3 + L + x 2 x3 + x 2 x 4 + L + x n −1 x n = an − 2

∑x x
i< j
i j an

x k = x1 x 2 x3 + x1 x3 x 4 + L + x 2 x3 x 4 + x 2 x 4 x5 + L + x n − 2 x n −1 x n = − aan n−3
i< j <k
i j

x1 x 2 x3 L x n −1 x n = (− 1)
M
n a0
an

Persamaan kuadrat x2 + 5x − 7 = 0 memiliki akar-akar x1 dan x2. Tent ukan nilai x13 + x23.
Cont oh 51 :

Dari persamaan x2 + 5x − 7 = 0 didapat nilai A = 1, B = 5 dan C = −7


Solusi :

x1 + x2 = − BA = −5
x1x2 = C
= −7
= (x1 + x2) 3 − 3x12x2 − 3x1x22
A
x13 x23
= (x1 + x2) 3 − 3x1x2(x1 + x2)
+
x13 x23
= (−5) 3 − 3(−7)(−5) = −125 − 105
+
x13 x23
= −230.
+
x13 + x23

Persamaan suku banyak x4 − 5x3 − 16x2 + 41x − 15 = 0 memiliki akar-akar a, b, c dan d. Maka nilai
Cont oh 52 :

dari
a. a2 + b2 + c2 + d2
b. 1a + 1b + 1c + d1
adalah ⋅⋅⋅⋅⋅⋅

Eddy Hermanto, ST 26 Aljabar


Pembinaan Olimpiade Matematika
Solusi :

ab + ac + ad + bc + bd + cd = −16
a+b+c+d=5

abc + abd + acd + bcd = −41


abcd = −15
a. a2 + b2 + c2 + d2 = (a + b + c + d) 2 − 2(ab + ac + ad + bc + bd + cd) = 52 − 2(−16) = 57
b. 1a + 1b + 1c + d1 = abc + abdabcd
+ acd + bcd
= −−15
41 41
= 15

(OSP 2005/ Canadian MO 1996) Jika α, β dan γ adalah akar-akar x3 − x − 1 = 0 maka t ent ukan nilai
Cont oh 53 :

1+α 1+ β 1+γ
1−α + 1− β + 1−γ .

Dengan melihat Ax3 + Bx2 + Cx + D = 0 dan x3 − x − 1 = 0 didapat A = 1, B = 0, C = −1 dan D = −1.


Solusi :

α + β +γ = − B
=0

αβ + αγ + βγ =
A

C
A = −1
1 = −1

αβγ = − D
A = −
(−1)
1
=1

1+α 1+ β 1+γ (1+α )(1− β )(1−γ )+ (1+ β )(1−α )(1−γ )+ (1+γ )(1−α )(1− β )
1−α + 1− β + 1−γ = (1−α )(1− β )(1−γ )
3−(α + β +γ )−(αβ +αγ + βγ )+ 3αβγ
= 1−(α + β +γ )+ (αβ +αγ + βγ )−αβγ

3− (0 )− ( −1)+ 3(1)
= 1− (0 )+ ( −1)− (1)

= −7

LATIHAN 4 :

1. Jika f(x) dibagi dengan (x − 2) sisanya 24, sedangkan j ika dibagi dengan (x + 5) sisanya 10. Jika f(x)
dibagi dengan x2 + 3x − 10 sisanya adalah ⋅⋅⋅⋅⋅⋅

2. Jika v(x) dibagi x2 − x dan x2 + x bert urut -t urut akan bersisa 5x + 1 dan 3x + 1, maka bila v(x) dibagi
x2 − 1 sisanya adalah ⋅⋅⋅⋅

3. (OSP 2006) Jika (x − 1)2 membagi ax4 + bx3 + 1, maka ab = ⋅⋅⋅⋅

4. (OSK 2008) Jika a dan b adalah bilangan-bilangan bulat dan x2 − x − 1 merupakan fakt or dari ax3 +
bx2 + 1, maka b = ⋅⋅⋅⋅⋅⋅⋅⋅⋅⋅

5. (AHSME 1999) Tent ukan banyaknya t it ik pot ong maksimal dari dua grafik y = p(x) dan y = q(x)
dengan p(x) dan q(x) keduanya adalah suku banyak berderaj at empat dan memenuhi koefisien x4
dari kedua suku banyak t ersebut adalah 1.

6. Suku banyak f(x) dibagi (x + 1) sisanya −2 dan dibagi (x − 3) sisanya 7. Sedangkan suku banyak g(x)
j ika dibagi (x + 1) akan bersisa 3 dan j ika dibagi (x − 3) akan bersisa 2. Diket ahui h(x) = f(x) ⋅ g(x).
Jika h(x) dibagi x2 − 2x − 3, maka sisanya adalah ⋅⋅⋅⋅⋅⋅

Eddy Hermanto, ST 27 Aljabar


Pembinaan Olimpiade Matematika

7. (OSP 2009) Misalkan p(x) = x2 − 6 dan A = {x ∈ R⏐p(p(x)) = x}. Nilai maksimal dari {⏐x⏐ : x ∈ A}
adalah ⋅⋅⋅⋅⋅⋅

8. Jika persamaan (3x2 − x + 1)3 dij abarkan dalam suku-sukunya maka akan menj adi persamaan
polinomial a6x6 + a5x5 + a4x4 + a3x3 + a2x2 + a1x + a0. Tent ukan nilai dari :

b) a6 − a5 + a4 − a3 + a2 − a1 + a0
a) a6 + a5 + a4 + a3 + a2 + a1 + a0

c) a6 + a5 + a4 + a3 + a2 + a1
d) a6 + a4 + a2 + a0

9. (OSK 2010) Polinom P(x) = x3 − x2 + x − 2 mempunyai t iga pembuat nol yait u a, b, dan c. Nilai dari
a3 + b3 + c3 adalah ⋅⋅⋅⋅⋅⋅⋅⋅

10. Tent ukan semua nilai m sehingga persamaan x4 − (3m + 2)x2 + m2 = 0 memiliki 4 akar real yang
membent uk barisan arit mat ika.

11. (OSP 2008) Misalkan a, b, c, d bilangan rasional. Jika diket ahui persamaan x4 + ax3 + bx2 + cx + d = 0
mempunyai 4 akar real, dua di ant aranya adalah 2 dan 2008 . Nilai dari a + b + c + d adalah ⋅⋅

12. (OSP 2010) Persamaan kuadrat x2 − px − 2p = 0 mempunyai dua akar real α dan β. Jika α3 + β3 = 16,
maka hasil t ambah semua nilai p yang memenuhi adalah ⋅⋅⋅⋅⋅⋅

13. (AIME 1996) Akar-akar x3 + 3x2 + 4x − 11 = 0 adalah a, b dan c. Persamaan pangkat t iga dengan akar-
akar a + b, a + c dan b + c adalah x3 + rx2 + sx + t = 0. Tent ukan nilai t .

14. (OSK 2003/ Sout h Carolina MC 1996) Misalkan bahwa f(x) = x5 + ax4 + bx3 + cx2 + dx + c dan bahwa
f(1) = f(2) = f(3) = f(4) = f(5). Berapakah nilai a ?

15. (AIME 1993) Misalkan po(x) = x3 + 313x2 − 77x − 8 dan pn(x) = pn−1(x − n). Tent ukan koefisien x dari
p20(x).

16. (OSP 2009) Misalkan a, b, c adalah akar-akar polinom x3 − 8x2 + 4x − 2. Jika f(x) = x3 + px2 + qx + r
adalah polinom dengan akar-akar a + b − c, b + c − a, c + a − b maka f(1) = ⋅⋅⋅⋅⋅⋅

17. (NAHC 1995-1996 Second Round) Misalkan p(x) = x6 + ax5 + bx4 + cx3 + dx2 + ex + f adalah polinomial
yang memenuhi p(1) = 1, p(2) = 2, p(3) = 3, p(4) = 4, p(5) = 5 dan p(6) = 6. Nilai dari p(7) adalah ⋅⋅⋅⋅⋅

18. (OSP 2010) Diberikan polinomial P(x) = x4 + ax3 + bx2 + cx + d dengan a, b, c, dan d konst ant a. Jika

P(12 ) + P(− 8)
P(1) = 10, P(2) = 20, dan P(3) = 30, maka nilai

= ⋅⋅⋅⋅⋅⋅⋅⋅⋅⋅
10
19. (AIME 2003 Bagian Kedua) Akar-akar persamaan x4 − x3 − x2 − 1 = 0 adalah a, b, c dan d. Tent ukan
nilai dari p(a) + p(b) + p(c) + p(d) j ika p(x) = x6 − x5 − x3 − x2 − x.

20. (Canadian MO 1970) Diberikan polinomial f(x) = xn + a1xn-1 + a2xn-2 + ⋅⋅⋅ + an-1x + an dengan koefisien
a1, a2, ⋅⋅⋅, an semuanya bulat dan ada 4 bilangan bulat berbeda a, b, c dan d yang memenuhi f(a) =
f(b) = f(c) = f(d) = 5. Tunj ukkan bahwa t idak ada bilangan bulat k yang memenuhi f(k) = 8.

Eddy Hermanto, ST 28 Aljabar


Pembinaan Olimpiade Matematika

5. PERSAMAAN
Ada beberapa persamaan yang akan dibahas, yait u :

A. Persamaan Kuadrat
Bent uk persamaan kuadrat adalah Ax2 + Bx + C = 0.
1) Pengert ian akar
Misalkan x1 dan x2 adalah nilai x yang memenuhi persamaan kuadrat di at as. Nilai x1 dan x2
dikenal j uga dengan akar-akar. Maka berlaku.
Ax12 + Bx1 + C = 0
Ax22 + Bx2 + C = 0

2) Menent ukan nilai akar-akar persamaan kuadrat


Unt uk mencari nilai x yang memenuhi dapat dicari dengan cara kuadrat sempurna,
memfakt orkan maupun dengan menggunakan rumus x1,2 = − B ± 2BA− 4 AC sebagaimana yang t elah
2

Persamaan B2 − 4AC dikenal dengan nama diskriminan. Nilai diskriminan ini menent ukan j enis-
didapat kan dari pelaj aran di kelas.

• Jika B2 − 4AC > 0 maka x1 dan x2 keduanya real dan berbeda.


j enis akar (nilai x1 dan x2). Ada t iga kemungkinan nilai diskriminan.

• Jika B2 − 4AC = 0 maka x1 = x2 sert a x1 dan x2 keduanya real.


• Jika B2 − 4AC < 0 maka x1 dan x2 keduanya t idak real.

3) Hubungan kedua akar

persamaan x2 − (x1 + x2)x + x1x2 = 0.


Persamaan kuadrat yang memiliki akar-akar x1 dan x2 dapat dit uliskan ke dalam bent uk

Misalkan t erdapat persamaan kuadrat Ax2 + Bx + C = 0 yang memiliki akar-akar x1 dan x2. Maka

x1 + x2 = − BA
hubungan ant ara x1 dan x2 adalah sebagai berikut .

x1 ⋅ x2 = C
A

4) Menent ukan persamaan kuadrat baru.


Misalkan persamaan kuadrat Ax2 + Bx + C = 0 memiliki akar-akar x1 dan x2. Ada beberapa cara
j ika ingin menent ukan persamaan kuadrat yang memiliki akar-akar x3 dan x4 dan memiliki

a. Membawa ke dalam persamaan x2 − (x3 + x4)x + x3x4 = 0.


hubungan t ert ent u dengan x1 dan x2.

Misalkan t erdapat persamaan kuadrat Ax2 + Bx + C = 0 yang memiliki akar-akar x1 dan x2.
Dari ket erangan sebelumnya akan didapat kan nilai dari x1 + x2 dan x1x2.
Jika dapat dit ent ukan nilai dari x3 + x4 dan x3x4 ke dalam bent uk x1 + x2 dan x1x2 maka

kuadrat yang memiliki akar-akar x3 dan x4 yait u x2 − (x3 + x4)x + x3x4 = 0.


berart i nilai dari x3 + x4 dan x3x4 dapat dit ent ukan sehingga akan didapat persamaan

b. Melakukan subt it usi set elah menghilangkan indeks


Jika dari hubungan x3 dan x4 yang memiliki hubungan t ert ent u dengan x1 dan x2 kit a
hilangkan indeksnya lalu kit a subt it usikan ke persamaan semula dan mendapat kan
persamaan kuadrat baru. Maka persamaan kudarat t ersebut memiliki akar-akar x3 dan x4.

5) Menent ukan nilai suat u bilangan yang berbent uk a + b + 2 ab dan a + b − 2 ab


a + b dan a − b keduanya dikuadrat kan akan didapat
( )
Jika

a+ b = a + b + 2 ab
( b)
2

a− = a + b − 2 ab
2

Eddy Hermanto, ST 29 Aljabar


Pembinaan Olimpiade Matematika

Sehingga dapat dit ent ukan nilai dari a + b + 2 ab dan a + b − 2 ab , yait u


a + b + 2 ab = a + b
a + b − 2 ab = a − b dengan syarat a ≥ b.

Jika salah sat u akar x2 + (a + 1)x + (3a + 2) = 0 adalah 5, maka akar lainnya adalah ⋅⋅⋅⋅
Cont oh 54 :

Solusi :

52 + (a + 1) ⋅ 5 + (3a + 2) = 0
Sesuai pengert ian akar maka akan didapat

a = −4

x2 + (−4 + 1)x − (3(−4) + 2) = 0


Persamaan kuadrat t ersebut adalah

x2 − 3x − 10 = 0
(x − 5)(x + 2) = 0
x1 = 5 dan x2 = −2
Jadi, akar lainnya adalah −2.

+
Cont oh 55 :
Jika x1 dan x2 akar-akar persamaan cx2 + bx + a = 0, maka 1 1
= ⋅⋅⋅⋅⋅⋅
x12 x22

Solusi :
x1 + x2 = − bc
a
x1x2 =

+
c
x12 + x22
( x1 x2 )2
1 1
=
x12 x22

( x1 + x2 )2 − 2 x1 x2
1
x12
+ 1
x22
=
( x1 x2 )2
1
x12
+ 1
x22
= b 2 − 2 ac
a2

Persamaan kuadrat yang akar-akarnya 3 lebihnya dari akar-akar persamaan kuadrat x2 + 5x − 24 = 0


Cont oh 56 :

adalah ⋅⋅⋅⋅⋅⋅⋅

Misalkan x1 dan x2 adalah akar-akar persamaan kuadrat x2 + 5x − 24 = 0.


Solusi :

Maksud soal adalah menent ukan persamaan kuadrat yang akar-akarnya x 3 = x1 + 3 dan x4 = x2 + 3.

x1 + x2 = −5 dan x1x2 = −24


Alt ernat if 1 :

x3 ⋅ x4 = (x1 + 3)(x2 + 3) = x1x2 + 3(x1 + x2) + 9 = −24 − 15 + 9 = −30


x3 + x4 = (x1 + x2) + 6 = 1

Persamaan kuadrat baru adalah x2 − (x3 + x4)x + x3x4 = 0.


Jadi, persamaan kuadrat yang dimint a adalah x2 − x − 30 = 0

Eddy Hermanto, ST 30 Aljabar


Pembinaan Olimpiade Matematika
Alt ernat if 2 :

Jika indeks dihilangkan akan didapat y = x + 3. Subt it usikan x = y − 3 ke persamaan semula.


Misalkan y3 = x1 + 3 dan y4 = x2 + 3

(y − 3) 2 + 5(y − 3) − 24 = 0
y2 − y − 30 = 0
y2 − y − 30 = 0 merupakan persamaan kuadrat yang akar-akarnya x1 + 3 dan x2 + 3.
Jadi, persamaan kuadrat yang dimint a adalah x2 − x − 30 = 0

Cont oh 57 :
8 + 4 3 = LL

Solusi :
8 + 4 3 = 8 + 2 12 = 6 + 2 + 2 6 ⋅ 2 . Memperhat ikan a + b + 2 ab = a + b , maka
8+4 3 = 6 + 2

Cont oh 58 :
(AIME 1983) Tent ukan hasil kali semua akar-akar real x 2 + 18 x + 30 = 2 x 2 + 18 x + 45 .

Solusi :
Misalkan y = x2 + 18x + 30 dan kuadrat kan kedua ruas persamaan semula, maka
y2 = 4(y + 15)
(y − 10)(y + 6) = 0
Karena akar dari suat u bilangan t ak mungkin negat if maka nilai y yang memenuhi hanya y = 10.
Jadi, t erdapat dua akar yang t idak real.
x2 + 18x + 30 = 10 sehingga x2 + 18x + 20 = 0
Jadi, hasil kali kedua akar persamaan x2 + 18x + 20 = 0 sama dengan 20.

Cont oh 59 :

b + b +10 a = 2
(OSK 2002) Misalkan a dan b bilangan real yang berbeda sehingga
a a +10 b

a
Tent ukan nilai b .

+ ba++10
Solusi :
a 10 b
Karena a = 2 maka

+ 1b+10 a = 2
b

a
a
+10

= x , maka
b
b

a
Misal
= 2− x
b
x +10
1+10 x
x + 10 = 2 − 10x2 + 19x
(5x − 4) (x − 1) = 0
x = 1 at au x = 54
Jadi, karena a ≠ b, maka x ≠ 1.
Jadi, ba = 54

Eddy Hermanto, ST 31 Aljabar


Pembinaan Olimpiade Matematika

LATIHAN 5.A

1. Persamaan kuadrat yang akar-akarnya dua lebih besar dari akar-akar x2 + px + 1 = 0 t api t iga
lebih kecil dari akar-akar persamaan 2x2 − 3x + q = 0 adalah ⋅⋅⋅⋅

1+ 3 x 2
maka bat as-bat as p supaya x real adalah ⋅⋅⋅
x− x2
2. Jika p =

3. Jika kedua akar persamaan kuadrat x2 − px + p = 0 bernilai real posit if, maka bat as-bat as nilai p
yang memenuhi adalah ⋅⋅⋅⋅

4. Jika x1 dan x2 akar-akar persamaan x2 + 2x + 4 = 0, maka persamaan kuadrat yang akar-akarnya


x −1 dan x −1 adalah ⋅⋅⋅⋅⋅
1 1
1 2

5. (OSK 2005) Misalkan a dan b adalah bilangan real t aknol yang memenuhi 9a2 − 12ab + 4b2 = 0.

( ) ( )
Tent ukan ba .

6. (AIME 1990) Tent ukan nilai dari 52 + 6 43 − 52 − 6 43


3/ 2 3/ 2
.

7. (AIME 1990) Tent ukan penyelesaian posit if x 2 −10 x − 29


1
+ x 2 −10 x − 45
1
= x 2 −10 x − 69
2
.

7−4 x
4 12
8. (OSK 2011 Tipe 1) Jumlah dari seluruh solusi persamaan x = adalah ....

9. Diket ahui α dan β adalah dua bilangan berbeda yang memenuhi α2 = 5α − 9 dan β2 = 5β − 9.
Nilai dari 1α + 1β adalah ⋅⋅⋅⋅⋅⋅

10. (ARML 1999 Individual) Jika a dan b adalah akar-akar persamaan kuadrat 11x2 − 4x − 2 = 0.

(1 + a + a2 + ⋅⋅⋅)(1 + b + b2 + ⋅⋅⋅)
hit unglah nilai dari :

11. (OSP 2002) Tinj au persamaan yang berbent uk x2 + bx + c = 0. Berapa banyakkah persamaan
demikian yang memiliki akar-akar real j ika koefisien b dan c hanya boleh dipilih dari himpunan
{1,2,3,4,5,6} ?

12. (OSK 2010) Jika a dan b bilangan bulat sehingga 2010 + 2 2009 merupakan solusi kuadrat
x2 + ax + b = 0 maka nilai a + b adalah ⋅⋅⋅⋅⋅

13. (OSK 2010) Diket ahui bahwa ada t epat 1 bilangan asli n sehingga n2 + n + 2010 merupakan
kuadrat sempurna. Bilangan asli n t ersebut adalah ⋅⋅⋅⋅⋅⋅⋅

14. Akar-akar persamaan kuadrat x2 + 6x + c = 0 adalah x1 dan x2 sedangkan akar-akar persamaan


kuadrat x2 + (x12 + x22)x + 4 = 0 adalah u dan v. Jika u + v = −uv maka nilai dari x13x2 + x1x23
adalah ⋅⋅⋅⋅⋅⋅⋅⋅

15. α dan β adalah akar-akar persamaan kuadrat x2 − 3(a − 1)x + 2a2 + 4b = 0. Jika α = 2β maka nilai
dari a + b = ⋅⋅⋅⋅⋅⋅⋅⋅

Eddy Hermanto, ST 32 Aljabar


Pembinaan Olimpiade Matematika
β
α
m
16. Jika suku pert ama deret geomet ri t ak hingga adalah 2 dan rasionya adalah r = = unt uk
nilai m > 0 dan α, β akar-akar x − (3m + 2) + (4m + 12) = 0, maka j umlah deret geomet ri t ak
m
2

hingga t ersebut adalah ⋅⋅⋅

17. Diket ahui x2 − (2p + 1)x + p = 0 dengan akar-akar x1 dan x2 sert a 3x2 − (q − 1)x − 1 = 0 dengan
akar-akar x3 dan x4. Jika x1x3 = 1 dan x2x4 = 1, maka nilai dari p − 2q + 13 = ⋅⋅⋅⋅⋅⋅⋅⋅⋅⋅

18. Jika a ≠ b dan j ika persamaan-persamaan x2 + ax + bc = 0 dan x2 + bx + ac = 0 mempunyai t epat


sebuah akar persekut uan, t unj ukkan bahwa akar-akar yang lain dari kedua persamaan t ersebut
memenuhi persamaan x2 + cx + ab = 0.

19. Misalkan α dan β adalah akar-akar persamaan x2 + px + 1 = 0 sedangkan γ dan δ adalah akar-akar
persamaan x2 + qx + 1 = 0. Bukt ikan bahwa (α − γ)(β − γ)(α + δ)(β + δ) = q2 − p2.

20. (AIME 1991) Misalkan k adalah penj umlahan semua nilai mut lak dari nilai-nilai x yang

x= 19 +
memenuhi 91 . Tent ukan nilai dari k2.

19 +
91
19 +
91
19 +
91
19 +
91
x

21. Diket ahui b1, c1, b2 dan c2 adalah bilangan real yang memenuhi b1b2 = 2(c1 + c2). Tunj ukkan
bahwa sedikit nya sat u dari dua persamaan x2 + b1x + c1 = 0 dan x2 + b2x + c2 = 0 memiliki akar-
akar real.

22. Diberikan a, b, c ∈ bilangan real sert a a dan 4a + 3b + 2c mempunyai t anda yang sama.
Tunj ukkan bahwa persamaan ax2 + bx + c = 0 kedua akarnya t idak mungkin t erlet ak pada
int erval (1, 2).

B. Persamaan Eksponen

ao = 1 unt uk a ≠ 0
Dalam pembahasan hanya akan disinggung t ent ang sifat -sifat pada eksponen, yait u :

a n = a1⋅ 42
⋅ a ⋅43
⋅ a unt uk n ∈ N.
(i)
(ii) a4 L4

a ⋅a = a
nkali
b c b+c
(iii)

unt uk a ≠ 0
ab b −c

(a )
(iv) c
= a
a
(v)
b c
= a bc
a −m = m unt uk a ≠ 0
1
(vi)
a

a = a dengan syarat a ≥ 0.
1
2
(vii)

am = a n
m
n
(viii)

Eddy Hermanto, ST 33 Aljabar


Pembinaan Olimpiade Matematika
Cont oh 60 :
Harga x yang memenuhi persamaan 4 x +3 = 4 8 x +5 adalah ⋅⋅⋅⋅⋅

Solusi :
4 x +3 = 4 8 x +5
3( x + 5 )
2 =2
2( x +3) 4
(sifat (v) dan sifat (viii))

x = − 95
8(x + 3) = 3(x + 5)

Jadi, nilai x yang memenuhi adalah x = − 95 .

Cont oh 61 :
Manakah yang lebih besar : 2175 at au 575 ? Bukt ikan.

Solusi :
2175 = (27) 25 = 12825 dan 575 = (53) 25 = 12525
12825 > 12525
2175 > 575
Jadi, 2175 lebih besar dari 575.

(2 )
Cont oh 62 :

(4 )
sama dengan ⋅⋅⋅⋅⋅
4 8
(OSK 2002) Bilangan
8 2

(2 )
Solusi :

(4 )
= = =1
4 8
2 32 2 32
8 2 416 2 32

( )
LATIHAN 5.B

1. Persamaan 3 ⋅ 27 2 x −1 = memberikan nilai x sama dengan ⋅⋅⋅⋅⋅


3x
3 1
243

(2 ) −(2 ) adalah ⋅⋅⋅⋅⋅⋅


(2 ) − (2 )
2010 2 2008 2
2. (OSK 2011 Tipe 3) Nilai dari 2011 2 2009 2

3. Jika 53x = 8, maka 53 + x = ⋅⋅⋅⋅⋅⋅

4. Jumlah akar-akar persamaan 5x+1 + 56−x = 11 adalah ⋅⋅⋅⋅⋅⋅⋅⋅⋅

5. Himpunan penyelesaian dari 5


8− 2 x
+ 49 ⋅ 5 3− x − 2 = 0 adalah ⋅⋅⋅⋅⋅

x 2 −3 x + 2
+ 3x −3 x
= 10 . Jika x1 dan x2 adalah penyelesaiannya, maka
2

= LL
6. Diberikan persamaan 3
x1 + x2
3

7. Persamaan 54(6x) + 3x = 6(18x) + 9 mempunyai penyelesaian x1 dan x2, maka (x1 ⋅ x2) 2 = ⋅⋅⋅⋅⋅

Eddy Hermanto, ST 34 Aljabar


Pembinaan Olimpiade Matematika

C. Persamaan Logarit ma
Pengert ian : Jika ab = c maka b = alog c.
Sifat -sifat pada logarit ma, yait u :

log b = = dengan syarat a, p ≠ 1 dan a, b, p > 0


p
a log b log b
(i) p
log a log a
(ii)
a
log a = 1 dengan syarat a > 0 dan a ≠ 1.
log b = b dengan a,b ≠ 1 dan a,b > 0
a 1
(iii)
log a
log b + alog c = alog (bc) dengan syarat a ≠ 1 dan a, b, c > 0
log b − a log c = a log ( bc ) dengan syarat a ≠ 1 dan a, b, c > 0
a
(iv)
a
(v)

log b m = a log b n = ⋅ log b dengan syarat a ≠ 1 dan a, b > 0


m
an ma
(vi)

log bn = n ⋅ alog b dengan syarat a ≠ 1 dan a, b > 0


n
a

log b⋅b log c = a log c dengan syarat a, b ≠ 1 dan a, b, c > 0


(vii)
a
(viii)

= b dengan syarat a ≠ 1 dan a, b > 0.


n
am
log b n m
(ix) a
Cat at an : Bent uk alog b kadang-kadang dit ulis dengan loga b.

Jika 5log 3 = a dan 3log 4 = b, maka 4log 15 = ⋅⋅⋅⋅⋅⋅


Cont oh 63 :

Berdasarkan sifat (viii) maka didapat 5log 3 ⋅ 3log 4 = 5log 4 = ab


Solusi :

Berdasarkan sifat (iv) maka 4log 15 = 4log 3 + 4log 5


Berdasarkan sifat (iii) maka 4log 15 = 1b + ab
1

a +1
Maka 4log 15 = ab

Cont oh 64 :
Himpunan penyelesaian pert idaksamaan :
log ( 2 x −3 )
− xlog (x + 6) +
2
1
x+2 =1
adalah ⋅⋅⋅⋅⋅
2
log x log x

Solusi :

log (2x − 3) − xlog (x + 6) + xlog (x + 2) = 1


Berdasarkan sifat (i) dan (iii) maka persoalan di at as ekivalen dengan

(( )
x

2 x − 3 )( x + 2 )
Berdasarkan sifat (iv) dan (v) maka
x+6
x
log =1

(2x − 3)(x + 2) = x(x + 6)


Berdasarkan kesamaan maka

2x2 + x − 6 = x2 + 6x
x2 − 5x − 6 = 0
(x − 6)(x + 1) = 0

Eddy Hermanto, ST 35 Aljabar


Pembinaan Olimpiade Matematika

x1 = 6 dan x2 = −1

x = −1 t idak memenuhi syarat basis pada xlog (x + 6) yait u bahwa x > 0 dan x ≠ 1.
Uj i kedua nilai x t ersebut ke persamaan awal.

Sedangkan x = 6 memenuhi syarat basis mapun numerus.


Jadi, himpunan semua nilai x yang memenuhi adalah {6}.

(OSK 2004/ AHSME 1952) Jika log p + log q = log (p + q), maka p dinyat akan dalam q adalah p = ⋅⋅⋅⋅
Cont oh 65 :

Solusi :
log p + log q = log (p + q)
log (pq) = log (p + q)

p(q − 1) = q
pq = p + q

q −1
q
Jadi, p =

Cont oh 66 :

= dan = c k , maka k = ⋅⋅⋅


b
log a 1 b
Jika c
log a 2 c

Solusi :
Berdasarkan sifat (i) akan didapat

= = =
b
log a log c log c 1
c k +1
log a log b log c 2
Maka k + 1 = 2
Jadi, k = 1

Cont oh 67 :
(OSP 2003) Berapakah nilai x yang memenuhi 4log (2log x) + 2log (4log x) = 2 ?

Solusi :
Berdasarkan sifat (vi) maka 4log (2log x) = 1 2 2
2 log ( log x) sehingga 4log ( 2log x) = 2log (2log x)1/ 2
Berdasarkan sifat (vi) maka 2log (4log x) = 2
log ( 12 2log x)
4 2 2 4 2 2 1/ 2
log ( log x) + log ( log x) = 2 sehingga log ( log x) + 2log ( 1 2
2 log x) = 2

Maka 2log (
2
log x ⋅ 12 ⋅ 2 log x ) = 2
log x ⋅ 12 ⋅ 2 log x = 2 2 = 4
( log x )
2

=8
2 3/ 2

2
log x = 4
x = 42
Jadi, x = 16

Eddy Hermanto, ST 36 Aljabar


Pembinaan Olimpiade Matematika
LATIHAN 5.C

1.
( 3
) (
log 45 − 3 log 5
2
)2

= ⋅⋅⋅⋅⋅⋅⋅
3 3
log 15

2. (OSK 2003) Misalkan 3a = 4, 4b = 5, 5c = 6, 6d = 7, 7e = 8, dan 8f = 9. Berapakah hasil kali abcdef ?

3. (AIME 1984) Bilangan real x dan y memenuhi 8log x + 4log y2 = 5 dan 8log y + 4log x2 = 7.
Tent ukan xy.

+ + +L+
4. (AHSME 1998) Tent ukan nilai dari
1 1 1 1

dengan n! = 1 x 2 x 3 x ⋅⋅⋅ x n.
2 3 4 100
log 100! log 100! log 100! log 100!

5. Bat as-bat as nilai p agar f(x) = 2log (px2 + px + 3) t erdefinisi pada set iap nilai x adalah ⋅⋅⋅⋅⋅

log x + 3 log y log x + 3 log y


= 3 unt uk x > y, maka x + y = ⋅⋅⋅⋅⋅
3 3
6. Jika x = 27 dan y
7. Diket ahui barisan 6 bilangan u1, u2, u3, u4, u5 dan u6 sert a u1 + u6 = 11 dan 10log u3 + 10log u4 = 1.
Jika unt uk set iap n = 1, 2, 3, ⋅⋅⋅, 5 berlaku un+1 = p⋅un dan p > 1, maka 10log p = ⋅⋅⋅⋅

8. (AIME 1983) Diket ahui x, y dan z adalah bilangan real lebih dari 1 dan w adalah bilangan real
posit if. Jika xlog w = 24, ylog w = 40 dan xyzlog w = 12, t ent ukan zlog w.

9. (AIME 1988) Diberikan 2log ( 8log x) = 8log (2log x). Tent ukan nilai dari ( 2log x)2.

10. (AHSME 1997) Unt uk bilangan asli n maka


8
log n j ika 8log n bilangan rasional
f(n) =

∑ f (n )
0 unt uk lainnya
1997
Nilai dari 3
n =1

11. (AHSME 1998) Ada berapa banyak bilangan prima yang merupakan fakt or dari N dan memenuhi
2
log (3log ( 5log ( 7log N))) = 11.

( ) ( log 5 ) + ( ) ( log 5 ) + ( ) ( log 5 ) + ...


12. (ARML 2000 Individual) Jika b = 2000, hit unglah nilai deret t ak hingga berikut :
b 0 b 40 b 1 b 41 b 2 b 42
log 2 log 2 log 2

13. (AIME 2002) Penyelesaian dari sist em persamaan 225log x + 64log y = 4 dan xlog 225 − ylog 64 = 1
adalah (x, y) = (x1, y1) dan (x2, y2). Nilai dari 30log (x1y1x2y2) adalah ⋅⋅⋅⋅⋅⋅

D. Persamaan Lingkaran
1) Persamaan Lingkaran berpusat di (0,0) dan (a,b)
Lingkaran adalah kumpulan t it ik-t it ik yang memiliki j arak yang sama t erhadap suat u t it ik
t ert ent u, yait u pusat lingkaran. Jadi ada dua hal yang sangat berkait an dengan lingkaran yait u
j ari-j ari lingkaran, R, dan pusat lingkaran.
Dari pengert ian lingkaran t ersebut j ika dit urunkan akan didapat persamaan :
x2 + y2 = r 2 yang merupakan persamaan lingkaran berpusat di (0,0) dan berj ari-j ari r.

Eddy Hermanto, ST 37 Aljabar


Pembinaan Olimpiade Matematika
(x − a)2 + (y − b) 2 = r 2 yang merupakan persamaan lingkaran berpusat di (a,b) dan berj ari-j ari r.
Jika persamaan (x − a)2 + (y − b)2 = r 2 dij abarkan akan didapat persamaan umum lingkaran yang
berbent uk :
x2 + y2 + Ax + By + C = 0
Salah sat u cara menent ukan persamaan lingkaran j ika diket ahui pusat lingkaran dan persamaan
garis yang menyinggung lingkaran t ersebut adalah dengan memanfaat kan rumus j arak t it ik ke
suat u garis lurus sebab j arak t it ik pusat ke garis singgung t ersebut adalah merupakan j ari-j ari
lingkaran. Misalkan suat u garis lurus memiliki persamaan Ax + By + C = 0. Maka rumus j arak t it ik
Ax1 + By1 + C
(x1, y1) ke garis t ersebut adalah d =
A2 + B 2
.

Misalkan t erdapat lingkaran dengan persamaan (x − a)2 + (y − b) 2 = r 2 dan t it ik (p, q). Maka
2) Hubungan ant ara t it ik dengan lingkaran

hubungan t it ik (p, q) dengan (x − a)2 + (y − b) 2 = r 2 akan memiliki t iga kemungkinan hubungan :


a) Jika (p − a)2 + (q − b) 2 < r 2 maka t it ik (p, q) t erlet ak di dalam lingkaran
b) Jika (p − a)2 + (q − b) 2 = r 2 maka t it ik (p, q) t erlet ak pada lingkaran
c) Jika (p − a)2 + (q − b) 2 > r 2 maka t it ik (p, q) t erlet ak di luar lingkaran

Misalkan diket ahui suat u garis lurus y = mx + c dan lingkaran (x − a)2 + (y − b)2 = r 2. Bagaimana
3) Hubungan ant ara garis lurus dengan lingkaran

Subt it usikan persamaan y = mx + c ke persamaan lingkaran (x − a) 2 + (y − b) 2 = r 2 sehingga


hubungan ant ara garis lurus dan lingkaran t ersebut ?

didapat suat u persamaan kuadrat dalam peubah x, yait u Ax2 + Bx + C = 0.


Dari persamaan t ersebut dapat dihit ung diskriminan = B2 − 4AC.
Jika B2 − 4AC < 0 maka garis lurus t idak memot ong lingkaran
(ii) Jika B2 − 4AC = 0 maka garis lurus menyinggung lingkaran
(i)

(iii) Jika B2 − 4AC > 0 maka garis lurus memot ong lingkaran di dua t it ik
Prinsip nilai diskriminan di at as t idak hanya dapat digunakan unt uk mencari hubungan ant ara
garis lurus dengan lingkaran t et api j uga hubungan ant ara garis lurus dengan irisan kerucut yang
lain sepert i parabola, elips maupun hiperbola.

4) Persamaan Garis Singgung pada Lingkaran


a) Garis singgung lingkaran dengan gradien t ert ent u
Misalkan diket ahui bahwa garis singgung t ersebut memiliki gradien m. Maka persamaan garis
singgung dapat dinyat akan dengan
(i) Unt uk lingkaran x2 + y2 = r 2
Persamaan Garis Singgung, y = mx ± r m + 1
2

Unt uk lingkaran (x − a)2 + (y − b) 2 = r 2


Persamaan Garis Singgung, y − b = m( x − a ) ± r m + 1
(ii)
2

b) Garis Singgung melalui t it ik pada lingkaran


Misalkan t it ik (x1, y1) t erlet ak pada lingkaran maka persamaan garis singgung yang melalui
t it ik t ersebut dapat dit ent ukan dengan
(i) Unt uk lingkaran x2 + y2 = r 2
Persamaan Garis Singgung, x1x + y1y = r 2
(ii) Unt uk lingkaran (x − a)2 + (y − b) 2 = r 2
Persamaan Garis Singgung, (x1 − a)(x − a) + (y1 − b)(y − b) = r 2

c) Persamaan Garis Singgung melalui t it ik di luar lingkaran


Unt uk menent ukan persamaan garis singgung ini dapat dilakukan dengan beberapa cara :

Eddy Hermanto, ST 38 Aljabar


Pembinaan Olimpiade Matematika
(i) Dengan mencari rumus diskriminan lalu memanfaat kan pengert ian hubungan ant ara
garis lurus dengan lingkaran
(ii) Dengan menggunakan persamaan garis polar
(iii) Dengan memanfaat kan persamaan garis singgung dengan gradien m unt uk mencari
nilai m

Cont oh 68 :
(OSK 2005) Tit ik A(a, b) disebut t it ik let is j ika a dan b keduanya adalah bilangan bulat . Banyaknya
t it ik let is pada lingkaran yang berpusat di O dan berj ari-j ari 5 adalah

Solusi :
Persamaan lingkaran yang berpusat di O dan berj ari-j ari 5 adalah x2 + y2 = 25
Karena 02 + 52 = 32 + 42 = 25 maka pasangan (x, y) bulat yang memenuhi ada 12, yait u (0, 5), (0, −5),
(5, 0), (−5, 0), (3, 4), (3, −4), (−3, 4), (−3, −4), (4, 3), (4, −3), (−4, 3) dan (−4, −3).
Jadi, banyaknya t it ik let is pada lingkaran yang berpusat di O dan berj ari-j ari 5 ada 12.

Persamaan lingkaran yang berpusat di (1, 4) dan menyinggung garis 3x − 4y − 2 = 0 adalah ⋅⋅⋅⋅⋅⋅
Cont oh 69 :

Jarak pusat (1, 4) ke garis 3x − 4y − 2 = 0 sama dengan j ari-j ari lingkaran t ersebut .
Solusi :

3(1) − 4(4 ) − 2
32 + 4 2
Jarak t ersebut = d = = 3.

(x − 1)2 + (y − 4) 2 = 9
Persamaan lingkaran berpusat di (1, 4) dan memiliki j ari-j ari 3 adalah

Cont oh 70 :
(OSK 2002) Unt uk nilai a yang manakah garis lurus y = 6x memot ong parabola y = x2 + a t epat di sat u
t it ik ?

Karena 6x = x2 + a maka x2 − 6x + a = 0
Solusi :

Diskriminan = 62 − 4(1)(a) = 36 − 4a
Syarat agar y = 6x memot ong parabola y = x2 + a di sat u t it ik adalah Diskriminan = 0
36 − 4a = 0
Jadi, a = 9

Persamaan garis singgung x2 + y2 − 6x + 4y − 12 = 0 di t it ik (7, −5) adalah ⋅⋅⋅⋅⋅


Cont oh 71 :

Subt it usi t it ik (7, −5) ke persamaan x2 + y2 − 6x + 4y − 12 = 0 didapat


Solusi :

(7) 2 + (−5) 2 − 6(7) + 4(−5) − 12 = 0


Art inya t it ik (7, −5) t erlet ak pada lingkaran x2 + y2 − 6x + 4y − 12 = 0.
Persamaan garis singgungnya adalah (x − 3)(7 − 3) + (y + 2)(−5 + 2) = 25
Jadi, persamaan garis singgung lingkaran x2 + y2 − 6x + 4y − 12 = 0 di t it ik (7, −5) adalah
4x − 3y = 43

Eddy Hermanto, ST 39 Aljabar


Pembinaan Olimpiade Matematika

Persamaan garis singgung yang dit arik dari t it ik (4,2) ke lingkaran x2 + y2 = 10 adalah ⋅⋅⋅⋅⋅⋅
Cont oh 72 :

Solusi :
Karena 42 + 22 > 10 maka t it ik (4,2) t erlet ak di luar lingkaran.

Persamaan garis melalui t it ik (4,2) dan gradien m adalah y − 2 = m(x − 4). Subt it usi garis t ersebut
Alt ernat if 1 :

x2 + (mx − 4m + 2) 2 = 10
ke persamaan lingkaran didapat

(m2 + 1)x2 + 2(−4m2 + 2m)x + 16m2 − 16m − 6 = 0


Diskriminan = 22(−4m2 + 2m) 2 − 4(m2 + 1)(16m2 − 16m − 6)
Agar y − 2 = m(x − 4) menyinggung lingkaran x2 + y2 = 10 maka diskriminan harus sama dengan 0.
22(−4m2 + 2m) 2 − 4(m2 + 1)(16m2 − 16m − 6) = 0
16m4 − 16m3 + 4m2 − 16m4 + 16m3 + 6m2 −16m2 + 16m + 6 = 0
3m2 − 8m − 3 = 0
(3m + 1)(m − 3) = 0
Jika m = − 13 maka garis singgung t ersebut memiliki persamaan y − 2 = − 13 (x − 4).
Jika m = 3 maka garis singgung t ersebut memiliki persamaan y − 2 = 3(x − 4).
Jadi, persamaan garis singgung yang dit arik dari t it ik (4,2) adalah x + 3y = 10 dan 3x − y = 10.

Alt ernat if 2 :
Misalkan t it ik (xo, yo) = (4, 2).
Persamaan garis polar t it ik (xo, yo) t erhadap lingkaran x2 + y2 = 10 adalah xox + yoy = 10 yait u
2x + y = 5
Subt it usikan persamaan garis polar t ersebut ke lingkaran x2 + y2 = 10 didapat
x2 + (5 − 2x) 2 = 10
x2 − 4x + 3 = 0
x1 = 1 at au x2 = 3
Jika x1 = 1 maka y1 = 3 sehingga t it ik singgung dari garis singgung t ersebut pada lingkaran adalah

Jika x2 = 3 maka y2 = −1 sehingga t it ik singgung dari garis singgung t ersebut pada lingkaran adalah
(1,3) sehingga persamaan garis singgungnya adalah x + 3y = 10.

(3,−1) sehingga persamaan garis singgungnya adalah 3x − y = 10.


Jadi, persamaan garis singgung yang dit arik dari t it ik (4,2) adalah x + 3y = 10 dan 3x − y = 10.

Alt ernat if 3 :
Misalkan gradien garis singgung t ersebut adalah m. Maka persamaan garis singgung t ersebut adalah
y = mx ± r m 2 + 1
Karena r 2 = 10 maka y = mx ± 10m + 10
2

10m 2 + 10 = ±(2 − 4m )
Karena garis t ersebut melalui t it ik (4,2) maka

10m2 + 10 = 4 − 16m + 16m2


(3m + 1)(m − 3) = 0
Jika m = − 13 maka garis singgung t ersebut memiliki persamaan y − 2 = − 13 (x − 4).
Jika m = 3 maka garis singgung t ersebut memiliki persamaan y − 2 = 3(x − 4).
Jadi, persamaan garis singgung yang dit arik dari t it ik (4,2) adalah x + 3y = 10 dan 3x − y = 10.

Eddy Hermanto, ST 40 Aljabar


Pembinaan Olimpiade Matematika
LATIHAN 5.D

1. Persamaan lingkaran dengan t it ik pusat (4,3) dan j ari-j ari = 4 adalah ⋅⋅⋅⋅⋅⋅

di t it ik (5,0) adalah ⋅⋅⋅⋅⋅


2. Persamaan lingkaran yang t it ik pusat nya t erlet ak pada garis y = x + 1 dan menyinggung sumbu X

persamaan ⋅⋅⋅⋅⋅⋅
3. Suat u lingkaran berj ari-j ari 5, melalui t it ik (0,0) dan pusat nya pada garis y = x + 1 mempunyai

4. Diket ahui t it ik A(−2,1) , B(4,−3) dan P(x,y) t erlet ak sedemikian sehingga (PA) 2 + (PB)2 = (AB) 2.

koordinat ⋅⋅⋅⋅⋅⋅⋅⋅
Maka P merupakan t it ik-t it ik yang t erlet ak pada busur lingkaran yang memot ong sumbu X pada

5. Persamaan garis singgung di t it ik (7,−1) pada lingkaran x2 + y2 = 50 adalah ⋅⋅⋅⋅⋅

6. Garis lurus 3x + 4y + k = 0 akan menyinggung lingkaran x2 + y2 + 6x + 8y = 0 j ika k bernilai ⋅⋅⋅⋅⋅⋅⋅

7. Jari-j ari lingkaran yang menyinggung sumbu x di (6,0) dan menyinggung garis y = √3 x adalah ⋅⋅⋅⋅

8. Persamaan garis singgung yang dit arik dari t it ik (7,−1) ke lingkaran x2 + y2 = 40 adalah ⋅⋅⋅⋅

9. Persamaan garis singgung pada lingkaran x2 + y2 = 36 yang t egak lurus garis 4y = −3x + 80 adalah

10. Jarak t erj auh dari t it ik (−12,5) ke lingkaran dengan persamaan x2 + y2 = 100 adalah ⋅⋅⋅⋅

11. Bilangan real x dan y memenuhi (x + 5) 2 + (y − 12) 2 = 142, maka nilai minimum dari x2 + y2
adalah ⋅⋅⋅⋅⋅⋅

12. (AHSME 1998) Kedua grafik x2 + y2 = 4 + 12x + 6y dan x2 + y2 = k + 4x + 12y memiliki t it ik pot ong
j ika k memenuhi a ≤ k ≤ b. Nilai dari b − a adalah ⋅⋅⋅⋅⋅

13. (AHSME 1996) Diberikan persamaan x2 + y2 = 14x + 6y + 6. Nilai t erkecil dari 3x + 4y yang
memenuhi adalah ⋅⋅⋅⋅⋅

Nilai mut lak dari x dit ulis dengan ⏐x⏐ dan memiliki pengert ian ⏐x⏐ = x j ika x ≥ 0 dan ⏐x⏐ = −x j ika
E. Persamaan Nilai mut lak

x < 0. Jika hanya memuat sat u t anda mut lak maka penyelesaian persamaan dapat dengan
menggunakan pengert ian nilai mut lak at au dapat j uga dengan mengkuadrat kan t anda mut lak.

Selesaikan persamaan ⏐x − 2⏐ = 8
Cont oh 73 :

Solusi :

Dari pengert ian didapat j ika x ≥ 2 maka x − 2 = 8 sehingga x = 10 yang memenuhi persamaan.
Alt ernat if 1 :

Sedangkan j ika x < 2 maka 2 − x = 8 sehingga x = −6 yang j uga memenuhi persamaan.


Jadi, penyelesaian x yang memenuhi adalah x = −6 at au x = 10.

Eddy Hermanto, ST 41 Aljabar


Pembinaan Olimpiade Matematika

Karena ⏐x − 2⏐ bernilai t ak negat if maka penyelesaiannya dapat dilakukan dengan mengkuadrat kan
Alt ernat if 2 :

(x − 2)2 = 64
kedua ruas.

x2 − 4x − 60 = 0
(x − 10)(x + 6) = 0
x = 10 at au x = −6

Persoalan menj adi lebih rumit apabila dalam persamaan t ersebut memuat lebih dari sat u t anda
mut lak. Penyelesaiannya dapat dilakukan dengan membagi kasus.

Cont oh 74 :

⏐x + 2⏐ + ⏐3x⏐ = 14
(OSP 2003) Apakah himpunan j awab dari persamaan

• Unt uk x ≤ −2, maka |x + 2| = −x − 2 dan |3x| = −3x


Solusi :

|x + 2| + |3x| = 14. Maka −x − 2 − 3x = 14 sehingga x = −4 (memenuhi bahwa x ≤ −2)


• Unt uk −2 ≤ x ≤ 0 maka |x + 2| = x + 2 dan |3x| = −3x
|x + 2| + |3x| = 14. Maka x + 2 − 3x = 14 sehingga x = −6 (t idak memenuhi bahwa −2 ≤ x ≤ 0)
• Unt uk x ≥ 0 maka |x + 2| = x + 2 dan |3x| = 3x
|x + 2| + |3x| = 14. Maka x + 2 + 3x = 14 sehingga x = 3 (memenuhi bahwa x ≥ 0)
Jadi, himpunan j awab dari persamaan |x + 2| + |3x| = 14 adalah = { −4, 3}

LATIHAN 5.E :

1. (OSK 2005) Tent ukan semua solusi persamaan ⏐x − 1⏐ + ⏐x − 4⏐ = 2.

2. (AIME 1983) Tent ukan nilai minimum dari ⎪x − p⎪ + ⎪x − 15⎪ + ⎪x − p − 15⎪ unt uk suat u nilai x
dalam bat as p ≤ x ≤ 15 dimana 0 < p < 15.

3. Banyaknya bilangan bulat berbeda y yang memenuhi ⏐x − 7⏐ − ⏐x + 11⏐ = y adalah ⋅⋅⋅⋅

4. (OSP 2006) Diberikan fungsi f(x) = ⎪⎪x − 2⎪ − a⎪ − 3. Jika grafik f memot ong sumbu-x t epat di
t iga t it ik, maka a = ····

5. (OSP 2006) Jika ⏐x⏐+ x + y = 10 dan x + ⏐y⏐ − y = 12, maka x + y = ⋅⋅⋅⋅⋅⋅

⎪a + b⎪ + c = 19 dan ab + ⎪c⎪ = 97
6. (AHSME 1997) Ada berapa banyak t ripel bilangan bulat (a,b,c) yang memenuhi

7. (AHSME 1977) Unt uk a, b, c bilangan real t aknol, semua kemungkinan nilai dari

+ + +
a b c abc
a b c abc
adalah ⋅⋅⋅⋅⋅

8. (AHSME 1999) Grafik y = −⎪x − a⎪ + b dan y = ⎪x − c⎪ + d berpot ongan di t it ik (2,5) dan (8,3).
Nilai a + c adalah ⋅⋅⋅⋅⋅⋅⋅⋅

Eddy Hermanto, ST 42 Aljabar


Pembinaan Olimpiade Matematika

memenuhi ⏐x⏐ + ⏐y⏐ ≤ 50.


9. (OSN 2011 SMP/ MTs) Jika x dan y adalah bilangan bulat , t ent ukan banyak pasangan (x, y) yang

10. (AIME 1988) xi adalah bilangan real yang memenuhi −1 < xi < 1 dan ⎪x1⎪ + ⎪x2⎪ + ⋅⋅⋅ + ⎪xn⎪ = 19 +
⎪x1 + x2 + ⋅⋅⋅ + xn⎪. Tent ukan nilai t erkecil n yang memenuhi.

f(x) = 1 − ⎪x − 2⎪ unt uk 1 ≤ x ≤ 3. Tent ukan bilangan posit if x t erkecil yang memenuhi


11. (AIME 2001) Fungsi f(x) memenuhi persamaan f(3x) = 3f(x) unt uk semua bilangan real x dan

f(x) = f(2001).

Eddy Hermanto, ST 43 Aljabar


Pembinaan Olimpiade Matematika

6. SISTEM PERSAMAAN
Sist em persamaan t erdiri dari lebih dari sat u persamaan dalam rangka mencari suat u penyelesaian.
A. Sist em Persamaan Linier
Sist em persamaan umum yang dikenal adalah sist em persamaan linier, yait u sist em persamaan yang
pangkat variabelnya t idak lebih dari sat u. Unt uk menyelesaikan n buah variabel dibut uhkan n buah
persamaan. Penyelesaian sist em persamaan dapat dilakukan dengan menggunakan subt it usi,
eliminasi maupun dengan memanfaat kan mat riks.

Harga x yang memenuhi : 3x + y = 29 dan x − y = 1 adalah ⋅⋅⋅


Cont oh 75 :

3x + y = 29 dan x − y = 1
Solusi :

x + y = 3log 29 ⋅⋅⋅⋅⋅⋅⋅⋅⋅⋅⋅⋅⋅⋅⋅⋅⋅⋅⋅⋅ (1)


x − y = 1 ⋅⋅⋅⋅⋅⋅⋅⋅⋅⋅⋅⋅⋅⋅⋅⋅⋅⋅⋅⋅ (2)
Dari persamaan (1) dan (2) didapat
3
log 29 +1 3
log 29 −1
x= 2 dan y = 2

Cont oh 76 :
(OSK 2003) Hari ini usiaku 13 kali usia ayahku. Lima t ahun yang lalu, usiaku 1
4 kali usia ayahku pada
wakt u it u. Berapakah usiaku sekarang ?

Solusi :
Misal usiaku saat ini = X dan usia ayahku saat ini = Y, maka :
X = 13 Y
X− 5 = 1
(Y − 5)
X− 5 = (3X − 5)
4
1

4X − 20 = 3X − 5
4

X = 15
Jadi, usiaku saat ini 15 t ahun

Cont oh 77 :
Selesaikan sist em persamaan
a+b+c+d=0
a+b+c+e=5
a+b+d+e=1
a+c+d+e=2
b+c+d+e=4

Solusi :
Jumlahkan semua persamaan di at as didapat
4(a + b + c + d + e) = 12
a+b+c+d+e=3

5 + d = 3 sehingga d = −2
Maka 0 + e = 3 sehingga e = 3

1 + c = 3 sehingga c = 2

4 + a = 3 sehingga a = −1
2 + b = 3 sehingga b = 1

Jadi, penyelesaian sist em persamaan t ersebut adalah (a, b, c, d, e) = (−1, 1, 2, −2, 3)

Eddy Hermanto, ST 44 Aljabar


Pembinaan Olimpiade Matematika

Cont oh 78 :
Tent ukan persamaan lingkaran yang melalui t it ik (0, 0), (6, 8) dan (7,7)

Solusi :
Persamaan umum lingkaran adalah
x2 + y2 + Ax + By + C = 0
Karena ada 3 t it ik yang diket ahui dan ada 3 variabel yang dicari yait u A, B dan C maka soal ini
merupakan sist em persamaan linier dengan 3 variabel (peubah).
Subt it usikan t it ik (0,0) ke persamaan lingkaran didapat
C= 0
Maka persamaan umum lingkaran menj adi x2 + y2 + Ax + By = 0
Berdasarkan t it ik (6, 8) maka 6A + 8B = −100
3A + 4B = −50 ⋅⋅⋅⋅⋅⋅⋅⋅⋅⋅⋅⋅⋅⋅⋅⋅⋅⋅⋅⋅⋅⋅⋅⋅⋅⋅⋅ (1)
Berdasarkan t it ik (7, 7) maka 7A + 7B = −98 ⋅⋅⋅⋅⋅⋅⋅⋅⋅⋅⋅⋅⋅⋅⋅ (2)
Dari persamaan (1) dan (2) didapat A = −6 dan B = −8
Maka persamaan umum lingkaran t ersebut adalah x2 + y2 − 6x − 8y = 0

B. Sist em Persamaan Tak Linier


Dalam sist em persamaan t ak linier pangkat variabel bisa lebih dari sat u at au merupakan perkalian
di ant ara variabel-variabel yang ada. Dalam sist em persamaan t ak linier maka persoalan menj adi
lebih sulit dan membut uhkan t eknik penyelesaian yang lebih t inggi.

Cont oh 79 :

(x − 1)(y − 2) = 12
Dit ent ukan 3 buah persamaan dengan x, y, z > 0

(y − 2)(z − 3) = 20
(z − 3)(x − 1) = 15
Tent ukan nilai 3x + 2y + 3z.

Solusi :

((x − 1)(y − 2)(z − 3)) 2 = 12 ⋅ 20 ⋅ 15 = (60) 2


Kalikan ket iga persamaan didapat

(x − 1)(y − 2)(z − 3) = 60 at au (x − 1)(y − 2)(z − 3) = −60


Niai x, y, z > 0 yang memenuhi adalah j ika z − 3 = 5, x − 1 = 3 dan y − 2 = 4
Didapat x = 4, y = 6 dan z = 8
Jadi, 3x + 2y + 3z = 48.

Jika persamaan-persamaan dalam sist em persamaan t ak linier dapat dibawa ke dalam persamaan-
persamaan sebagaimana t elah dij elaskan pada rumus Viet a maka hubungan suku banyak dengan
akar-akarnya dapat digunakan unt uk menyelesaikan persoalan sist em persamaan t ak linier.

Cont oh 80 :
Tent ukan nilai a, b dan c yang memenuhi sist em persamaan berikut :
a+b+c=9
ab + ac + bc = 26
abc = 24

Sesuai dengan rumus Viet a maka a, b, dan c adalah akar-akar persamaan x3 − 9x2 + 26x − 24 = 0
Solusi :

x3 − 9x2 + 26x − 24 = (x − 2)(x − 3)(x − 4) = 0


Dengan t eorema horner didapat

Tripel (a, b, c) yang memenuhi adalah (2, 3, 4) dan permut asinya.

Eddy Hermanto, ST 45 Aljabar


Pembinaan Olimpiade Matematika

Cont oh 81 :
(AIME 1991) m, n adalah bilangan asli yang memenuhi mn + m + n = 71 dan m2n + mn2 = 880,
t ent ukan m2 + n2.

Solusi :
mn + m + n = 71
m2n + mn2 = 880 sehingga mn(m + n) = 880
mn + 880
mn = 71
(mn) 2 − 71(mn) + 880 = 0
(mn − 16)(mn − 55) = 0

• Jika mn = 16 maka m + n = 71 − 16 = 55
mn = 16 at au mn = 55

Nilai (m, n) yang memenuhi mn = 16 adalah (1, 16), (2, 8), (4, 4), (8, 2) dan (16, 1) t et api t idak

• Jika mn = 55 maka m + n = 71 − 55 = 16
ada yang memenuhi m + n = 55.

Nilai (m, n) yang memenuhi mn = 55 adalah (1, 55), (5, 11), (11, 5), (55, 1).
Yang memenuhi m + n = 16 adalah m = 5 dan n = 11 at au m = 11 dan n = 5
m2 + n2 = 52 + 112 = 146
Jadi, nilai dari m2 + n2 sama dengan 146.

Cont oh 82 :
Tent ukan penyelesaian dari sist em persamaan
ab = 1 ; bc = 2 ; cd = 3 ; de = 4 dan ea = 6

Solusi :

(abcde)2 = 24 ⋅ 32
Kalikan kelima persamaan didapat

abcde = 12 at au abcde = −12


• Jika abcde = 12
3
Mengingat ab = 1 dan cd = 3 maka e = 4 sehingga d = 1 dan a = 2 .
2
Maka b = 3 dan c = 3.
3 2
Jadi, (a, b, c, d, e) = ( , , 3, 1, 4) merupakan penyelesaian.
• Jika abcde = −12
2 3

Mengingat ab = 1 dan cd = 3 maka e = −4 sehingga d = −1 dan a = − 32 .


Maka b = − 23 dan c = −3.
Jadi, (a, b, c, d, e) = (− 32 ,− 23 , −3, −1, −4) merupakan penyelesaian.
Jadi, penyelesaian (a, b, c, d, e) adalah ( 3
2 , 2
3 , 3, 1, 4) dan (− 32 ,− 23 , −3, −1, −4).

Cont oh 83 :

x2 − yz = 3
Selesaikan sist em persamaan :

y2 − xz = 4
z2 − xy = 5

Solusi :

x2 + y2 + z2 − xy − xz − yz = 12
Alt ernat if 1 :

Eddy Hermanto, ST 46 Aljabar


Pembinaan Olimpiade Matematika
2x2 + 2y2 + 2z2 − 2xy − 2xz − 2yz = 24
(x2 − 2xy + y2) + (x2 − 2xz + z2) + (y2 − 2yz + z2) = 24
(x − y) 2 + (x − z) 2 + (y − z) 2 = 24 ⋅⋅⋅⋅⋅⋅⋅⋅⋅⋅⋅ (1)
y2 − xz − (x2 − yz) = 1
(y + x)(y − x) + z(y − x) = 1
(y − x)(x + y + z) = 1 ⋅⋅⋅⋅⋅⋅⋅⋅⋅⋅⋅⋅⋅⋅⋅⋅⋅⋅⋅⋅⋅⋅⋅⋅⋅⋅⋅ (2)
z2 − xy − (y2 − xz) = 1
(z + y)(z − y) + x(z − y) = 1
(z − y)(x + y + z) = 1 ⋅⋅⋅⋅⋅⋅⋅⋅⋅⋅⋅⋅⋅⋅⋅⋅⋅⋅⋅⋅⋅⋅⋅⋅⋅⋅⋅ (3)
z2 − xy − (x2 − yz) = 2
(z + x)(z − x) + y(z − x) = 2
(z − x)(x + y + z) = 2 ⋅⋅⋅⋅⋅⋅⋅⋅⋅⋅⋅⋅⋅⋅⋅⋅⋅⋅⋅⋅⋅⋅⋅⋅⋅⋅⋅ (4)

( ) +( ) +( )
Subt it usikan persamaan (2), (3) dan (4) ke persamaan (1)
2 2 2
x+ y + z x+ y + z x+ y + z
1 1 2
= 24
2 1
(x + y + z) =

4

Jika x + y + z = 12

y−x=2
Dari persamaan (2), (3) dan (4)

z−y=2
z−x=4
x + (2 + x) + z = 12
2x + z = − 32
2x + (4 + x) = − 32 sehingga x = − 11
6

y = 2 + (− 11
6 ) =
1
6
13
z = 4 + (− 11
6 ) =

• − 12
6

Jika x + y + z =
y − x = −2
z − y = −2
z − x = −4
x + (−2 + x) + z = − 12
3
2x + z = 2
3 11
2x + (−4 + x) = sehingga x =
y = −2 + ( 11
2 6
1
6 ) =

z = −4 + ( 11
6
13
6 ) = 6
1 13 13
Tripel (x, y, z) yang memenuhi adalah ((− 11 11 1
6 , 6 , 6 )( 6 ,− 6 ,− 6 ))

x2 − yz + z2 − xy = 2y2 − 2xz
Alt ernat if 2 :

(x − y)(x + y) + (z − y)(z + y) + z(x − y) + x(z − y) = 0


(x + y + z)(x + z − 2y) = 0
* Jika x + y + z = 0

x2 − y(−x − y) = 3
Subt it usikan ke persamaan pada soal didapat

x2 + y2 + xy = 3 ⋅⋅⋅⋅⋅⋅⋅⋅⋅⋅⋅⋅⋅⋅⋅⋅⋅ (5)
(−x − y)2 − xy = 5

Eddy Hermanto, ST 47 Aljabar


Pembinaan Olimpiade Matematika
x2 + y2 + xy = 5 ⋅⋅⋅⋅⋅⋅⋅⋅⋅⋅⋅⋅⋅⋅⋅⋅⋅⋅⋅⋅⋅⋅ (6)

Jika x + z = 2y ⋅⋅⋅⋅⋅⋅⋅⋅⋅⋅⋅⋅⋅⋅⋅⋅⋅⋅⋅⋅⋅⋅⋅ (7)


Maka t idak ada t ripel (x, y, z) yang memenuhi.

x2 − y(2y − x) = 3
*

x2 − 2y2 + xy = 3 ⋅⋅⋅⋅⋅⋅⋅⋅⋅⋅⋅⋅⋅⋅⋅⋅⋅⋅⋅ (8)


y2 − x(2y − x) = 4
x2 + y2 − 2xy = 4 ⋅⋅⋅⋅⋅⋅⋅⋅⋅⋅⋅⋅⋅⋅⋅⋅⋅⋅⋅⋅ (9)
4(x2 − 2y2 + xy) = 3(x2 + y2 − 2xy)
x2 − 11y2 + 10xy = 0
(x + 11y)(x − y) = 0
• Jika x = y

x = y sehingga x = y = z yang t idak memenuhi x2 − yz = 3


Subt it usikan ke persamaan (7) maka

• Jika x = −11y
Subt it usikan ke persamaan (7) maka

y2 − xz = 4
z = 13y

y2 + 143y2 = 4
144y2 = 4
y = ± 16
1
* Jika y =
Maka x = − 11
6
13
6 dan z =

Jika y = − 16
6

*
Maka x = 11
6 dan z = − 136
1 13 13
Tripel (x, y, z) yang memenuhi adalah ((− 11 11 1
6 , 6 , 6 )( 6 ,− 6 ,− 6 ))

LATIHAN 6 :

1. Lingkaran yang melalui t it ik-t it ik (2,2), (2,−4) dan (5,−1) mempunyai j ari-j ari ⋅⋅⋅⋅⋅⋅

x( x+ y )
2. Tent ukan semua nilai x + y real yang memenuhi sist em persamaan :
x
x+y+ y
= 232 dan y = 2007

3. Tent ukan semua penyelesaian pasangan (x, y) real yang memenuhi


x2 + y2 + x + y = 12
xy + x + y = 3

4. (OSP 2011) Tent ukan semua nilai k yang mungkin sehingga t idak ada pasangan bilangan real (x, y)

x2 − y2
yang memenuhi sist em persamaan

(x − k) 2 + y2 = 1
= 0

x− z
y x+ y x
5. (OSK 2005) Diberikan t iga bilangan posit if x, y dan z yang semuanya berbeda. Jika = z = y
,
x
maka nilai y
sama dengan

Eddy Hermanto, ST 48 Aljabar


Pembinaan Olimpiade Matematika
6. (Canadian MO 1969) Tunj ukkan bahwa j ika
a1
b1 = a2
b2 = a3
b3
dan p1, p2 dan p3 semuanya t idak sama
dengan nol, maka :
⎛ a1 ⎞ p a n + p 2 a 2n + p 3 a 3n
⎜⎜ ⎟⎟ = 1 1n
n

⎝ b1 ⎠ p1b1 + p 2 b2n + p 3 b3n

7. (AIME 1989/ OSN 2004) Diberikan


x1 + 4x2 + 9x3 + 16x4 + 25x5 + 36x6 + 49x7 = 1
4x1 + 9x2 + 16x3 + 25x4 + 36x5 + 49x6 + 64x7 = 12
9x1 + 16x2 + 25x3 + 36x4 + 49x5 + 64x6 + 81x7 = 123.
Tent ukan nilai dari 16x1 + 25x2 + 36x3 + 49x4 + 64x5 + 81x6 + 100x7.

8. (Irish MO 1999) Selesaikan sist em persamaan berikut :


y2 = (x + 8)(x2 + 2)
y2 − (8 + 4x)y + (16 + 16x − 5x2) = 0

9. (Canadian MO 1970) Tent ukan semua t ripel (x, y, z) yang memenuhi bahwa salah sat u bilangan j ika
dit ambahkan dengan hasil kali kedua bilangan yang lain hasilnya adalah 2.

x3 − xyz = 2
10. Misalkan (a, b, c) adalah solusi real unt uk sist em persamaan

y3 − xyz = 6
z3 − xyz = 20
Nilai t erbesar a3 + b3 + c3 dapat dit ulis ke dalam bent uk mn dengan m dan n adalah bilangan bulat
posit if yang saling relat if prima. Nilai dari m + n adalah ⋅⋅⋅⋅⋅⋅⋅

x − yz = 42
11. (COMC 1997) Tent ukan bilangan real x, y dan z yang memenuhi sist em persamaan :

y − xz = 6
z − xy = −30

⎛ 1 ⎞
12. (Viet namese MO 1996) Selesaikan sist em persamaan berikut :

3 x ⎜⎜1 + ⎟=2
⎝ x + y ⎟⎠
⎛ 1 ⎞
7 y ⎜⎜1 − ⎟⎟ = 4 2
⎝ x+ y⎠

Eddy Hermanto, ST 49 Aljabar


Pembinaan Olimpiade Matematika

7. KETAKSAMAAN
A. Konsep Urut an dan Sifat -sifat dasar dari konsep urut an
Sifat pent ing pada bilangan-bilangan real adalah adanya urut an sehingga dapat membandingkan dua
bilangan sehingga didapat apakah kedua bilangan t ersebut sama at au t idak sama.
Sifat -sifat dari dari konsep urut an pada sist em bilangan real :
(1) Set iap bilangan real a hanya memenuhi sat u dan hanya sat u dari kemungkinan
(i) a = 0
(ii) a > 0
(iii) a < 0
(2) Set iap bilangan real a dan b hanya memenuhi sat u dan hanya sat u dari kemungkinan
(i) a = b
(ii) a > b
(iii) a < b
(3) Jika a > 0 dan b > 0 maka a + b > 0
(4) Jika a > 0 dan b > 0 at au a < 0 dan b < 0 maka ab > 0

(6) Jika a < b maka a ± c < b ± c unt uk set iap bilangan real c
(5) Jika a < b dan b < c maka a < c

(7) Jika a < b dan c < d maka a + c < b + d


(8) Jika a < b dan c > 0 maka ac < bc
(9) Jika a < b dan c < 0 maka ac > bc
(10) Jika a > 0 maka 1a > 0
a
(11) Jika a > 0 dan b > 0 maka >0
(12) Jika 0 < a < b at au a < b < 0 maka b1 < 1a .
b

(13) Jika a > 0 dan b > 0 sert a a2 > b2 maka a > b.


Sifat -sifat t ersebut j uga berlaku j ika t anda < digant i dengan t anda ≤ kecuali unt uk sifat (12) yang
mensyarat kan bahwa a dan b keduanya t ak nol.

< < < dc .


Cont oh 84 :
a+c
b+d
a c a
Bukt ikan bahwa j ika a, b, c dan d adalah bilangan posit if yang memenuhi b d maka b

Solusi :

<
Alt ernat if 1 :
a c
Karena bd posit if sert a maka
ad < bc ⋅⋅⋅⋅⋅⋅⋅⋅⋅⋅⋅⋅⋅⋅⋅ (sifat 8)
b d

ad + ab < bc + ab ⋅⋅⋅⋅⋅⋅⋅⋅⋅⋅⋅⋅⋅⋅⋅⋅⋅⋅⋅ (sifat 6)


a(b + d) < b(a + c)

b < b+d
Karena b(b + d) posit if maka
a a +c
⋅⋅⋅⋅⋅⋅⋅⋅⋅⋅⋅⋅⋅⋅⋅⋅⋅⋅⋅⋅⋅ (sifat 8) ⋅⋅⋅⋅⋅⋅⋅⋅⋅⋅⋅⋅⋅⋅⋅⋅⋅⋅⋅⋅⋅⋅⋅⋅⋅⋅⋅⋅⋅⋅⋅⋅⋅⋅⋅⋅⋅⋅ (1)
Karena bd posit if sert a a
< c
maka
ad < bc ⋅⋅⋅⋅⋅⋅⋅⋅⋅⋅⋅⋅⋅⋅⋅ (sifat 8)
b d

ad + cd < bc + cd ⋅⋅⋅⋅⋅⋅⋅⋅⋅⋅⋅⋅⋅⋅⋅⋅⋅⋅⋅ (sifat 6)


d(a + c) < c(b + d)

b+d < d
Karena d(b + d) posit if maka
a+c c
⋅⋅⋅⋅⋅⋅⋅⋅⋅⋅⋅⋅⋅⋅⋅⋅⋅⋅⋅⋅⋅ (sifat 8) ⋅⋅⋅⋅⋅⋅⋅⋅⋅⋅⋅⋅⋅⋅⋅⋅⋅⋅⋅⋅⋅⋅⋅⋅⋅⋅⋅⋅⋅⋅⋅⋅⋅⋅⋅⋅⋅⋅ (2)

b < b+d < d


Dari persamaan (1) dan (2) didapat
a a+c c
(t erbukt i)

Eddy Hermanto, ST 50 Aljabar


Pembinaan Olimpiade Matematika

Karena ba < dc maka bc > ad


Alt ernat if 2 :

a+c
b+ d − ba = bc − ad
b (b + d )

b + d − b = b (b + d ) > 0
Karena bc > ad sedangkan b dan (b + d) posit if maka
a+c a bc − ad

Jadi, a +c
b+d
a
> ⋅⋅⋅⋅⋅⋅⋅⋅⋅⋅⋅⋅⋅⋅⋅⋅⋅⋅⋅⋅⋅⋅⋅⋅⋅⋅⋅⋅⋅⋅⋅⋅ (3)

b
a+c bc − ad
b + d = d (b + d )
c
d

d − b + d = d (b + d ) > 0
Karena bc > ad sedangkan d dan (b + d) posit if maka
c a+c bc − ad

Jadi, dc > ba++dc ⋅⋅⋅⋅⋅⋅⋅⋅⋅⋅⋅⋅⋅⋅⋅⋅⋅⋅⋅⋅⋅⋅⋅⋅⋅⋅⋅⋅⋅⋅ (4)

b < b+d < d


Dari persamaan (3) dan (4) didapat
a a+c c
(t erbukt i)

Cont oh 85 :
Jika a ≥ b dan x ≤ y maka bukt ikan bahwa ≤ ⋅
ax+ by a+ b x+ y
2 2 2

( a −b )( y − x )
Solusi :
⋅ −
a+ b x+ y ax+ by ax + ay + bx + by − 2 ax − 2 by
= =
Karena a ≥ b dan x≤ y maka (a − b)(y − x) ≥ 0.
2 2 2 4 4

⋅ − ≥ 0.
a+ b x+ y ax+ by
Jadi, Tanda kesamaan t erj adi j ika a = b at au x = y.
≤ a+2 b ⋅
2 2 2
ax+ by x+ y
Terbukt i bahwa 2 2 .

x + 10 − x + 2 < 2 , maka bat as-bat as nilai x yang memenuhi adalah ⋅⋅⋅


Cont oh 86 :
Jika

Solusi :
Ket aksamaan pada soal memiliki syarat bahwa bilangan dalam t anda akar t idak boleh negat if

x ≥ −10 dan x ≥ −2 yang berakibat syarat pada soal adalah x ≥ −2 ⋅⋅⋅⋅⋅⋅⋅⋅⋅⋅⋅⋅⋅⋅⋅⋅⋅⋅⋅⋅⋅⋅⋅⋅⋅⋅⋅ (1)
sehingga

x + 10 < 2 + x + 2
Akar dari suat u bilangan selalu posit if sehingga j ika dikuadrat kan t idak akan mempengaruhi t anda

x+2
ket aksamaan.
x + 10 < 4 + x + 2 + 4
1< x+2

x > −1 ⋅⋅⋅⋅⋅⋅⋅⋅⋅⋅⋅⋅⋅⋅⋅⋅⋅⋅⋅⋅⋅⋅⋅⋅⋅⋅⋅⋅ (2)


1<x+2

Jadi, bat as-bat as nilai x yang memenuhi adalah x > −1.

B. Kuadrat Sebarang Bilangan Real Selalu Tak Negat if


Sebagaimana kit a ket ahui bahwa kuadrat dari suat u bilangan real t idak mungkin negat if. Konsep ini

Jika a sebarang bilangan real maka a2 ≥ 0. Tanda kesamaan t erj adi hanya j ika a = 0.
pent ing unt uk menyelesaikan suat u persoalan.

Eddy Hermanto, ST 51 Aljabar


Pembinaan Olimpiade Matematika

Bukt ikan bahwa a2 + b2 ≥ 2ab unt uk bilangan real a dan b. Kapan t anda kesamaan t erj adi ?
Cont oh 87 :

Solusi :

(a − b) 2 ≥ 0
Karena bilangan kuadrat t idak mungkin negat if maka

a2 + b2 − 2ab ≥ 0
Tanda kesamaan t erj adi j ika a = b

a2 + b2 ≥ 2ab (t erbukt i)

Cont oh 88 :
Diket ahui a, b, c > 0 sert a a + b + c = 2. Bukt ikan bahwa ab + bc t idak lebih dari 1.

ab + bc = b(a + c) = b(2 − b)
Solusi :

ab + bc = 1 − (b − 1) 2
Karena bilangan kuadrat t idak mungkin negat if maka ab + bc ≤ 1 dengan t anda kesamaan t erj adi
j ika b = 1.
Jadi, t erbukt i bahwa ab + bc t idak lebih dari 1.

Sifat bilangan kuadrat t idak mungkin negat if t idak hanya digunakan unt uk menyelesaikan masalah
pert idaksamaan t et api j uga menyangkut persamaan.

(AHSME 1997) Jika x, y dan z adalah bilangan real yang memenuhi (x − 3) 2 + (y − 4) 2 + (z − 5) 2 = 0


Cont oh 89 :

maka nilai dari x + y + z adalah

Karena bilangan kuadrat t idak mungkin negat if maka penyelesaian (x − 3)2 + (y − 4)2 + (z − 5)2 = 0
Solusi :

hanya didapat j ika x − 3 = 0, y − 4 = 0 dan z − 5 = 0.


Maka, penyelesaian (x, y, z) yang memenuhi adalah x = 3, y = 4 dan z = 5.
Jadi, nilai dari x + y + z = 3 + 4 + 5 = 12.

C. Ket aksamaan Rat aan Kuadrat (QM), Rat aan Arit mat ik (AM), Rat aan Geomet ri (GM) dan Rat aan
Harmonik (HM)

Misalkan x1, x2, x3, ⋅⋅⋅, xn adalah bilangan real posit if.
Perlu dij elaskan t erlebih dahulu pengert ian masing-masing rat aan.

x12 + x 22 + x32 + L + x n2
Rat aan Kuadrat (QM) =

x1 + x 2 + x3 + L + x n
n

Rat aan Arit mat ik (AM) =


n
Rat aan Geomet ri (GM) = x1 x 2 x3 L x n
n

n
+ + +L+
Rat aan Harmonik (HM) =
1 1 1 1
x1 x 2 x3 xn

Eddy Hermanto, ST 52 Aljabar


Pembinaan Olimpiade Matematika

Cont oh 90 :
Hit unglah QM, AM, GM dan HM dari bilangan-bilangan 2, 3 dan 7.

Solusi :
QM = 2 2 + 32 + 7 2
3 = 62
3
2 + 3+ 7
AM = =4
2 ⋅ 3 ⋅ 7 = 3 42
3
3
GM =

+ +
3 126
HM = 1 1 1 = 41
2 3 7

QM ≥ AM ≥ GM ≥ HM
Hubungan ant ara QM, AM, GM dan HM adalah

Tanda kesamaan t erj adi j ika x1 = x2 = x3 = ⋅⋅⋅ = xn

Bukt ikan bahwa a2 + b2 ≥ 2ab unt uk bilangan real posit if a dan b.


Cont oh 91 :

Solusi :
Dengan memanfaat kan ket aksamaan AM-GM didapat
≥ a b = ab
a 2 +b 2 2 2
2
Tanda kesamaan t erj adi j ika a2 = b2 sehingga a = b.
a2 + b2 ≥ 2ab (t erbukt i)

+ + 8zx dan kapan nilai t erkecil it u t erj adi ?


Cont oh 92 :
x y
Tent ukan nilai minimal dari 2y 4z

Solusi :
Berdasarkan ket aksamaan AM-GM maka
x
+ y
+ 8zx ≥ 3 ⋅ 3 x
⋅ 4yz ⋅ 8zx = 3

+ + 8zx adalah
2y 4z 2y 4
x y 3
Jadi, nilai minimal dari .

= =
2y 4z 4
x y z
Tanda kesamaan t erj adi j ika .

= =
2y 4z 8x
x y z
Misalkan = k maka

⋅ 4yz ⋅ 8zx =
2y 4z 8x
x
k3 = 2y
1
64
1
k= 4
x 1
Jadi, 2y
= 4 sehingga y = 2x
z 1
8x= sehingga z = 2x
4

+ + 8zx adalah
Jadi, t anda kesamaan t erj adi j ika y = z = 2x
x y 3
Maka nilai minimal dari 2y 4z 4 yang t erj adi j ika y = z = 2x.

Eddy Hermanto, ST 53 Aljabar


Pembinaan Olimpiade Matematika

Unt uk a, b dan c bilangan posit if, bukt ikan ket aksamaan (a + b)(b + c)(c + a) ≥ 8abc.
Cont oh 93 :

Berdasarkan ket aksamaan AM ≥ GM maka


Solusi :

a+ b
2 ≥ ab
a + b ≥ 2 ab
Dengan cara yang sama maka
a + c ≥ 2 ac
b + c ≥ 2 bc
Tanda kesamaan t erj adi j ika a= b = c.
(a + b)(a + c)(b + c) ≥ 2 ab ⋅ 2 ac ⋅ 2 bc
(a + b)(b + c)(c + a) ≥ 8abc (t erbukt i)

Bukt ikan bahwa unt uk x, y, z posit if maka x2 + y2 + z2 ≥ xy + xz + yz dan kapan t anda kesamaan
Cont oh 94 :

t erj adi.

Solusi :

x2 + y2 ≥ 2xy
Berdasarkan ket aksamaan AM-GM maka

x2 + z2 ≥ 2xz
y2 + z2 ≥ 2yz
Tanda kesamaan t erj adi j ika x = y = z

2x2 + 2y2 + 2z2 ≥ 2xy + 2xz + 2yz


Jumlahkan ket iga persamaan didapat

x2 + y2 + z2 ≥ xy + xz + yz (t erbukt i)

Cont oh 95 :
(OSP 2009/ AIME 1983) Tent ukan nilai minimal dari 9 x 2 sin 2 x + 4
x sin x unt uk 0 < x < π.

Solusi :
9 x 2 sin 2 x + 4 4
= 9x sin x + x sin
x sin x x

≥ 2 9 x sin x ⋅
Berdasarkan ket aksamaan AM-GM maka
9 x 2 sin 2 x + 4 4 4
x sin x = 9x sin x + x sin x x sin x = 12
9 x sin x + 4
2 2
Maka nilai minimum dari x sin x sama dengan 12.

LATIHAN 7 :

1. (OSK 2010) Bilangan bulat yang memenuhi pert idaksamaan x4 ≤ 8x2 − 16 sebanyak ⋅⋅⋅⋅⋅

2. (OSP 2004) Tent ukan semua bilangan real x yang memenuhi x2 < ⎪2x − 8⎪

Eddy Hermanto, ST 54 Aljabar


Pembinaan Olimpiade Matematika

memenuhi 158 < n +k < 137 .


3. (AIME 1987) Tent ukan bilangan bulat t erbesar n sehingga t erdapat bilangan bulat unik k yang
n

x 2 sin x + x cos x + x 2 + 12 > 0


4. (India RMO 1995) Tunj ukkan bahwa unt uk sembarang bilangan real x maka

5. (OSP 2009) Banyaknya bilangan real x yang memenuhi persamaan x4 − 2x3 + 5x2 − 176x + 2009 = 0
adalah ⋅⋅⋅⋅⋅⋅

x + y +2 z−2 + u + v = x+ y+ z+u+v
6. Selesaikan persamaan berikut dalam bilangan real

( 2x − 1 + 2 y − 1 )
7. (Balt ic Way 2000) Tent ukan semua bilangan real posit if (x, y) yang memenuhi persamaan :
x+y− 1
x − 1
y
+4=2


8. Bukt ikan bahwa unt uk bilangan real x maka
x 3 −1 x 4 −1
3 4

9. (ME V2N4) Misalkan a, b dan c adalah bilangan posit if yang memenuhi persamaan a2 + b2 − ab = c2.
Bukt ikan bahwa (a − c)(b − c) ≤ 0

x 8 − x 5 − 1x + ≥0
10. (Irish MO 1998) Tunj ukkan bahwa j ika x bilangan real t ak nol maka :
1
x4

11. (OSP 2008) Diberikan f(x) = x2 + 4. Misalkan x dan y adalah bilangan-bilangan real posit if yang
memenuhi f(xy) + f(y − x) = f(y + x). Nilai minimum dari x + y adalah ⋅⋅⋅⋅⋅⋅

12. Tent ukan nilai t erkecil dari f(x) j ika f(x) = 4 x 2 +8 x +13
6 (1+ x ) unt uk x ≥ 0. Tent ukan j uga x yang
menyebabkan nilai minimum t ersebut .

⎜ ∑ ai ⎟ ⋅ ⎜⎜ ∑
⎛ n ⎞ ⎛ n 1 ⎞
⎟⎟ ≥ n 2 unt uk ai > 0.
⎝ i =1 ⎠ ⎝ i =1 ai ⎠
13. Bukt ikan bahwa

14. (OSP 2003) Bukt ikan bahwa 999! < 500999.

+ b4 ≥
15. Jika a dan b adalah bilangan real posit if maka bukt ikan bahwa
a +b
1 9
a
Kapan t anda kesamaan t erj adi ?

16. (Aust rian MO 2000 : Beginner Compet it ion) Jika a dan b adalah bilangan real posit if maka bukt ikan
bahwa
( a + b )3
a 2b
≥ 27
4
Kapan t anda kesamaan t erj adi ?

Eddy Hermanto, ST 55 Aljabar


Pembinaan Olimpiade Matematika

( )( )
17. (Canadian MO 1971) Diket ahui x dan y adalah bilangan real posit if yang memenuhi x + y = 1.

1 + 1x 1 + 1y ≥ 9
Bukt ikan bahwa

b + c + a =3
18. (OSP 2009) Bilangan rasional a < b < c membent uk barisan hit ung (arit mat ika) dan
a b c

Banyaknya bilangan posit if a yang memenuhi adalah ⋅⋅⋅⋅⋅⋅⋅⋅⋅⋅⋅⋅

19. x, y dan z adalah bilangan real yang memenuhi x + y + z = 1. Bukt ikan bahwa xy + yz + xz ≤ 1
3 .

+ 1y + 1z > + +
20. Misalkan x, y dan z adalah bilangan posit if berbeda. Bukt ikan bahwa
1 1 1 1
x xy xz yz

21. (OSP 2011) Jika a ≥ b > 1, maka nilai t erbesar yang mungkin unt uk alog (a/ b) + blog (b/ a) adalah ⋅⋅⋅⋅⋅⋅

22. (OSP 2011) Misalkan x, y, dan z adalah bilangan real posit if dengan sifat xyz = 1. Nilai t erkecil dari

t ercapai saat x + y + z bernilai ⋅⋅⋅⋅


(x + 2y)(y + 2z)(xz + 1)

23. (Hongkong PSC 1988) Jika a + b + c + d + e = 8 dan a2 + b2 + c2 + d2 + e2 = 16 maka t ent ukan nilai
maksimal dari e.

24. Diberikan persamaan x4 + px3 + qx2 + rx + s = 0 yang mempunyai empat akar real posit if. Bukt ikan

a) pr − 16s ≥ 0
bahwa :

b) q2 − 36s ≥ 0
dengan t anda kesamaan t erj adi bila keempar akarnya sama.

+ + ≥3
25. Bukt ikan bahwa unt uk a, b dan c bilangan real posit if maka
b+c a+c a +b
2a 2b 2c

Kapan t anda kesamaan t erj adi ?

(
a +b + c ≤ 2 a +b + b + c + c + a
)
26. (Irish MO 1998) Tunj ukkan bahwa j ika a, b, c adalah bilangan real posit if maka :

( 1a + b1 + 1c )
9 1 1 1
(i)
(ii) a +b
1
+ b +1 c + c +1 a ≤ 1
2

+ ba+2c + cb+2a ≥ 2( a1 + b1 + 1c )
27. Bukt ikan bahwa unt uk bilangan real posit if a, b, dan c sebarang berlaku
a +b
c2

Eddy Hermanto, ST 56 Aljabar


Pembinaan Olimpiade Matematika

8. STATISTIK SEDERHANA
St at ist ik yang dibahas dalam buku ini hanya bersifat sederhana yait u berupa dat a t unggal. Hal yang
dibahas adalah pengert ian dari rat a-rat a (rat aan), modus, median dan j angkauan.

Rat a-rat a at au disebut j uga dengan mean at au dikenal j uga dengan rat aan arit mat ika. Rat a-rat a diberi
simbol x (dibaca x bar). Cara menghit ung mean adalah membagi ant ara j umlah semua dat a dengan
banyaknya dat a.

Modus adalah dat a yang paling sering muncul at au dat a dengan frekuensi paling banyak.

Median (diberi simbol Q2) adalah nilai t engah dari gugusan dat a yang t elah diurut kan dari nilai yang
t erendah sampai t ert inggi. Jika banyaknya dat anya ganj il maka median adalah dat a yang berada di
t engah sedangkan j ika banyaknya dat anya genap maka median adalah rat a-rat a hit ung dari dua dat a
yang berada di t engah. Nilai t engah dari dat a-dat a yang berada di sebelah kanan median disebut j uga
kuart il at as (diberi simbol Q3) sedangkan nilai t engah dari dat a-dat a yang berada di sebelah kiri median
disebut j uga kuart il bawah (diberi simbol Q1).

Jangkauan adalah selisih nilai t ert inggi dan t erendah.

Cont oh 96 :
Diberikan dat a : 1, 2, 6, 6, 8, 9, 10. Hit unglah mean, modus, median, kuart il at as, kuart il bawah dan
j angkauan.

Solusi :
Mean = rat aan = 1+ 2 + 6 + 67+8+9 +10 = 6.
Nilai yang paling banyak muncul adalah 6 yait u muncul sebanyak 2 kali. Jadi, modus = 6.
Banyaknya dat a = 7. Jadi, median adalah nilai dat a ke-4. Median = 6.

Jangkauan = 10 − 1 = 9
Ada masing-masing 3 dat a disebelah kiri dan kanan median. Maka kuart il bawah = 2 dan kuart il at as = 9.

Cont oh 97 :
Diberikan dat a : 1, 8, 4, 5, 6, 6, 5, 6, 3, 9. Hit unglah mean, modus, median, kuart il at as, kuart il bawah
dan j angkauan.

Solusi :
Set elah dat anya diurut kan didapat : 1, 3, 4, 5, 5, 6, 6, 6, 8, 9.
Mean = rat aan = 1+ 3+ 4 +5+ 510
+ 6 + 6+ 6 +8+9 53
= 10 .
Nilai yang paling banyak muncul adalah 6 yait u muncul sebanyak 3 kali. Jadi, modus = 6.
Banyaknya dat a = 10. Jadi, median adalah rat a-rat a dat a ke-5 dan ke-6. Median = 5+2 6 = 11
2 .

Jangkauan = 9 − 1 = 8
Ada masing-masing 5 dat a disebelah kiri dan kanan median. Maka kuart il bawah = 4 dan kuart il at as = 6.

Cont oh 98 :

t ingginya 170 cm. Tinggi badan rat a-rat a ke-30 orang t ersebut adalah ⋅⋅⋅⋅⋅⋅ cm.
Sepuluh orang wanit a mempunyai rat a-rat a t inggi badan 160 cm sedang 20 orang pria rat a-rat a

Jumlah t inggi badan 10 wanit a = 10 ⋅ 160 = 1600 cm. Jumlah t inggi badan 20 pria = 20 ⋅ 170 = 3400 cm.
Solusi :

+ 3400
Tinggi badan rat a-rat a 30 orang t ersebut = 160030 = 500
3 cm.

Eddy Hermanto, ST 57 Aljabar


Pembinaan Olimpiade Matematika

Cont oh 99 :
(OSK 2004) Di suat u hot el, rat a-rat a 96% kamar t erpakai sepanj ang sebulan liburan kenaikan kelas dan
rat a-rat a 72% kamar t erpakai sepanj ang sebelas bulan lainnya. Maka rat a-rat a pemakaian kamar
sepanj ang t ahun di hot el t ersebut adalah

Solusi :
Rat a-rat a %pemakaian kamar set ahun = 1⋅96%1++11
11
⋅72%
= 74 %
Jadi, Rat a-rat a pemakaian kamar sepanj ang t ahun di hot el t ersebut adalah 74 %.

LATIHAN 8

t ahun. Tiga anak yang lain bert urut -t urut berumur X + 2, X + 4 dan 2X − 3. Bila rat a-rat a hit ung
1. Suat u keluarga mempunyai lima orang anak. Anak t ermuda berumur X t ahun dan yang t ert ua 2X

umur mereka adalah 16 t ahun, maka anak t ermuda berumur ⋅⋅⋅⋅⋅⋅ t ahun.

2. Kumpulan dat a : 1, 3, 5, 7, 2, 4, 5, 8, x, 1, −1, 2, 3. Jika dat a t ersebut memiliki j angkauan sama


dengan 10, maka nilai x adalah ⋅⋅⋅⋅⋅⋅⋅⋅⋅

3. Rat aan dari a − 2, b + 3 dan c + 5 adalah 6. Rat aan dari a + 4, b + 6 dan c − 1 adalah ····

4. Umur rat a-rat a dari suat u kelompok yang t erdiri dari dokt er dan j aksa adalah 40 t ahun. Jika umur

perbandingan banyaknya dokt er dan banyaknya j aksa adalah ⋅⋅⋅⋅


rat a-rat a para dokt er adalah 35 t ahun dan umur rat a-rat a para j aksa adalah 50 t ahun, maka

5. (OSP 2003) Agung mendapat kan bahwa nilai rat a-rat a dari t iga ulangan mat emat ika yang diikut inya
adalah 81. Nilai ulangan pert ama adalah 85. Nilai ulangan ket iga lebih rendah 4 dari nilai ulangan
kedua. Berapakah nilai ulangan kedua Agung ?

6. Nilai rat a-rat a uj ian sekelompok siswa yang berj umlah 40 orang adalah 51. Jika seorang siswa dari

nilai rat a-rat a uj ian akan menj adi ⋅⋅⋅⋅⋅⋅⋅


kelompok ini yang mendapat nilai 90 t idak dimasukkan dalam perhit ungan rat a-rat a t ersebut , maka

7. (OSP 2008) Serat us siswa suat u Provinsi di Pulau Jawa mengikut i seleksi t ingkat Provinsi dan skor
rat a-rat anya adalah 100. Banyaknya siswa kelas II yang mengikut i seleksi t ersebut 50% lebih banyak

II. Skor rat a-rat a siswa kelas III adalah ⋅⋅⋅⋅⋅⋅


dari siswa kelas III, dan skor rat a-rat a siswa kelas III 50% lebih t inggi dari skor rat a-rat a siswa kelas

8. (OSP 2005) Misalkan A dan B dua himpunan, masing-masing beranggot akan bilangan-bilangan asli

adalah 5002. Jika A ∩ B = {2005}, t ent ukan unsur t erbesar yang mungkin dari himpunan A ∪ B.
yang berurut an. Jumlah rat a-rat a arit mat ika unsur-unsur A dan rat a-rat a arit mat ika unsur-unsur B

9. Terdapat 8 buah dat a yang semuanya adalah bilangan bulat dan set elah diurut kan dari t erendah ke
t ert inggi adalah a, b, c, d, e, f, g, dan h. Median dari dat a t ersebut = 6,5 dan kuart il at as = 7. Jika

t ersebut sama dengan ⋅⋅⋅⋅⋅⋅


modus = 3 dan 7 sert a a, d dan h membent uk barisan arit mat ika maka rat a-rat a seluruh dat a

10. (OSP 2007) Bilangan-bilangan asli 1, 2, ···, n dit uliskan di papan t ulis, kemudian salah sat u bilangan
dihapus. Rat a-rat a arit mat ika bilangan yang t ert inggal adalah 602
17 . Bilangan n yang memungkinkan
ini t erj adi adalah ⋅⋅⋅⋅⋅

Eddy Hermanto, ST 58 Aljabar


Pembinaan Olimpiade Matematika

BAB II
TEORI BILANGAN

1. SIFAT-SIFAT PENJUMLAHAN DAN PERKALIAN DUA BILANGAN

1. Bilangan Ganj il ± Bilangan Ganj il = Bilangan Genap


Sifat -sifat dalam penj umlahan dua bilangan adalah :

2. Bilangan Ganj il ± Bilangan Genap = Bilangan Ganj il


3. Bilangan Genap ± Bilangan Ganj il = Bilangan Ganj il
4. Bilangan Genap ± Bilangan Genap = Bilangan Genap

Sifat -sifat dalam perkalian dua bilangan adalah :


1. Bilangan Ganj il x Bilangan Ganj il = Bilangan Ganj il
2. Bilangan Ganj il x Bilangan Genap = Bilangan Genap
3. Bilangan Genap x Bilangan Ganj il = Bilangan Genap
4. Bilangan Genap x Bilangan Genap = Bilangan Genap

Dari sifat -sifat perkalian dua bilangan akan didapat bahwa bilangan genap t idak mungkin membagi
bilangan ganj il sedangkan bilangan ganj il mungkin membagi bilangan genap. Ket erbagian akan dibahas
lebih lanj ut dalam sub bab selanj ut nya.

Cont oh 1 :

ganj il t erbesar yang memenuhi syarat t ersebut adalah ⋅⋅⋅⋅⋅


(OSK 2003 SMP/ MTs) Hasil kali suat u bilangan genap dengan suat u bilangan ganj il adalah 820. Bilangan

820 = 22 ⋅ 5 ⋅ 41
Solusi :

Perkalian dua bilangan yang menghasilkan bilangan ganj il hanya didapat j ika kedua bilangan t ersebut
adalah ganj il.

Bilangan ganj il t erbesar yang memenuhi adalah 5 ⋅ 41 = 205.


Fakt or ganj il dari 820 selain 1 adalah 5 dan 41.

Jadi, bilangan ganj il t erbesar yang memenuhi adalah 205.

LATIHAN 1 :

bahwa 1945 ≤ p ≤ 2005. Banyaknya nilai p bulat yang memenuhi persamaan t ersebut adalah ⋅⋅⋅⋅⋅⋅⋅⋅
1. Diket ahui a + p⋅b = 19452005 dengan a dan b masing-masing adalah bilangan ganj il sert a diket ahui

2. p dan q adalah bilangan prima dan p > q. Jika p + q = 2005, maka berapakah p − q ?

3. (OSP 2011) Jika kedua akar persamaan x2 − 2013x + k = 0 adalah bilangan prima, maka nilai k yang
mungkin adalah ⋅⋅⋅⋅⋅⋅

4. Tent ukan bilangan prima t erkecil yang membagi 192004 + 452005.

5. Diket ahui persamaan a = 1 + b + c + d sert a c − d = 6 dengan a, b, c, d semuanya bilangan prima dan


a < 100. Tent ukan nilai a maksimum yang memenuhi.

Eddy Hermanto, ST 59 Teori Bilangan


Pembinaan Olimpiade Matematika

6. (Canadian MO 1971) Diberikan polinomial p(x) = xn + a1xn-1 + a2xn-2 + ⋅⋅⋅ + an-1x + an dengan koefisien a1,
a2, ⋅⋅⋅, an semuanya bilangan bulat . Jika p(0) dan p(1) keduanya bilangan ganj il, t unj ukkan bahwa p(x)
t idak mempunyai akar bilangan bulat .

7. Diket ahui (bd + cd) adalah bilangan ganj il. Tunj ukkan bahwa polinomial x3 + bx2 + cx + d t idak dapat
diubah menj adi (x + r)(x2 + px + q) dengan b, c, d, p, q dan r semuanya bilangan bulat .

8. Jika a, b dan c adalah bilangan ganj il, bukt ikan bahwa akar-akar persamaan kuadrat ax2 + bx + c = 0
t idak dapat merupakan bilangan rasional.

Eddy Hermanto, ST 60 Teori Bilangan


Pembinaan Olimpiade Matematika

2. SIFAT-SIFAT KETERBAGIAN

sehingga b = a ⋅ k. Beberapa hal berkait an dengan pembagian adalah sebagai berikut :


Definisi : Sebuah bilangan bulat a dikat akan membagi b (dit ulis a⎪b) j ika t erdapat bilangan bulat k

1.1 Misalkan a, b, c, x dan y bilangan bulat , maka sifat -sifat di bawah ini berlaku :
(1) a⏐a (semua bilangan bulat membagi dirinya sendiri)
(2) a⏐0 (semua bilangan bulat membagi 0)

(4) Jika a⏐1 maka a = ±1


(3) 1⏐a (sat u membagi semua bilangan bulat )

(5) Jika a⏐b maka a⏐xb


(6) Jika ab⏐c maka a⏐c dan b⏐c
(7) Jika a⏐b dan b⏐c maka a⏐c
(8) Jika a⏐b dan a⏐c maka a⏐(bx + cy)

(10) Jika a⏐b dan b ≠ 0 maka ⏐a⏐ ≤ ⏐b⏐


(9) Jika a⏐b maka xa⏐xb

(11) Jika a⏐b dan b⏐a maka a = ±b


(12) Jika a⏐bc dan FPB(a, b) = 1 maka a⏐c
(13) 0⏐a hanya j ika a = 0
1.2 Jika suat u bilangan habis dibagi a dan j uga habis dibagi b, maka bilangan t ersebut akan habis dibagi
ab dengan syarat a dan b relat if prima. Berlaku sebaliknya.
Dua bilangan dikat akan prima relat if, j ika fakt or persekut uan t erbesarnya (FPB) dua bilangan
t ersebut sama dengan 1.
Cont oh : 36 habis dibagi 4 dan 3, maka 36 akan habis dibagi 12.

12 habis dibagi 4 dan 12 j uga habis dibagi 6 t et api 12 t idak habis dibagi 4 ⋅ 6 = 24 sebab 4 dan 6
45 habis dibagi 15. Maka 45 j uga habis dibagi 3 dan 45 j uga habis dibagi 5.

t idak relat if prima, FPB (4, 6) = 2

dengan t anda “ !” menyat akan fakt orial. n! = 1 ⋅ 2 ⋅ 3 ⋅ ⋅⋅⋅ ⋅ n.


1.3 Bilangan yang dapat diubah menj adi perkalian n bilangan bulat berurut an akan habis dibagi n!

Cont oh : 3x4x5x6 = 360 merupakan perkalian 4 bilangan bulat berurut an maka habis dibagi 4! = 24.

(i) (an − bn) = (a − b)(an-1 + an-2b + an-3b2 + ⋅⋅⋅ + abn-2 + bn-1) dengan n ∈ bilangan asli
1.4 Mengingat penj abaran pada dua persamaan berikut :

(ii) (an + bn) = (a + b)(an-1 − an-2b + an-3b2 − ⋅⋅⋅ − abn-2 + bn-1) dengan n ∈ bilangan ganj il
Maka (a − b) membagi (an − bn) unt uk semua a, b bulat dan n bilangan asli
(a + b) membagi (an + bn) unt uk semua a, b bulat dan n bilangan ganj il

(OSN 2003 SMP/ MTs) Bukt ikan bahwa (n − 1)n(n3 + 1) senant iasa habis dibagi oleh 6 unt uk semua bilangan
Cont oh 2 :

asli n.

Solusi :

Berdasarkan 1.2 didapat bahwa j ika (n − 1)n(n3 + 1) habis dibagi 6 maka (n − 1)n(n3 + 1) habis akan dibagi
Alt ernat if 1 :

2 dan j uga habis dibagi 3. Jadi, j ika dapat dibukt ikan bahwa (n − 1)n(n3 + 1) habis dibagi 2 dan j uga habis
dibagi 3 maka dapat dibukt ikan (n − 1)n(n3 + 1) senant iasa habis dibagi oleh 6 unt uk semua bilangan asli

(n − 1) dan n adalah 2 bilangan bulat berurut an maka (n − 1)n akan habis dibagi 2.
n.

Berdasarkan 2.1 poin (1) maka (n − 1)n(n3 + 1) habis dibagi 2.

Jika n = 3k maka 3 membagi n sehingga 3⏐(n − 1)n(n3 + 1)


Sebuah bilangan bulat dapat diklasifikasikan ke dalam salah sat u bent uk dari 3k, 3k + 1 at au 3k + 2.

Jika n = 3k + 1 maka 3⏐(n − 1) sehingga 3⏐(n − 1)n(n3 + 1).


Jika n = 3k + 2 maka n3 + 1 =(3k + 2) 3 + 1 = 3(9k3 + 18k2 + 12k + 3) sehingga 3⏐(n3 + 1).
Maka 3⏐(n − 1)n(n3 + 1).

Eddy Hermanto, ST 61 Teori Bilangan


Pembinaan Olimpiade Matematika
Didapat bahwa (n − 1)n(n3 + 1) habis dibagi 2 dan j uga habis dibagi 3. Karena 2 dan 3 relat if prima maka
(n − 1)n(n3 + 1) habis dibagi 2 ⋅ 3 = 6.
Jadi, (n − 1)n(n3 + 1) habis dibagi 6.

(n − 1)n(n3 + 1) = (n − 1)n(n + 1)(n2 − n + 1)


Alt ernat if 2 :

Karena n − 1, n dan n t iga bilangan asli berurut an maka (n − 1)n(n + 1)(n2 − n + 1) habis dibagi oleh 3!= 6.
Jadi, (n − 1)n(n3 + 1) habis dibagi 6.

Cont oh 3 :

b = −2, nilai dari 5a + 11b adalah 43. Manakah dari t iga bilangan 37, 254 dan 1986 yang t idak dapat
(OSK 2005 SMP/ MTS) Bilangan 43 dapat dinyat akan ke dalam bent uk 5a + 11b karena unt uk a = 13 dan

dinyat akan dalam bent uk 5a + 11b ?


A. 1983 B. 254 C. 254 dan 1986 D. semua E. t ak ada

Perhat ikan bahwa 1 dapat dinyat akan ke dalam bent uk 5a + 11b dengan a = −2 dan b = 1. Karena 1
Solusi :

membagi semua bilangan bulat maka semua bilangan dapat dinyat akan ke dalam bent uk 5a + 11b.

Misalkan diinginkan 5a + 11b = k maka kesamaan akan t erj adi saat a = −2k dan b = k.
(Jawaban : D)

Cont oh 4 :
Bukt ikan bahwa 7, 13 dan 181 adalah fakt or dari 3105 + 4105

Solusi :
Karena 105 ganj il maka 3105 + 4105 habis dibagi 3 + 4 = 7.
3105 + 4105 = (33) 35 + (43) 35 = 2735 + 6435
Karena 35 ganj il maka 3105 + 4105 habis dibagi 27 + 64 = 91.
Karena 91 = 7 ⋅ 13 maka 3105 + 4105 habis dibagi 13.
3105 + 4105 = (35) 21 + (45) 21 = 24321 + 102421
Karena 21 ganj il maka 3105 + 4105 habis dibagi 243 + 1024 = 1267. Karena 1267 = 7 ⋅ 181 maka 3105 + 4105
habis dibagi 181.

Cont oh 5 :
(OSK 2004 SMP/ MTS) Semua n sehingga n dan n +3
n −1 keduanya merupakan bilangan bulat adalah ⋅⋅⋅⋅⋅

Solusi :

= = 1+
Alt ernat if 1 :
n+3 n −1+ 4
n −1 n −1 n −1
4
Perhat ikan bahwa
Agar 1 + n −1
4
merupakan bilangan bulat maka n − 1 haruslah merupakan fakt or dari 4.
Maka nilai dari n − 1 adalah ±1, ±2 dan ±4.
Nilai n yang memenuhi adalah −3, −1, 0, 2, 3 dan 5.

Alt ernat if 2 :
Selain dengan menggunakan sifat ket erbagian, soal t ersebut j uga bisa diselesaikan dengan memfakt orkan.
Misalkan m = nn+−13 unt uk suat u bilangan bulat n dan m.

n + 3 = mn − m
Persamaan di at as ekivalen dengan

(m − 1)(n − 1) = 4.

Eddy Hermanto, ST 62 Teori Bilangan


Pembinaan Olimpiade Matematika
n − 1 haruslah merupakan fakt or dari 4.
Maka nilai dari n − 1 adalah ±1, ±2 dan ±4.
Nilai n yang memenuhi adalah −3, −1, 0, 2, 3 dan 5.

+ n3 = 1 adalah ⋅⋅⋅⋅⋅⋅⋅⋅⋅
Cont oh 6 :
2
(OSP 2005 SMP/ MTs) Semua pasangan bilangan asli m dan n yang memenuhi m

Solusi :

(m − 2)(n − 3) = 6
Persamaan pada soal ekivalen dengan 2n + 3m = mn

Dengan demikian m − 2 dan n − 3 keduanya merupakan fakt or dari 6.


Karena m dan n bilangan asli maka m − 2 > −2 dan n − 3 > −3
Maka m − 2 = 1, 2, 3 at au 6. Jadi m = 3, 4, 5 at au 8.
Jadi, pasangan (m, n) yang memenuhi adalah (3, 9), (4, 6), (5, 5), (8, 4).

LATIHAN 2 :

1. (OSK 2002) Bilangan n t erbesar sehingga 8n membagi 4444 adalah

2. (OSK 2002) Berapa banyak pasang bilangan bulat posit if (a,b) yang memenuhi 1
a + 1b = 16 .

3. (OSK 2003) Jika a dan b bilangan bulat sedemikian sehingga a2 − b2 = 2003, maka berapakah nilai dari
a2 + b2 ?
(Diket ahui bahwa 2003 merupakan bilangan prima)

4. (AIME 1986) Tent ukan nilai n t erbesar sehingga n + 10 membagi n3 + 100.

5. (MATNC 2001) Jumlah N bilangan kuadrat sempurna pert ama merupakan kelipat an 41. Tent uan nilai
minimal dari N.

6. (OSP 2009) Diket ahui k, m, dan n adalah t iga bilangan bulat posit if yang memenuhi

+ =
k m 1

Bilangan m t erkecil yang memenuhi adalah ⋅⋅⋅⋅⋅⋅


m 4n 6

7. (AIME 1987/ OSP 2008) m dan n adalah bilangan bulat yang memenuhi m2 + 3m2n2 = 30n2 + 517. Nilai
dari 3m2n2 adalah ⋅⋅⋅⋅⋅

8. (AIME 1989) Lima bilangan asli berurut an memenuhi bahwa j umlahnya merupakan bilangan kubik dan
j umlah t iga bilangan di t engah merupakan bilangan kuadrat . Tent ukan nilai t erkecil dari bilangan
yang di t engah.

9. Bilangan prima t erbesar kurang dari 100 yang merupakan fakt or dari 332 − 232 adalah ⋅⋅⋅⋅⋅⋅

10. (AIME 1989) Misalkan k ∈ N sehingga 36 + k, 300 + k, 596 + k adalah kuadrat dari t iga bilangan yang
membent uk barisan arit mat ika. Tent ukan nilai k.

11. (OSP 2002) Berapakah bilangan bulat posit if t erbesar yang membagi semua bilangan 15 −1, 25 −2, ⋅⋅⋅,
n5 −n, ⋅⋅?

Eddy Hermanto, ST 63 Teori Bilangan


Pembinaan Olimpiade Matematika
12. Jika n adalah bilangan bulat lebih dari 1, bukt ikan bahwa n6 − n2 habis dibagi 60.

13. Tunj ukkan bahwa 15 + 25 + 35 + ⋅⋅⋅ + 995 + 1005 habis dibagi 10100, namun t idak habis dibagi 3.

+ 149 ab
14. Banyaknya pasangan bilangan bulat posit if (a, b) yang saling relat if prima dan memenuhi
a
b

merupakan bilangan bulat adalah ⋅⋅⋅⋅⋅⋅⋅⋅⋅

dan a + d = b + c sert a bc − ad = 93.


15. (AIME 1993) Tent ukan banyaknya t upel bilangan bulat (a,b,c,d) yang memenuhi 0 < a < b < c < d < 500

16. Bukt ikan bahwa j ika a, b dan c bilangan asli dan b adalah kelipat an a, c adalah kelipat an b sert a a
adalah kelipat an c maka a = b = c.

17. (AIME 2001) Tent ukan penj umlahan semua bilangan asli dua angka yang habis dibagi oleh masing-
masing digit nya.

a. Jika p dan q bilangan bulat dan memenuhi 10p + q kelipat an 7, bukt ikan bahwa p − 2q j uga
18. Bilangan bulat n dikat akan merupakan kelipat an 7 j ika memenuhi n = 7k dengan k bilangan bulat .

b. Jika c dan d bilangan bulat dan memenuhi 5c + 4d kelipat an 7, bukt ikan bahwa 4c − d j uga
kelipat an 7.

kelipat an 7.

19. Tent ukan bilangan bulat posit if terbesar x yang memenuhi dua persyarat an berikut :
a. x t idak habis dibagi 10
b. Jika dua angka t erakhir dari x2 dibuang maka bilangan t ersisa j uga merupakan bilangan kuadrat .

20. (Canadian MO 1971) Unt uk n bilangan bulat , t unj ukkan bahwa n2 + 2n + 12 bukan kelipat an 121.

21. (ME V1N2) Jumlah dua bilangan bulat posit if adalah 2310. Tunj ukkan bahwa hasil kali keduanya t idak
habis dibagi 2310.

Eddy Hermanto, ST 64 Teori Bilangan


Pembinaan Olimpiade Matematika

3. UJI HABIS DIBAGI


Sebuah bilangan memiliki sifat khusus j ika dibagi oleh suat u bilangan t ert ent u. Beberapa sifat t ersebut
adalah :
a. Suat u bilangan habis dibagi 5 j ika dan hanya j ika digit t erakhir dari bilangan t ersebut adalah 0 at au 5.
Cont oh : 67585 dan 457830 adalah bilangan-bilangan yang habis dibagi 5.
b. Suat u bilangan habis dibagi 2n j ika dan hanya j ika n digit t erakhir dari bilangan t ersebut habis dibagi
2n.
Cont oh : 134576 habis dibagi 8 = 23 sebab 576 habis dibagi 8 (576 : 8 = 72)
4971328 habis dibagi 16 = 24 sebab 1328 habis dibagi 16
c. Suat u bilangan habis dibagi 3 j ika dan hanya j ika j umlah digit bilangan t ersebut habis dibagi 3.
Cont oh : 356535 habis dibagi 3 sebab 3 + 5 + 6 + 5 + 3 + 5 = 27 dan 27 habis dibagi 3.
d. Suat u bilangan habis dibagi 9 j ika dan hanya j ika j umlah digit bilangan t ersebut habis dibagi 9.
Cont oh : 23652 habis dibagi 9 sebab 2 + 3 + 6 + 5 + 2 = 18 dan 18 habis dibagi 9.
e. Suat u bilangan habis dibagi 11 j ika dan hanya j ika selisih ant ara j umlah digit dari bilangan t ersebut

Cont oh : 945351 habis dibagi 11 sebab (9 + 5 + 5) − (4 + 3 + 1) = 11 dan 11 habis dibagi 11. Cont oh
pada posisi ganj il dengan j umlah digit dari bilangan t ersebut pada posisi genap habis dibagi 11.

bilangan lain yang habis dibagi 11 adalah 53713 dan 245784.

Cont oh 7 :
(OSK 2003) Ada berapa banyak diant ara bilangan-bilangan 20000002, 20011002, 20022002, 20033002 yang
habis dibagi 9 ?

Solusi :
Penj umlahan digit 20000002 = 2 + 0 + 0 + 0 + 0 + 0 + 0 + 2 = 4 (t idak habis dibagi 9)
Penj umlahan digit 20011002 = 2 + 0 + 0 + 1 + 1 + 0 + 0 + 2 = 6 (t idak habis dibagi 9)
Penj umlahan digit 20022002 = 2 + 0 + 0 + 2 + 2 + 0 + 0 + 2 = 8 (t idak habis dibagi 9)
Penj umlahan digit 20033002 = 2 + 0 + 0 + 3 + 3 + 0 + 0 + 2 = 10 (t idak habis dibagi 9)
Karena semua penj umlahan digit t idak ada yang habis dibagi 9 maka t idak ada bilangan-bilangan t ersebut
yang habis dibagi 9.

Cont oh 8 :
(Canadian MO 1980) Jika a679b adalah bilangan lima angka yang habis dibagi 72, t ent ukan nilai a dan b.

72 = 9 ⋅ 8. Karena 9 dan 8 relat if prima maka a679b harus habis dibagi 8 dan 9. Karena a679b habis dibagi
Solusi :

8 maka 79b habis dibagi 8. Agar 790 + b habis dibagi 8 maka b = 2.


Karena a6792 habis dibagi 9 maka a + 6 + 7 + 9 + 2 habis dibagi 9. Nilai a yang memenuhi hanya 3.
Jadi bilangan t ersebut adalah 36792.

LATIHAN 3 :

1. (MATNC 2001) Di ant ara empat bilangan : 5256, 7018, 18623, 32571, yang habis dibagi 99 adalah ⋅⋅⋅⋅⋅

2. (OSK 2010) Nilai n t erkecil sehingga bilangan


20102010
14424 ...2010
43
habis dibagi 99 adalah ⋅⋅⋅⋅⋅
n buah 2010

Eddy Hermanto, ST 65 Teori Bilangan


Pembinaan Olimpiade Matematika
3. N adalah bilangan bulat t erbesar dengan semua digit nya berbeda dan N merupakan bilangan kelipat an
8. Tent ukan t iga angka t erakhir dari N ?

4. Jika dihit ung maka didapat 17! = 3a56874280b6000. Tent ukan digit a dan b t anpa menggunakan

(Tanda “ !” menyat akan fakt orial. n! = 1 ⋅ 2 ⋅ 3 ⋅ ⋅⋅⋅ n. Cont oh 3! = 1 ⋅ 2 ⋅ 3 = 6 ; 4! = 24; 5! = 120)


kalkulat or maupun alat hit ung lainnya.

5. (AIME 1983) Tent ukan bilangan asli t erkecil n sehingga angka-angka 15n hanya t erdiri dari 0 dan 8.

6. Tent ukan bilangan asli t erkecil yang merupakan kelipat an 84 yang angka-angkanya hanya 6 at au 7.

7. (Flanders MO 2000 Final Round) Bilangan asli n t erdiri dari 7 angka berbeda dan n habis dibagi oleh
masing-masing angkanya. Tent ukan t iga angka yang bukan angka dari n.

Eddy Hermanto, ST 66 Teori Bilangan


Pembinaan Olimpiade Matematika

4. FAKTOR PERSEKUTUAN TERBESAR (FPB) DAN PERSEKUTUAN TERKECIL (KPK)


Pengert ian :
FPB (a, b) adalah bilangan asli t erbesar d sehingga d⏐a dan d⏐b.
KPK (a,b) adalah bilangan asli t erkecil m sehingga a⏐m dan b⏐m.

Misalkan M = p1a1 ⋅ p2a2 ⋅ p3a3 ⋅ ⋅⋅⋅ pnan dan N = p1b1 ⋅ p2b2 ⋅ p3b3 ⋅ ⋅⋅⋅ pnbn dengan pi adalah bilangan prima dan ai

a. Fakt or Persekut uan Terbesar dari M dan N dit ulis FPB (M, N) = p1c1 ⋅ p2c2 ⋅ p3c3 ⋅ ⋅⋅⋅ pncn
sert a bi adalah bilangan asli maka :

b. Kelipat an Persekut uan Terkecil dari M dan N dit ulis KPK (M, N) = p1d1 ⋅ p2d2 ⋅ p3d3 ⋅ ⋅⋅⋅ pndn
Dengan c1 = min (a1, b1) ; c2 = min (a2, b2) ; c2 = min (a3, b3) ; ⋅⋅⋅ ; cn = min (an, bn)
d1 = maks (a1, b1) ; d2 = maks (a2, b2) ; d3 = maks (a3, b3) ; ⋅⋅ ; dn = maks (an, bn)

Beberapa hal berkait an dengan FPB adalah :

b. FPB(a, 0) = ⏐a⏐
a. FPB(0,0) = 0

c. FPB (a, b) = FPB (⏐a⏐, ⏐b⏐)


d. FPB (a,b) = FPB(b,a)
e. Fakt or Persekut uan Terbesar (FPB) dari dua bilangan asli berurut an adalah 1.

g. Misalkan a = mp dan b = mq maka FPB(a, b) = m ⋅ FPB(p, q)


f. Jika d = FPB(a, b) maka d⏐a dan d⏐b

h. Jika a ≠ 0 dan b ≠ 0 maka 0 ≤ FPB(a,b) ≤ min (⏐a⏐,⏐b⏐)


i. Misalkan a > b > 0 dan a = bq + r unt uk bilangan asli a, b, p dan r maka
FPB(a,b) = FPB(b,r)
j . Dua bilangan dikat akan prima relat if, j ika fakt or persekut uan t erbesarnya (FPB) sama dengan 1.
k. Bezout ’ s Lemma : Unt uk set iap bilangan bulat a dan b t erdapat bilangan bulat x dan y yang
memenuhi ax + by = FPB(a, b)

(OSK 2003 SMP/ MTs) Kelipat an Persekut uan Terkecil dari 210, 42 dan 70 adalah ⋅⋅⋅⋅⋅
Cont oh 9 :

A. 14 B. 210 C. 420 D. 7 E. 1260

210 = 2 ⋅ 3 ⋅ 5 ⋅ 7
Solusi :

42 = 2 ⋅ 3 ⋅ 7
70 = 2 ⋅ 5 ⋅ 7
Maka KPK (210, 42, 70) = 2 ⋅ 3 ⋅ 5 ⋅ 7 = 210. (Jawaban : B)

Cont oh 10 :
Bila KPK dan FPB dari empat bilangan berbeda 18, 24, 18n dan 72 adalah 72 dan 6, t ent ukan nilai n asli
yang memenuhi.

KPK(18, 24, 18n, 72) = 72 = 23 ⋅ 32


Solusi :

FPB(18, 24, 18n, 72) = 6 = 2 ⋅ 3


18 = 2a3b ; 24 = 2c3d ; 18n = 2 ⋅ 32n = 2e3f dan 72 = 2g3h
maks (a, c, e, g) = 3 ; maks (b, d, f, h) = 2 ; min (a, c, e, g) = 1 ; min (b, d, f, h) = 1
Karena a = 1, c = 3, g = 3 maka e = 1, 2 at au 3
Karena b = 2, d = 1, h = 2 maka f = 1 at au 2.

18n = 2 ⋅ 32n = 2e32


Tet api f min = 2 maka f = 2

Jika e = 1 maka n = 1 sehingga 18n = 18

Eddy Hermanto, ST 67 Teori Bilangan


Pembinaan Olimpiade Matematika
Jika e = 2 maka n = 2 sehingga 18n = 36
Jika e = 3 maka n = 4 sehingga 18n = 72
Karena keempat bilangan t ersebut berbeda maka 18n = 36
n=2
Jadi, bilangan asli n yang memenuhi adalah n = 2.

Cont oh 11 :
(OSP 2006) Misalkan d = FPB(7n + 5, 5n + 4), dimana n adalah bilangan asli.
a. Bukt ikan bahwa unt uk set iap bilangan asli n berlaku d = 1 at au 3.
b. Bukt ikan bahwa d = 3 j ika dan hanya j ika n = 3k + 1, unt uk suat u bilangan asli k.

Solusi :
d = FPB(7n + 5, 5n + 4)
a. Maka d⏐7n + 5 dan d⏐5n + 4
Karena d membagi 7n + 5 maka d j uga membagi 5(7n + 5)

Akibat nya d j uga membagi 7(5n + 4) − 5(7n + 5) = 3


Karena d membagi 5n + 4 maka d j uga membagi 7(5n + 4)

Karena d⏐3 maka d = 1 at au 3 (t erbukt i)

Jika n = 3k maka 7n + 5 = 21k + 5 ≡ 2 (mod 3) dan 5n + 4 = 15k + 4 ≡ 1 (mod 3)


b. Sebuah bilangan akan t ermasuk ke dalam salah sat u bent uk dari 3k, 3k + 1 at au 3k + 2

Jika n = 3k + 1 maka 7n + 5 = 21k + 12 ≡ 0 (mod 3) dan 5n + 4 = 15k + 9 ≡ 0 (mod 3)


Jika n = 3k + 2 maka 7n + 5 = 21k + 19 ≡ 1 (mod 3) dan 5n + 4 = 15k + 14 ≡ 2 (mod 3)
Hanya bent uk n = 3k + 1 yang menyebabkan kedua bilangan 7n + 5 dan 5n + 4 habis dibagi 3 unt uk n
bilangan asli.
Jadi, t erbukt i bahwa d = 3 j ika dan hanya j ika n = 3k + 1, unt uk suat u bilangan asli k

LATIHAN 4 :

adalah ⋅⋅⋅⋅⋅
1. (OSK 2008) Diket ahui FPB (a, 2008) = 251. Jika a > 2008 maka nilai t erkecil yang mungkin bagi a

adalah ⋅⋅⋅⋅⋅
2. (OSK 2011) Bilangan asli t erkecil lebih dari 2011 yang bersisa 1 j ika dibagi 2, 3, 4, 5, 6, 7, 8, 9, 10

∑ FPB(k ,7 ) adalah ⋅⋅⋅⋅⋅⋅⋅⋅


2009
3. (OSK 2009) Nilai dari
k =1

bilangan asli a1 < a2 < a3 memenuhi fpb(a1, a2, a3) = 1, t et api fpb(ai , aj ) > 1 j ika i ≠ j , i, j = 1, 2, 3.
4. (OSP 2004) Not asi fpb(a, b) menyat akan f akt or persekut uan t erbesar dari bilangan bulat a dan b. Tiga

Tent ukan (a1, a2, a3) agar a1 + a2 + a3 minimal.

5. (OSP 2006) Dari set iap bilangan sat u-angka a, bilangan N dibuat dengan menyandingkan ket iga
bilangan a + 2, a + 1, a yait u N = (a + 2)(a + 1)a . Sebagai cont oh, unt uk a = 8, N = 1098. Kesepuluh
bilangan N semacam it u memiliki fakt or persekut uan t erbesar ⋅⋅⋅⋅⋅

S = {gcd(n3 + 1, n2 + 3n + 9)⏐n ∈ Z}
6. (OSP 2010) Banyaknya anggot a himpunan

adalah ⋅⋅⋅⋅⋅

7. (AIME 1998) Ada berapa banyak nilai k sehingga KPK(66, 88, k) = 1212 ?

Eddy Hermanto, ST 68 Teori Bilangan


Pembinaan Olimpiade Matematika

8. Jumlah dua bilangan asli sama dengan 52 sedangkan Kelipat an Persekut uan Terkecilnya sama dengan
168. Tent ukan selisih posit if dua bilangan t ersebut .

9. Dua bilangan memiliki j umlah 145. Misalkan Kelipat an Persekut uan Terkecil (KPK) kedua bilangan
t ersebut adalah k dan Fakt or Persekut uan Terbesar (FPB) kedua bilangan t ersebut adalah d. Jika
perbandingan k : d = 168, maka t ent ukan selisih posit if kedua bilangan t ersebut ?

10. (ME V5N4) Tent ukan semua pasangan bilangan bulat posit if (a, b) yang memenuhi :
FPB(a, b) + KPK(a, b) = a + b + 6

21n + 4
11. (IMO 1959) Bukt ikan bahwa pecahan 14 n + 3 t idak dapat disederhanakan unt uk semua nilai n bilangan
asli.

n 2 + n −1
n2 + 2n
12. (Mexican MO 1987) Bukt ikan bahwa pecahan t idak dapat disederhanakan unt uk semua nilai n
bilangan asli.

13. (AIME 1985) Misalkan d(n) adalah fakt or persekut uan t erbesar dari 100 + n2 dan 100 + (n + 1) 2 unt uk
set iap n = 1, 2, 3, ⋅⋅⋅. Tent ukan nilai d(n) yang t erbesar.

Eddy Hermanto, ST 69 Teori Bilangan


Pembinaan Olimpiade Matematika

Misalkan M = p1a1 ⋅ p2a2 ⋅ p3a3 ⋅ ⋅⋅⋅ pnan unt uk bilangan asli M sert a p1, p2, p3, ⋅⋅⋅, pn semuanya adalah bilangan
5. BANYAKNYA FAKTOR POSITIF

Banyaknya fakt or posit if dari M adalah (a1 + 1)(a2 + 1)(a3 + 1) ⋅⋅⋅ (an + 1)
prima maka :

Cont oh 12 :
(OSK 2004 SMP/ MTs) Joko mengalikan t iga bilangan prima berbeda sekaligus. Ada berapa fakt or berbeda
dari bilangan yang dihasilkan ?
A. 3 B. 4 C. 5 D. 6 E. 8

Solusi :
Misalkan t iga bilangan prima t ersebut adalah a, b dan c dan N = a x b x c.
Maka sesuai t eori, banyaknya fakt or posit if dari N adalah (1 + 1)(1 + 1)(1 + 1) = 8. (Jawaban : E)
Kedelapan fakt or t ersebut adalah 1, a, b, c, ab, ac, bc dan abc.
Jadi, banyaknya fakt or berbeda adalah 8.

(OSK 2004) Bilangan 2004 memiliki fakt or selain 1 dan 2004 sendiri sebanyak ⋅⋅⋅⋅⋅
Cont oh 13 :

2004 = 22 ⋅ 501
Solusi :

2004 = 22 ⋅ 3 ⋅ 167 dan 167 adalah bilangan prima.

Banyaknya fakt or 2004 selain 1 dan 2004 adalah = 12 − 2 = 10


Maka banyaknya fakt or posit if dari 2004 t ermasuk 1 dan 2004 = (2 +1)(1 + 1)(1 + 1) = 12

Fakt or dari 2004 selain 1 dan 2004 adalah : 2, 3, 4, 6, 12, 167, 334, 501, 668, 1002.
Bilangan 2004 memiliki fakt or selain 1 dan 2004 sendiri sebanyak 10

LATIHAN 5 :

1. (OSK 2008) Banyaknya fakt or posit if dari 5! adalah

berbeda t erbanyak adalah ⋅⋅⋅⋅⋅⋅⋅


2. (OSP 2007) Di ant ara bilangan-bilangan 2006, 2007 dan 2008, bilangan yang memiliki fakt or prima

3. (MATNC 2001) Tent ukan bilangan asli t erkecil yang memiliki t epat 12 fakt or posit if.

4. (MATNC 2001) Tent ukan bilangan asli t erkecil yang memiliki t epat 12 fakt or posit if dan t idak habis
dibagi 3.

5. (OSP 2002) Misalkan M dan m bert urut -t urut menyat akan bilangan t erbesar dan bilangan t erkecil di

t erbesar dari M − m ?
ant ara semua bilangan 4-angka yang j umlah keempat angkanya adalah 9. Berapakah fakt or prima

kelipat an 2009. Fakt or prima t erkeci dari n adalah ⋅⋅⋅⋅⋅⋅


6. (OSP 2009) Misalkan n bilangan asli t erkecil yang mempunyai t epat 2009 fakt or dan n merupakan

7. (AIME 1990) n adalah bilangan asli t erkecil yang merupakan kelipat an 75 dan memiliki t epat 75 fakt or
n
posit if. Tent ukan nilai dari 75 .

Eddy Hermanto, ST 70 Teori Bilangan


Pembinaan Olimpiade Matematika

banyaknya fakt or posit if yang dimiliki 6n adalah ⋅⋅⋅⋅⋅⋅⋅


8. Misalkan n bilangan asli. 2n mempunyai 28 fakt or posit if dan 3n punya 30 fakt or posit if maka

9. (OSK 2011) Ada berapa fakt or posit if dari 27355372 yang merupakan kelipat an 10 ?

10. (AIME 1994) Tent ukan fakt or prima t erbesar dari p(1) + p(2) + ⋅⋅⋅ + p(999) dimana p(n) adalah hasil kali
semua angka-angka t aknol dari n.

11. (MATNC 2001) Tent ukan penj umlahan semua fakt or posit if dari 84.

12. (AIME 1995) Tent ukan banyaknya fakt or posit if dari n2 yang kurang dari n t et api t idak membagi n j ika
n = 231 319 .

13. (AIME 2000) Tent ukan bilangan asli t erkecil yang memiliki 12 fakt or posit if genap dan 6 fakt or posit if
ganj il.

14. (OSN 2004) Berapa banyaknya pembagi genap dan pembagi ganj il dari 56 − 1 ?

Eddy Hermanto, ST 71 Teori Bilangan


Pembinaan Olimpiade Matematika
6. KEKONGRUENAN

dapat dinyat akan ke dalam bent uk N = pq + r at au N − r = pq dengan p, q, r adalah bilangan bulat dan r
Konsep kekongruenan bilangan dikembangkan berdasarkan konsep bahwa set iap bilangan bulat posit if

berada pada 0 ≤ r < p. Persamaan N = pq + r dengan p menyat akan pembagi, q menyat akan hasil bagi dan

Persamaan di at as sering pula dit ulis N ≡ r (mod p)


r menyat akan sisa.

Dari hal t ersebut didapat definisi bahwa a ≡ b (mod m) j ika m⏐(a − b) unt uk bilangan bulat a, b dan m.

(1) 25 ≡ 1 (mod 4) sebab 4⏐24


Cont oh :

(2) 1 ≡ −3 (mod 4) sebab 4⏐4

Beberapa sifat berkait an dengan modulu adalah sebagai berikut . Misalkan a, b, c, d dan m adalah

a ≡ a (mod m)
bilangan-bilangan bulat dengan d > 0 dan m > 0, berlaku :

(ii) Jika a ≡ b (mod m), maka b ≡ a (mod m)


(i)

(iii) Jika a ≡ b (mod m) dan b ≡ c (mod m) maka a ≡ c (mod m)


(iv) Jika a ≡ b (mod m) dan d⏐m maka a ≡ b (mod d)
Jika a ≡ b (mod m) maka ak ≡ bk (mod m) unt uk semua k bilangan asli
(vi) Jika a ≡ b (mod m) dan f(x) = anxn + an-1xn-1 + ⋅⋅⋅ + ao maka f(a) ≡ f(b) (mod m)
(v)

(vii) Jika a ≡ b (mod m) dan c ≡ d (mod m) maka a + c ≡ b + d (mod m)


(viii) Jika a ≡ b (mod m) dan c ≡ d (mod m) maka ac ≡ bd (mod m)
(ix) (am + b)k ≡ bk (mod m) unt uk semua k bilangan asli
(x) Dari sifat (viii) didapat (am + b) k ⋅ (cm + d) n ≡ bk ⋅ dn (mod m) unt uk semua k dan n bilangan asli
(xi) Jika ca ≡ cb (mod m) dan FPB(c, m) = 1 maka a ≡ b (mod m)
(xii) Misalkan n ∈ N dan S(n) adalah penj umlahan digit -digit dari n maka berlaku n ≡ S(n) (mod 9).
(xiii) n5 ≡ n (mod 10) unt uk set iap n ∈ N.

Cont oh 14 :

merupakan sisa a x b j ika dibagi oleh 5. Bilangan yang dit unj ukkan oleh <−3, 4> adalah ⋅⋅⋅⋅⋅⋅
(OSP 2004 SMP/ MTs) Unt uk bilangan bulat a dan b, <a, b> art inya bilangan bulat t ak negat if yang

Karena −3 x 4 = − 12 = 5 x (−3) + 3 maka <−3, 4> = 3


Solusi :

Jadi, <−3, 4> = 3.

Cont oh 15 :
(OSN 2003 SMP/ MTs) Unt uk menarik minat pelanggan, suat u rest oran penj ual makanan cepat saj i
memberikan kupon berhadiah kepada set iap orang yang membeli makanan di rest oran t ersebut dengan
nilai lebih dari Rp. 25.000,-. Di balik set iap kupon t ersebut t ert era salah sat u dari bilangan-bilangan
berikut : 9, 12, 42, 57, 69, 21, 15, 75, 24 dan 81. Pembeli yang berhasil mengumpulkan kupon dengan
j umlah bilangan di balik kupon t ersebut sama dengan 100 akan diberi hadiah berupa TV 21” . Kalau
pemilik rest oran t ersebut menyediakan sebanyak 10 buah TV 21” , berapa banyak yang harus diserahkan
kepada para pelanggannya ?

Solusi :
Bilangan-bilangan 9, 12, 42, 57, 69, 21, 15, 75, 24 dan 81 semuanya habis dibagi 3.
Maka penj umlahan bilangan-bilangan mana pun di ant ara 9, 12, 42, 57, 69, 21, 15, 75, 24 dan 81 akan
menghasilkan suat u bilangan yang habis dibagi 3.
Tet api 100 j ika dibagi 3 akan bersisa 1.
Maka t idak ada TV yang diserahkan.

Eddy Hermanto, ST 72 Teori Bilangan


Pembinaan Olimpiade Matematika

Cont oh 16 :
77
Tent ukan angka sat uan dari 7 .

Solusi :

yang bersisa 1 at au −1 j ika dibagi dengan m agar memudahkan dalam perhit ungan sebab 1k = 1 sedangkan
Dalam persoalan bent uk perpangkat an dengan modulu m selalu diusahakan agar didapat suat u bilangan

(−1)k sama dengan 1 at au −1 bergant ung dari parit as k unt uk k suat u bilangan bulat .
Angka sat uan merupakan sisa j ika suat u bilangan dibagi 10.
Alt ernat if 1 :
Perlu diingat bahwa 74 = 2401 yang bersisa 1 j ika dibagi 10.

72 ≡ 1 (mod 4)
Maka akan dibuat pangkat dari bilangan pokok 7 dinyat akan dalam modulu 4.

77 = (72) 3 ⋅ 7 ≡ 13 ⋅ 7 (mod 4) ≡ 3 (mod 4)

( )
Sehingga 77 = 4k + 3 unt uk suat u bilangan asli k.
7 7 = 7 4 k +3 = 7 4 ⋅ 7 3 = (240 ⋅ 10 + 1) k ⋅ (34 ⋅ 10 + 3) ≡ 1k ⋅ 3 (mod 10)
7 k

7 7 ≡ 3 (mod 10)
7

77
Jadi, angka sat uan dari 7 adalah 3.

72 = 49 yang bersisa −1 j ika dibagi 10.


Alt ernat if 2 :

Mngingat bahwa (−1) k bernilai posit if hanya j ika k genap maka akan dibuat pangkat dari bilangan pokok 7

72 ≡ 1 (mod 4)
dinyat akan dalam modulu 4.

77 = (72) 3 ⋅ 7 ≡ 13 ⋅ 7 (mod 4) ≡ 3 (mod 4)


Sehingga 77 = 4k + 3 unt uk suat u bilangan asli k.
7 7 = 74k+3 = (72)2k ⋅ 73 ≡ (5 ⋅ 10 − 1)2k ⋅ (343) (mod 10) ≡ (−1)2k ⋅ (3) (mod 10) ≡ 3 (mod 10)
7

77
Jadi, angka sat uan dari 7 adalah 3.
Cara lain penyelesaian soal t ersebut adalah dengan melihat pola dari angka sat uan yang selalu berulang.
Cara ini t idak dibahas dalam cont oh ini dan dipersilakan kepada Pembaca unt uk menyelesaikannya.

Cont oh 17 :
Tent ukan angka sat uan dari 20072009.

Solusi :

20072009 = (200 ⋅ 10 + 7) 2009


Mencari angka sat uan dari suat u bilangan sama dengan mencari sisa j ika bilangan t ersebut dibagi 10.

20072009 ≡ 72009 (mod 10) (menggunakan sifat (ix)

20072009 ≡ (74) 502 ⋅ 71 (mod 10)


Alt ernat if 1 :

20072009 ≡ (240 ⋅ 10 + 1) 502 ⋅ 71 (mod 10)


20072009 ≡ 1502 ⋅ 7 (mod 10) (menggunakan sifat (x)
20072009 ≡ 7 (mod 10)
Jadi, angka sat uan 20072009 adalah 7.

20072009 ≡ (72) 1004 ⋅ 71 (mod 10)


Alt ernat if 2 :

20072009 ≡ (5 ⋅ 10 − 1)1004 ⋅ 71 (mod 10)


20072009 ≡ (−1) 1004 ⋅ 7 (mod 10) (menggunakan sifat (x)
20072009 ≡ 7 (mod 10)
Jadi, angka sat uan 20072009 adalah 7.

Eddy Hermanto, ST 73 Teori Bilangan


Pembinaan Olimpiade Matematika

(OSK 2004 SMP/ MTs) Jika 213 dibagi dengan 13, maka akan memberikan sisa sama dengan ⋅⋅⋅⋅⋅
Cont oh 18 :

Solusi :

213 = 8192 = 13 ⋅ 630 + 2


Alt ernat if 1 :

Maka sisa j ika 213 dibagi dengan 13 adalah 2.

Alt ernat if 2 :
Alt ernat if 1 bisa digunakan unt uk suat u bilangan-bilangan kecil. Tet api j ika pangkat dari 2 merupakan

213 =(26) 2 ⋅ 2 = (13 ⋅ 5 − 1) 2 ⋅ 2 ≡ (−1) 2 ⋅ 2 (mod 13) ≡ 2 (mod 13)


bilangan yang besar maka cara t ersebut t idak ef ekt if. Alt ernat if berikut bisa dipert imbangkan.

Maka sisa j ika 213 dibagi dengan 13 adalah 2.

Tent ukan sisa pembagian 3 ⋅ 53 + 272010 oleh 7.


Cont oh 19 :

53 = (8 ⋅ 7 − 3) ≡ −3 (mod 7)
Solusi :

3 ⋅ 53 ≡ 3 (−3) (mod 7) ≡ −9 (mod 7) ≡ −2 (mod 7)


27 ≡ −1 (mod 7) sehingga 272010 ≡ (−1)2010 (mod 7) ≡ 1 (mod 7)
3 ⋅ 53 + 272010 ≡ −2 + 1 (mod 7) ≡ −1 (mod 7) ≡ 6 (mod 7)
Jadi, sisa pembagian 3 ⋅ 53 + 272010 oleh 7 adalah 6.

(MATNC 2001) N adalah bilangan asli yang memenuhi N ≡ 2 (mod 3) dan N ≡ 1 (mod 2). Tent ukan sisanya
Cont oh 20 :

j ika N dibagi 6.

Solusi :
Alt ernat if 1 :
Karena N j ika dibagi 3 bersisa 2 maka N j ika dibagi 6 akan bersisa 2 at au 5.
Karena N bersisa 1 j ika dibagi 2 maka t idak mungkin N akan berbent uk N = 6k + 2.
Maka N = 6k + 5.
Jadi, j ika N dibagi 6 maka akan bersisa 5.

Karena N ≡ 1 (mod 2) maka N = 2p − 1 unt uk suat u bilangan bulat p.


Alt ernat if 2 :

2p − 1 ≡ 2 (mod 3) sehingga 2p ≡ 0 (mod 3)


Karena 2 dan 3 relat if prima maka p ≡ 0 (mod 3)

N = 6q − 1 ≡ −1 (mod 6) ≡ 5 (mod 6)
Maka didapat p = 3q unt uk suat u bilangan bulat q.

Jadi, sisanya j ika N dibagi 6 adalah 5.

Karena N ≡ 1 (mod 2) maka N = 2p + 1 unt uk suat u bilangan bulat p.


Alt ernat if 3 :

2p + 1 ≡ 2 (mod 3) sehingga 2p ≡ 1 (mod 3)


Karena 2 dan 3 relat if prima maka p ≡ 2 (mod 3)

N = 2(3q + 2) + 1 ≡ 5 (mod 6)
Maka didapat p = 3q + 2 unt uk suat u bilangan bulat q.

Jadi, sisanya j ika N dibagi 6 adalah 5.

Eddy Hermanto, ST 74 Teori Bilangan


Pembinaan Olimpiade Matematika

Karena N ≡ 2 (mod 3) maka N = 3p + 2 unt uk suat u bilangan bulat p.


Alt ernat if 4 :

3p + 2 ≡ 1 (mod 2) sehingga 3p ≡ 1 (mod 2)


Karena 2 dan 3 relat if prima maka p ≡ 1 (mod 2)

N = 3(2q + 1) + 2 ≡ 5 (mod 6)
Maka didapat p = 2q + 1 unt uk suat u bilangan bulat q.

Jadi, sisanya j ika N dibagi 6 adalah 5.

Cont oh 21 :
Tent ukan dua angka t erakhir dari 32009.

32009 = (35) 400 ⋅ 39 = (243)400 ⋅ 39


Solusi :

32009 ≡ (43)400 ⋅ 39 (mod 100)


32009 ≡ (1849) 200 ⋅ 39 (mod 100)
32009 ≡ (49)200 ⋅ 19683 (mod 100)
32009 ≡ (2401) 100 ⋅ 83 (mod 100)
32009 ≡ (1)100 ⋅ 83 (mod 100)
32009 ≡ 83 (mod 100)
Jadi, dua angka t erakhir dari 32009 adalah 83.

LATIHAN 6 :

1. (OSK 2009) Jika 10999999999 dibagi oleh 7, maka sisanya adalah ⋅⋅⋅⋅⋅⋅⋅⋅⋅

2. (MATNC 2001) N adalah bilangan asli yang memenuhi N ≡ 2 (mod 3) dan N ≡ 1 (mod 2). Tent ukan
sisanya j ika N dibagi 6.

3. (OSK 2003) Misalkan N adalah bilangan bulat t erkecil yang bersifat : bersisa 2 j ika dibagi 5, bersisa 3
j ika dibagi oleh 7, dan bersisa 4 j ika dibagi 9. Berapakah hasil penj umlahan digit -digit dari N ?

4. (MATNC 2001) Tent ukan angka puluhan dari 7707.

4343
5. (OSP 2002) Berapakah sisa pembagian 43 oleh 100 ?

6. (OSP 2003) Berapakah sisa pembagian 1 ⋅ 1! + 2 ⋅ 2! + 3 ⋅ 3! + ⋅⋅⋅ + 99 ⋅ 99! + 100 ⋅ 100! oleh 101 ?

7. (MATNC 2001) Jika 10a ≡ 1 (mod 13) maka 17a j ika dibagi 13 akan bersisa ⋅⋅⋅⋅⋅

55 66 88 99 1010
8. (OSK 2005) Mana di ant ara 5 ekspresi 5 5 , 6 6 , 88 , 9 9 dan 1010 yang angka t erakhirnya
bert urut -t urut bukan 5, 6, 8, 9 at au 0 ?

9. (OSP 2011) Bilangan asli n yang memenuhi (−2004)n − 1900n + 25n − 121n habis dibagi 2000 adalah ⋅⋅⋅⋅

10. (AIME 1989) Diberikan 1335 + 1105 + 845 + 275 = k5 dengan k bilangan bulat . Tent ukan nilai k.

11. (MATNC 2001) Tent ukan sisanya j ika 5301 dibagi 8.

12. (MATNC 2001) Jika N ≡ 2 (mod 4) dan M ≡ 8 (mod 16) maka sisanya j ika MN dibagi 32 adalah ⋅⋅⋅⋅

Eddy Hermanto, ST 75 Teori Bilangan


Pembinaan Olimpiade Matematika
13. (AIME 1994) Fungsi f memenuhi f(x) + f(x − 1) = x2 unt uk semua bilangan real x. Diket ahui f(19) = 94.
Tent ukan sisanya j ika f(94) dibagi 1000.

14. (MATNC 2001) Tent ukan sisanya j ika 337.500.000 dibagi 128.

15. (AIME 1994) Barisan 3, 15, 24, 48, ⋅⋅⋅⋅ adalah barisan bilangan asli yang merupakan kelipat an 3 dan
kurang 1 dari suat u bilangan kuadrat . Tent ukan sisanya j ika suku ke-1994 dibagi 1000.

16. Berapakah sisanya j ika 1 ⋅ 3 ⋅ 5 ⋅ ⋅⋅⋅ ⋅ 2005 dibagi 1000 ?

20112011
17. (OMITS 2011) Tent ukan sisanya j ika 2011 dibagi 14.

18. Tent ukan semua kemungkinan sisa j ika N100 dibagi 125 dengan N adalah bilangan bulat ?

19. (AIME 1988) Tent ukan bilangan asli t erkecil yang j ika dipangkat t igakan akan berakhiran dengan 888.

Eddy Hermanto, ST 76 Teori Bilangan


Pembinaan Olimpiade Matematika

7. BILANGAN BULAT, RASIONAL DAN PRIMA

Secara umum bilangan dibagi menj adi dua yait u bilangan real dan bilangan t idak real.
Bilangan real dibagi menj adi dua yait u bilangan rasional dan bilangan t ak rasional.

Bilangan rasional adalah bilangan real yang dapat diubah ke dalam bent uk ba dengan a dan b keduanya
bilangan bulat dan b ≠ 0 sedangkan bilangan t ak rasional adalah bilangan real yang t idak dapat diubah ke
dalam bent uk ba dengan a dan b keduanya bilangan bulat dan b ≠ 0.
Cont oh bilangan t ak rasional adalah √2, π, e, 2log 3 dan sebagainya.

Bilangan rasional dapat dibagi menj adi dua yait u bilangan bulat dan bilangan pecahan.
Sebuah bilangan bulat posit if dapat diuraikan menj adi dalam bent uk angka-angkanya. Misalkan
ABCDEL N adalah suat u bilangan yang t erdiri dari n digit , maka dapat diuraikan menj adi A⋅10n-1 +
B⋅10n-2 + C⋅10n-3 + D⋅10n-4 + ⋅⋅⋅ + N.
Sebuah bilangan bulat selalu dapat diubah menj adi bent uk pq + r dengan 0 ≤ r < p. Sehingga j ika sebuah
bilangan bulat dibagi oleh p maka kemungkinan sisanya ada p yait u 0, 1, 2, ⋅⋅⋅, p − 1.
Sebagai cont oh j ika sebuah bilangan bulat dibagi oleh 3 maka kemungkinan sisanya adalah 0, 1 at au 2.
Maka set iap bilangan bulat dapat diubah menj adi salah sat u bent uk 3k, 3k + 1 at au 3k + 2 unt uk suat u
bilangan bulat k.

Bilangan bulat posit if p > 1 merupakan bilangan prima j ika hanya memiliki t epat dua fakt or posit if yait u 1
dan p it u sendiri sedangkan bilangan bulat n merupakan bilangan komposit j ika n memiliki lebih dari dua
fakt or posit if.
Bilangan prima genap hanya ada sat u yait u 2.
Beberapa sifat bilangan prima :
(1) Bilangan prima p hanya memiliki dua fakt or posit if yait u 1 dan p
(2) Bilangan prima t idak mungkin memiliki angka sat uan 0 sedangkan bilangan prima yang memiliki angka
sat uan sama dengan 5 hanya ada 1 yait u 5.
(3) Jika p prima maka unt uk sebarang bilangan asli n berlaku p⏐n at au FPB(p, n) = 1.
(4) Jika bilangan prima p membagi n2 unt uk suat u bilangan asli n maka p⏐n.

(6) Semua bilangan prima p > 3 memiliki bent uk 6k ± 1.


(5) Jika bilangan prima p memenuhi p⏐ab unt uk a dan b bilangan asli maka p⏐a at au p⏐b.

(7) Jika n bukan bilangan prima maka fakt or prima t erkecil dari n selalu ≤ √n. Tanda sama dengan t erj adi
j ika n meupakan kuadrat dari suat u bilangan prima.

Cont oh 22 :
m
(OSP 2002) Bilangan real 2,525252⋅⋅⋅ adalah bilangan rasional, sehingga dapat dit ulis dalam bent uk ,
dimana m, n bilangan-bilangan bulat , n ≠ 0. Jika dipilih m dan n yang relat if prima, berapakah m + n ?
n

Solusi :
Misal X = 2,525252⋅⋅⋅

100X − X = 252,525252⋅⋅⋅ − 2,525252⋅⋅⋅


100X = 252,525252⋅⋅⋅

99X = 250
X = 250
99
Karena 250 dan 99 relat if prima, maka m = 250 dan n = 99
Jadi, m + n = 349.

Eddy Hermanto, ST 77 Teori Bilangan


Pembinaan Olimpiade Matematika
Cont oh 23 :
Tent ukan semua kemungkinan sisa j ika bilangan kuadrat dibagi oleh 5.

Solusi :
Sebuah bilangan bulat past i t ermasuk ke dalam salah sat u bent uk 5k, 5k + 1, 5k + 2, 5k + 3 at au 5k + 4

Jika n = 5k maka n2 = (5k) 2 ≡ 0 (mod 5)


unt uk suat u bilangan bulat k.

Jika n = 5k + 1 maka n2 = (5k + 1) 2 ≡ 12 (mod 5) ≡ 1 (mod 5)


Jika n = 5k + 2 maka n2 = (5k + 2) 2 ≡ 22 (mod 5) ≡ 4 (mod 5)
Jika n = 5k + 3 maka n2 = (5k + 3) 2 ≡ 32 (mod 5) ≡ 4 (mod 5)
Jika n = 5k + 4 maka n2 = (5k + 4) 2 ≡ 42 (mod 5) ≡ 1 (mod 5)
Jadi, j ika bilangan kuadrat dibagi oleh 5 maka kemungkinan sisanya adalah 0, 1 at au 4.

Cont oh 24 :
Carilah bilangan bulat yang t erdiri dari 6 angka dengan angka t erakhir 7 yang menj adi 5 kali bilangan
semula j ika angka t erakhir t ersebut t empat nya dipindahkan menj adi angka pert ama.

Solusi :
Misal bilangan t ersebut adalah N = ABCDE7, maka
700000 + 10000A + 1000B + 100C + 10D + E = 5 (100000A + 10000B + 1000C + 100D + 10E + 7)
490000A + 49000B + 4900C + 490D + 49E = 699965
10000A + 1000B + 100C + 10D + E = 14285
Maka : A = 1 ; B = 4 ; C = 2 ; D = 8 ; E = 5
Jadi, bilangan t ersebut adalah 142857

Cont oh 25 :
Suat u bilangan t erdiri dari 2 angka. Bilangan t ersebut sama dengan 4 kali j umlah kedua angka t ersebut .
Jika angka kedua dikurangi angka pert ama sama dengan 2, t ent ukan bilangan t ersebut .

Solusi :

Karena b − a = 2 maka 2a − a = 2 sehingga a = 2 dan b = 4


Misal bilangan it u adalah ab maka 10a + b = 4(a + b) sehingga 2a = b

Jadi bilangan t ersebut adalah 24.

Cont oh 26 :
Berapakah banyaknya bilangan 3 angka abc (dengan a > 0) sehingga nilai a2 + b2 + c2 membagi 26 ?

Solusi :
Karena a2 + b2 + c2 membagi 26 maka a2 + b2 + c2 memiliki 4 kemungkinan nilai.
• Jika a2 + b2 + c2 = 1

• Jika a2 + b2 + c2 = 2
Tripel (a, b, c) yang memenuhi adalah (1, 0, 0) sebanyak 1 bilangan.

Tripel (a, b, c) yang memenuhi adalah (1, 1, 0) besert a permut asinya kecuali unt uk a = 0 yang t erdiri

• Jika a2 + b2 + c2 = 13
dari 2 bilangan, yait u 110, 101.

Tripel (a, b, c) yang memenuhi adalah (2, 3, 0) besert a permut asinya kecuali unt uk a = 0. Banyaknya

• Jika a2 + b2 + c2 = 26
bilangan ada 4 bilangan, yait u 203, 230, 302, 320.

Tripel (a, b, c) yang memenuhi adalah (1, 5, 0) dan (1, 3, 4) besert a permut asinya kecuali unt uk a =
0. Banyaknya bilangan ada 10 bilangan, yait u 105, 150, 501, 510, 134, 143, 314, 341, 413, 431.
Jadi, banyaknya bilangan = 1 + 2 + 4 + 10 = 17.

Eddy Hermanto, ST 78 Teori Bilangan


Pembinaan Olimpiade Matematika

LATIHAN 7 :

j umlah angka pembent uk n. Jika n bilangan dua angka dan n + p(n) + s(n) = 69 , maka n adalah ⋅⋅⋅⋅⋅⋅⋅⋅
1. (OSK 2011 Tipe 3) Unt uk bilangan asli n, p(n) dan s(n) bert urut -t urut menyat akan hasil kali dan

2. Suat u bilangan t erdiri dari 3 angka. Bilangan t ersebut sama dengan 12 kali j umlah ket iga angkanya.
Tent ukan bilangan t ersebut .

angkanya adalah 2007. Banyaknya bilangan yang dit emukan Nanang t idak akan lebih dari ⋅⋅⋅⋅⋅⋅
3. (OSK 2006) Nanang mencari semua bilangan empat -angka yang selisihnya dengan j umlah keempat

4. (OSK 2010) Pasangan bilangan asli (x, y) yang memenuhi 2x + 5y = 2010 sebanyak ⋅⋅⋅⋅⋅⋅

5. Jika j umlah digit -digit dari bilangan asli m adalah 30, maka penj umlahan semua kemungkinan j umlah
dari digit -digit m + 3 sama dengan ….

6. (OSK 2002) Berapa banyak bilangan posit if yang kurang dari 10.000 yang berbent uk x8 + y8 unt uk suat u
bilangan bulat x > 0 dan y > 0 ?

7. (AIME 1986) abc adalah bilangan asli t iga angka. Jika acb + bca + bac + cab + cba = 3194, t ent ukan
nilai dari abc.

8. Unt uk n bilangan asli, s(n) adalah penj umlahan angka-angka n dalam desimal. Tent ukan nilai n
maksimal yang memenuhi n = 7 s(n).

9. (QAMT 2001) Tent ukan semua bilangan t iga angka yang merupakan penj umlahan dari fakt orial digit -
digit nya.

9 ⋅ M ⋅ N = abcde, maka t ent ukan semua pasangan (M, N) yang memenuhi.


10. (AIME 1997) M adalah bilangan asli dua angka ab sedangkan N adalah bilangan asli t iga angka cde. Jika

1001, 1002, 1003, ⋅⋅⋅, 2000} sehingga j ika dij umlahkan maka t idak ada ’ simpanan’ ? (Sebagai cont oh
11. (AIME 1992) Ada berapa banyak pasangan bilangan asli berurutan yang diambil dari himpunan {1000,

j ika 1004 dij umlahkan dengan 1005 maka t idak ada ’ simpanan’ , t et api j ika 1005 dij umlahkan dengan
1006 maka saat menj umlahkan 5 dengan 6 maka hasilnya adalah 1 dan ’ simpanan’ 1).

12. (AIME 1989) Unt uk suat u digit d, maka 0,d25d25d25⋅⋅⋅ = n


810 dimana n ∈ N. Tent ukan n.

13. (OSP 2003) Tent ukan semua bilangan bulat a dan b sehingga bilangan
2+ a
3+ b
merupakan bilangan rasional.

a
14. (AIME 1992) Tent ukan penj umlahan semua bilangan rasional posit if 30 yang merupakan bent uk paling
sederhana sert a nilainya < 10.

0,abcabcabcabc ⋅⋅⋅ dimana bilangan asli a, b dan c t idak harus berbeda. Jika semua anggot a S dit ulis
15. (AIME 1992) Misalkan S adalah himpunan semua bilangan rasional yang dapat dit ulis ke dalam bent uk

ke dalam bent uk rs dalam bent uk yang paling sederhana, maka ada berapa banyaknya nilai r berbeda
yang muncul.

Eddy Hermanto, ST 79 Teori Bilangan


Pembinaan Olimpiade Matematika
16. (Canadian MO 1971) Misalkan n adalah bilangan lima angka dan m adalah bilangan empat angka yang
didapat dengan menghapus angka yang ada di t engah dari bilangan n. Tent ukan semua nilai n yang
memenuhi bahwa mn adalah bilangan bulat .

17. a. Diket ahui bahwa x + y dan x + y2 keduanya bilangan rasional. Apakah dapat dipast ikan x dan y
keduanya rasional ? Jelaskan.
b. Diket ahui bahwa x + y, x + y2 dan x + y3 ket iganya bilangan rasional. Apakah dapat dipast ikan x
dan y keduanya rasional ? Jelaskan.

18. (OSP 2009) Diberikan n adalah bilangan asli. Misalkan x = 6 + 2009 n . Jika x 2009 − x
x 3 −1
merupakan
bilangan rasional, t unj ukkan bahwa n merupakan kuadrat dari suat u bilangan asli.

19. (OSK 2006) Jumlah t iga bilangan prima pert ama yang lebih dari 50 adalah

adalah ⋅⋅⋅⋅
20. (OSP 2006) Bilangan prima dua angka t erbesar yang merupakan j umlah dua bilangan prima lainnya

21. (OSK 2009) Banyaknya pasangan bilangan asli (x, y) sehingga x4 + 4y4 merupakan bilangan prima
adalah ⋅⋅⋅⋅⋅

22. (AIME 1999) Tent ukan bilangan t erkecil a5 sehingga a1, a2, a3, a4, a5 membent uk barisan arit mat ika
dengan a1, a2, a3, a4, a5 semuanya bilangan prima.

23. (OSK 2002) Bilangan bulat posit if p ≥ 2 disebut bilangan prima j ika ia hanya mempunyai fakt or 1 dan
p. Tent ukan nilai penj umlahan semua bilangan prima diant ara 1 dan 100 yang sekaligus bersifat : sat u
lebihnya dari suat u bilangan kelipat an 5 dan sat u kurangnya dari suat u bilangan kelipat an 6.

24. (OSP 2010) Bilangan prima p sehingga p2 + 73 merupakan bilangan kubik sebanyak ⋅⋅⋅⋅⋅⋅

25. (OSP 2009) Bilangan prima p yang memenuhi (2p − 1)3 + (3p)2 = 6p ada sebanyak ⋅⋅⋅⋅⋅⋅

26. (OSK 2011 Tipe 1) Tent ukan semua bilangan bulat posit if p sedemikian sehingga p, p + 8, p + 16
adalah bilangan prima.

27. Unt uk n bilangan bulat berapakah sehingga 11 ⋅ 14n + 1 adalah bilangan prima ?

28. (AIME 1983) Tent ukan bilangan prima dua angka t erbesar yang membagi 200C100. Cat at an : nCr

didefinisikan ( n −nr!)!⋅ r ! dengan n! = 1 ⋅ 2 ⋅ 3 ⋅ ⋅⋅⋅ ⋅ n. Cont oh 1! = 1, 2! = 2, 3! = 6, 4! = 24, 5! = 120.

29. (OSK 2010) Diket ahui p adalah bilangan prima sehingga t erdapat pasangan bilangan bulat posit if (x, y)
yang memenuhi x2 + xy = 2y2 + 30p. Banyaknya pasangan bilangan bulat posit if (x, y) yang memenuhi
ada sebanyak ⋅⋅⋅⋅⋅⋅

6c > 12d dan a2 − b2 + c2 − d2 = 1749. Tent ukan semua kemungkinan nilai dari a2 + b2 + c2 + d2.
30. (Balt ic Way 1999 Mat hemat ical Team Cont est ) a, b, c dan d bilangan prima yang memenuhi a > 3b >

31. (Brit ish MO 2006/ 2007 Round 1) Tent ukan 4 bilangan prima kurang dari 100 yang merupakan fakt or
dari 332 − 232.

bulat posit if n, bilangan prima (t idak harus berbeda) q1, q2, q3, ⋅⋅⋅, qk, dan bilangan prima berbeda p1,
32. (OSP 2010) Diket ahui k adalah bilangan bulat posit if t erbesar, sehingga dapat dit emukan bilangan

p2, p3, ⋅⋅⋅⋅⋅, pk yang memenuhi

Eddy Hermanto, ST 80 Teori Bilangan


Pembinaan Olimpiade Matematika
1
p1 + 1
p2 +L+ 1
pk = 7 + nq1q2 Lq k
2010
Tent ukan banyaknya n yang memenuhi.

33. (OSP 2009) Diket ahui p adalah bilangan prima sehingga persamaan 7p = 8x2 − 1 dan p2 = 2y2 − 1
mempunyai solusi x dan y berupa bilangan bulat . Tent ukan semua nilai p yang memenuhi.

Eddy Hermanto, ST 81 Teori Bilangan


Pembinaan Olimpiade Matematika

8. BILANGAN KUADRAT SEMPURNA.


Bilangan kuadrat sempurna adalah bilangan bulat yang dapat diubah ke dalam bent uk n2 dengan n adalah
bilangan bulat .
Cont oh, 16 adalah bilangan kuadrat sempurna sebab 16 = 42.
Beberapa sifat bilangan kuadrat adalah :
a. Angka sat uan dari bilangan kuadrat adalah 0, 1, 4, 5, 6, 9.
b. Bilangan kuadrat j ika dibagi 3 akan bersisa 0 at au 1.
c. Bilangan kuadrat j ika dibagi 4 akan bersisa 0 at au 1
d. Bilangan kuadrat j ika dibagi 5 akan bersisa 0, 1, at au 4.
e. Bilangan kuadrat j ika dibagi 8 akan bersisa 0, 1, at au 4. Dan set erusnya.

Cont oh 27 :
Tent ukan bilangan kuadrat 4 angka dengan angka pert ama sama dengan angka kedua dan angka ket iga
sama dengan angka keempat .

Solusi :
Misal bilangan t ersebut adalah aabb.
Karena aabb kuadrat maka nilai b yang memenuhi adalah 0, 1, 4, 5, 6, at au 9. Tet api 11, 55, 99 j ika
dibagi 4 bersisa 3 sedangkan 66 j ika dibagi 4 bersisa 2 yang membuat aabb t idak mungkin merupakan

Jika b = 0 maka aa00 = 102(10a + a) yang berakibat 10a + a harus bilangan kuadrat . Tet api 11, 22, 33, ⋅⋅⋅,
bilangan kuadrat . Jadi nilai b yang mungkin adalah 0 at au 4.

99 t idak ada sat upun yang merupakan bilangan kuadrat . Sehingga t idak mungkin aa00 kuadrat . Jadi,b ≠ 0.
Jika b = 4 maka aa44 = 11(100a + 4). Karena aa44 bilangan kuadrat maka 100a + 4 = 11k2. Sesuai dengan
sifat bilangan habis dibagi 11 maka a + 4 − 0 habis dibagi 11. Nilai a yang memenuhi hanya 7.
Jadi bilangan t ersebut adalah 7744.

LATIHAN 8 :

1. Tent ukan bilangan asli t erkecil yang j ika dikalikan dengan 420 maka hasilnya adalah bilangan kuadrat
sempurna.

2. (AIME 2001) Tent ukan bilangan asli t erbesar sehingga dua angka berurut an membent uk kuadrat
sempurna. Sebagai cont oh adalah 364 sebab 36 dan 64 merupakan bilangan kuadrat sempurna.

3. a, b, c dan d adalah digit -digit suat u bilangan. Bilangan 7 angka berikut :

merupakan kuadrat sempurna, kecuali ⋅⋅⋅⋅⋅⋅⋅⋅


a0bc225 ; abcd756 ; 1abc584 ; ab3c289 ; 4abc899

4. Bukt ikan bahwa t idak ada pasangan bilangan bulat (a, b) yang memenuhi
a2 = 2005b2 + 2

11, 111, 1111, ⋅⋅⋅, 111⋅⋅⋅1111


5. Adakah di ant ara bilangan-bilangan

22, 222, 2222, ⋅⋅⋅, 222⋅⋅⋅2222


33, 333, 3333, ⋅⋅⋅, 333⋅⋅⋅3333
⋅⋅⋅⋅⋅⋅⋅⋅
99, 999, 9999, ⋅⋅⋅, 999⋅⋅⋅9999
yang merupakan bilangan kuadrat sempurna ?

6. (Canadian MO 1978) n adalah bilangan bulat . Jika angka puluhan n2 adalah t uj uh, apakah angka
sat uan dari n2 ?

Eddy Hermanto, ST 82 Teori Bilangan


Pembinaan Olimpiade Matematika

7. (OSK 2009) Banyaknya bilangan asli kurang dari 1000 yang dapat dinyat akan dalam bent uk x2 − y2
unt uk suat u bilangan ganj il x dan y adalah ⋅⋅⋅⋅⋅⋅⋅

8. Misalkan a dan b adalah bilangan bulat yang memenuhi a + 2b dan b + 2a keduanya bilangan kuadrat
sempurna. Bukt ikan bahwa a dan b keduanya merupakan kelipat an 3.

dengan a ≠ 0 dan d ≠ 0 yang memenuhi abcdef = (def) 2.


9. (Flanders MO 1999 Final Round) Tent ukan semua bilangan asli t erdiri dari 6 angka, misalkan abcdef,

a2 + b2 − 8c = 6.
10. (Canadian MO 1969) Tunj ukkan bahwa t idak ada bilangan bulat a, b dan c yang memenuhi persamaan

11. abac adalah bilangan 4 digit yang merupakan kuadrat dari sebuah angka 2 digit . Jika kit a naikan
semua nilai digit dari abac dengan 1 maka bilangan hasil j uga merupakan kuadrat dari angka 2 digit
yang lain. Berapakah nilai dari a + b + c ?

12. (AIME 1999) Tent ukan penj umlahan semua n ∈ N sehingga n2 − 19n + 99 merupakan bilangan kuadrat
sempurna.

13. Bukt ikan bahwa 2n6k + 4n2k + 11 t idak mungkin bilangan kuadrat .

14. Tent ukan pasangan bilangan bulat posit if x dan n yang memenuhi persamaan x2 + 615 = 2n.

15. (Balt ic Way 1994 Mat hemat ical Team Cont est ) Tent ukan semua pasangan bulat posit if (a, b) yang
memenuhi 2a + 3b adalah bilangan kuadrat sempurna.

Eddy Hermanto, ST 83 Teori Bilangan


Pembinaan Olimpiade Matematika

Perhat ikan fungsi y = f(x) = ⎣x⎦ dengan t anda ⎣α⎦ menyat akan bilangan bulat t erbesar kurang dari at au
9. FUNGSI TANGGA DAN CEILING

sama dengan α.
Jika x bernilai 3,7 maka y = ⎣3,7⎦ = 3.
Jika x bernilai 4 maka y = ⎣4⎦ = 4
Jika x bernilai −2,5 maka y = ⎣−2,5⎦ = −3
Jika 4 ≤ x < 5 maka y = f(x) = 4.
Jika 5 ≤ x < 6 maka y = f(x) = 5. Dan set erusnya.
Jika fungsi t ersebut digambarkan dalam koordinat kart esian maka

Perhat ikan fungsi y = f(x) = ⎡x⎤ dengan t anda ⎡α⎤ menyat akan bilangan bulat t erkecil lebih dari at au sama
Selain it u ada j uga yang disebut fungsi ceiling yang merupakan kebalikan dari fungsi t angga.

dengan α.
Jika x bernilai 3,7 maka y = ⎡3,7⎤ = 4.
Jika x bernilai 4 maka y = ⎡4⎤ = 4
Jika x bernilai −2,5 maka y = ⎡−2,5⎤ = −2
Jika 4 ≤ x < 5 maka y = f(x) = 5.
Jika 5 ≤ x < 6 maka y = f(x) = 6. Dan set erusnya.
Grafik fungsinya pun agak mirip dengan fungsi t angga.

(x − 1) < ⎣x⎦ ≤ x
Dari pengert ian t ersebut akan didapat kan

(ii) ⎣x⎦ ≤ ⎡x⎤.


(i)

Tanda ⎣x⎦ dapat digunakan unt uk menghit ung pangkat t ert inggi bilangan prima dari suat u bilangan n!
Tanda kesamaan t erj adi hanya saat x adalah bilangan bulat .

Misalkan diket ahui n! = 1 x 2 x ⋅⋅⋅ x n dan p suat u bilangan prima. Akan dicari nilai k maksimal sehingga
⎣⎦
dengan t anda “ !” menyat akan fakt orial.

pk⏐n!. Banyaknya bilangan di ant ara 1, 2, 3, ⋅⋅⋅, n yang merupakan kelipat an prima adalah np . Tet api

⎣ ⎦ unt uk p
⎣ ⎦ masih harus dit ambahkan dengan ⎣ ⎦ . Tet api nilai
kmaks > n 2
≤ n sebab masih ada bilangan kelipat an p yang fakt ornya baru dihit ung sat u kali.
2

Maka unt uk mencari kmaks dengan p2 ≤ n, nilai

⎣ ⎦ + ⎣ ⎦ unt uk p
p
n n
p p2

≤ n sebab masih ada bilangan kelipat an p3 yang fakt ornya baru dihit ung dua

⎣ ⎦ + ⎣ ⎦ masih harus dit ambahkan dengan ⎣ ⎦.


n n 3
kmaks > p p2

kali. Maka unt uk mencari kmaks dengan p3 ≤ n, nilai n n n

⎣ ⎦ ⎣ ⎦+ ⎣ ⎦+ L
p p2 p3

Jadi, nilai kmaks = np +


Demikian set erusnya.
n n
p2 p3

Eddy Hermanto, ST 84 Teori Bilangan


Pembinaan Olimpiade Matematika

⎣ ⎦
Cont oh 28 :
522007 sama dengan ⋅⋅⋅⋅⋅⋅⋅⋅

⎣ ⎦
522007 = ⎣722,LL⎦
Solusi :

⎣ ⎦
7222 = 521284 < 522007 ; 7232 = 522729 > 522007 maka

522007 = 722

Bilangan 2010! = 1 ⋅ 2 ⋅ 3 ⋅ ⋅⋅⋅ ⋅ 2010 habis dibagi oleh 7k unt uk suat u bilangan asli k t ert ent u. Tent ukan
Cont oh 29 :

nilai maksimal dari k.

Di ant ara 2010 bilangan 1, 2, 3, ⋅⋅⋅, 2010 t erdapat ⎣ 2010


7 ⎦ = 287 bilangan yang habis dibagi 7.
Solusi :

⎣ ⎦
2
Jika kmaks = 287 maka akan ada bilangan kelipat an 7 yang fakt or 7-nya hanya dihit ung sat u kali. Maka nilai
k t ersebut haruslah dit ambahkan dengan 2010 = 41.

⎣ ⎦
72
Tet api fakt or 7 dari bilangan kelipat an 73 = 343 hanya dihit ung dua kali padahal seharusnya t iga kali.
Maka hasil sebelumnya harus dit ambahkan dengan 2010 = 5. Karena t idak ada bilangan kelipat an 74 dari

7 ⎦ + ⎣ 7 2 ⎦ + ⎣ 73 ⎦
73

k maks = ⎣ 2010
2010 bilangan t ersebut maka perhit ungan t elah lengkap.
2010 2010

kmaks = 333.

LATIHAN 9 :

1. (OSK 2011) Ani mempunyai sangat banyak dadu dengan ukuran 3 cm x 3 cm x 3 cm. Jika ia
memasukkan dadu- dadu t ersebut ke dalam sebuah kardus dengan ukuran 50 cm x 40 cm x 35 cm
maka berapa banyak dadu yang bisa masuk ke dalam kardus t ersebut ?

2. (OSK 2003) Unt uk set iap bilangan real α, kit a definisikan ⎣α ⎦


at au sama dengan α. Sebagai cont oh ⎣4,9⎦ = 4 dan ⎣7 ⎦ = 7. Jika x dan y bilangan real sehingga ⎣ x⎦
sebagai bilangan bulat yang kurang dari

= 9 dan ⎣ y ⎦ = 12, maka nilai t erkecil yang mungkin dicapai oleh ⎣ y − x⎦ adalah ?
3. (OSK 2007) Jika n adalah bilangan asli sehingga 3n adalah fakt or dari 33!, maka nilai n t erbesar yang
mungkin adalah ⋅⋅⋅⋅

4. (OSP 2009) Pada bagian kanan 100! t erdapat digit 0 bert urut -t urut sebanyak ⋅⋅⋅⋅⋅⋅⋅⋅

bilangan 26!. (Maksud soal ini adalah 26! = ⋅⋅⋅⋅⋅⋅0000. Ada berapa banyak angka nol yang t erlet ak pada
5. Angka t erakhir dari 26! Past i 0. Tent ukan banyaknya angka 0 berurut an yang t erlet ak pada akhir

akhir bilangan t ersebut ).

6. (AIME 1994) Tent ukan bilangan asli n yang memenuhi ⎣2log 1⎦ + ⎣2log 2⎦ + ⎣2log 3⎦ + ⋅⋅⋅ + ⎣2log n⎦ = 1994.

7. (ARML 2000 Individual) Jika dilihat dari kiri ke kanan 7 digit t erakhir dari n! adalah 8000000. Tent ukan
nilai n. ( Ingat n! adalah n fakt orial, n! = 1⋅2⋅3⋅4⋅⋅⋅⋅(n−1)⋅n )

Eddy Hermanto, ST 85 Teori Bilangan


Pembinaan Olimpiade Matematika

8. (COMC 2003) Lambang ⎣a⎦ memiliki art i bilangan bulat t erbesar kurang dari at au sama dengan a.
Sebagai cont oh, ⎣5,7⎦ = 5, ⎣4⎦ = 4 dan ⎣−4,2⎦ = −5. Tent ukan semua bilangan real x yang memenuhi
⎣ 3x ⎦ + ⎣4x ⎦ = 5 .
9. Tanda ⎣x⎦ menyat akan bilangan bulat t erbesar kurang dari at au sama dengan x. Sebagai cont oh
adalah ⎣4,2⎦ = 4, ⎣π⎦ = 3. Jika M = ⎣ 10 66
10 33 + 3 ⎦ , berapakah sisanya j ika M dibagi 1000 ?

⎣ ⎦ ⎣ 3⎦,
10. (OSP 2005) Unt uk sembarang bilangan real a, not asi ⎣a⎦ menyat akan bilangan bulat t erbesar yang
lebih kecil dari at au sama dengan a. Jika x bilangan real yang memenuhi x + 3 = ⎣x ⎦ +
maka x − ⎣x⎦ t idak akan lebih besar dari ⋅⋅⋅⋅⋅

11. (AIME 1991) Bilangan real x memenuhi ⎣x + 0,19⎦ + ⎣x + 0,20⎦ + ⎣x + 0,21⎦ + ⋅⋅⋅ + ⎣x + 0,91⎦ = 546.
Tent ukan nilai dari ⎣100x⎦.

12. Tent ukan nilai dari ⎣1x2008


2007
⎦ + ⎣2 2008
x 2007
⎦ + ⎣3 x2008
2007
⎦ + L + ⎣20072008
x 2007
⎦.
13. Tent ukan semua nilai x real yang memenuhi persamaan ⎣5+86 x ⎦ = 15 x5−7 .
14. (AIME 2002) Tent ukan bilangan asli n t erkecil sehingga t idak ada x bulat yang memenuhi ⎣2002
x ⎦ = n.

15. (OSP 2009) Misalkan q= 5 +1


dan ⎣x⎦ menyat akan bilangan bulat t erbesar yang lebih kecil at au sama
dengan x. Nilai ⎣q⎣qn⎦⎦ − ⎣q n⎦ unt uk sebarang n ∈ N adalah ⋅⋅⋅⋅⋅⋅
2
2

16. (OSP 2010) Unt uk sebarang bilangan real x didefinisikan ⎣x⎦ sebagai bilangan bulat t erbesar yang
kurang dari at au sama dengan dengan x. Bilangan asli n sehingga persamaan x ⎣ 1x ⎦ + 1x ⎣x ⎦ = nn+1
mempunyai t epat 2010 solusi real posit if adalah ⋅⋅⋅⋅⋅⋅

Eddy Hermanto, ST 86 Teori Bilangan


Pembinaan Olimpiade Matematika

BAB III
GEOMETRI

1. TRIGONOMETRI
Trigonomet ri pada bidang geomet ri ini merupakan alat bant u unt uk menyelesaikan suat u persoalan.
Rumus-rumus t rigonomet ri yang perlu diingat adalah :
sin2x + cos2x = 1
t an2x + 1 = sec2x

sec x =
cot 2x + 1 = cosec2x
1

csc x =
cos x
1

cot x =
sin x
1

sin (90o − x) = cos x


tan x

cos (90o − x) = sin x


t an (90o − x) = cot x
sin (90o + x) = cos x
cos (90o + x) = −sin x
cos (180o + x) = − cos x
t an(180o + x) = t an x
sin (x ± y) = sin x cos y ± cos x sin y
cos (x ± y) = cos x cos y ∓ sin x sin y
t an (x ± y) =
tan x ± tan y
1m tan x tan y

cos 2x = cos2x − sin2x


sin 2x = 2 sin x cos x

cos 2x = 2 cos2x − 1
cos 2x = 1 − 2 sin2x

tan 2 x =
2 tan x
1 − tan 2 x
sin 12 x = ± 1−cos x

cos 12 x = ±
2

1+ cos x

tan 12 x = ± = ± 1−sincosx x = ± 1+sincosx x


2

( )cos( )
1− cos x
1+ cos x

sin x + sin y = 2 sin


sin x − sin y = 2 cos( )sin ( )
x+ y x− y

cos x + cos y = 2 cos ( )cos ( )


2 2
x+ y x− y

cos x − cos y = −2 sin ( )sin ( )


2 2
x+ y x− y
2 2
x+ y x− y

2 sin x cos y = sin (x + y) + sin (x − y)


2 2

2 cos x sin y = sin (x + y) − sin (x − y)


2 cos x cos y = cos (x + y) + cos (x − y)
2 sin x sin y = −(cos (x + y) − cos (x − y))

Eddy Hermanto, ST 87 Geometri


Pembinaan Olimpiade Matematika

Nilai dari sin 15o adalah ⋅⋅⋅⋅⋅⋅


Cont oh 1 :

sin (x − y) = sin x cos y − cos x sin y


Solusi :

sin (45o − 30o) = sin 45o cos 30o − cos 45o sin 30o
2 ⋅ 12 3 − 12 2 ⋅ 12
( )
sin 15o = 1

6− 2
( )
2

sin 15o = 1

6− 2
4

Jadi, sin 15o = 1


4

Cont oh 2 :
Tent ukan nilai dari cos 105o.

cos (x + y) = cos x cos y − sin x sin y


Solusi :

cos (60o + 45o) = cos 60o cos 45o − sin 60o sin 45o
⋅ 12 2 − 12 3 ⋅ 12 2
( )
cos 105o = 1

2− 6
( )
2

cos 105o = 1

2− 6
4

Jadi, cos 105o = 1


4

Cont oh 3 :
Tent ukan nilai dari cos 15o sin 75o

Solusi :
cos 15o sin 75o = 1
(sin (15 + 75) o − sin (15 − 75)o)
(sin 90o − sin (−60o))
2

cos 15o sin 75o = 1


2

cos 15o sin 75o = 1


2 (sin 90o + sin 60o)
cos 15o sin 75o = 1
4 (2 + 3)
Jadi, cos 15o sin 75o = 1
4 (2 + 3)

Cont oh 4 :
Jika t an 4o = p maka t ent ukan nilai dari t an 49o dinyat akan dalam p.

Solusi :
t an 49o = t an (45o + 4o)
45° + tan 4°
− tan 45° tan 4°
t an 49o = 1tan
Karena t an 45o = 1 maka
1+ p
t an 49o = 1− p
1+ p
Jadi, t an 49o = 1− p

Eddy Hermanto, ST 88 Geometri


Pembinaan Olimpiade Matematika

Cont oh 5 :
Jika sin A sin B = 28
125 dan t an A t an B = 7
18 , maka nilai cos (A − B) = ⋅⋅⋅⋅⋅

Solusi :
sin A sin B
t an A t an B =

cos A cos B
sin A sin B 28 18 72
cos A cos B = = =
cos (A − B) = cos A cos B + sin A sin B
tan A tan B 125 7 125

cos (A − B) = 125
72 28
+ 125
cos (A − B) = 4
5

Diberikan segit iga ABC yang siku-siku di C. Jika cos (A + C) = k, maka sin A + cos B = ⋅⋅⋅⋅
Cont oh 6 :

Solusi :
Karena siku-siku di C maka A + B = 90o.
sin A = sin (90o − B) = cos B
cos A = cos (90o − B) = sin B
k = cos (A + C) = cos (A + 90o) = −sin A
sin A + cos B = sin A + sin A = −2k
Jadi, sin A + cos B = −2k

Cont oh 7 :
Diket ahui t an x = 4
3 dan t an y = 5
12 dengan x dan y adalah sudut lancip. Nilai t an (2x + y) = ⋅⋅⋅⋅

Solusi :

1− tan 2 x
2 tan x
t an 2x =

= − 24
8
3
t an 2x = 1− 169 7

tan 2 x + tan y
1− tan 2 x⋅tan y = (− 253 7
t an (2x + y) = 84 )( 17 )

t an (2x + y) = − 204
253

Cont oh 8 :
Hit unglah t an 10o t an 50o t an 70o t anpa menggunakan kalkulat or.

sin 10° (sin 50° sin 70° )


Solusi :
t an 10o t an 50o t an 70o = cos 10° (cos 50° cos 70° )
sin 10° ( 2 cos 20° +1)
t an 10o t an 50o t an 70o = cos 10° ( 2 cos 20° −1)
2 sin 10° cos 20° + sin 10°
t an 10o t an 50o t an 70o = 2 cos 10° cos 20° − cos 10°
sin 30° −sin 10°°+ sin 10°
t an 10o t an 50o t an 70o = cos 30° + cos 10° − cos 10°
3
t an 10o t an 50o t an 70o = t an 30o = 3
3
Jadi, t an 10o t an 50o t an 70o = 3

Eddy Hermanto, ST 89 Geometri


Pembinaan Olimpiade Matematika

LATIHAN 1 :

1. Jika A + B = 45o dan cos A sin B = 1


6 2 maka cos (B − A) = ⋅⋅⋅⋅⋅

2. Diket ahui bahwa A, B dan C adalah sudut -sudut dalam segit iga ABC sert a berlaku bahwa j umlah sudut -
sudut dalam suat u segit iga sama dengan 180o. Maka bukt ikan bahwa pada segit iga ABC t ersebut
berlaku t an A + t an B + t an C = t an A ⋅ t an B ⋅ t an C.

3. α, β dan γ adalah besar sudut -sudut suat u segit iga. Jika cot α = −3 dan cot β = 1, maka cot γ = ⋅⋅

(a) sin 3x = 3 sin x − 4 sin3x


4. Bukt ikan bahwa

(b) cos 3x = 4 cos3x − 3 cos x

5. cos 75o + cos 15o = ⋅⋅⋅⋅⋅

6. Bukt ikan bahwa tan 2 x + cos2 x


sin x + sec x = sec x − sin x.

cos 3 x −sin 6 x − cos 9 x


7. Bukt ikan bahwa sin 9 x − cos 6 x −sin 3 x = t an 6x.

8. Jika cos A + cos B = cos C, bukt ikan bahwa cos 3A + cos 3B − cos 3C = −12 cos A cos B cos C.

9. Jika A + B + C = 180o, bukt ikan bahwa t an 12 A t an 12 B + t an 12 A t an 12 C + t an 12 B t an 12 C = 1.

10. (AHSME 1999) Misalkan x ∈ R yang memenuhi sec x − t an x = 2. Nilai dari sec x + t an x adalah ⋅⋅⋅⋅

11. (OSP 2009/ AIME 1986) Jika t an x + t an y = 25 dan cot x + cot y = 30, maka nilai t an (x + y) adalah ⋅⋅⋅⋅⋅⋅

12. (AIME 1995) Jika (1 + sin t )(1 + cos t ) = 5


4 maka nilai dari (1 − sin t )(1 − cos t ) adalah ⋅⋅⋅⋅⋅⋅

13. Tent ukan nilai eksak dari t an 1o ⋅ t an 2o ⋅ t an 3o ⋅ ⋅⋅⋅ ⋅ t an 89o.

14. (OSK 2005) Nilai sin875o − cos875o = ⋅⋅⋅⋅⋅⋅

15. (OSK 2011 Tipe 1) Misalkan A dan B adalah sudut -sudut lancip yang memenuhi
t an (A + B) = 12 dan t an (A − B) = 13 maka besar sudut A adalah ....

16. (OSK 2008) Diket ahui bahwa a dan b adalah besar dua sudut pada sebuah segit iga. Jika sin a + sin b =
1
2 6 dan cos a + cos b = 1
2 2 , maka sin (a + b) = ⋅⋅⋅⋅⋅⋅⋅⋅⋅⋅

(1 − tan )(1 − tan )L (1 − tan ) = 2


17. (OSP 2011) Jika
2 x 2 x 2 x 2011 x
3 tan 2 2011
maka sin 2x = ⋅⋅⋅⋅⋅⋅
2 2011 2 2010 2

18. Nilai dari cos π


7 ⋅ cos 2π
7 ⋅ cos 4π
7 adalah ⋅⋅⋅⋅⋅

Eddy Hermanto, ST 90 Geometri


Pembinaan Olimpiade Matematika

cos 96° + sin 96°


19. (AIME 1996) Tent ukan nilai n bulat posit if t erkecil yang memenuhi t an 19no = cos 96° −sin 96° .

cosec 10o + cosec 50o − cosec 70o


20. Hit unglah t anpa menggunakan kalkulat or

21. Hit unglah nilai dari sin26o + sin242o + sin266o + sin278o.

22. (OSP 2009) Jika x1, x2, ⋅⋅⋅, x2009 bilangan real, maka nilai t erkecil dari
cos x1 sin x2 + cos x2 sin x3 + ⋅⋅⋅ + cos x2009 sin x1
adalah ⋅⋅⋅⋅⋅⋅⋅⋅⋅⋅

23. Tent ukan nilai eksak dari sin 18o.

Eddy Hermanto, ST 91 Geometri


Pembinaan Olimpiade Matematika

2. GARIS
Garis memiliki panj ang t ak t erhingga sedangkan ruas garis dibat asi oleh dua buah t it ik sebagai uj ung-
uj ung koordinat .

A. Jarak 2 titik
Misalkan AB adalah suat u ruas garis dengan koordinat A(xA, yA) dan B(xB, yB).

Jarak t it ik A dan t it ik B dapat didefinisikan dengan panj ang ruas garis lurus yang menghubungkan t it ik

panj ang ruas garis t ersebut pada arah sumbu X sama dengan xB − xA panj ang ruas garis t ersebut pada
A dengan t it ik B. Jika ruas garis t ersebut diproyeksikan ke sumbu X dan sumbu Y akan didapat bahwa

arah sumbu Y sama dengan yB − yA. Karena sumbu X dan sumbu Y t egak lurus maka dengan dalil

( x B − x A )2 + ( y B − y A )2 .
pit agoras akan didapat
Panj ang ruas AB =

Lingkaran C1 memiliki pusat di (17, 8) sedangkan lingkaran C2 memiliki pusat di (10, −16). Maka j arak
Cont oh 9 :

ant arpusat kedua lingkaran t ersebut adalah ⋅⋅⋅⋅⋅

( )2 ( )2
Solusi :
Jarak ant arpusat = 17 − 10 + 8 + 16 = 25.
Jadi, j arak ant arpusat kedua lingkaran t ersebut sama dengan 25.

B. Kemiringan (Gradien) Garis


Kemiringan garis disebut dengan dengan gradien yang dinot asikan dengan m.
Sebelumnya t elah dij elaskan t ent ang perhit ungan proyeksi sebuah ruas garis t erhadap sumbu X dan Y.
Kemiringan garis dapat didefinisikan dengan perbandingan proyeksi ruas garis t erhadap sumbu Y
dengan proyeksi ruas garis t ersebut t erhadap sumbu X.
yB − y A
xB − x A
Jadi, kemiringan garis, m =
Dengan memperhat ikan bahwa proyeksi ruas garis t erhadap sumbu X dan sumbu Y saling t egak lurus
sert a j uga memperhat ikan pengert ian t angen suat u sudut maka didapat
t an α =
yB − y A
xB − x A
=m
dengan α adalah sudut garis t erhadap sumbu X posit if. Pengukuran dimulai dari sumbu X posit if
berlawanan arah j arum j am ke garis dimaksud.

(OSK 2003) Suat u garis melalui t it ik (m, −9) dan (7, m) dengan kemiringan m. Berapakah nilai m ?
Cont oh 10 :

Eddy Hermanto, ST 92 Geometri


Pembinaan Olimpiade Matematika

Solusi :
yB − y A
xB − x A
Gradien =
m − ( −9 )
7−m
m=
m + 9 = 7m − m2
m2 − 6m + 9 = 0
(m − 3)2 = 0
Jadi, m = 3

C. Persamaan garis
Persamaan dari suat u garis lurus dapat dit ent ukan j ika diket ahui sedikit nya sat u dari 2 kemungkinan
berikut :
a. Jika diket ahui 2 t it ik sebarang yang t erlet ak pada garis
Misalkan diket ahui 2 t it ik dengan koordinat A(x1, y1) dan B(x2, y2). Persamaan garis t ersebut dapat
dit ent ukan dengan menganggap t erdapat sebuah t it ik P(x, y) yang j uga t erlet ak pada garis

=
t ersebut . Karena berada pada sat u garis lurus maka gradien ruas AP dan AB akan sama. Maka
y − y1 y 2 − y1
x − x1 x2 − x1
Dari persamaan t ersebut akan dapat dit ent ukan persamaan garis.

b. Jika diket ahui kemiringan garis t ersebut sert a t it ik yang t erlet ak pada garis t ersebut
Misalkan sebuah garis lurus memiliki kemiringan m sert a sebuah t it ik dengan koordinat A(x1, y1)
t erlet ak pada garis t ersebut . Persamaan garis t ersebut dapat dit ent ukan dengan menganggap
t erdapat sebuah t it ik P(x, y) yang j uga t erlet ak pada garis t ersebut . Berdasarkan pengert ian
gradien maka
y − y1
x − x1
=m
y = m(x − x1) + y1
Persamaan di at as set ara dengan y = mx + c dengan c = y1 − mx1. Persamaan y = mx + c dikenal
j uga dengan persamaan umum garis lurus.

Sebuah garis lurus melalui t it ik (2, 4) dan (6, 7). Persamaan garis t ersebut adalah ⋅⋅⋅⋅
Cont oh 11 :

Solusi :
y −4 7−4
x−2
= 6−2
4(y − 4) = 3(x − 2)
4y − 16 = 3x − 6
4y = 3x + 10
Jadi, persamaan garis t ersebut adalah 4y = 3x + 10.

Sebuah garis melalui t it ik (2, −4) sert a sudut garis t ersebut t erhadap sumbu X posit if adalah 45o.
Cont oh 12 :

Persamaan garis t ersebut adalah ⋅⋅⋅⋅

Solusi :
Gradien garis = m = t an 45o = 1.
y + 4 = m(x − 2)
y=x−6
Jadi, persamaan garis t ersebut adalah y = x − 6.

Eddy Hermanto, ST 93 Geometri


Pembinaan Olimpiade Matematika

D. Hubungan Gradien 2 Garis


Pada bidang (2 dimensi), hubungan 2 buah garis lurus dapat t erj adi dalam 2 kemungkinan : sej aj ar
(t ermasuk berhimpit ) dan berpot ongan.

a. Dua buah garis sej aj ar


Dua buah garis dikat akan sej aj ar j ika t idak memiliki t it ik persekut uan (sej aj ar t idak berhimpit )
at au memiliki t ak berhingga t it ik persekut uan (berhimpit ).

Dua buah garis dikat akan sej aj ar adalah apabila memiliki kemiringan yang sama.
Jadi, j ika garis pert ama dengan gradien m1 dan garis kedua dengan gradien m2 sej aj ar maka
berlaku
m1 = m2

b. Dua buah garis berpot ongan


Dua buah garis lurus dikat akan berpot ongan j ika t erdapat sat u buah t it ik persekut uan.

Misalkan sebuah garis memiliki sudut α t erhadap sumbu X posit if.


Gambar di at as menunj ukkan bahwa garis l1 dan l2 berpot ongan.

Maka kemiringan garis t ersebut adalah m1 = t an θ.

kedua t ersebut t erhadap sumbu X posit if adalah 90o + θ.


Jika sebuah garis memot ong t egak lurus garis t ersebut maka t ent unya sudut yang dibent uk garis

Maka kemiringan garis kedua t ersebut adalah m2 = t an (90o + θ).


m2 = t an (90o + θ)
m2 = −cot θ = − tan1 α = − m1
1

Eddy Hermanto, ST 94 Geometri


Pembinaan Olimpiade Matematika

m1m2 = −1
Jadi, syarat dua buah garis berpot ongan t egak lurus adalah

Jika dua buah garis berpot ongan t idak t egak lurus akan dibahas pada bagian sudut ant ara dua
buah garis.

(OSK 2006) Sebuah garis l1 mempunyai kemiringan −2 dan melalui t it ik (p, −3). Sebuah garis lainnya
Cont oh 13 :

l2, t egaklurus t erhadap l1 di t it ik (a, b) dan melalui t it ik (6, p). Bila dinyat akan dalam p, maka a =

Persamaan garis l1 adalah y + 3 = −2(x − p)


Solusi :

1
Karena l2 t egak lurus l1 maka gradien garis l2 adalah .
Persamaan garis l2 adalah y − p = 12 (x − 6)
2

Kedua garis berpot ongan di t it ik (a, b) maka :


b + 3 = −2(a − p) dan b − p = 12 (a − 6)
Eliminasi b pada kedua persamaan didapat 3 + p = −2(a − p) − 1
(a − 6)
6 + 2p = −4a + 4p − a + 6
2

Jadi, a = 52 p

Cont oh 14 :

Nilai a sama dengan ⋅⋅⋅⋅


Ket iga garis lurus ax + 2y + 3 = 0 ; x + y + 1 = 0 dan 2x + 3y + 4 = 0 melalui sebuah t it ik yang sama.

Solusi :

Subt it usikan yA = −1 − xA ke persamaan 2xA + 3yA + 4 = 0 didapat


Misalkan t it ik pot ong garis x + y + 1 = 0 dan 2x + 3y + 4 = 0 adalah t it ik A(xA, yA).

2xA + 3(−1 − xA) + 4 = 0 sehingga xA = 1 dan yA = −1 − 1 = −2.


Garis ax + 2y + 3 = 0 melalui t it ik (1, −2) maka
a(1) + 2(−2) + 3 = 0
a=1
Jadi, nilai a sama dengan 1.

E. Jarak Titik ke Garis


Jarak ant ara t it ik dengan garis l adalah panj ang t erpendek ant ara t it ik t ersebut dengan t it ik –t it ik
yang t erlet ak pada garis l t ersebut . Panj ang t erpendek didapat j ika garis yang melalui t it ik t ersebut
dengan t it ik acuan yang t erlet ak pada garis l t egak lurus dengan garis l.

Misalkan t erdapat t it ik A(xo, yo) dan sebuah garis ax + by + c = 0 dan akan dicari j arak t it ik A dengan
garis t ersebut dengan j arak t ersebut sama dengan d.

Eddy Hermanto, ST 95 Geometri


Pembinaan Olimpiade Matematika
Buat dua garis bert urut -t urut sej aj ar sumbu X dan sumbu Y sert a memot ong garis ax + by + c = 0 di
t it ik C dan B.
− axo −c
Karena koordinat A(xo, yo) dan persamaan garis ax + by + c = 0 maka koordinat B(xo, b ) dan
− byo −c

( ) =
koordinat C( a , yo).

Panj ang ⏐AB⏐ =


− axo − c
− yo axo + byo + c

(x ) =
2
b b

Panj ang ⏐AC⏐ = − −byao −c axo + byo + c

(x ) +( )
2
o a

− −byao −c − yo
axo + byo + c a 2 +b 2
Panj ang ⏐BC⏐ =
2 − axo − c 2
o b = ab

⏐AB⏐ ⋅ ⏐AC⏐ = ⏐BC⏐ ⋅ d


Dengan luas segit iga mapun perbandingan segit iga akan didapat

axo + by o + c
a 2 +b 2
d=

axo + by o + c
a 2 +b 2
Jadi, j arak t it ik A(xo, yo) ke garis ax + by + c = 0 adalah d = .

Sebuah garis lurus dengan persamaan 4y = 3x + 7 menyinggung lingkaran yang berpusat di (2, −3).
Cont oh 15 :

Tent ukan j ari-j ari lingkaran t ersebut .

Solusi :

Garis 4y = 3x + 7 set ara dengan 3x − 4y + 7 = 0


Persoalan ini sama aj a dengan menent ukan j arak pusat lingkaran ke garis 4y = 3x + 5.

3( 2 )− 4 ( − 3 )+ 7
Jadi, j arak t it ik (2, −3) ke garis 3x − 4y + 7 = 0 adalah d =
32 + 4 2
= 5.
Jadi, j ari-j ari lingkaran t ersebut sama dengan 5.

Misalkan sudut ant ara dua garis berpot ongan adalah θ, sudut garis pert ama t erhadap sumbu X posit if
F. Sudut Antara Dua Garis

adalah α dan sudut garis kedua t erhadap sumbu X posit if adalah β maka

θ=α−β
tan α − tan β
t an θ = t an (α − β) = 1+ tan α tan β
Pada penj elasan sebelumnya t elah disebut kan bahwa gradien suat u garis sama dengan t angen sudut
garis t ersebut t erhadap sumbu X posit if. Maka
t an θ =
m1 − m2
1+ m1m2

Jika diket ahui kemiringan dari kedua garis t ersebut maka t ent unya θ dapat dihit ung.
Pada kasus khusus misalkan dua buah garis saling t egak lurus, maka θ = 90o sehingga didapat syarat
yang memenuhi adalah 1 + m1m2 = 0.

Eddy Hermanto, ST 96 Geometri


Pembinaan Olimpiade Matematika

m1m2 = −1
Jadi, syarat dua buah garis t egak lurus adalah

Sudut t erkecil yang dibent uk garis y = −2x + 7 dan garis y = 3x + 2 adalah ⋅⋅⋅⋅⋅
Cont oh 16 :

Solusi :

Garis y = −2x + 7 memiliki gradien, m2 = −2


Garis y = 3x + 2 memiliki gradien, m1 = 3

Misalkan sudut ant ara kedua garis sama dengan θ maka


3−( −2 )
t an θ = = −1
m1 − m2
1+ m1m2 = 1+ (3 )( − 2 )
θ = 135 sehingga sudut t erkecil yang dibent uk kedua garis adalah 45o.
o

Jadi, sudut yang dibent uk garis y = −2x + 7 dan garis y = 3x + 2 adalah 45o.

G. Sudut antara Beberapa Garis


Bagaimana hubungan sudut -sudut di ant ara beberapa garis ? Diberikan dua buah garis sej aj ar sert a
sebuah garis yang memot ong kedua garis t ersebut .

Dua sudut berpelurus sama dengan 180o


Sebagai cont oh ∠A2 + ∠A3 = 180o
(i)

Sebagai cont oh ∠A1 = ∠A3


(ii) Dua sudut bert olak belakang sama besar

Sebagai cont oh ∠A1 = ∠B1


(iii) Dua sudut yang sehadapan sama besar

Sebagai cont oh ∠A2 = ∠B4


(iv) Dua sudut dalam berseberangan selalu sama besar

Sebagai cont oh ∠A1 = ∠B3


(v) Dua sudut luar berseberangan selalu sama besar

(vi) Dua sudut dalam sepihak j umlah sudut nya 180o


Sebagai cont oh ∠A2 + ∠B1 = 180o
(vii) Dua sudut dalam sepihak j umlah sudut nya 180o
Sebagai cont oh ∠A1 + ∠B2 = 180o

Cont oh 17 :
(Alabama MC 2002) Ruas AB sej aj ar dengan ruas CD. Tit ik F t erlet ak pada ruas AB dan t it ik G t erlet ak

dalam suat u segit iga sama dengan 180o. Jika ∠EFB = 132o dan ∠EGD = 112o maka besar ∠FEG sama
pada ruas CD sedangkan t it ik E t erlet ak di ant ara ruas AB dan CD. Diket ahui bahwa j umlah sudut

dengan ⋅⋅⋅⋅⋅⋅

Eddy Hermanto, ST 97 Geometri


Pembinaan Olimpiade Matematika

Solusi :

Misalkan ∠FEG = x.

∠EFA berpelurus dengan ∠EFB sehingga ∠EFA = 180o − 132o = 48o.


Perpanj ang garis FE sehingga memot ong ruas CD di H.

Karena AB sej aj ar CD maka ∠GHE = ∠EFA = 48o.


∠HGE berpelurus dengan ∠EGD sehingga ∠HGE = 180o − 112o = 68o.
∠GEH berpelurus dengan ∠FEG sehingga ∠GEH = 180o − x
Jumlah sudut dalam ∆EGH = 180o maka
∠GHE + ∠HGE + ∠GEH = 180o
(48o) + (68o) + (180o − x) = 180o
x = 116o
Jadi, besar ∠FEG sama dengan 116o.

H. Pembagian Ruas Garis dalam Perbandingan Bagian


Misalkan t erdapat ruas garis AB dengan koordinat A(xA, yA) dan B(xB, yB) dan sebuah t it ik P(xP, yP) yang
t erlet ak pada ruas AB sehingga AP : PB = m : n.

Perhat ikan bahwa ruas AP dan AB memiliki kemiringan yang sama sehingga j ika keduanya
diproyeksikan ke sumbu X dan sumbu Y maka perbandingan panj ang proyeksi kedua ruas t ersebut

Jadi, (xP − xA) : (xB − xP) = m : n


akan t et ap sama dengan semula yait u m : n.

nxP − nxA = mxB − mxP


nx A + mxB
xP = m+n
ny + my
Dengan cara yang sama didapat yP = Am + n B .
Jadi, j ika t erdapat ruas garis AB dengan koordinat A(xA, yA) dan B(xB, yB) dan sebuah t it ik P(xP, yP)
nx A + mxB ny A + my B
yang t erlet ak pada ruas AB sehingga AP : PB = m : n maka koordinat P( m+n , m+n ).

Cont oh 18 :
Tit ik A(2, 4) dan t it ik B(−4, 12) adalah uj ung-uj ung diamet er suat u lingkaran. Tent ukan persamaan
lingkaran t ersebut

Eddy Hermanto, ST 98 Geometri


Pembinaan Olimpiade Matematika

( )2
Solusi :
Panj ang AB = 2 + 4 + (4 − 12) = 10
2

Maka j ari-j ari lingkaran t ersebut , r = 5


Karena AB adalah diamet er lingkaran maka pusat lingkaran t ersebut t erlet ak di t engah-t engah AB.
Koordinat pusat lingkaran P( 2−2 4 , 4+212 ) = P(−1, 8)
Maka persamaan lingkaran t ersebut adalah (x + 1) 2 + (y − 8) 2 = 25

LATIHAN 2 :

1. Persamaan garis melalui (4,5) dan sej aj ar dengan garis y + 2x = 4 adalah ⋅⋅⋅⋅⋅⋅

2. Garis l melalui t it ik pot ong garis x + y + 1 = 0 dan 3x + 2y − 1 = 0 sert a t egak lurus garis yang
menghubungkan t it ik (8, 5) dan (−4, 7). Persamaan garis l adalah ⋅⋅⋅⋅⋅

Persamaan garis melalui R dan bersudut 45o t erhadap sumbu X posit if adalah ⋅⋅⋅⋅⋅⋅
3. Dit ent ukan t it ik P(−3,−2) dan t it ik Q(7,3). Tit ik R t erlet ak pada garis PQ sehingga PR : RQ = 3 : 2.

4. Jika garis g ≡ ax + 2y = 8, garis x − 5y = 10 dan garis 3x + 7y = 8 melalui sat u t it ik yang sama, maka
garis g akan memot ong sumbu X dengan absis sama dengan ⋅⋅⋅⋅

5. Jika j arak t it ik P(3,6) ke garis 12x + 5y − 40 = 0 sama dengan j arak t it ik P ke t it ik Q(a, 4), maka nilai a
yang memenuhi adalah ⋅⋅⋅⋅⋅⋅

6. Sebuah persegi (buj ur sangkar) dengan sepasang sisinya t erlet ak pada garis 5x − 12y − 65 = 0 dan
garis 5x − 12y + 26 = 0. Luas buj ur sangkar t ersebut adalah ⋅⋅⋅⋅

7. Garis berat adalah garis yang dit arik dari salah sat u t it ik sudut memot ong pert engahan sisi di

segit iga t ersebut yang dit arik dari t it ik C adalah ⋅⋅⋅⋅⋅⋅


hadapannya. ABC adalah sebuah segit iga dengan A(2, 0); B(4, 0) dan C(7, 5). Persamaan garis berat

sehingga PQ t egak lurus t erhadap garis y = x dan PQ = 2. Maka koordinat Q adalah ⋅⋅⋅⋅⋅
8. (OSP 2007) Tit ik P t erlet ak di kuadran I pada garis y = x. Tit ik Q t erlet ak pada garis y = 2x demikian

9. (OSK 2008) Tit ik A dan B t erlet ak pada parabola y = 4 + x − x2. Jika t it ik asal O merupakan t it ik t engah
ruas garis AB, maka panj ang AB adalah ⋅⋅⋅⋅⋅

Eddy Hermanto, ST 99 Geometri


Pembinaan Olimpiade Matematika

3. SEGITIGA
Segit iga dibent uk dari t iga buah garis lurus dengan t idak ada garis yang sej aj ar.
Jumlah ket iga sudut dalam segit iga sama dengan 180o.

Segit iga Lancip Segit iga Tumpul Segit iga Siku-siku

Jika salah sat u sudut segit iga ada yang lebih dari 90o maka disebut segit iga t umpul sedangkan j ika t idak
ada sat upun sudut yang lebih dari 90o maka disebut segit iga lancip. Segit iga dikat akan siku-siku j ika salah
sat u sudut nya sama dengan 90o.

Selain nama-nama t ersebut ada j uga beberapa segit iga yang perlu unt uk dikenal berkait an dengan
panj ang sisinya.
a. Segit iga sama sisi.
Sesuai dengan namanya maka sisi-sisi segit iga sama panj ang. Selain it u, ket iga sudut segit iga t ersebut
j uga sama besar yait u 60o.
b. Segit iga sama kaki.
Misalkan ∆ABC dengan sisi-sisinya a, b dan c. Segit iga ABC dikat akan sama kaki j ika t erdapat sepasang
sisi misalkan a dan b sehingga a = b. Akibat dari a = b maka ∠A = ∠B.

Hal yang pent ing j uga adalah misalkan ∆ABC dengan a = b maka sebuah garis dari t it ik sudut C akan
memot ong t egak lurus pert engahan sisi c = AB.

Cont oh 19 :

BX = BZ dan CX = CY dengan besar sudut XZY = 40o dan ∠ZYX = 75o. Besar sudut A adalah ⋅⋅⋅⋅
Pada segit iga ABC t it ik-t it ik X, Y, Z masing-masing t erlet ak pada sisi BC, AC dan AB sehingga AY = AZ,

Misal ∠AYZ = α maka ∠AZY = α


Solusi :

Misal ∠BXZ = β maka ∠BZX = β


Misal ∠CXY = γ maka ∠CYX = γ
∠YXZ = 180o − 40o − 75o = 65o
α + γ = (180 − 75)o = 105o ⋅⋅⋅⋅⋅⋅⋅⋅⋅⋅⋅⋅⋅⋅⋅⋅⋅⋅⋅⋅⋅⋅⋅ (1)
α + β = (180 − 40) o = 140o ⋅⋅⋅⋅⋅⋅⋅⋅⋅⋅⋅⋅⋅⋅⋅⋅⋅⋅⋅⋅⋅⋅⋅ (2)
β + γ = (180 − 65)o = 115o ⋅⋅⋅⋅⋅⋅⋅⋅⋅⋅⋅⋅⋅⋅⋅⋅⋅⋅⋅⋅⋅⋅⋅ (3)
Dari ket iga persamaan di at as didapat α = 65o
∠A = (180 − 65 − 65)o = 50o
Jadi, besar sudut A adalah 50o.

Eddy Hermanto, ST 100 Geometri


Pembinaan Olimpiade Matematika

A. Dalil Cosinus dan Sinus


Pada set iap segit iga sebarang selalu berlaku dalil cosinus. Misalkan segit iga ABC memiliki sisi-sisi yang

a2 = b2 + c2 − 2bc cos A
panj angnya a, b, c dengan sudut di hadapannya secara berurut an adalah A, B, C, maka berlaku :

b2 = a2 + c2 − 2ac cos B
c2 = a2 + b2 − 2ab cos C
Jika salah sat u sudut segit iga t ersebut siku-siku misalkan di A maka
a2 = b2 + c2
yang dikenal dengan dalil pit agoras.

• Jika a2 > b2 + c2 dengan a adalah sisi t erpanj ang maka segit iga t ersebut adalah segit iga t umpul
Dari dalil cosinus t ersebut akan didapat

• Jika a2 < b2 + c2 dengan a adalah sisi t erpanj ang maka segit iga t ersebut adalah segit iga lancip

Pada segit iga ABC t ersebut j uga berlaku dalil sinus

= = = 2R
a b c
sin A sin B sin C
dengan R adalah j ari-j ari lingkaran luar ∆ABC.

Dari dalil sinus j uga didapat bahwa sisi di hadapan sudut yang t erbesar merupakan sisi t erpanj ang.

Cont oh 20 :
Pada segit iga ABC diket ahui panj ang AC = 5, AB = 6 dan BC = 7. Dari t it ik C dibuat garis t egak lurus sisi
AB memot ong sisi AB di t it ik D. Tent ukan panj ang CD.

Solusi :

Misalkan panj ang AD = x sehingga BD = 6 − x


Alt ernat if 1 :

CD2 = AC2 − AD2 = BC2 − BD2


52 − x2 = 72 − (6 − x)2
24 = 36 − 12x + x2 − x2 sehingga x = 1
CD2 = 52 − 12
CD = 2 6

Alt ernat if 2 :
s = 12 (5 + 6 + 7) = 9

Luas ∆ABC = s ( s − a)( s − b)( s − c)


Luas ∆ABC = 9(9 − 5)(9 − 6)(9 − 7) = 6 6
Luas ∆ABC = 1
2 ⋅ AB ⋅ CD = 3CD
3 ⋅ CD = 6 6
Jadi, panj ang CD = 2 6 .

Eddy Hermanto, ST 101 Geometri


Pembinaan Olimpiade Matematika

Cont oh 21 :
(OSK 2002) Pada suat u segit iga ABC, sudut C t iga kali besar sudut A dan sudut B dua kali besar sudut
A. Berapakah perbandingan (rasio) ant ara panj ang AB dengan BC ?

∠C = 3∠A dan ∠B = 2∠A


Solusi :

Karena ∠A + ∠B + ∠C = 180o maka


∠A + 2∠A + 3∠A = 180o sehingga ∠A = 30o
∠C = 3∠A = 90o
sin ∠C = sin ∠A
AB BC

sin 90°
sin 30°
AB
Jadi, BC = = 2.

LATIHAN 3.A

sehingga AP = PC = CB, maka besarnya sudut A adalah ⋅⋅⋅⋅⋅⋅


1. (OSK 2010) Diberikan segit iga ABC, AB = AC. Jika t it ik P diant ara A dan B sedemikian rupa

2. (NAHC 1995-1996 First Round) Pada segit iga siku-siku diket ahui panj ang sisi-sisinya adalah a, a + b
dan a + 9b unt uk suat u bilangan posit if a dan b. Tent ukan nilai dari ba .

3. (OSK 2003) Segit iga ABC adalah segit iga sama sisi dengan panj ang sisi 1 sat uan. Melalui B dibuat
garis yang t egak lurus BC. Garis t ersebut berpot ongan dengan perpanj angan garis AC di t it ik D.
Berapakah panj ang BD?

(a + b + c)(a + b − c) = 3ab, maka besar sudut yang menghadapi sisi dengan panj ang c adalah ⋅⋅⋅⋅⋅⋅⋅⋅
4. (OSP 2010) Jika a, b, dan c menyat akan panj ang sisi-sisi suat u segit iga yang memenuhi

5. (OSP 2010) Pada segit iga ABC dimisalkan a, b, dan c bert urut -t urut merupakan panj ang sisi BC,

=
CA, dan AB. Jika
2a b
tan A tan B
sin 2 A − sin 2 B
adalah ⋅⋅⋅⋅⋅⋅
cos 2 A + cos 2 B
Maka nilai

6. (OSP 2011) Diberikan persegi panj ang (siku empat ) ABCD dengan AB = a dan BC = b. Tit ik O adalah
perpot ongan ant ara kedua diagonalnya. Perpanj ang sisi BA sehingga sehingga AE = AO, j uga
perpanj ang diagonal DB sehingga BZ = BO. Asumsikan segit iga EZC samasisi. Bukt ikan bahwa
i. b = a 3
ii. EO t egak lurus ZD

7. (AIME 1983) Tit ik A dan C t erlet ak pada lingkaran yang berpusat di O dan berj ari-j ari 50 . Tit ik
B t erlet ak di dalam lingkaran yang memenuhi ∠ABC = 90o, AB = 6 dan BC = 2. Tent ukan panj ang
OB.

8. (AIME 1983/ Hongkong PSC 1988) Dua lingkaran yang masing-masing berj ari-j ari 8 dan 6
mempunyai j arak ant ar pusat 12. Melalui t it ik P yang merupakan salah sat u t it ik perpot ongan
kedua lingkaran dibuat t ali busur PQ dan PR dengan Q, P, R segaris. Jika PQ = PR, t ent ukan PQ2.

Eddy Hermanto, ST 102 Geometri


Pembinaan Olimpiade Matematika

bilangan bulat berurut an sert a ∠C = 2∠A.


9. (ME V7N1) Tent ukan semua kemungkinan sisi-sisi segit iga ABC dengan sisi-sisinya membent uk 3

AB = AC dan AD = DC. Pada ∆ABC besar ∠BAC = 20o sedangkan pada ∆ADC berlaku ∠ADC = 100o.
10. (Flanders MO 1996 Final Round) Misalkan ABC dan DAC adalah dua buah segit iga sama kaki dengan

Bukt ikan bahwa AB = BC + CD.

Eddy Hermanto, ST 103 Geometri


Pembinaan Olimpiade Matematika

B. Kesebangunan Segitiga
Dua buah segit iga dikat akan sebangun apabila sisi-sisinya memiliki perbandingan yang sama sedangkan
segit iga yang memiliki sisi-sisi yang sama dikat akan kongruen (sama dan sebangun).

Ket iga sudut nya sama. Dengan kat a lain ∠A = ∠D, ∠B = ∠E dan ∠C = ∠F. Jika diperhat ikan
Dua buah segit iga ABC dan DEF dikat akan sebangun j ika memenuhi salah sat u syarat berikut :
(i)
syarat sebenarnya hanyalah dua buah sudut nya sama sebab sudut ket iga akan sama j ika dua

= DF = BC
sudut lainnya sama.
AB AC
(ii) Sisi-sisinya memiliki perbandingan yang sama, DE EF .
(iii) Dua sisi memiliki perbandingan yang sama sert a sudut yang mengapit kedua sisi t ersebut j uga

DE = DF dan ∠A = ∠D.
sama.
AB AC

Pada segit iga ABC dan segit iga DEF berlaku ∠BAC = ∠EDF dan ∠ABC = ∠DEF. Diket ahui panj ang sisi-
Cont oh 22 :

sisi AB = 8, AC = 6, DE = 4 dan EF = 6. Jumlah keliling dua segit iga t ersebut sama dengan ⋅⋅⋅⋅

Solusi :

Karena ∠BAC = ∠EDF dan ∠ABC = ∠DEF maka kedua segit iga t ersebut sebangun.
AB AC
DE = DF
8 6
4 = DF
DF = 3
AB BC
DE = EF
8
4= BC6
BC = 12
Keliling ∆ABC + keliling ∆DEF = (6 + 8 + 12) + (3 + 4 + 6) = 39
Jadi, j umlah keliling dua segit iga t ersebut sama dengan 39.

Segit iga ABC memiliki sisi dengan panj ang AB = 16, AC = 12 dan BC = 8 sert a ∠BAC = α. Segit iga DEF
Cont oh 23 :

memiliki sisi-sisi yang panj angnya DE = 12, DF = 9 sert a ∠EDF = α. Panj ang EF sama dengan ⋅⋅⋅⋅

=
Solusi :
AB AC 4
Karena DE DF = 3 dan sudut yang mengapit sisi AB dan AC sama dengan sudut yang mengapit sisi
DE dan DF sama besar maka ∆ABC dan ∆DEF sebangun.

Eddy Hermanto, ST 104 Geometri


Pembinaan Olimpiade Matematika
EF DE 3
= =
EF = 34 BC = ⋅ 8 = 6
BC AB 4
3
4
Jadi, panj ang EF = 6

Cont oh 24 :
(OSK 2002) Garis AB dan CD sej aj ar dan berj arak 4 sat uan. Misalkan AD memot ong BC di t it ik P
diant ara kedua garis. Jika AB = 4 dan CD = 12, berapa j auh P dari garis CD ?

Solusi :
Dibuat garis EF t egak lurus AB maupun CD sert a melalui t it ik P.

Karena ∠CPD = ∠APB dan AB sej aj ar dengan CD, maka ∆APB sebangun dengan ∆CPD.
PF
= CD
= 124 = 3
⋅⋅⋅⋅⋅⋅⋅⋅⋅⋅ (1)
EP AB
1
EP = 3 PF
EP + PF = 4
1
3 PF + PF = 4
Jadi, PF = 3 sat uan

Cont oh 25 :
Trapesium ABCD dengan AB sej aj ar DC memiliki panj ang AB = 8 dan DC = 4. Diagonal AC dan BD
berpot ongan di t it ik X. Diket ahui perbandingan panj ang AX dengan XD adalah 4 : 1 dan panj ang
diagonal AC = 12. Tent ukan keliling segit iga CXD.

Solusi :
Misalkan panj ang XD = y maka AX = 4y

Karena AB sej aj ar DC maka ∆ABX dan ∆DCX sebangun.


AB AX BX
DC = XC = XD
8 4y BX
4 = XC
= y
Didapat bahwa XC = 2y dan BX = 2y
AC = AX + XC = 4y + 2y
12 = 6y sehingga y = 2
XC = 2y = 4 dan XD = y = 2
Keliling segit iga CXD = DC + XD + XC = 4 + 2 + 4 = 10
Jadi, keliling segit iga CXD = 10.

Eddy Hermanto, ST 105 Geometri


Pembinaan Olimpiade Matematika

LATIHAN 3.B

1. ABCD adalah persegi panj ang dengan AB = 4 dan BC = 3. Tent ukan j arak dari t it ik A ke garis BD.

2. ABCD adalah persegi panj ang dengan panj ang sisi AB = 16 dan AD = 12. Dari t it ik D dibuat garis
memot ong t egak lurus diagonal AC di t it ik P. Dari t it ik B j uga dibuat garis yang memot ong t egak
lurus diagonal AC di t it ik Q. Hit unglah panj ang PQ.

AB. Panj ang AB = 16, DE = 10 dan AD = 6. Panj ang DC adalah ⋅⋅⋅⋅⋅⋅


3. (Alabama MC 1999) Tit ik D dan E bert urut -t urut t erlet ak pada sisi AC dan BC sehingga DE sej aj ar

4. Pada sebuah segit iga siku-siku dengan sisi siku-siku 4 dan 6 dibuat set engah lingkaran dengan
pusat lingkaran t erlet ak pada hipot enusa dan menyinggung kedua sisi siku-siku segit iga t ersebut .
Tent ukanlah j ari-j ari lingkaran t ersebut ?

berpot ongan di t it ik E. Jika panj ang CD = 3 dan AB = 6, maka j arak t it ik E ke AD sama dengan ⋅
5. Pada t rapesium ABCD diket ahui AB t egak lurus AD dan AB sej aj ar DC. Diagonal AC dan BD

6. Pada j aj aran genj ang ABCD, E t erlet ak pada sisi BC. Garis DE memot ong diagonal AC di t it ik G.
Perpanj angan DE dan perpanj angan AB saling berpot ongan di t it ik F. Jika panj ang DG = 6 dan
panj ang EG = 4, t ent ukan panj ang EF.

Jika t it ik E dan F bert urut -t urut adalah t it ik t engah BD dan CD, bukt ikan bahwa AE ⊥ BF.
7. (OSP 2006) Misalkan segit iga ABC siku-siku di B. Garis t inggi dari B memot ong sisi AC di t it ik D.

Eddy Hermanto, ST 106 Geometri


Pembinaan Olimpiade Matematika

C. Garis-garis pada segitiga


Ada empat garis yang akan dibahas.
a. Garis Bagi.
Garis bagi adalah suat u garis yang dit arik dari salah sat u t it ik sudut dan membagi sudut t ersebut
menj adi dua bagian yang sama besar.

Sifat -sifat yang berhubungan dengan ket iga garis bagi dalam ∆ABC :
(i) Ket iga garis bagi bert emu di sat u t it ik.
(ii) Pert emuan ket iga garis bagi merupakan t it ik pusat lingkaran dalam ∆ABC. Lingkaran dalam
segit iga adalah lingkaran yang menyinggung bagian dalam ket iga sisi segit iga.

(iii) Misalkan garis bagi dalam dibuat dari t it ik A memot ong sisi BC di D maka berlaku BA
AC = BD
DC .

= DC
(iv) Misalkan j uga garis bagi luar dibuat dari t it ik A memot ong perpanj angan sisi BC di D maka
BA BD
j uga berlaku AC .

Eddy Hermanto, ST 107 Geometri


Pembinaan Olimpiade Matematika
b. Garis Tinggi.
Garis t inggi adalah suat u garis yang dit arik dari salah sat u t it ik sudut dan memot ong t egak lurus
sisi di hadapannya.

Sifat -sifat yang berhubungan dengan ket iga garis t inggi dalam ∆ABC :
(i) Ket iga garis t inggi bert emu di sat u t it ik.
(ii) Misalkan AD adalah garis t inggi dari ∆ABC maka ∠BDA = ∠CDA = 90o.

c. Garis Berat .
Garis Berat (disebut j uga median) adalah suat u garis yang dit arik dari salah sat u t it ik sudut dan
memot ong pert engahan sisi di hadapannya.

Sifat -sifat yang berhubungan dengan ket iga garis berat dalam ∆ABC :
(i) Ket iga garis berat bert emu di sat u t it ik.
(ii) Perpot ongan ket iga garis berat merupakan t it ik berat ∆ABC.
(iii) Misalkan ket iga garis berat (garis AD, BE dan CF) berpot ongan di t it ik G maka berlaku

( )
AG : GD = BG : GE = CG : GF = 2 : 1.
(iv) Misalkan koordinat t it ik sudut ∆ABC adalah A(xA, yA), B(xB, yB) dan C(xC, yC) maka koordinat
x A + x B + xC
t it ik berat G 3 , y A + y3B + yC .

d. Garis Sumbu.
Garis Sumbu adalah suat u garis yang dit arik t egak lurus dari pert engahan salah sat u sisi dan
memot ong sisi di hadapannya.

Pada gambar di at as, t it ik D, E dan F bert urut -t urut adalah pert engahan sisi AB, BC dan AC.
Sifat -sifat yang berhubungan dengan ket iga garis sumbu dalam ∆ABC :
(i) Ket iga garis sumbu bert emu di sat u t it ik.
(ii) Perpot ongan ket iga garis sumbu merupakan pusat lingkaran luar ∆ABC.

Eddy Hermanto, ST 108 Geometri


Pembinaan Olimpiade Matematika

Cont oh 26 :

∠ACD = ∠DCB dan AC = 10 maka panj ang BC = ⋅⋅⋅⋅⋅


Dari t it ik C pada segit iga ABC dit arik garis memot ong sisi AB di t it ik D sehingga AD = 6 dan DB = 3. Jika

Solusi :

Karena ∠ACD = ∠DCB maka CD adalah garis bagi sehingga berlaku


AC AD
BC = DB
10
BC = 63
Jadi, BC = 5

Cont oh 27 :
(OSP 2004 / OSK 2010) Diberikan segit iga ABC dengan perbandingan panj ang sisi AC : CB = 3 : 4. Garis
bagi sudut luar C memot ong perpanj angan BA di P (t it ik A t erlet ak di ant ara t it ik-t it ik P dan B).
Tent ukan perbandingan panj ang PA : AB.

Solusi :
3
Karena CP adalah garis bagi maka berlaku AC : CB = PA : PB. Maka PA = 4 PB.

PB = PA + AB
4
3 PA = PA + AB.
PA = 3 AB
Jadi, perbandingan panj ang PA : AB = 3 : 1

Cont oh 28 :

AD adalah garis t inggi dari A dan E adalah t it ik t engah AD. Nilai dari BE + CE adalah ⋅⋅⋅⋅⋅⋅⋅
(OSK 2009) Diket ahui ABC adalah segit iga siku-siku di A dengan AB = 30 cm dan AC = 40 cm. Misalkan

Eddy Hermanto, ST 109 Geometri


Pembinaan Olimpiade Matematika
Solusi :
Karena ABC siku-siku di A maka BC = 50 cm.

BD = 30 ⋅ 30
50 = 18 cm sehingga DC = 50 − 18 = 32 cm.
30 x 40
AD = 50 = 24 cm

BE2 = BD2 + DE2 = 182 + 122 = 62 ⋅ 13


DE = 12 cm

CE2 = CD2 + DE2 = 322 + 122 = 42 ⋅ 73


BE + CE = 4 73 + 6 13
Jadi, nilai dari BE + CE adalah 4 73 + 6 13 cm.

Cont oh 29 :
AD dan BE adalah garis berat suat u segit iga ABC. Kedua garis berat ini saling t egak lurus. Hit ung AB
j ika AC = 8 dan BC = 6.

Solusi :
Misal G adalah t it ik berat segit iga
Misal AD = 3x maka AG = 2x dan DG = x

Pada ∆DGB berlaku : x2 + (2y) 2 = 9 sehingga x2 + 4y2 = 9 ⋅⋅⋅⋅⋅⋅⋅⋅⋅⋅⋅⋅⋅ (1)


Misal BE = 3y maka BG = 2y dan EG = y

Pada ∆EGA berlaku : y2 + (2x)2 = 16 sehingga 4x2 + y2 = 16 ⋅⋅⋅⋅⋅⋅⋅⋅⋅⋅⋅⋅⋅ (2)


Jumlahkan persamaan (1) + (2) didapat 5x2 + 5y2 = 25 sehingga x2 + y2 = 5
Pada ∆ABG berlaku (AB)2 = (2x)2 + (2y)2 = 4(x2 + y2) = 20
AB = 2 5

LATIHAN 3.C

1. (OSK 2010) AB, BC dan CA memiliki panj ang 7, 8, 9 bert urut -t urut . Jika D merupakan t it ik t inggi
dari B, t ent ukan panj ang AD.

2. Pada segit iga ABC diket ahui panj ang AB = 5, BC = 7 dan AC = 9. Tit ik D t erlet ak pada AC sehingga
panj ang BD = 5. Tent ukan perbandingan AD : DC.

3. Diket ahui ∆ABC dengan AC = 2BC = 10 cm. Dari t it ik C dibuat garis bagi sudut ACB, sehingga
memot ong AB di t it ik D. Dibuat garis DE t egak lurus pada AB, sehingga BC = EB. Dari t it ik D dibuat
garis t egak lurus pada EB dan memot ong EB di t it ik F. Jika panj ang AD = 8 cm. Hit unglah panj ang
EF.

4. Segit iga sama sisi ABC ket iga t it ik sudut nya t erlet ak pada lingkaran berj ari-j ari 1. Tit ik M dan N
berurut an adalah pert engahan AC dan BC. Perpanj angan MN memot ong lingkaran di t it ik P dengan
panj ang NP < MP. Maka panj ang NP adalah ……

Eddy Hermanto, ST 110 Geometri


Pembinaan Olimpiade Matematika

BD = 1, maka CD = ⋅⋅⋅⋅⋅
5. (OSP 2006) Pada segit iga ABC, garis bagi sudut A memot ong sisi BC di t it ik D. Jika AB = AD = 2 dan

6. Pada ∆ABC, diket ahui AB = 5, AC = 6, BC = 4. Tit ik D t erlet ak pada sisi AB sehingga panj ang AD =
2. Dari t it ik D dibuat garis t egak lurus AC di E dan dibuat sebuah garis lagi dari D t egak lurus BC di
t it ik F. Tent ukan nilai DE : DF.

memot ong AC di t it ik D sehingga BC = BD + AD. Besar sudut CAB adalah ⋅⋅⋅⋅⋅⋅


7. (OSP 2011) Diberikan segit iga samakaki ABC dengan AB = AC. Misalkan garis bagi sudut ABC

8. (OSK 2009) Diberikan segit iga ABC t umpul (∠ABC > 90o), AD dan AE membagi sudut BAC sama

dari sisi segit iga ABC adalah ⋅⋅⋅⋅⋅⋅


besar. Panj ang segmen garis BD, DE dan EC bert urut -t urut adalah 2, 3, dan 6. Panj ang t erpendek

9. (AIME 1992) ABCD adalah t rapesium dengan AB sej aj ar DC, diket ahui panj ang AB = 92, BC = 50, CD
= 19, DA = 70. P adalah sebuah t it ik yang t erlet ak pada sisi AB sehingga dapat dibuat sebuah
lingkaran yang berpusat di P yang menyinggung AD dan BC. Tent ukan panj ang AP.

10. Tit ik E t erlet ak pada sisi AB sehingga AE : EB = 1 : 3 dan t it ik D t erlet ak pada sisi BC sehingga
EF AF
CD : DB = 1 : 2. Garis AD dan CE berpot ongan di F. Tent ukan nilai dari FC + FD .

11. Tit ik M adalah t it ik t engah sisi BC dari segit iga ABC dengan AM : BC = 3 : 2. Bukt ikan bahwa garis
berat dari t it ik B dan C saling t egak lurus.

12. Garis t inggi AP, BQ dan CR dari segit iga ABC berpot ongan di t it ik H. Jika panj ang AH = BC maka
bukt ikan bahwa PR dan PQ t egak lurus.

Eddy Hermanto, ST 111 Geometri


Pembinaan Olimpiade Matematika

D. Luas Segitiga
a. Diket ahui alas dan t inggi segit iga.

Misalkan ∆ABC memiliki panj ang alas = a dan t inggi = t maka


Luas segit iga = [ABC] = 12 at
Dari persamaan di at as akan didapat
(i) Dua buah segit iga yang alas dan t ingginya sama panj ang akan memiliki luas yang sama.

Sebagai cont oh, perhat ikan gambar. Garis l1 dan l2 adalah dua garis yang sej aj ar. Akibat nya
t inggi ∆ABC, ∆ABD akan sama. Karena panj ang alasnya sama yait u AB maka ∆ABC, ∆ABD
keduanya memiliki luas yang sama. Sebagai t ambahan, misalkan perpot ongan kedua segit iga
di t it ik E, maka luas ∆ACE = Luas ∆BDE.

(ii) Dua buah segit iga yang alas at au t ingginya sama maka perbandingan luasnya bert urut -t urut
dapat dinyat akan sebagai perbandingan t inggi at au alasnya.

Sebagai cont oh, perhat ikan gambar. Garis l1 dan l2 adalah dua garis yang sej aj ar. Akibat nya
t inggi ∆ABC, ∆ADE akan sama. Maka perbandingan luas ∆ABC dan ∆ADE dapat dinyat akan
sebagai perbandingan alas. Luas ∆ABC : Luas ∆ADE = panj ang AB : AD.

Cont oh 30 :
Hit unglah luas daerah yang diarsir

Eddy Hermanto, ST 112 Geometri


Pembinaan Olimpiade Matematika

Luas ∆ABD + Luas ∆ABE − 2 ⋅ Luas ∆ABC


Solusi :
Luas daerah yang diarsir =
2 ⋅ 4⋅ 6+ 2 ⋅ 4⋅ 9− 2⋅ 2 ⋅ 4⋅ 3
1 1 1
=
= (12 + 18 − 12) cm2
= 18 cm2

Cont oh 31 :

luas segit iga AED = 6 maka luas segit iga ABD sama dengan ⋅⋅⋅⋅⋅
Pada t rapesium ABCD sisi AB sej aj ar DC. Tit ik E t erlet ak pada sisi AB sehingga AE : EB = 3 : 5. Jika

Solusi :

Segit iga AED dan segit iga ABD memiliki t inggi yang sama, maka perbandingan luas dapat
dinyat akan sebagai perbandingan alas.
[AED] : [ABD] = AE : AB = 3 : 8
[ABD] = 83 [AED] = 83 ⋅ 6 = 16
Jadi, luas segit iga ABD sama dengan 16.

b. Diket ahui dua sisi dan sat u sudut yang mengapit kedua sisi t ersebut .

Misalkan ∆ABC memiliki sisi-sisi a, b dan c sert a t it ik sudut A, B dan C.


Luas segit iga ABC = [ABC] = 12 ab sin C = 12 ac sin B = 12 bc sin A

Cont oh 32 :
(OSP 2002) Segit iga ABC memiliki panj ang sisi AB = 10, BC = 7, dan CA = 12. Jika set iap sisi
diperpanj ang menj adi t iga kali panj ang semula, maka segit iga yang t erbent uk memiliki luas
berapa kali luas ∆ABC ?

Solusi :
1
Luas segit iga semula = ab sin C
(3a)(3b)sin C = 9 ⋅
2
1 1
Luas segit iga akhir = ab sin C
Luas segit iga akhir = 9 ⋅ Luas segit iga semula
2 2

Jadi, perbandingan luas segit iga akhir dengan luas segit iga semula adalah = 9

Berdasarkan cont oh 32 kit a akan dapat kan fakt a bahwa j ika dua buah segit iga yang sebangun
memiliki perbandingan sisi sama dengan k maka perbandingan luasnya akan sama dengan k 2.

Eddy Hermanto, ST 113 Geometri


Pembinaan Olimpiade Matematika

Cont oh 33 :

1 : 2 dan AE : EC = 2 : 3. Jika luas segit iga ABC sama dengan 15 maka luas segit iga ADE adalah ⋅⋅⋅⋅⋅
Tit ik D dan E bert urut -t urut t erlet ak pada sisi AB dan AC dari suat u segit iga ABC dengan AD : DB =

Solusi :
2 AD ⋅ AE sin ∠BAC
1
[ADE] =

2 ⋅ 3 AB ⋅ 5 AC sin ∠BAC
1 1 2
[ADE] =

15 ⋅ 2 AB ⋅ AC sin ∠BAC
2 1
[ADE] =

15 [ABC] = 15 ⋅ 15 = 2
2 2
[ADE] =
Jadi, luas segit iga ADE adalah 2.

c. Diket ahui ket iga sisi.

Misalkan ∆ABC memiliki sisi-sisi a, b dan c

s(s − a )(s − b )(s − c )


Luas segit iga ABC dapat dihit ung dengan menggunakan rumus Heron yait u
Luas segit iga = [ABC] =
1
dengan s = 2 (a + b + c)

Segit iga ABC memiliki sisi-sisi yang panj angnya 5, 6 dan 7. Luas segit iga ABC t ersebut adalah ⋅⋅⋅⋅⋅
Cont oh 34 :

Solusi :

s(s − a )(s − b )(s − c ) = 9(9 − 5)(9 − 6)(9 − 7 ) = 6 6


s = 12 (a + b + c) = 1
2 (5 + 6 + 7) = 9

Luas segit iga = [ABC] =


Jadi, luas segit iga ABC sama dengan 6 6

LATIHAN 3.D

1. Pada segit iga ABC diket ahui a = 2√2, b = 2√3 dan sudut A = 45o, maka luas segit iga it u adalah ⋅⋅⋅⋅

2. (Alabama MC 2003) ABCD adalah t rapesium dengan AB sej aj ar DC. Panj ang AB = 12 dan CD = 9.

CXD adalah ⋅⋅⋅⋅⋅


Diagonal AC dan BD berpot ongan di t it ik X. Diket ahui bahwa luas segit iga AXB = 64. Luas segit iga

3. (OSP 2004) Pada sisi-sisi SU, TS dan UT dari ∆STU dipilih t it ik-t it ik P, Q dan R bert urut -t urut
sehingga SP = 14 SU, TQ = 12 TS dan UR = 13 UT. Jika luas segit iga STU adalah 1, berapakah luas
segit iga PQR ?

Eddy Hermanto, ST 114 Geometri


Pembinaan Olimpiade Matematika

4. Diket ahui koordinat t it ik A(4,7), B(6,13) dan C(7,9). Berapakah luas segit iga ABC ?

5. (OSP 2009/ AIME 1988) Diberikan segit iga ABC dengan t an ∠CAB = 227 . Melalui t it ik sudut A dit arik

dan 17. Luas segit iga ABC adalah ⋅⋅⋅⋅⋅⋅⋅⋅


garis t inggi sedemikian rupa sehingga membagi sisi BC menj adi segmen-segmen dengan panj ang 3

6. (Canadian MO 1969) Misalkan ABC adalah sebuah segit iga dengan sisi-sisinya a, b dan c. Garis bagi
yang dit arik dari t it ik C memot ong AB di D. Bukt ikan bahwa panj ang
2 ab cos C2
CD = a +b

7. (OSP 2008/ Hongkong PSC) Diberikan segit iga ABC dengan sisi-sisi a, b, dan c. Nilai a2 + b2 + c2
sama dengan 16 kali luas segit iga ABC. Besarnya nilai ct g A + ct g B + ct g C adalah ⋅⋅⋅⋅⋅

8. Segi empat ABCD memiliki panj ang sisi-sisi AB = 9, BC = 12, CD = 13 dan DA = 14. Panj ang diagonal
AC adalah 15. Dari t it ik B dan D dibuat garis t egak lurus AC dan memot ong AC bert urut -t urut di
t it ik P dan Q. Hit unglah panj ang PQ.

9. ABCD adalah sebuah persegi panj ang dengan luas 1. Diagonal AC dan BD berpot ongan di E. Tit ik F
t erlet ak pada pert engahan BC. Jika AF berpot ongan dengan diagonal BD di G, maka berapakah
luas segit iga AEG ?

panj ang BF dan FE dinyat akan dalam x dan y adalah ⋅⋅⋅⋅⋅⋅⋅⋅⋅⋅


10. (OSP 2011) Pada gambar di bawah ini, panj ang AE = x, EC = y, dan DC = 2BD. Perbandingan

11. (OSN 2004) Bukt ikan bahwa suat u segit iga ABC siku-siku di C dengan a menyat akan sisi dihadapan

diamet er lingkaran dalam = a + b − c.


sudut A, b menyat akan sisi di hadapan sudut B, c menyat akan sisi di hadapan sudut C memiliki

12. Pada persegi panj ang ABCE, t it ik F dan G t erlet ak pada sisi AB sehingga AF = FG = GB dan t it ik E

panj ang ABCD sama dengan 70 maka luas segit iga EHJ adalah ⋅⋅⋅⋅⋅
merupakan pert engahan sisi DC. Diagonal AC memot ong sisi EF di H dan EG di J. Jika luas persegi

13. (OSN 2011 SMP/ MTs) Bangun dat ar ABCD di samping adalah
t rapesium dengan AB sej aj ar CD. Tit ik E dan F t erlet ak
pada CD sehingga AD sej aj ar BE dan AF sej aj ar BC. Tit ik H
adalah perpot ongan AF dengan BE dan t it ik G adalah
perpot ongan AC dengan BE. Jika panj ang AB adalah 4 cm
dan panj ang CD adalah 10 cm hit unglah perbandingan luas
segit iga AGH dengan luas t rapesium ABCD.

14. (OSN 2009 SMP/ MTs) Diket ahui segit iga ABC dengan A sebagai puncak dan BC sebagai alas. Tit ik P
t erlet ak pada sisi CA. Dari t it ik A dit arik garis sej aj ar PB dan memot ong perpanj angan alas di t it ik
D. Tit ik E t erlet ak pada alas sehingga CE : ED = 2 : 3. Jika F adalah t engah-t engah ant ara E dan C,
dan luas segit iga ABC sama dengan 35 cm2, berapakah luas segit iga PEF ?

Eddy Hermanto, ST 115 Geometri


Pembinaan Olimpiade Matematika
15. (Alabama MC 2003) Pada segit iga ABC, t it ik E dan F bert urut -t urut adalah pert engahan AC dan

berpot ongan di J. Jika luas segit iga ABC = 120 maka luas segit iga EFJ adalah ⋅⋅⋅⋅⋅
pert engahan BC. Tit ik H dan I t erlet ak pada ruas AB sehingga AH = HI = IB. Garis IE dan HF

16. (AIME 1988) P adalah t it ik di dalam segit iga ABC. Perpanj angan PA memot ong sisi BC di D,

PD = PE = PF = 3 dan PA + PB + PC = 43 t ent ukan nilai dari PA ⋅ PB ⋅ PC.


perpanj angan PB memot ong sisi AC di E dan perpanj angan PC memot ong sisi AB di F. Jika panj ang

17. (OSK 2006) Pada segit iga ABC, t it ik F membagi sisi AC dalam perbandingan 1 : 2. Misalkan G t it ik
t engah BF dan E t it ik perpot ongan ant ara sisi BC dengan AG. Maka t it ik E membagi sisi BC dalam
perbandingan

18. Pada persegi ABCD dengan panj ang sisi 1, t it ik E pada AB dan t it ik F pada BC sehingga segit iga
DEF adalah segit iga sama sisi. Tent ukan luas segit iga DEF.

19. (OSK 2006) Pada segit iga ABC yang t umpul di C, t it ik M adalah t it ik t engah AB. Melalui C dibuat

lurus di D. Jika luas segit iga ABC adalah 54 sat uan luas, maka luas segit iga BED adalah ⋅⋅⋅⋅⋅
garis t egak lurus pada BC yang memot ong AB di t it ik E. Dari M t arik garis memot ong BC t egak

20. Diket ahui segit iga siku-siku ABC, sisi AB t egak lurus sisi AC. Panj ang AB = 3 dan panj ang AC = 4.
Tit ik P t erlet ak di dalam segit iga ABC. Tit ik D, E dan F masing-masing t erlet ak pada sisi BC, AC

PF + PE + PD = 12 , hit unglah panj ang PE, PF dan PD.


dan AB sehingga PD t egak lurus BC, PE t egak lurus AC dan PF t egak lurus AB. Jika
AB AC BC

t erlet ak di dalam segit iga ABC sehingga ∠PAC = ∠PBA = ∠PCB = ϕ. Nilai dari t an ϕ = ⋅⋅⋅⋅⋅⋅
21. Pada segit iga ABC diket ahui panj ang sisi-sisinya adalah AB = 13, BC = 14 dan AC = 15. Tit ik P

22. P adalah sebuah t it ik di dalam segit iga ABC. Tiga buah garis dibuat melalui t iit k P yang sej aj ar
dengan ket iga sisi segit iga ABC. Perpot ongan garis-garis t ersebut dengan sisi-sisi segit iga
membent uk segit iga kecil. Luas ket iga segit iga t ersebut adalah p2, q2 dan r 2. Bukt ikan bahwa luas
segit iga ABC adalah (p + q + r) 2.

23. S adalah t it ik yang t erlet ak di dalam segit iga ABC sehingga luas ∆SAB, ∆SBC dan ∆SCA sama.
Tunj ukkan bahwa S adalah t it ik berat segit iga ABC.

24. (Flanders MO 2001 Final Round) Pada segit iga ABC t it ik D dan E bert urut -t urut t erlet ak pada sisi
AC dan BC. Garis BD dan AE berpot ongan di t it ik F. Misalkan [XYZ] menyat akan luas segit iga XYZ.
Jika [ADF] = 4, [ABF] = 8 dan [BEF] = 7 maka t ent ukan luas daerah CDFE.

Eddy Hermanto, ST 116 Geometri


Pembinaan Olimpiade Matematika

E. Hubungan antara luas segitiga dengan jari-jari lingkaran dalam dan jari-jari lingkaran luar segitiga
Ada hubungan ant ara luas segit iga dengan j ari-j ari lingkaran dalam dan j ari-j ari lingkaran luar.
Luas segit iga ABC = [ABC] = 12 r(a + b + c) = rs
Luas segit iga ABC = [ABC] = abc
4R
Sebagai bahan pembelaj aran, silakan Pembaca membukt ikan kedua rumus di at as dengan
menggunakan rumus-rumus luas yang ada dit ambah dengan rumus-rumus yang lainnya.

Cont oh 35 :

ABC adalah ⋅⋅⋅⋅


(OSK 2004) Jika luas segit iga ABC sama dengan kelilingnya, maka j ari-j ari lingkaran dalam segit iga

Solusi :
Misal j ari-j ari lingkaran dalam sama dengan r dan ket iga sisinya adalah a, b dan c, maka :
Luas segit iga = 12 r (a + b + c)
Luas segit iga = 12 r ⋅ Keliling segit iga
Karena luas segit iga sama dengan keliling segit iga maka r = 2
Jadi, j ari-j ari lingkaran dalam segit iga ABC adalah 2.

Cont oh 36 :

segit iga ABC t ersebut adalah ⋅⋅⋅⋅⋅


(Alabama MC 1999) ABC adalah segit iga samasisi dengan panj ang sisi 12. Jari-j ari lingkaran dalam

Solusi :
Misalkan sisi-sisi segit iga t ersebut adalah a, b dan c dengan a = b = c = 12.
[ABC] = 12 r (a + b + c)
1
2 ⋅ 12 ⋅ 12 sin 60o = 1
2 r ⋅ (12 + 12 + 12)
r = 2√3
Jadi, j ari-j ari lingkaran dalam segit iga ABC t ersebut adalah 2√3.

LATIHAN 3.E

1. Jika r dan R menyat akan j ari-j ari lingkaran dalam dan lingkaran luar segit iga yang panj ang sisi-
sisinya adalah 5, 6 dan 7 maka t ent ukan nilai dari hasil kali rR.

2. (OSK 2008) Lingkaran T merupakan lingkaran luar bagi segit iga ABC dan lingkaran dalam bagi
segit iga PQR. Jika ABC dan PQR keduanya segit iga samasisi, maka rasio keliling ∆ABC t erhadap
keliling ∆PQR adalah

3. (OSP 2009) Diket ahui segit iga siku-siku ABC dengan panj ang sisi-sisinya a, b, dan c sert a a < b < c.

r (a +b + c )
= 3 maka nilai dari
Misalkan r dan R bert urut -t urut menyat akan panj ang j ari-j ari lingkaran dalam dan lingkaran
luarnya. Jika a +b+ c
r
adalah ⋅⋅⋅⋅⋅⋅⋅⋅
R2

Eddy Hermanto, ST 117 Geometri


Pembinaan Olimpiade Matematika

F. Ketaksamaan Segitiga
Pada set iap segit iga haruslah berlaku bahwa panj ang set iap sisi selalu kurang dari j umlah panj ang dua
sisi yang lain.
Misalkan panj ang sisi-sisi segit iga ABC adalah a, b dan c maka berlaku
a < b + c ; b < a + c dan c < a + b

Cont oh 37 :
Ada berapa banyak nilai n bulat j ika 5, 6 dan n + 4 merupakan sisi-sisi suat u segit iga ?

* Misalkan 6 adalah sisi t erpanj ang maka 6 < 5 + n + 4 akan didapat n > −3
Solusi :

Selain it u n + 4 ≤ 6 sehingga n ≤ 2.
Nilai n yang memenuhi adalah −2, −1, 0, 1, 2

Selain it u n + 4 ≥ 6 sehingga n ≥ 2
* Misalkan n + 4 adalah sisi t erpanj ang maka n + 4 < 5 + 6 akan didapat n < 7

Jadi, nilai n yang memenuhi adalah −2, −1, 0, 1, 2, 3, 4, 5 dan 6.


Nilai n yang memenuhi adalah 2, 3, 4, 5, 6

Jadi, banyaknya nilai n yang memenuhi ada 9.

Cont oh 38 :
Misalkan t it ik T t erlet ak pada segit iga ABC. Bukt ikan bahwa
TA + TB + TC > 12 Keliling ∆ABC

Solusi :

Berdasarkan ket aksamaan segit iga maka


Pada ∆TAB berlaku TA + TB > AB ⋅⋅⋅⋅⋅⋅⋅⋅⋅⋅⋅⋅⋅⋅⋅⋅⋅⋅⋅⋅⋅ (1)
Pada ∆TAC berlaku TA + TC > AC ⋅⋅⋅⋅⋅⋅⋅⋅⋅⋅⋅⋅⋅⋅⋅⋅⋅⋅⋅⋅⋅ (2)
Pada ∆TBC berlaku TB + TC > BC ⋅⋅⋅⋅⋅⋅⋅⋅⋅⋅⋅⋅⋅⋅⋅⋅⋅⋅⋅⋅⋅ (1)
Jumlahkan ket iga persamaan (1), (2) dan (3) maka
2(TA + TB + TC) > AB + AC + BC
TA + TB + TC > 12 Keliling ∆ABC (t erbukt i)

Cont oh 39 :
(OSK 2007) Keliling sebuah segit iga adalah 8. Jika panj ang sisi-sisinya adalah bilangan bulat , maka
luas segit iga t ersebut sama dengan

Solusi :
a + b + c = 8 dengan a, b, dan c semuanya bilangan asli.
Kombinasi t ripel (a,b,c) yang mungkin adalah (6,1,1), (5,2,1), (4,3,1), (4,2,2), (3,3,2).
Syarat : panj ang salah sat u sisi selalu kurang dari j umlah kedua sisi yang lain.
Yang memenuhi a < b + c hanya t ripel (a,b,c) = (3,3,2)

Eddy Hermanto, ST 118 Geometri


Pembinaan Olimpiade Matematika
1
s= (a + b + c) = 4

s (s − a )(s − b )(s − c ) = 2 2
2

Dengan rumus Heron, Luas ∆ =


Luas ∆ = 2 2

LATIHAN 3.F :

10 adalah ⋅⋅⋅⋅⋅
1. (OSP 2009) Banyaknya segit iga t umpul dengan sisi bilangan asli yang memiliki sisi-sisi t erpanj ang

2. (OSP 2009) Diberikan segit iga dengan panj ang dari ket iga garis t inggi segit iga it u merupakan

t inggi ket iga adalah ⋅⋅⋅⋅⋅⋅


bilangan bulat . Jika panj ang kedua garis t ingginya adalah 10 dan 6, maka panj ang maksimum garis

berpot ongan t egak lurus. Nilai minimum ct g B + ct g C adalah ⋅⋅⋅⋅⋅


3. (OSP 2006) Pada segit iga ABC, garis-garis berat dari t it ik sudut B dan t it ik sudut C saling

4. Buj ur sangkar ABCD memiliki sisi yang panj angnya a dan diagonal yang panj angnya d. Segit iga APQ
dibuat sedemikian sehingga t it ik P pada sisi BC dan Q pada sisi AB dengan DP = DQ. Jika keliling
segit iga DPQ = k, bukt ikan bahwa 2d < k < 4a.

5. Panj ang sisi-sisi suat u segi empat merupakan bilangan asli. Panj ang masing-masing sisi membagi
j umlah panj ang ket iga sisi yang lain. Bukt ikan bahwa t erdapat sedikit nya dua sisi dengan panj ang
yang sama.

6. (OSP 2009) Diberikan segit iga ABC dan t it ik D pada sisi AC. Misalkan r 1, r 2 dan r bert urut -t urut
menyat akan j ari-j ari lingkaran dalam dari segit iga-segit iga ABD, BCD, dan ABC. Bukt ikan bahwa
r 1 + r 2 > r.

Eddy Hermanto, ST 119 Geometri


Pembinaan Olimpiade Matematika

4. SEGIEMPAT
Ada beberapa bangun segiempat dalam dua dimensi yang akan dibahas.
a. Persegi Panj ang.

Sifat -sifat persegi panj ang :


(i) Dua sisi berhadapan sej aj ar.
Dari gambar didapat AB ⁄⁄ DC dan AD ⁄⁄ BC.
(ii) Dua buah sisi berhadapan sama panj ang.
AB = DC dan AD = BC.
(iii) Masing-masing keempat t it ik sudut sama dengan 90o.
∠A = ∠B = ∠C = ∠D = 90o.

(iv) Kedua diagonal berpot ongan dan saling membagi dua sama panj ang.
AO = OC = BO = OD.

Misalkan persegi panj ang memiliki sisi yang panj angnya p dan l maka berlaku

Luas persegi panj ang = p ⋅ l


Keliling persegi panj ang = 2(p + l)

b. Persegi.

Sifat -sifat persegi :


(i) Dua sisi berhadapan sej aj ar.
Dari gambar didapat AB ⁄⁄ DC dan AD ⁄⁄ BC.
(ii) Keempat sisi sama panj ang.
AB = DC = AD = BC.
(iii) Masing-masing keempat t it ik sudut sama dengan 90o.
∠A = ∠B = ∠C = ∠D = 90o.
(iv) Kedua diagonal saling t egak lurus.
AC t egak lurus BD.
(v) Kedua diagonal sama panj ang dan saling membagi dua sama panj ang.
AC = BD dan AO = OC = BO = OD.

Misalkan persegi memiliki sisi yang panj angnya s maka berlaku


Keliling persegi = 4s
Luas persegi = s2

Eddy Hermanto, ST 120 Geometri


Pembinaan Olimpiade Matematika
c. Jaj aran Genj ang.

Sifat -sifat j aj aran genj ang :


(i) Dua sisi berhadapan sej aj ar dan sama panj ang.
Dari gambar didapat AB ⁄⁄ DC dan AD ⁄⁄ BC sert a AB = DC dan AD = BC.

∠BAD = ∠BCD dan ∠ADC = ∠ABC.


(ii) Sudut yang berhadapan sama besar.

(iii) Kedua diagonal saling membagi dua sama panj ang.


AO = OC dan BO = OD.

Misalkan j aj aran genj ang memiliki sisi yang panj angnya a dan b sert a j arak dua sisi sej aj ar a sama
dengan t maka berlaku

Luas j aj aran genj ang = a ⋅ t


Keliling j aj aran genj ang = 2(a + b)

d. Belah Ket upat .

Sifat -sifat belah ket upat :


(i) Dua sisi berhadapan sej aj ar.
Dari gambar didapat AB ⁄⁄ DC dan AD ⁄⁄ BC.
(ii) Semua sisi sama panj ang.
AB = DC = AD = BC.

∠BAD = ∠BCD dan ∠ADC = ∠ABC.


(iii) Sudut yang berhadapan sama besar.

(iv) Kedua diagonal berpot ongan t egak lurus dan saling membagi sama panj ang.
AO = OC dan BO = OD.

Misalkan belah ket upat memiliki sisi-sisi yang panj angnya a sert a panj ang kedua diagonalnya d1 = AC
dan d2 = BD maka berlaku
Keliling belah ket upat = 4a
Luas belah ket upat = 12 ⋅ d1 ⋅ d2

Eddy Hermanto, ST 121 Geometri


Pembinaan Olimpiade Matematika

e. Trapesium.

Sifat -sifat t rapezium :


(i) Memiliki t epat sepasang sisi yang sej aj ar.
Dari gambar didapat AB ⁄⁄ DC.
(ii) Sudut ant ara dua sisi sej aj ar sama dengan 180o.
∠BAD + ∠ADC = 180o dan ∠ABC + ∠BCD = 180o.

Misalkan t rapesium memiliki sisi-sisi yang panj angnya a, b, c dan d dengan a dan c sej aj ar sert a j arak
dua sisi sej aj ar sama dengan t maka berlaku
Keliling t rapesium = a + b + c + d
Luas t rapesium = 12 ⋅ (a + c) ⋅ t

f. Layang-layang.

Sifat -sifat layang-layang :


(i) Memiliki dua pasang sisi sama panj ang.
AB = BC dan AD = CD.

Diagonal BD ⊥ AC.
(ii) Kedua diagonal berpot ongan t egak lurus.

(iii) Diagonal t erpanj ang membagi diagonal t erpendek sama panj ang.
Diagonal t erpanj ang adalah BD sehingga AO = OC.

Misalkan layang-layang memiliki sisi-sisi yang panj angnya AB = BC = a dan AD = CD = b sert a panj ang
kedua diagonalnya d1 = AC dan d2 = BD maka berlaku
Keliling layang-layang = 2(a + b)
Luas belah ket upat = 12 ⋅ d1 ⋅ d2

Cont oh 40 :

adalah 20 cm, maka keliling A, dalam cent imet er, adalah ⋅⋅⋅⋅
(OSK 2005) Diberikan dua buah persegi, A dan B, dimana luas A adalah separuh dari luas B. Jika keliling B

Solusi :
Luas B = 2 Luas A, maka B = 2A
Misalkan panj ang sisi A = x dan panj ang sisi B = y maka
Luas B = y2 = 2x2 sehingga y = x√2

Eddy Hermanto, ST 122 Geometri


Pembinaan Olimpiade Matematika
Keliling B = 4y.
5
Maka 4x√2 = 20 sehingga x = 2 2
Keliling A = 4x = 10√2
Jadi, keliling A = 10√2 cm

Cont oh 41 :
(OSK 2008) Pada t rapesium ABCD, sisi AB sej aj ar sisi DC dan rasio luas segit iga ABC t erhadap luas segit iga
ACD adalah 13 . Jika E dan F bert urut -t urut adalah t it ik t engah BC dan DA, maka rasio luas ABEF t erhadap
luas EFDC adalah

Solusi :

∆ABC dan ∆ACD memiliki t inggi yang sama maka perbandingan luas keduanya dapat dinyat akan sebagai
perbandingan alas.
AB : DC = 1 : 3
Misalkan panj ang sisi AB = x maka panj ang sisi DC = 3x.
E adalah pert engahan BC dan F pert engahan DA sehingga FE sej aj ar AB dan DC.
Maka FE = 12 (x + 3x) = 2x

( AB + FE )

Misalkan t inggi t rapesium = t .
t 3tx
Luas ABEF = =
⋅ = 5tx4
( FE + DC ) t
2 2 4

Luas EFDC = 2 2
Rasio luas ABEF t erhadap luas EFDC = 3 : 5.
3
Jadi, rasio luas ABEF t erhadap luas EFDC adalah 5 .

LATIHAN 4 :

1. (OSK 2007) Sepot ong kawat dipot ong menj adi 2 bagian,dengan perbandingan panj ang 3 : 2. Masing-
masing bagian kemudian dibent uk menj adi sebuah persegi. Perbandingan luas kedua persegi adalah

2. (Alabama MC 1999) Sebuah belah ket upat memiliki sisi yang panj angnya 10. Salah sat u sudut dalam
belah ket upat t ersebut besarnya sama dengan 60o. Panj ang diagonal t erpendek belah ket upat
t ersebut adalah ⋅⋅⋅⋅⋅⋅

3. (OSP 2003) Dalam sebuah segit iga ABC siku-siku sama kaki, dibuat persegi PQRS sebagai berikut : Tit ik
P pada sisi AB, t it ik Q pada sisi AC, sedangkan t it ik-t it ik R dan S pada sisi miring BC. Jika luas segit iga
ABC adalah x, berapakah luas persegi PQRS ?

4. (OSP 2005) Misalkan ABCD adalah sebuah t rapesium dengan BC║AD. Tit ik-t it ik P dan R bert urut -t urut
adalah t it ik t engah sisi AB dan CD. Tit ik Q t erlet ak pada sisi BC sehingga BQ : QC = 3 : 1, sedangkan

t rapesium ABCD adalah ⋅⋅⋅⋅⋅⋅


t it ik S t erlet ak pada sisi AD sehingga AS : SD = 1 : 3. Maka rasio luas segiempat PQRS t erhadap luas

Eddy Hermanto, ST 123 Geometri


Pembinaan Olimpiade Matematika

D(0,0). Luas j aj aran genj ang ABCD sama dengan ⋅⋅⋅⋅⋅⋅


5. (OSK 2007) Diket ahui empat t it ik pada bidang dengan koordinat A(1,0), B(2008,2007), C(2007,2007),

6. ABC adalah segit iga siku-siku dengan AB adalah hipot enusa. Panj ang AC = 6 dan BC = 8. Bangun ABLH

CBLH adalah ⋅⋅⋅⋅⋅⋅⋅⋅


adalah persegi (buj ur sangkar) dengan t it ik C t idak t erlet ak di dalam persegi t ersebut . Luas segiempat

7. Pada suat u j aj aran genj ang, dua diagonalnya membent uk sudut 60o. Panj ang sisi-sisinya adalah 6 dan
8. Luas j aj aran genj ang t ersebut adalah

8. ABCD adalah t rapesium dengan AB sej aj ar CD. Diagonal AC dan BD berpot ongan di t it ik O. Luas
segit iga AOB = 992 sedangkan luas segit iga COD = 192. Tent ukan luas t rapesium t ersebut .

adalah belah ket upat . Jika AB = 16 dan BC = 12, maka panj ang EF sama dengan ⋅⋅⋅⋅⋅⋅⋅
9. Tit ik E dan F secara berurut an t erlet ak pada sisi AB dan CD suat u persegi panj ang ABCD sehingga DFBE

10. (OSP 2004/ Hongkong PSC 1988) Pada sebuah t rapesium dengan t inggi 4, kedua diagonalnya saling
t egak lurus. Jika salah sat u dari diagonal t ersebut panj angnya 5, berapakah luas t rapesium t ersebut ?

11. (Bulgarian MO 1995 : Spring MC Grade 8) Misalkan M adalah t it ik t engah sisi BC pada j aj aran genj ang
ABCD sedangkan N adalah perpot ongan AM dan diagonal BD. Perpanj angan DA dan CN berpot ongan di
t it ik P.
a. Bukt ikan bahwa AP = AD
b. Jika AB = AC maka bukt ikan CP = BD

Eddy Hermanto, ST 124 Geometri


Pembinaan Olimpiade Matematika

5. SEGI-N BERATURAN
Segi-n berat uran adalah suat u bangun dat ar yang memiliki sisi sebanyak n dan panj ang semua sisinya
sama.

Gambar di at as adalah cont oh segi-n berat uran yait u segi-12 berat uran.

Keliling segi-n berat uran = n ⋅ s


Misalkan panj ang sisi suat u segi-n berat uran adalah s maka

Bagaimana caranya menghit ung luas ?


Segi-n berat uran dapat dibagi menj adi n buah segit iga sama kaki dengan salah sat u sisi panj angnya s dan
dua sisi yang lain sama panj ang.
Karena sat u put aran = 360o maka besarnya sudut pada segit iga di hadapan sisi s besarnya dapat dihit ung
°
yait u = 360
n .
Karena salah sat u sat u sisi diket ahui dan sudut di hadapan sisi t ersebut diket ahui sert a dua sisi yang lain

Luas segi-n berat uran = n ⋅ Luas segit iga


sama panj ang maka luas segit iga t ersebut dapat dihit ung.

LATIHAN 5 :

1. (OSK 2004) Pada sebuah segi6 berat uran, rasio panj ang ant ara diagonal t erpendek t erhadap diagonal
t erpanj ang adalah

2. (OSP 2005) Sebuah segienam berat uran dan sebuah segit iga sama sisi mempunyai keliling yang sama.
Jika luas segit iga adalah 3 , maka luas segienam adalah ⋅⋅⋅⋅

3. Diket ahui bahwa ABCDEF adalah segienam berat uran. Tent ukan perbandingan luas segienam
berat uran ABCDEF dengan luas daerah diarsir.

j am. Diket ahui koordinat A(0, 4), B(4, 0) dan E(p, q). Maka p − q = ⋅⋅⋅⋅
4. ABCDEFGH adalah segidelapan berat uran dengan penyusunan huruf disusun berlawanan arah j arum

Eddy Hermanto, ST 125 Geometri


Pembinaan Olimpiade Matematika

6. LINGKARAN
Lingkaran adalah kumpulan t it ik-t it ik yang memiliki j arak yang sama t erhadap suat u t it ik t ert ent u, yait u
pusat lingkaran.
Jadi ada dua hal yang sangat berkait an dengan lingkaran yait u j ari-j ari lingkaran, R, dan pusat lingkaran.
Unsur-unsur pada lingkaran dapat dilihat pada gambar berikut .

a. Tit ik O disebut sebagai pusat lingkaran


b. OA, OB, OC, OD, OE disebut sebagai j ari-j ari lingkaran
c. Ruas garis lurus AB yang melalui pusat lingkaran disebut diamet er lingkaran
d. Ruas garis DE disebut t ali busur
e. Garis lengkung DE dan AC disebut busur lingkaran
f. Daerah arsiran yang dibat asi dua j ari-j ari (pada gambar dibat asi OA dan OC sert a berwarna hit am)
disebut j uring
g. Daerah yang dibat asi t alibusur DE dan busur DE disebut t embereng
h. Garis OF yang t egak lurus DE disebut apot ema

Misalkan r adalah j ej ari lingkaran dan d adalah diamet er lingkaran dengan d = 2r


Luas lingkaran = πr 2 = 14 πd2.
Keliling Lingkaran = 2πr
° ⋅ πr dengan n adalah sudut pusat diukur dalam deraj at .
n 2
Luas Juring = 360

Panj ang Busur = 360°


n
⋅ 2πr dengan n adalah sudut pusat diukur dalam deraj at .
Luas t embereng DE = Luas Juring ODE − Luas ∆ODE

Cont oh 42 :
Perhat ikan gambar. AB dan CD adalah diamet er lingkaran dengan AB = CD = 8 sert a AB dan CD saling t egak
lurus. Busur AC, CB, BD dan DA adalah 4 busur yang kongruen dengan dua busur yang berdekat an saling
bersinggungan. Tent ukan luas daerah yang diarsir.

Solusi :
Alt ernat if 1 :
Buat persegi EFGH dengan A, B, C dan D adalah pert engahan sisi-sisinya.

Eddy Hermanto, ST 126 Geometri


Pembinaan Olimpiade Matematika
Luasarsir = Luaspersegi EFGH − 4 ⋅ Luas1/ 4 lingkaran
Luasarsir = 8 ⋅ 8 − 4 (¼ π 42)
Luasarsir = 64 − 16π

Alt ernat if 2 :

Luast embereng AC = Luas1/ 4 lingkaran − Luas ∆AOC


Misal perpot ongan garis AB dan CD di t it ik O

Luast embereng AC = ¼ ⋅ π ⋅ 42 − ½ ⋅ 4 ⋅ 4
Luast embereng AC = 4π − 8
Luas arsir = Luas lingkaran − 8 ⋅ Luas t embereng
Luas arsir = π ⋅ 42 − 8 ⋅ (4π − 8)
Luas arsir = 64 − 16π

Dalam pembahasan berikut ini, Penulis t idak menj elaskan persamaan lingkaran, t et api lebih
mengedepankan kepada dalil-dalil yang berhubungan dengan lingkaran.

(a) Misalkan garis l menyinggung lingkaran yang berpusat di O pada t it ik A maka OA akan t egak lurus
garis l.

(b) Misalkan t it ik A t erlet ak di luar lingkaran L maka dari t it ik A dapat dibuat dua buah garis singgung
yang j araknya t erhadap t it ik singgungnya sama panj ang.

Tit ik A t erlet ak di luar lingkaran. Dari A dibuat dua garis yang menyinggung lingkaran di t it ik P dan
Q maka panj ang AP = AQ.

menyinggung lingkaran di t it ik A dan B. Maka ∠APO = ∠BPO.


(c) Misalkan t it ik P t erlet ak di luar lingkaran L yang berpusat di O dan garis yang dit arik dari t it ik P

Berdasarkan kesimet rian akan didapat ∠APO = ∠BPO.

(d) Sebuah lingkaran berpusat di A menyinggung di luar sebuah lingkaran berpusat di B pada t it ik P.
Maka A, P dan B berada pada sat u garis lurus.

Eddy Hermanto, ST 127 Geometri


Pembinaan Olimpiade Matematika

Buat garis singgung melalui t it ik P. Maka garis singgung t ersebut akan t egak lurus AP dan PB
berakibat AP dan PB akan sej aj ar. Jadi, A, P dan B berada pada sat u garis lurus.

(e) Garis yang menghubungkan pusat dua lingkaran akan memot ong t egak lurus pert engahan t alibusur
persekut uannya.

Misalkan lingkaran yang berpusat di A berpot ongan di t it ik P dan Q dengan lingkaran yang berpusat
di B. Maka AB akan berpot ongan t egak lurus dengan PQ di t it ik T yang merupakan pert engahan PQ.

(f) Besar sudut pusat sama dengan dua kali sudut keliling.

Misalkan AB adalah t alibusur dan O pusat lingkaran. Maka ∠AOB disebut sebagai sudut pusat .
Misalkan j uga t it ik C t erlet ak pada lingkaran t ersebut , maka ∠ACB disebut sudut keliling.

∠AOB = 2∠ACB
Hubungan ant ara sudut pusat dan sudut keliling t ersebut adalah

Berlaku j uga bahwa j ika ∠AOB = 2 ∠ACB maka dapat dibuat sebuah lingkaran melalui A, B dan C
sert a berpusat di O.

(g) Besar sudut keliling yang menghadap t alibusur yang sama akan sama besar.

∠ACB = ∠ADB
Misalkan AB adalah t alibusur dan t it ik C dan D t erlet ak pada lingkaran. Maka

∠ACB = 90o
(h) Misalkan AB adalah diamet er suat u lingkaran dan C t erlet ak pada lingkaran t ersebut , maka berlaku

Eddy Hermanto, ST 128 Geometri


Pembinaan Olimpiade Matematika

(i) Misalkan AB adalah t alibusur suat u lingkaran yang berpusat di O dan t it ik P adalah pert engahan AB
maka OP akan t egak lurus AB

Karena O adalah pusat lingkaran maka OA = OB = j ari-j ari lingkaran. Jadi ∆AOB adalah segit iga sama
kaki. Karena OAB segit iga sama kaki maka garis dari O akan memot ong t egak lurus pert engahan sisi
AB.

AX ⋅ XB = CX ⋅ XD. Berlaku sebaliknya, j ika dua buah garis AB dan CD berpot ongan di t it ik X dan
(j ) Misalkan dua t alibusur AB dan CD pada sat u lingkaran saling berpot ongan di t it ik X maka berlaku

memenuhi AX ⋅ XB = CX ⋅ XD maka keempat t it ik A, B, C dan D t erlet ak pada sat u lingkaran.


Perhat ikan gambar.

Dari hubungan garis didapat bahwa ∠AXD = ∠CXB


Perhat ikan bahwa ruas AC j uga merupakan t alibusur sehingga dari dalil sebelumnya maka ∠ADC =
∠ABC.
Dengan cara yang sama akan didapat bahwa ∠BAD = ∠BCD.
Karena ket iga sudut ∆ADX dan ∆BCX sama maka kedua segit iga t ersebut sebangun. Akibat nya
CX
berlaku AX
XD = XB sehingga
AX ⋅ XB = CX ⋅ XD
Berlaku kebalikannya.

(k) Misalkan t it ik A, B, C dan D semuanya t erlet ak pada sat u lingkaran dengan AC dan BD adalah kedua

AB ⋅ DC + AD ⋅ BC = AC ⋅ BD
diagonal.Maka berlaku :

Persamaan di at as dikenal dengan dalil Pt olomeus.

(l) Pada segiempat t alibusur, j umlah sudut sehadapan sama dengan 180o berlaku j uga bahwa j ika
j umlah sudut sehadapan sama dengan 180o maka segiempat t ersebut merupakan segiempat
t alibusur.
Perlu dij elaskan bahwa segiempat t alibusur adalah segiempat yang keempat t it ik sudut nya t erlet ak
pada sat u lingkaran.

Eddy Hermanto, ST 129 Geometri


Pembinaan Olimpiade Matematika

Karena t it ik-t it ik A, B, C dan D semuanya t erlet ak pada sat u lingkaran maka ABCD adalah segiempat

∠ABC + ∠ADC = 180o


t ali busur. Maka berlaku

∠BAD + ∠BCD = 180o

Cont oh 43 :
ABC adalah sebuah segit iga dengan panj ang AB = 6. Dibuat sebuah lingkaran dalam yang menyinggung sisi
AB di K, sisi AC di L dan sisi BC di M (lihat gambar). Jika diket ahui panj ang LC = 5, t ent ukan keliling
segit iga ABC.

Solusi :
Perhat ikan bahwa CM = CL, BM = BK dan AL = AK
Keliling ∆ABC = BK + KA + AL + LC + CM + MB
Keliling ∆ABC = BK + KA + KA + LC + LC + BK
Keliling ∆ABC = 2(BK + KA) + 2LC
Keliling ∆ABC = 2AB + 2LC
Keliling ∆ABC = 2 ⋅ 6 + 2 ⋅ 5
Keliling ∆ABC = 22

Pada segiempat ABCD besar ∠ABC = ∠ADC = 90o. Jika ∠ADB = 35o maka ∠ACB sama dengan ⋅⋅⋅⋅⋅
Cont oh 44 :

Karena ∠ABC + ∠ADC = 180o maka ABCD adalah segiempat t alibusur sehingga t it ik-t it ik A, B, C dan D
Solusi :

semuanya t erlet ak pada sat u lingkaran.

∠ADB dan ∠ACB menghadap t alibusur yang sama yait u sisi AB sehingga kedua sudut t ersebut harus sama

Jadi, ∠ACB = 35o.


besar.

Eddy Hermanto, ST 130 Geometri


Pembinaan Olimpiade Matematika

Cont oh 45 :

CD sama dengan ⋅⋅⋅⋅⋅


(Alabama MC 2003) Talibusur AB dan CD berpot ongan di O sehingga CO = 2, AB = 8 dan OA = OB. Panj ang

Solusi :

Karena AB = 8 dan OA = OB maka OA = OB = 4

AO ⋅ OB = CO ⋅ OD
Karena t alibusur AB dan CD berpot ongan di O maka berlaku

4 ⋅ 4 = 2 ⋅ OD
OD = 8
CD = CO + OD = 2 + 8 = 10
Jadi, panj ang CD sama dengan 10.

Cont oh 46 :

dan BX bert urut -t urut memot ong lingkaran di t it ik D dan C. Diket ahui besar ∠AOB = 100o dan ∠COD = 60o.
Tit ik A dan B t erlet ak pada suat u lingkaran yang berpusat di O. Tit ik X t erlet ak di luar lingkaran. Garis AX

Besar ∠AXB adalah ⋅⋅⋅⋅⋅

Solusi :

Karena sudut keliling sama dengan set engah sudut pusat maka
∠ACB = 12 ∠AOB = 50o
Karena sudut keliling sama dengan set engah sudut pusat maka
∠CAD = 12 ∠COD = 30o
Karena ∠ACX dan ∠ACB saling berpelurus maka
∠ACX = 180o − ∠ACB = 130o
Pada ∆AXC berlaku
∠AXC + ∠ACX + ∠CAX = 180o
∠AXB + 130o + 30o = 180o
∠AXB = 20o
Jadi, besar ∠AXB adalah 20o.

Eddy Hermanto, ST 131 Geometri


Pembinaan Olimpiade Matematika

Cont oh 47 :
(OSP 2002) Garis t engah sebuah set engah lingkaran berimpit dengan alas AB dari ∆ABC. Tit ik sudut C
bergerak sedemikian rupa, sehingga t it ik t engah sisi AC selalu t erlet ak pada set engah lingkaran. Berupa
apakah lengkungan t empat kedudukan t it ik C ?

Solusi :

AB adalah diamet er dan D t erlet ak pada lingkaran. Maka ∠ADB = 90o


Karena AD = CD dan BD ⊥ AC maka ∆ABC adalah segit iga sama kaki dengan AB = BC.
Karena BC = AB = diamet er lingkaran yang berart i bernilai t et ap dan B adalah t it ik yang t et ap maka
lengkung yang t erj adi adalah berupa set engah lingkaran dengan pusat t it ik B.
Jadi, lengkung yang t erj adi adalah berupa set engah lingkaran

LATIHAN 6 :

1. Tent ukan sudut t erkecil yang dibent uk oleh j arum panj ang (menit ) dan j arum pendek (j am) pada
pukul 20 : 06.

2. (OSK 2002) Suat u persegi panj ang berukuran 8 kali 2√2 mempunyai t it ik pusat yang sama dengan
suat u lingkaran berj ari-j ari 2. Berapakah luas daerah irisan ant ara persegi panj ang dan lingkaran
t ersebut ?

3. (OSP 2004) Sant i dan Tini berlari sepanj ang sebuah lint asan yang berbent uk lingkaran. Keduanya
mulai berlari pada saat yang sama dari t it ik P, t et api mengambil arah berlawanan. Sant i berlari 1½

berpapasan unt uk pert ama kalinya di t it ik R, berapa deraj at kah besar ∠RPQ ?
kali lebih cepat daripada Tini. Jika PQ adalah garis t engah lingkaran lint asan dan keduanya

keempat sisi t rapesium dapat dibuat . Jika AB = 75 dan DC = 40, maka keliling t rapesium ABCD = ⋅⋅⋅⋅⋅
4. (OSP 2006) Pada t rapesium ABCD, sisi AB sej aj ar dengan DC. Sebuah lingkaran yang menyinggung

5. Garis AD adalah diamet er set engah lingkaran dengan M adalah t it ik t engah diamet er t ersebut . Tit ik B

Jika diket ahui ∠CAD = 50o, hit unglah sudut yang dibent uk ant ara garis AC dan BD.
dan t it ik C keduanya t erlet ak pada set engah lingkaran sedemikian sehingga garis AC t egak lurus BM.

6. (NHAC 1994-1995 Second Round) Sebuah lingkaran menyinggung bagian dalam suat u segienam

adalah ⋅⋅⋅⋅⋅⋅⋅⋅
ABCDEF. Jika diket ahui panj ang sisi-sisi AB = 1, BC = 2, CD = 3, DE = 4 dan EF = 5 maka panj ang sisi FA

7. (Canadian MO 1971) DEB adalah t ali busur suat u lingkaran dengan DE = 3 dan EB = 5. Misalkan O
adalah pusat lingkaran. Hubungkan OE dan perpanj angan OE memot ong lingkaran di t it ik C. Diket ahui
EC = 1. Tent ukan radius lingkaran t ersebut .

8. (OSP 2010) Dua lingkaran (t idak sama besar) bersinggungan di luar. Tit ik A dan A1 t erlet ak pada

singgung persekut uan kedua lingkaran t ersebut . Jika PA = AB = 4, maka luas lingkaran kecil adalah ⋅⋅⋅⋅
lingkaran kecil; sedangkan B dan B1 pada lingkaran besar. Garis PAB dan PA1B1 merupakan garis

Eddy Hermanto, ST 132 Geometri


Pembinaan Olimpiade Matematika

9. ABCD adalah persegi dengan panj ang sisi 9. Tit ik P t erlet ak pada sisi AB sehingga AP : PB = 7 : 2.
Sebuah seperempat lingkaran dibuat dengan C sebagai t it ik pusat dan CB j ej arinya. Dari t it ik P dibuat

Panj ang QD adalah ⋅⋅⋅⋅⋅⋅


sebuah garis yang menyinggung seperempat lingkaran t ersebut dan memot ong sisi AD di t it ik Q.

10. (OSP 2011) Misalkan ABC suat u segit iga dan P t it ik di dalam segit iga. Misalkan D, E, F bert urut -t urut

AB. Jika segit iga DEF sama sisi dan ∠APB = 70o, maka ∠ACB = ⋅⋅⋅⋅⋅
t it ik di sisi-sisi BC, CA, AB sedemikian sehingga PD t egaklurus BC, PE t egaklurus CA dan PF t egaklurus

11. (OSP 2011) Misalkan Γ lingkaran luar segit iga ABC. Talibusur AD adalah garis bagi ∠BAC yang
BL
memot ong BC di t it ik L. Talibusur DK t egaklurus pada AC dan memot ongnya di t it ik M. Jika LC = 12 ,
maka perbandingan AM
MC = ⋅⋅⋅⋅⋅⋅

12. Jika ∠ACB = ∠ADB maka bukt ikan bahwa dapat dibuat sebuah lingkaran yang melalui A, B, C dan D.

13. LM adalah t ali busur suat u lingkaran dengan K adalah pert engahan LM. Dari t it ik K dibuat garis yang
memot ong lingkaran di t it ik D dan J. Dengan DJ sebagai diamet er dibuat set engah lingkaran. Sebuah
garis melalui t it ik K dan t egak lurus DJ memot ong set engah lingkaran di t it ik S. Bukt ikan bahwa
panj ang KS = KL.

14. (Canadian MO 1975) Tit ik-t it ik A, B, C dan D bert urut -t urut t erlet ak pada sat u buah lingkaran pada
arah put aran yang sama. Tit ik-t it ik P, Q, R dan S bert urut -t urut pert engahan busur-busur AB, BC, CD
dan DA. Bukt ikan bahwa PR t egak lurus QS.

15. Tit ik P t erlet ak di luar sebuah lingkaran. Dari t it ik P dit arik sebuah garis memot ong lingkaran di t it ik
A dan B dengan PA < PB. Dari t it ik P j uga dit arik sebuah garis lain yang memot ong lingkaran di t it ik C
dan D dengan PC < PD. Sat u buah garis lagi dit arik dari t it ik P menyinggung lingkaran di t it ik T.

PA ⋅ PB = PC ⋅ PD = PT2
Bukt ikan bahwa

Eddy Hermanto, ST 133 Geometri


Pembinaan Olimpiade Matematika

7. GARIS (LANJUTAN)
Pembahasan pada bagian ini adalah melanj ut kan apa yang t elah dibahas pada bagian t ent ang garis, yait u
memanfaat kan semua yang t elah dibahas pada bagian t ent ang garis t ermasuk j uga sifat -sifat bangun
dat ar pada bidang sepert i segit iga, segiempat , segi n berat uran maupun lingkaran unt uk menghit ung
maupun membukt ikan persoalan.
Solusi yang akan dibahas pada bagian ini adalah dengan memanfaat kan koordinat -koordinat yang ada
maupun yang dit ent ukan unt uk mencapai t uj uan yang akan dicapai. Penent uan koordinat yang diambil
adalah yang paling mudah namun dengan t et ap t idak mengurangi keumuman soal.
Mengingat penent uan koordinat menj adi suat u yang cukup pent ing maka perlunya banyak lat ihan.

Cont oh 48 :
Bukt ikan bahwa kedua diagonal persegi (buj ur sangkar) saling t egak lurus.

Solusi :
Misalkan persegi t ersebut adalah ABCD. Langkah awalnya adalah menent ukan koordinat .

Hal yang paling mudah adalah dengan menganggap koordinat A(0, 0) sert a B(a, 0). Hal ini t idak
mengurangi keumuman soal sebab bagaimana pun posisi persegi kit a selalu dapat membuat sumbu
kart esian sedemikian hingga salah sat u sisi akan berhimpit dengan sumbu X . Berdasarkan pemisalan t adi,
kit a sudah menganggap bahwa panj ang sisi persegi t ersebut adalah a. Akibat pemisalan t adi, maka dua
buah sisi persegi akan sej aj ar dengan sumbu X dan dua sisi lagi t ent u saj a akan sej aj ar dengan sumbu Y.
Pemisalan koordinat A dan B t ersebut akan menyebabkan koordinat C(a, a) dan D(0, a).

Persamaan garis yang melalui B(a, 0) dan D(0, a) adalah y = −x dengan gradien, m2 = −1.
Persamaan garis yang melalui A(0, 0) dan C(a, a) adalah y = x dengan gradien, m1 = 1.

Maka akan didapat m1m2 = −1.


Sebagaimana yang t elah dibahas sebelumnya, karena m1m2 = −1 maka hal t ersebut memiliki art i garis AC
dan BD saling t egak lurus.
Jadi, t erbukt i bahwa kedua diagonal persegi (buj ur sangkar) saling t egak lurus.

Diket ahui t it ik A(1, −1), B(3, 5) dan C(7, 17). Bukt ikan bahwa ket iga t it ik A, B dan C kolinier (t erlet ak
Cont oh 49 :

pada sat u garis lurus).

Solusi :
Unt uk membukt ikan bahwa 3 t it ik t erlet ak pada sat u garis maka cukup dibukt ikan bahwa dua di ant ara
ruas garis – ruas garis t ersebut memiliki kemiringan yang sama.
Kemiringan AB, mAB = 53+−11 = 3
Kemiringan AC, mAC = 177 −+11 = 3
Jadi, t erbukt i bahwa ket iga t it ik A, B dan C kolinier (t erlet ak pada sat u garis lurus).

Cont oh 50 :

pada BC dengan AE = FB = 5. Misalkan P adalah t it ik pot ong DE dan AF. Luas DCFP adalah ⋅⋅⋅⋅⋅⋅
(OSK 2009 yang disesuaikan) Diberikan persegi ABCD dengan panj ang sisi 10. Misalkan E pada AB dan F

Eddy Hermanto, ST 134 Geometri


Pembinaan Olimpiade Matematika

Solusi :
Misalkan koordinat A(0,0), B(10,0) maka C(10,10) dan D(0,10). Maka koordinat E(5, 0) dan F(10, 5)

1
Persamaan garis AF adalah y = x.
Persamaan garis ED adalah y = −2x + 10
2

Perpot ongan garis ED dan AF adalah t it ik P maka


2 xp = −2xp + 10 didapat xP = 4 sehingga yP = 2.
1

Misalkan [ABCD] menyat akan luas bangunan ABCD.


[AEP] = 12 ⋅ AE ⋅ yP = 12 ⋅ 5 ⋅ 2 = 5
[ABF] = 1
⋅ AB ⋅ BF = 1
⋅ 10 ⋅ 5 = 25
⋅ AD ⋅ AE = ⋅ 10 ⋅ 5 = 25
2 2
1 1
[AED] =
[DCFP] = 100 − [ABF] − [AED] + [AEP] = 100 − 25 − 25 + 5
2 2

Jadi, luas bangun DCFP adalah 55.

Cont oh 51 :
Diket ahui koordinat t it ik A(0, 0), t it ik B(a, 0) dan C(b, c).
a. Tent ukan koordinat t it ik berat segit iga ABC.
b. Bukt ikan bahwa ket iga garis berat melalui sat u t it ik.
c. Misalkan t it ik berat segit iga ABC t ersebut adalah G, bukt ikan bahwa AG : GD = 2 : 1, BG : GE = 2 : 1
dan CG : GF = 2 : 1.

Solusi :

a. Misalkan AD, BE dan CF adalah garis berat segit iga ABC. Karena D, E dan F bert urut -t urut adalah
pert engahan BC, AC dan AB maka
Koordinat D( a+2 b , 2c ), koordinat E( b2 , 2c ) dan koordinat F( a2 , 0).

a +b
c
Persamaan garis AD adalah y = x.

=
−0
(x − a).
y −0
c

x−a − b−2a
2 c
Persamaan garis BE adalah b
a
yang set ara dengan y =
2

Misalkan garis AD dan BE berpot ongan di t it ik G.


a + b xG = b − 2 a (xG − a)
c c

(b − 2a)xG = (a + b)xG − a(a + b)


(a + b) = 3xG

Eddy Hermanto, ST 135 Geometri


Pembinaan Olimpiade Matematika
a+ b c
xG = 3 sehingga yG = 3 .
Maka koordinat t it ik berat segit iga adalah G( a+3 b , 3c ).
b. Unt uk membukt ikan apa yang dit anyakan dalam soal maka hanya perlu membukt ikan bahwa t it ik G

=
t erlet ak pada garis CF.
(2x − a).
y −0 c −0
x − a2 b − a2 2b − a
c
Persamaan garis CF adalah yang set ara dengan y =
a+ b c
Uj i t it ik ( 3 , 3 ) pada garis CF.
y = 2bc− a ( 2 a+ 2 b

a) = 3c
3
Jadi, t erbukt i bahwa ket iga garis berat melalui sat u t it ik.
c. Karena A(0, 0) dan D( a+2 b , 2c ) sedangkan G( a+3 b , 3c ) maka AG : GD = 2 : 1.
b c a+ b c
Karena B(a, 0) dan E( 2 , 2 ) sedangkan G( 3 , 3 ) maka BG : GE = 2 : 1.
a a+ b c
Karena C(b, c) dan F( ,0) sedangkan G(
2 3 , 3 ) maka CG : GF = 2 : 1.
Jadi, t erbukt i bahwa AG : GD = 2 : 1, BG : GE = 2 : 1 dan CG : GF = 2 : 1.

Cont oh 52 :
Pada segit iga ABC diket ahui koordinat t it ik A(0, 0), t it ik B(a, 0) dan C(b, c).
a. Tent ukan koordinat perpot ongan garis t inggi segit iga ABC.
b. Bukt ikan bahwa ket iga garis t inggi melalui sat u t it ik

Solusi :

a. Misalkan garis t inggi dari A memot ong BC di D, garis t inggi dari B memot ong AC di E dan garis t inggi
dari C memot ong AB di F.
Kemiringan garis BC adalah mBC = b −c a sehingga kemiringan AD, mAD = a−c b
Karena A(0, 0) maka persamaan garis AD adalah y = a−c b x
Karena garis AB sej aj ar sumbu X maka CF akan sej aj ar sumbu Y.
Jadi, persamaan garis CF adalah x = b.
Misalkan perpot ongan garis CF dengan AD adalah di t it ik H.
a−b ab −b 2
Maka xH = b dan yH = c xH = c .
ab −b 2
Jadi, koordinat perpot ongan garis t inggi segit iga, H(b, c ).
c
b. Kemiringan garis AC adalah mAC =
Maka kemiringan garis BE, mBE = −
b
b

Karena koordinat B(a, 0) maka persamaan garis BE adalah y = − bc (x − a)


c

Uj i t it ik H(b, ab−c b ) ke persamaan garis BE sehingga akan didapat bahwa t it ik H t erlet ak pada garis
2

BE.
Jadi, t erbukt i bahwa ket iga garis t inggi melalui sat u t it ik.

Eddy Hermanto, ST 136 Geometri


Pembinaan Olimpiade Matematika

Cont oh 53 :
Pada segit iga ABC diket ahui koordinat t it ik A(0, 0), t it ik B(a, 0) dan C(b, c).
a. Tent ukan pusat lingkaran luar segit iga ABC
b. Bukt ikan bahwa ket iga garis sumbu segit iga melalui sat u t it ik.

Solusi :
Lingkaran luar suat u segit iga merupakan pert emuan garis sumbu.

a. Misalkan garis sumbu yang t egak lurus AB memot ong AB di D, garis sumbu yang t egak lurus BC
memot ong BC di E dan garis sumbu yang t egak lurus AC memot ong AC di F.
Jelas bahwa D, E dan F bert urut -t urut adalah pert engahan AB, BC dan AC.
Koordinat koordinat D( a2 , 0), koordinat E( a+2 b , 2c ) dan koordinat F( b2 , 2c ).
a
Karena AB sej aj ar sumbu X maka persamaan garis sumbu melalui D adalah x = .
sehingga gradien garis sumbu yang melalui F, mF = −
2
c b
Gradien AC, mAc = .
Persamaan garis sumbu yang melalui F adalah y − = − (x − ).
b c
c b b
2 c 2
Misalkan perpot ongan garis sumbu melalui F dan garis sumbu melalui D adalah t it ik O yang j uga
merupakan pusat lingkaran luar segit iga ABC.
yO − 2c = − bc ( a2 − b2 )
b 2 − ab + c 2
yO = 2c
a b 2 − ab + c 2
Maka pusat lingkaran luar segit iga ABC adalah O( 2 , 2c )

b−a
c
b. Kemiringan garis BC, mBC =
a−b
Maka gradien garis sumbu yang melalui t it ik E, mE = .
Persamaan garis sumbu yang melalui t it ik E adalah y − (x −
c
c a−b a+ b
2 = c 2 ).

Uj i t it ik O( a2 , b − 2abc + c ) t erhadap garis sumbu yang melalui E sehingga akan didapat bahwa t it ik O
2 2

t erlet ak pada garis sumbu t ersebut .


Jadi, t erbukt i bahwa ket iga garis sumbu segit iga melalui sat u t it ik.

Cont oh 54 :
Pada segit iga ABC diket ahui koordinat t it ik A(0, 0), t it ik B(a, 0) dan C(b, c). Tent ukan pusat lingkaran
dalam segit iga ABC.

Solusi :
Misalkan pusat lingkaran dalam segit iga t ersebut adalah t it ik I.
Perhat ikan gambar.

Eddy Hermanto, ST 137 Geometri


Pembinaan Olimpiade Matematika

Jelas bahwa pusat lingkaran dalam segit iga ABC adalah I(xI, r) dengan r adalah j ari-j ari lingkaran dalam
segit iga ABC.

(a − b)2 + c 2
Luas segit iga ABC = ac

b2 + c2 +
2

(a − b)2 + c 2 ) =
Keliling segit iga ABC = a +

Maka 1
2 r(a + b2 + c2 + ac
2

( a − b )2 + c 2
ac
a+ b2 +c2 +
r=

Persamaan garis AC adalah by − cx = 0.


Jarak t it ik I ke garis AC sama dengan r maka
⎛ ⎞
b ⎜⎜ ⎟−c ( xI )
⎝ a+ ( a − b )2 + c 2 ⎟⎠
ac
b2 +c2 +

( a − b )2 + c 2
ac
a+ b2 +c2 + b2 +c2
=

Karena b 2 + c 2 > b dan dengan menganggap bahwa xI > 0 maka


c ( x I )−
( a −b )2 + c 2
abc
a + b2 +c2 +

(a − b )
ac
a+ b +c + +c b +c
=

(a − b)2 + c 2 )
2 2 2 2 2 2

a( b 2 + c 2 + b) = xI(a + b 2 + c 2 +
a ⎛⎜ b 2 + c 2 + b ⎞⎟
⎝ ⎠
a+ b2 +c2 + ( a − b )2 + c 2
xI =

a ⎛⎜ b 2 + c 2 + b ⎞⎟
⎝ ⎠
( a − b )2 + c 2 ( a − b )2 + c 2
ac
a+ b2 +c2 + a+ b2 +c2 +
Jadi, pusat lingkaran dalam segit iga ABC adalah I( , )
Alt ernat if lain adalah dengan memot ongkan garis bagi AI dengan BI.
Unt uk menent ukan persamaan garis AI adalah dengan mencari kemiringan garis AI yang didapat dari
kemiringan garis AC. Kemiringan garis AC, mAC = bc .
Misalkan ∠CAB = 2α maka ∠IAB = α.
tan α
α
t an 2α = mAC = bc = 1−2tan yang set ara dengan persamaan
c t an2α + 2b t an α − c = 0
2

Dengan mengambil t an α > 0 maka


t an α = mAI = − 2b + 4b 2 + 4 c 2 −b + b 2 + c 2 c
b+ b2 +c2
2c
= c
= .
Karena kemiringan garis AI diket ahui dan garis AI melalui A(0, 0) maka persamaan garis AI dapat
dit ent ukan. Dengan cara yang sama dapat dit ent ukan persamaan garis BI. Perpot ongan kedua garis AI dan
Bi adalah t it ik I yang merupakan pusat lingkaran dalam segit iga. Penulis menyerahkan kepada Pembaca
unt uk menyelesaikan alt ernat if penyelesaian ini. Hal yang perlu diperhat ikan adalah bahwa sudut
kemiringan diukur t erhadap sumbu X posit if.

Eddy Hermanto, ST 138 Geometri


Pembinaan Olimpiade Matematika
Cont oh 55 :
(OSK 2006/ Lat ihan 3D Nomor 10) Pada segit iga ABC, t it ik F membagi sisi AC dalam perbandingan 1 : 2.
Misalkan G t it ik t engah BF dan E t it ik perpot ongan ant ara sisi BC dengan AG. Maka t it ik E membagi sisi BC
dalam perbandingan

Solusi :

Tanpa mengurangi keumuman misalkan koordinat A(0, 0), C(a, 0) dan B(b, c).
Karena AF : FC = 1 : 2 maka koordinat F( 13 a, 0).
a+ 3b c
Karena G adalah pert engahan BF sedangkan koordinat B(b, c) maka koordinat G( 6 , 2 ).

a + 3b sehingga persamaan garis AG adalah y = a + 3b x.


3c 3c
Kemiringan garis AG adalah
Kemiringan garis BC adalah b − a dan melalui C(a, 0) sehingga persamaan garis BC adalah
c
y= b−a
c
(x − a).
Garis AG berpot ongan dengan BC di E sehingga
a + 3b xE = b − a (xE − a)
3c c

a + 3b
a+ 3b 3c a+ 3b 3c
xE = 4 sehingga yE = ( 4 )= 4

Karena E( a+43b , 3c4 ) sedangkan C(a, 0) dan B(b, c) maka BE : EC = 1 : 3.


Jadi, t it ik membagi sisi BC dalam perbandingan 1 : 3.

Cont oh 56 :
(OSP 2004/ Hongkong PSC 1988/ Lat ihan 4 Nomor 3) Pada sebuah t rapesium dengan t inggi 4, kedua
diagonalnya saling t egak lurus. Jika salah sat u dari diagonal t ersebut panj angnya 5, berapakah luas
t rapesium t ersebut ?

Solusi :

Tanpa mengurangi keumuman misalkan koordinat A(0, 0) dan B(a, 0). Karena t inggi t rapesium sama

mAC ⋅ mBD = −1
dengan 4 maka ordinat C dan D sama dengan 4. Misalkan koordinat C(b, 4) dan D(c, 4).

b ⋅ c− a = −1
4 4

b(a − c) = 16
• Jika panj ang AC = 5
b2 + 42 = 52 sehingga b = 3
Maka a − c = 163
( a )+ (b − c )
Luas t rapesium = AB+ CD
2 ⋅ t = 2
⋅ 4 = ( 163 + 3) ⋅ 2 = 50
3

Eddy Hermanto, ST 139 Geometri


Pembinaan Olimpiade Matematika
50
Jadi, luas t rapesium ABCD =

3

(a − c)2 + 42 = 52 sehingga a − c = 3
Jika panj ang BD = 5

Maka b = 163
( a )+ (b − c )
Luas t rapesium = AB+ CD
2 ⋅ t = 2 ⋅ 4 = ( 163 + 3) ⋅ 2 = 50
3
50
Jadi, luas t rapesium ABCD = 3
50
Jadi, luas t rapesium ABCD = 3 .

LATIHAN 7

1. Persegi panj ang ABCD memiliki ukuran AB = 10 dan BC = 5. Tit ik E t erlet ak pada sisi DC sehingga DE :
EC = 9 : 1. Tit ik F pada sisi BC sehingga BF : FC = 2 : 3. Bukt ikan bahwa garis AF t egak lurus BE.

2. Segiempat ABCG dibagi menj adi 12 buah persegi yang sama. Panj ang AB = 4 dan panj ang BC = 3.
Berapakah luas irisan ant ara segiempat DEFG dengan segit iga ABC ?

3. Pada persegi ABCD dengan panj ang sisi k t erdapat t it ik P dan Q yang masing-masing pada sisi AB dan
BC sedemikian sehingga AP : PB = 2 : 1 dan BQ : QC = 3 : 1. Garis DP dan garis AQ berpot ongan di t it ik
R. Hit unglah luas segit iga ARP dinyat akan dalam k.

4. (OSP 2003) Suat u garis vert ikal membagi segit iga dengan t it ik sudut (0,0), (1,1) dan (9,1) menj adi dua
daerah dengan luas yang sama. Apakah persamaan garis t ersebut ?

5. Diberikan persegi panj ang (siku empat ) ABCD dengan AB = a dan BC = b. Tit ik O adalah perpot ongan
ant ara kedua diagonalnya. Perpanj ang sisi BA sehingga sehingga AE = AO, j uga perpanj ang diagonal DB
sehingga BZ = BO. Asumsikan segit iga EZC samasisi. Bukt ikan bahwa
i. b = a 3
ii. EO t egak lurus ZD

AC di t it ik D. Jika t it ik E dan F bert urut -t urut adalah t it ik t engah BD dan CD, bukt ikan bahwa AE ⊥ BF.
6. (OSP 2006/ Lat ihan 3B Nomor 5) Misalkan segit iga ABC siku-siku di B. Garis t inggi dari B memot ong sisi

7. (OSP 2010) Diberikan segit iga ABC. Andaikan P dan P1 t it ik-t it ik pada BC, Q pada CA, dan R pada AB,
sedemikian rupa sehingga

= = =
AR BP CQ CP1
RB PC QA P1 B
Misalkan G t it ik berat segit iga ABC dan K = AP1 ∩ RQ. Bukt ikan, bahwa t it ik-t it ik P, G, dan K kolinier
(t erlet ak pada sat u garis)

8. (Bulgarian MO 1995 : Spring MC Grade 8/ Lat ihan 4 No 9) Misalkan M adalah t it ik t engah sisi BC pada
j aj aran genj ang ABCD sedangkan N adalah perpot ongan AM dan diagonal BD. Perpanj angan DA dan CN
berpot ongan di t it ik P.

Eddy Hermanto, ST 140 Geometri


Pembinaan Olimpiade Matematika
a. Bukt ikan bahwa AP = AD
b. Jika AB = AC maka bukt ikan CP = BD

9. Lingkaran L memiliki pusat di M. Tit ik A t erlet ak di luar lingkaran. Dari t it ik A dibuat garis yang
memot ong lingkaran di t it ik B dan C dengan B t erlet ak di ant ara A dan C. Lalu dibuat garis CM yang
pepanj angannya memot ong lingkaran di t it ik D. Ternyat a AD menyinggung lingkaran. Tit ik E t erlet ak
pada ruas AD sehingga AE = 1. Jika panj ang AB = 2 dan BC = 6, maka bukt ikan bahwa garis AM, BD dan
CE berpot ongan di sat u t it ik.

10. (AIME 1983/ Hongkong PSC 1988) Dua lingkaran yang masing-masing berj ari-j ari 8 dan 6 mempunyai
j arak ant ar pusat 12. Melalui t it ik P yang merupakan salah sat u t it ik perpot ongan kedua lingkaran
dibuat t ali busur PQ dan PR dengan Q, P, R segaris. Jika PQ = PR, t ent ukan PQ2.

11. Tit ik D dan E bert urut -t urut t erlet ak pada sisi BC dan AC sehingga AD dan BE adalah garis t inggi.
Bukt ikan bahwa garis yang menyinggung lingkaran luar ∆ABC di t it ik C akan sej aj ar dengan garis yang
melalui DE.

Eddy Hermanto, ST 141 Geometri


Pembinaan Olimpiade Matematika

BAB IV
KOMBINATORIK

1. KAIDAH PENCACAHAN DAN PENJABARAN BINOM NEWTON


Ada beberapa ilust rasi persoalan yang berhubungan dengan cara yang mungkin t erj adi sepert i sebagai
berikut :

Cont oh 1 :
Misalkan t erdapat 3 buah celana dan 4 buah baj u. Permasalahannya adalah ada berapa banyak cara
seseorang memilih celana dan baj u yang akan dipakai ?

Cont oh 2 :
Misalkan ada 3 buku : Mat emat ika, Fisika dan Biologi. Jika seseorang ingin menumpuk dua buku secara
vert ikal, ada berapa cara ia melakukan penumpukan ?

Masalah-masalah t ersebut dapat diselesaikan dengan Kaidah Pencacahan yang dapat dit empuh dengan
menggunakan sat u at au beberapa cara berikut :
- at uran pengisian t empat (f illing slot )
- permut asi
- kombinasi

A. Aturan pengisian tempat (filling slot s)


Misalkan ada n t empat t ersedia dengan k1 adalah banyaknya cara mengisi t empat pert ama, k2 adalah

t empat ke-n. Maka banyaknya cara mengisi t empat adalah k1 x k2 x ⋅⋅⋅ x kn.
banyaknya cara mengisi t empat kedua, dan set erusnya hingga kn adalah banyaknya cara mengisi

Cara ini disebut sebagai at uran pengisian t empat dan sering disebut dengan kaidah perkalian.

Sebagai ilust rasi penyelesaian soal cont oh 1 adalah sebagai berikut :


Tempat pert ama adalah memilih celana. Karena banyaknya celana ada 3, maka banyaknya cara

pasangan celana dan baj u ada 3 ⋅ 4 = 12 cara.


memilih celana ada 3 sedangkan banyaknya cara memilih baj u ada 4. Maka banyaknya cara memilih

Unt uk soal pada cont oh 2, banyaknya cara memilih t empat pert ama ada 3 cara karena bukunya ada 3.

pert ama. Banyaknya cara memilih dua buku adalah 3 ⋅ 2 = 6 cara.


Unt uk memilih buku yang kedua hanya t inggal 2 cara karena sat u buku sudah dipilih pada t empat

Cont oh 3 :
Berapa banyak cara menyusun huruf-huruf R, A, J, I, N j ika
a) huruf pert ama dimulai dari huruf hidup (vokal)
b) huruf pert ama dimulai dari huruf mat i (konsonan)

Solusi :
a) Banyaknya cara memilih huruf pert ama ada 2 yait u A at au I. Karena huruf A at au I sudah dipakai
sebagai huruf pert ama maka banyaknya cara memilih huruf kedua t inggal 4 cara. (Misalkan huruf
pert ama adalah A maka kemungkinan huruf kedua ada 4 yait u R, J, I at au N.) Banyaknya cara
memilih huruf ket iga ada 3 cara, huruf keempat ada 2 cara dan huruf kelima t inggal 1 cara.
Banyaknya cara menyusun huruf t ersebut ada 2 x 4 x 3 x 2 x 1 = 48 cara.
b) Banyaknya cara memilih huruf pert ama ada 3 yait u R, J at au N. Banyaknya cara memilih huruf
kedua, ket iga, keempat dan kelima bert urut -t urut ada 4, 3, 2, dan 1 cara.
Banyaknya bilangan yang dapat dibent uk ada 3 x 4 x 3 x 2 x 1 = 72 cara.

Eddy Hermanto, ST 142 Kombinatorik


Pembinaan Olimpiade Matematika
Cont oh 4 :
Sembilan orang siswa akan duduk pada 5 kursi sej aj ar. Ada berapa cara susunan mereka ?

Solusi :
Kursi pert ama ada 9 kemungkinan. Karena seorang siswa t idak akan mungkin duduk pada 2 kursi
dalam wakt u yang bersamaan maka banyaknya kemungkinan yang duduk pada kursi kedua t inggal 8.
Dan set erusnya.
Banyaknya cara susunan mereka adalah 9 x 8 x 7 x 6 x 5 = 15120.

Cont oh 5 :
Dari lima angka 0, 3, 4, 5, 7 akan dibent uk sebuah bilangan yang t erdiri dari 4 angka. Berapa banyak
bilangan yang dapat dibent uk j ika :
a) angka-angkanya boleh berulang
b) angka-angkanya t idak boleh berulang

Solusi :
a) Angka pert ama sebagai ribuan dapat dipilih 4 kemungkinan yait u 3, 4, 5 at au 7. Angka 0 t idak
mungkin menj adi angka pert ama sebab akan menyebabkan bilangan yang dibent uk hanya t erdiri
dari 3 angka. Karena boleh berulang maka angka rat usan, puluhan dan sat uan masing-masing
dapat dipilih 5 kemungkinan.
Banyaknya bilangan yang t erbent uk ada 4 x 5 x 5 x 5 = 500 bilangan.
b) Angka pert ama sebagai ribuan dapat dipilih 4 cara. Karena t idak boleh berulang sedangkan sat u
angka sudah dipakai pada angka pert ama maka banyaknya cara memilih angka kedua hanya
t inggal 4 cara. (Misalkan angka pert ama dipilih 3 maka pilihan pada angka kedua adalah 0, 4, 5
at au 7.) Banyaknya pilihan pada angka ket iga ada 3 cara dan banyaknya pilihan pada angka
keempat ada 2 cara.
Banyaknya bilangan yang dapat dibent uk ada 4 x 4 x 3 x 2 = 96 bilangan.

Cont oh 6 :
Denny akan membent uk bilangan genap 3 angka yang angka-angkanya diambil dari 2, 3, 4, 5, 6, 7, 8.
Berapa banyak bilangan yang dapat dibent uk j ika :
a) angka-angkanya boleh berulang
b) angka-angkanya t idak boleh berulang

Solusi :
a) Angka pert ama sebagai rat usan dapat dipilih 7 kemungkinan. Angka kedua dapat dipilih j uga dari
7 kemungkinan. Karena bilangan t ersebut genap maka angka sat uan hanya dapat dipilih dari 4
kemungkinan yait u 2, 4, 6 at au 8.
Banyaknya bilangan yang t erbent uk ada 7 x 7 x 4 = 196 bilangan.
b) Sebuah bilangan dikat akan genap at au ganj il cukup dengan melihat angka sat uannya. Karena
bilangan t ersebut genap maka pemilihan pert ama dilakukan pada angka sat uan. Angka sat uan
dapat dipilih dari 4 kemungkinan, yait u 2, 4, 6 at au 8. Angka puluhan dapat dipilih dari 6 cara
sedangkan angka rat usan dapat dipilih dari 5 kemungkinan.
Banyaknya bilangan yang dapat dibent uk ada 5 x 6 x 4 = 120 bilangan.

Cont oh 7 :
Dari t uj uh angka 1, 3, 5, 6, 7, 8, 9, Furkan akan membent uk sebuah bilangan 3 angka dan lebih dari
600. Berapa banyak bilangan yang dapat dibent uk j ika :
a) angka-angkanya boleh berulang
b) angka-angkanya t idak boleh berulang

Eddy Hermanto, ST 143 Kombinatorik


Pembinaan Olimpiade Matematika
Solusi :
Sebuah bilangan 3 angka dikat akan lebih dari 600 j ika digit rat usan sekurang-kurangnya 6.
a) Angka pert ama sebagai rat usan dapat dipilih 4 kemungkinan, yait u 6, 7, 8 at au 9. Angka kedua
dapat dipilih j uga dari 7 kemungkinan. Angka sat uan j uga dapat dipilih dari 7 kemungkinan.
Banyaknya bilangan yang t erbent uk ada 4 x 7 x 7 = 196 bilangan.
b) Angka pert ama sebagai rat usan dapat dipilih 4 kemungkinan, yait u 6, 7, 8 at au 9. Angka puluhan
dapat dipilih dari 6 cara sedangkan angka sat uan dapat dipilih dari 5 kemungkinan.
Banyaknya bilangan yang dapat dibent uk ada 4 x 6 x 5 = 120 bilangan.

Cont oh 8 :
Hansen mendapat kan t ugas membent uk sebuah bilangan t iga angka kurang dari 500 yang angka-
angkanya adalah 2, 3, 4, 5, 6, 7 at au 9. Berapa banyak bilangan yang dapat dibent uk j ika :
a) angka-angkanya boleh berulang
b) angka-angkanya t idak boleh berulang

Solusi :
Sebuah bilangan 3 angka dikat akan kurang dari 500 j ika digit rat usan kurang dari 5.
a) Angka pert ama sebagai rat usan dapat dipilih 3 kemungkinan, yait u 2, 3 at au 4. Angka kedua dapat
dipilih dari 7 kemungkinan. Angka sat uan j uga dapat dipilih dari 7 kemungkinan.
Banyaknya bilangan yang t erbent uk ada 3 x 7 x 7 = 147 bilangan.
b) Angka pert ama sebagai rat usan dapat dipilih 3 kemungkinan, yait u 2, 3 at au 4. Angka puluhan
dapat dipilih dari 6 cara sedangkan angka sat uan dapat dipilih dari 5 kemungkinan.
Banyaknya bilangan yang dapat dibent uk ada 3 x 6 x 5 = 90 bilangan.

Cont oh 9 :
Dari t uj uh angka 1, 3, 4, 5, 6, 8, 9 akan dibent uk sebuah bilangan 3 angka dan lebih dari 500. Berapa
banyak bilangan genap yang dapat dibent uk j ika :
a) angka-angkanya boleh berulang
b) angka-angkanya t idak boleh berulang

Solusi :
a) Angka pert ama sebagai rat usan dapat dipilih 4 kemungkinan, yait u 5, 6, 8 at au 9. Angka kedua
dapat dipilih dari 7 kemungkinan. Angka sat uan dapat dipilih dari 3 kemungkinan.
Banyaknya bilangan yang t erbent uk ada 4 x 7 x 3 = 84 bilangan.
b) Pada bagian inilah t imbul sebuah permasalahan.
Jika kit a menj awab banyaknya bilangan adalah 4 x 5 x 3 dengan alasan bahwa banyaknya bilangan
yang mungkin unt uk angka rat usan ada 4 dan angka sat uan ada 3 sedangkan sisa bilangan t inggal 5
maka j awaban t ersebut adalah keliru. Jika angka rat usan yang dipilih adalah 5 at au 9 maka
banyaknya kemungkinan angka sat uan memang benar ada 3 yait u 4, 6 at au 8. Tet api bila angka
rat usan yang dipilih adalah 6 at au 8 maka angka sat uan yang mungkin dipilih hanya t inggal 2.
Sedangkan j ika kit a menj awab banyaknya bilangan adalah 4 x 5 x 2 j uga mengandung kesalahan
dengan alasan bahwa j ika angka rat usan yang kit a pilih adalah 5 at au 9 maka kemungkin angka
sat uan yang dipilih adalah t et ap 3. Lalu bagaimana cara kit a menj awab soal ini ?
Ada dua alt ernat if yang akan dibahas.

Alt ernat if 1 :
Sudah dij elaskan bahwa banyaknya kemungkinan unt uk angka rat usan ada 4 namun pemilihan
angka rat usan t ernyat a menimbulkan dampak yang berbeda unt uk angka sat uan. Maka
penyelesaian soal ini adalah dengan membagi kasus t erhadap pemilihan angka rat usan.
Kasus pert ama adalah j ika angka rat usannya adalah 5 at au 9. Banyaknya cara memilih angka
rat usan ada 2. Banyaknya kemungkinan angka sat uan t et ap ada 3 sedangkan angka puluhan
t inggal 5 kemungkinan. Banyaknya bilangan unt uk kasus pert ama ini adalah 2 x 5 x 3 = 30
bilangan.

Eddy Hermanto, ST 144 Kombinatorik


Pembinaan Olimpiade Matematika
Kasus kedua adalah j ika angka rat usannya adalah 6 at au 8. Banyaknya cara memilih angka rat usan
ada 2, yait u 6 at au 8 t ersebut . Banyaknya kemungkinan angka sat uan t inggal 2. Penj elasannya
adalah j ika angka rat usan yang dipilih adalah 6 maka kemungkinan angka sat uannya adalah 4 at au
8 sedangkan j ika angka rat usan yang dipilih adalah 8 maka kemungkinan angka sat uannya adalah 4
at au 6. Sedangkan angka puluhan t inggal 5 kemungkinan. Banyaknya bilangan unt uk kasus kedua
ini adalah 2 x 5 x 2 = 20 bilangan.
Banyaknya bilangan yang dapat dibent uk ada 30 + 20 = 50 bilangan.

Alt ernat if 2 :
Caranya sebenarnya sama dengan alt ernat if 1, t et api kit a memulainya dari angka sat uan.
Kit a bagi kasus pemilihan angka sat uan menj adi 2 kasus.
Kasus pert ama adalah j ika angka sat uan yang dipilih adalah 4. Banyaknya cara memilih hanya ada
1. Angka rat usan yang dipilih t et ap ada 4 kemungkinan yait u 5, 6, 8 at au 9. Sedangkan angka
puluhan t inggal 5 kemungkinan. Banyaknya bilangan unt uk kasus pert ama ini adalah 1 x 5 x 4 = 20
bilangan.
Kasus kedua adalah j ika angka sat uan yang dipilih adalah 6 at au 8. Banyaknya cara memilih ada 2.
Angka rat usan yang dipilih t inggal 3 kemungkinan. Sedangkan angka puluhan t inggal 5
kemungkinan. Banyaknya bilangan unt uk kasus kedua ini adalah 2 x 5 x 3 = 30 bilangan.
Banyaknya bilangan yang dapat dibent uk ada 20 + 30 = 50 bilangan.

LATIHAN 1.A

1. Berapa banyak cara menyusun huruf-huruf K, A, N, T, O, R j ika


a. huruf pert ama dimulai dari huruf hidup (vokal)
b. huruf pert ama dimulai dari huruf mat i (konsonan)

2. Tuj uh orang siswa akan duduk pada 7 kursi sej aj ar. Ada berapa cara susunan mereka ?

3. Suat u keluarga t erdiri dari 2 orang put era dan 3 orang put eri. Apabila kelima orang t ersebut
berdiri sej aj ar dengan posisi yang put ra selalu mengapit yang put ri, maka ada berapa formasi
yang mungkin ?

4. Sebagai panit ia perlombaan sepakbola, Furkan mencoba menyusun t uj uh buah bendera A, B, C, D,


E, F dan G pada posisi sej aj ar. Ada berapa banyak cara penyusunan j ika diinginkan bendera A dan
B berada di uj ung ?

5. Weki mencoba membent uk sebuah bilangan 3 angka dengan angka-angkanya t idak boleh ada yang
sama dan angka-angka t ersebut diambil dari 3, 5, 6, 7, 8 dan 9. Ada berapa banyak bilangan yang
dapat dibent uk ?

6. Ada berapa banyak bilangan posit if genap t erdiri dari 5 angka berbeda dapat dibuat j ika t idak ada
sat u pun angka 5 sert a angka ribuan harus angka 0 ?

7. Pada sebuah klub dansa, t erdapat 6 laki-laki dan 6 perempuan yang akan melakukan lat ihan.
Dalam lat ihan ini, laki-laki harus dipasangkan dengan perempuan. Ada berapa banyak carakah
membent uk 6 pasangan ini ?

8. Edwin sedang menyusun suat u bilangan t iga angka dengan angka-angka : 1, 2, 3, 4, 5, 7, 8. Jika
bilangan it u t idak memuat angka yang sama, maka ada berapa banyaknya bilangan yang dapat
dibent uk dengan syarat bilangan t ersebut genap ?

bilangan-bilangan t ersebut yang t erlet ak ant ara 300 dan 800 ada sebanyak ⋅⋅⋅⋅⋅⋅⋅⋅⋅
9. Dari angka 3, 5, 6, 7 dan 8 dibuat bilangan yang t erdiri dari t iga angka berbeda. Di ant ara

Eddy Hermanto, ST 145 Kombinatorik


Pembinaan Olimpiade Matematika

10. Dari angka-angka : 1, 2, 4, 5, 7, 8 akan disusun suat u bilangan yang t erdiri at as 3 angka. Jika
bilangan it u t idak memuat angka yang sama, maka ada berapa banyaknya bilangan yang dapat
dibent uk dengan syarat bilangan t ersebut lebih dari 245 ?

11. (OSK 2003) Ada berapa banyak bilangan 4-angka (digit ) yang semua angkanya genap dan bukan
merupakan kelipat an 2003 ?

12. Sebuah bilangan 4 angka dibent uk dengan 3 angka di ant aranya adalah 3, 4 dan 6. Jika keempat
angkanya berbeda sert a bilangan t ersebut habis dibagi 3, maka ada berapa bilangan yang dapat
dibent uk ?

13. Hansen sedang membent uk sebuah bilangan 3 angka kurang dari 600 yang angka-angkanya diambil
dari 0, 3, 4, 6, 7, 8 dan 9. Ada berapa banyak bilangan yang dapat dibent uk j ika :
a. angka-angkanya boleh berulang
b. angka-angkanya t idak boleh berulang

14. Ada berapa banyak bilangan yang dapat dibent uk oleh Furkan dengan syarat : bilangan t ersebut 4
angka, lebih dari 4000 dan angka-angkanya diambil dari 0, 1, 3, 4, 5, 6, 7, dan 9 ?
a. angka-angkanya boleh berulang
b. angka-angkanya t idak boleh berulang

15. Put u Wira sedang merancang sebuah bendera 6 st rip vert ikal. Warna masing-masing st rip vert ikal
harus menggunakan sebagian at au keseluruhan warna kuning, hij au, biru at au merah. Dalam
berapa banyak rancangan ini dapat dibuat bila dua st rip berdekat an t idak boleh berwarna sama ?

16. Semua susunan huruf-huruf yang t erdiri dari 3 angka disusun. Jika AAA berada pada urut an
pert ama sedangkan AAB pada urut an kedua maka berada pada urut an berapakah susunan huruf
OSN ?

17. Dari angka-angka : 1, 2, 3, 4, 5, 7, 8 akan disusun suat u bilangan ganj il yang t ediri at as 3 angka.
Jika bilangan it u t idak memuat angka yang sama dan kurang dari 500, maka ada berapa
banyaknya bilangan yang dapat dibent uk ?

18. Ada berapa banyak bilangan genap 3 angka, angka-angkanya t idak berulang dan kurang dari 600
dapat dibent uk j ika angka-angkanya diambil dari 0, 1, 2, 3, 4, 6, 7, 8 ?

19. (OSK 2004) Nomor polisi mobil-mobil di suat u daerah selalu t erdiri dari 4 angka. Jika j umlah
keempat angka pada set iap nomor j uga harus genap sert a angka 0 t idak boleh menj adi angka
pert ama, maka ada berapa banyak sist em penomoran mobil yang dapat dibent uk ?

20. Buah bent eng pada permainan cat ur mempunyai kemampuan unt uk bergerak at au ” memakan”
buah lawan pada pet ak-pet ak yang berada pada sat u garis horizont al at au sat u garis vert ikal
dengan dirinya. Hansen mencoba menyusun 3 buah bent eng yang ket iganya dianggap berbeda
warna pada papan cat ur 8 x 8 sehingga ket iga bent eng t ersebut t idak saling ” makan” . Ada berapa
cara penyusunan yang dapat dilakukannya ?

21. Pada suat u t urnamen diikut i oleh 6 t im dengan sist em pert andingan sebagai berikut : Tim F
melawan Tim E dan t im yang kalah sebagai j uara 6 sedangkan yang menang menghadapi t im D.
Yang kalah sebagai j uara 5 sedangkan pemenangnya menghadapi t im C. Yang kalah sebagai j uara
4 sedangkan pemenangnya menghadapi t im B. Yang kalah sebagai j uara 3 sedangkan
pemenangnya menghadapi t im A. Yang kalah sebagai j uara 2 sedangkan pemenangnya j uara 1.
Ada berapa banyak susunan j uara yang dapat dibuat ?

Eddy Hermanto, ST 146 Kombinatorik


Pembinaan Olimpiade Matematika

B. Permutasi
Sebelum dibahas lebih j auh mengenai permut asi, akan diperkenalkan t erlebih dahulu definisi dan
not asi fakt orial.
Unt uk set iap n bilangan asli, didefinisikan :

n! = 1 x 2 x 3 x ⋅⋅⋅ x (n − 2) x (n − 1) x n

Not asi n! dibaca n fakt orial


Dedefinisikan j uga 1! = 1 dan 0! = 1.

n! = n x (n − 1)! = n x (n − 1) x (n − 2)! = n x (n − 1) x (n − 2) x (n − 3)! dan set erusnya.


Sebagai ilust rasi 2! = 1 x 2 = 2, 3! = 1 x 2 x 3 = 6, 4! = 1 x 2 x 3 x 4 = 24, 5! = 120.

Permut asi r obyek yang diambil dari n obyek berbeda, dengan r ≤ n adalah nPr yang didefinisikan
1) Permut asi dari Unsur-unsur Yang Berbeda

dengan :

Pr = ⋅⋅⋅⋅⋅⋅⋅⋅⋅⋅⋅⋅⋅⋅⋅⋅⋅⋅⋅⋅⋅⋅⋅⋅⋅⋅⋅⋅⋅⋅⋅
n!
(n − r )!
n (1.B.1.1)

Perhat ikan bahwa dalam permut asi urut an sangat diperhat ikan. Ini berbeda dengan kombinasi
yang t idak memperhat ikan urut an yang nant inya akan dibahas pada bagian lain.

Cont oh 10 :
Misalkan dari huruf-huruf P, Q dan R akan dibuat susunan yang t erdiri dari 3 huruf maka ada
berapa banyak susunan yang dapat dibuat ?

Solusi :
Dengan at uran pengisian t empat yang t elah dipelaj ari sebelumnya dapat diket ahui bahwa
banyaknya susunan adalah 3 x 2 x 1 = 6 susunan.
Jika kit a daft arkan sat u-sat u maka susunan t ersebut adalah PQR, PRQ, QPR, QRP, RPQ, RQP
yang semuanya ada 6 susunan.
Perhat ikan bahwa PQR dan PRQ hanya menggunakan huruf P, Q dan R. Tet api kedua susunan
t ersebut dianggap berbeda karena urut annya diperhat ikan. Maka kit a dapat menggunakan
permut asi 3 unsur yang diambil dari 3 unsur.
Banyaknya susunan 3P3 = (3−3!3 )! = 03!! = 6 susunan sebab 3! = 6 dan 0! = 1.

Cont oh 11 :
Misalkan dari huruf-huruf A, B, C dan D akan dibuat susunan yang t erdiri dari 2 huruf maka ada
berapa banyak susunan yang dapat dibuat ?

Solusi :
Dengan at uran pengisian t empat , banyaknya susunan adalah 4 x 3 = 12 susunan.
Jika kit a daft arkan sat u-sat u maka susunan t ersebut adalah AB, AC, AD, BA, BC, BD, CA, CB, CD,
DA, DB dan DC yang semuanya ada 12 susunan.
Perhat ikan bahwa AB dan BA hanya menggunakan huruf A dan B. Tet api kedua susunan t ersebut
dianggap berbeda karena urut annya diperhat ikan. Maka kit a dapat menggunakan permut asi 2
unsur yang diambil dari 4 unsur.
Banyaknya susunan 4P2 = (4 −4!2 )! = 42!! = 12 susunan sebab 4! = 24 dan 2! = 2.

Eddy Hermanto, ST 147 Kombinatorik


Pembinaan Olimpiade Matematika
Cont oh 12 :
Ada berapa banyak bilangan t erdiri dari 3 angka berbeda yang disusun dari angka-angka 1, 2, 3,
4 dan 5 ?

Solusi :
Dari bahasa soal, kit a dapat menarik kesimpulan bahwa j ika menggunakan at uran pengisian
t empat maka bilangan t ersebut masuk dalam kat egori t idak berulang (lihat penj elasan
sebelumnya).
Karena 123 dianggap berbeda dengan 231 maka berart i urut an diperhat ikan.
Kit a dapat menghit ung banyaknya susunan dengan permut asi 3 unsur yang diambil dari 5 unsur
berbeda.
Banyaknya bilangan adalah 5P3 = 60 bilangan.

Cont oh 13 :
Berapa banyak bilangan dapat dibent uk dari sebagian at au semua angka 2, 3, 4, 5 j ika t idak
boleh ada angka yang diulang ?

Solusi :
Bilangan yang digunakan dapat hanya t erdiri dari 1, 2, 3 at au 4 angka.
Jika bilangan t ersebut t erdiri dari 1 angka saj a maka banyaknya bilangan = 4P1.
Jika bilangan t ersebut t erdiri dari 2 angka saj a maka banyaknya bilangan = 4P2.
Jika bilangan t ersebut t erdiri dari 3 angka saj a maka banyaknya bilangan = 4P3.
Jika bilangan t ersebut t erdiri dari 4 angka maka banyaknya bilangan = 4P4.
Banyaknya bilangan dapat dibent uk adalah 4P1 + 4P2 + 4P3 + 4P4 = 4 + 12 + 24 + 24 = 64

Cont oh 14 :
Ada berapa banyak cara memilih 3 orang siswa dari 8 orang siswa yang akan dit unj uk sebagai
Ket ua, Wakil Ket ua dan Sekret aris ?

Solusi :
Perhat ikan bahwa pada soal ini urut an diperhat ikan. Misalkan 3 siswa yang t erpilih adalah A, B
dan C. Dengan int uisi, j elas bahwa j ika A sebagai Ket ua, B sebagai Wakil Ket ua dan C sebagai
Sekret aris berbeda susunannya j ika A sebagai Ket ua, C sebagai Wakil Ket ua dan B sebagai
Sekret aris.
Maka penyelesaian soal ini dapat dilakukan dengan permut asi 3 obyek yang dipilih dari 8 obyek.
Banyaknya cara adalah 8P3 = (8 −8!3 )! = 8 x 7 5x!6 x 5! = 8 x 7 x 6 = 336 cara.

Cont oh 15 :
Sembilan orang siswa akan duduk pada lima kursi sej aj ar. Ada berapa cara susunan yang dapat
mereka buat ?

Solusi :
Banyaknya susunan adalah 9P5 = 15120.

2) Permut asi Yang Memuat Beberapa Unsur Yang Sama


Pada cont oh 10, huruf-huruf yang disediakan semuanya berbeda yait u P, Q dan R. Bagaimana
j ika huruf-huruf yang disediakan ada yang sama ? Misalkan pada cont oh berikut :

Cont oh 16 :
Misalkan dari huruf-huruf P, P dan Q akan dibuat susunan yang t erdiri dari 3 huruf maka ada
berapa banyak susunan yang dapat dibuat ?

Eddy Hermanto, ST 148 Kombinatorik


Pembinaan Olimpiade Matematika
Solusi :
Kit a t idak bisa langsung menj awab bahwa banyaknya susunan adalah 3P3 = 6 karena dalam
kenyat aannya banyaknya susunan hanya ada 3, yait u PPQ, PQP dan QPP.

yang sama dengan k + m + p ≤ n dit ent ukan dengan rumus


Banyaknya permut asi n unsur yang memuat k unsur yang sama, m unsur yang sama dan p unsur

P= ⋅⋅⋅⋅⋅⋅⋅⋅⋅⋅⋅⋅⋅⋅⋅⋅⋅⋅⋅⋅⋅⋅⋅⋅⋅⋅⋅⋅⋅⋅⋅
n!
k!⋅m!⋅ p!
(1.B.2.1)

Pada cont oh 16, ada 3 unsur yait u P, P dan Q dengan t erdapat 2 unsur P yang sama maka
banyaknya susunan adalah 32!! = 3.

Cont oh 17 :
Ada berapa banyak susunan yang dapat dibent uk dari huruf-huruf T, E, R, C, E, C, E, R?

Solusi :
Banyaknya unsur ada 8 dengan t erdapat 3 huruf E yang sama, 2 huruf R yang sama dan 2 huruf C
yang sama, maka banyaknya susunan = 3!⋅28!!⋅2! = 8(3x 7x 2x 6x1x)(5 2x 4x1x)(32x 2x1x)1 = 1680 susunan.

Cont oh 18 :
Ada berapa banyak bilangan 7 angka berbeda yang dapat dibent uk dengan cara mengubah
susunan angka dari 3464383 (t ermasuk bilangan 3464383 it u sendiri) ?

Solusi :
Banyaknya angka ada 7 dengan t erdapat angka 3 muncul 3 kali dan angka 4 muncul 2 kali.
Maka banyaknya bilangan yang dapat dibent uk adalah = 3!7⋅2! ! = 420.

Cont oh 19 :
Perhat ikan gambar.

Jika seseorang akan berj alan dari t it ik A ke t it ik B. Ada berapa banyak cara j alan t erpendek
yang dapat dipilihnya ?

Solusi :
Jalan t erpendek yang bisa dipilih orang t ersebut adalah j ika ia memilih j alan ke kanan at au ke
at as t anpa berj alan ke kiri at au ke bawah.
Misalkan j ika berj alan ke kanan diberi t anda 1 dan j ika ke at as diberi t anda 2.
Jadi j ika 12111211212 maka orang t ersebut berj alan ke kanan lalu ke at as lalu ke kanan t iga
kali lalu ke at as lalu ke kanan dua kali lalu ke at as lalu ke kanan lalu ke at as.

Eddy Hermanto, ST 149 Kombinatorik


Pembinaan Olimpiade Matematika
Maka persoalan di at as adalah mencari banyaknya susunan 12111211212 yang merupakan
permut asi berulang 11 angka dengan t erdapat 7 angka 1 yang sama dan 4 angka 2 yang sama.
Banyaknya susunan adalah 711 !⋅4! = 330.
!

Jadi, banyaknya cara j alan t erpendek yang dapat dipilih orang t ersebut adalah 330.

3) Permut asi Siklis


Bagaimana j ika t erdapat beberapa orang yang duduk dalam suat u lingkaran (siklis) ? Ada berapa
cara menyusun semuanya ? Persoalan inilah yang berhubungan dengan permut asi siklis.
Misalkan t ersedia n unsur yang berbeda.
Banyaknya permut asi siklis dari n unsur t ersebut dirumuskan dengan :

P(siklis) = (n − 1)! ⋅⋅⋅⋅⋅⋅⋅⋅⋅⋅⋅⋅⋅⋅⋅⋅⋅⋅⋅⋅⋅⋅⋅⋅⋅⋅⋅⋅⋅⋅⋅ (1.B.3.1)

Cont oh 20 :
Jika t erdapat t iga orang yang duduk pada t iga kursi yang membent uk suat u lingkaran, maka ada
berapa banyak susunan yang mungkin t erj adi ?

Solusi :
Jika mereka duduk pada kursi yang sej aj ar maka dengan kaidah perkalian didapat banyaknya
susunan adalah 3 x 2 x 1 = 6. At au j ika dengan permut asi didapat 3P3 = 6.
Tet api karena kursi yang mereka duduki membent uk lingkaran maka hal t ersebut berbeda.

Perhat ikan gambar di bawah!

Gambar 1 Gambar 2

Pada gambar 1, j ika kit a membacanya searah j arum j am maka :


a. Jika A sebagai urut an pert ama maka didapat susunan ABC
b. Jika B sebagai urut an pert ama maka didapat susunan BCA
c. Jika C sebagai urut an pert ama maka didapat susunan CAB
Ket iga susunan ABC, BCA dan CAB adalah susunan yang sama yang dalam permut asi siklis baru
dianggap sebagai sat u susunan.

Pada gambar 2, j ika kit a membacanya searah j arum j am maka :


a. Jika A sebagai urut an pert ama maka didapat susunan ACB
b. Jika B sebagai urut an pert ama maka didapat susunan BAC
c. Jika C sebagai urut an pert ama maka didapat susunan CBA
Ket iga susunan ACB, BAC dan CBA adalah susunan yang sama yang dalam permut asi siklis
sehingga baru dianggap sebagai sat u susunan.

Banyaknya susunan 3 orang yang duduk pada kursi yang membent uk lingkaran = (3 − 1)! = 2! = 2
susunan.

Eddy Hermanto, ST 150 Kombinatorik


Pembinaan Olimpiade Matematika
Kalau kit a perhat ikan hal t ersebut , maka didapat langkah-langkah dalam membuat suat u
susunan pada permut asi siklis adalah :
1. Tet apkan sebuah obyek (unsur) sebagai pedoman
2. Kemudian permut asikan unsur-unsur yang t ersisa sepert i pada persoalan sebelumnya.

Pada cont oh 20 kit a misalkan A sebagai pat okan, maka sisanya dapat kit a permut asikan (dapat
j uga diselesaikan dengan at uran pengisian t empat at au kaidah perkalian). Sisa unsur yang ada
t inggal 2 yait u A dan B. Banyaknya susunan B dan C adalah 2 x 1 = 2, yait u BC dan CB. Maka
semua susunan yang mungkin adalah ABC dan ACB.

Cont oh 21 :
Jika t erdapat empat orang yang duduk pada empat kursi yang membent uk suat u lingkaran,
maka ada berapa banyak susunan yang mungkin t erj adi ?

Dengan rumus kit a dapat kan banyaknya susunan adalah (4 − 1)! = 3! = 6 susunan.
Solusi :

Jika kit a misalkan keempat orang t ersebut adalah A, B, C dan D dan kit a misalkan A sebagai
pedoman, maka t iga unsur sisanya yait u B, C dan D dapat disusun dengan 3 x 2 x 1 = 6 cara
yait u BCD, BDC, CBD, CDB, DBC dan DCB.

Jadi kemungkinan susunan empat orang t ersebut adalah ABCD, ABDC, ACBD, ACDB, ADBC dan
ADCB. Gambar berikut merupakan susunan ke-6 kemungkinan t ersebut .

LATIHAN 1.B

( n + 5 )!
1. Tent ukan nilai n yang yang memenuhi persamaan (n + 4 )! = 6n.

2. Jika diket ahui nP4 = 30 ⋅ nP2, maka t ent ukan nilai n.

3. Ada berapa banyaknya susunan huruf yang dapat dibent uk dari huruf-huruf :
a. F, U, R, K, A, N ?
b. D, E, N, N, Y ?
c. H, A, N, S, E, N ?
d. A, H, M, A, D, I ?

Eddy Hermanto, ST 151 Kombinatorik


Pembinaan Olimpiade Matematika
e. R, O, M, B O, N, G, A, N ?
f. O, L, I, M, P, I, A, D, E ?
g. M, A, T, E, M, A, T, I, K, A ?

4. (OSP 2006) Ada berapa banyaknya bilangan 7 angka berbeda yang dapat dibent uk dengan cara
mengubah susunan angka dari 2504224 ?

5. Banyaknya bilangan t iga angka yang memiliki sedikit nya sat u buah angka 4 dan sat u buah angka 5
adalah ······

6. Banyaknya bilangan 5 angka yang memenuhi hasil kali angka-angkanya sama dengan 45 ada ⋅⋅⋅⋅⋅⋅

7. Dalam suat u rapat OSIS yang t erdiri dari 6 orang siswa ( 2 di ant ara kakak beradik ) dalam posisi
melingkar. Ada berapa formasi duduk melingkar yang bisa t erbent uk j ika kakak beradik t ersebut
harus berdekat an ?

8. Sama dengan nomor 5 t et api kakak beradik t ersebut t idak boleh berdekat an.

9. (OSP 2003) Empat pasang suami ist eri menont on pagelaran orkest ra. Tempat duduk mereka harus
dipisah ant ara kelompok suami dan kelompok ist eri. Unt uk masing-masing kelompok disediakan 4
buah t empat duduk bersebelahan dalam sat u barisan. Ada berapa banyak cara memberikan
t empat duduk kepada mereka ?

10. (OSP 2009) Seekor semut hendak melangkah ke makanan yang berada sej auh 10 langkah di
depannya. Semut t ersebut sedang mendapat kan hukuman, ia hanya boleh melangkah ke depan
sebanyak kelipat an t iga langkah dan selebihnya harus melangkah ke belakang. Tent ukan
banyaknya cara melangkah agar bisa mencapai makanan, j ika ia harus melangkah t idak lebih dari
dua puluh langkah. (Cat at an : j ika semut melangkah dua kali dimana masing-masing melangkah
sekali ke belakang, maka dianggap sama saj a dengan dua langkah ke belakang.)

C. Kombinasi
Definisi dari kombinasi :
Suat u kombinasi r unsur yang diambil dari n unsur yang t ersedia (t iap unsur t ersebut berbeda) adalah
suat u pilihan dari r unsur t adi t anpa memperhat ikan urut annya.
Kat a kunci yang membedakan ant ara kombinasi dan permut asi adalah memperhatikan at au t idak

Banyaknya kombinasi r unsur yang diambil dari n unsur yang t ersedia dengan r ≤ n dirumuskan
memperhat ikan urut an.

dengan:

Cr = ⋅⋅⋅⋅⋅⋅⋅⋅⋅⋅⋅⋅⋅⋅⋅⋅⋅⋅⋅⋅⋅⋅⋅⋅⋅⋅⋅⋅⋅⋅⋅
n!
(n − r )!⋅r!
n (1.C.1)

bahwa nPr ≥ nCr . Kapan t anda kesamaan t erj adi ?


Dengan memperhat ikan rumus di at as dan membandingkannya dengan rumus permut asi didapat

⎛n⎞
Pada buku lain penulisan nCr dapat dit uliskan dengan not asi C r at au ⎜⎜ ⎟⎟ at au C(n, r) yang memiliki
⎝ ⎠
n

r
pengert ian yang sama.

Eddy Hermanto, ST 152 Kombinatorik


Pembinaan Olimpiade Matematika

Cont oh 22 :
Ada berapa banyak cara memilih 3 orang siswa dari 8 orang siswa yang akan dit unj uk sebagai
Pengurus Kelas.

Solusi :
Persoalan ini berbeda dengan cont oh 14. Pada cont oh 14, urut an diperhat ikan karena ada Ket ua,
Wakil Ket ua dan Sekret aris. Pada cont oh 22, j ika kit a memilih A, B dan C sebagai pengurus kelas maka
hal t ersebut sama saj a dengan memilih A, C dan B sebagai pengurus kelas. Perhat ikan bahwa urut an
t idak diperhat ikan.
Maka banyaknya cara ada 8C3 = (8−83!)!⋅3! = 8 x 75!x⋅36!x 5! = 8 x37!x 6 = 56 cara.

Cont oh 23 :
Ada berapa cara membent uk t im t erdiri dari 2 laki-laki dan 2 wanit a yang diambil dari 8 orang laki-
laki dan 6 orang wanit a ?

Solusi :
Banyaknya cara memilih 2 laki-laki dari 8 laki-laki adalah 8C2 = 28.
Banyaknya cara memilih 2 wanit a dari 6 wanit a adalah 6C2 = 15.
Dengan kaidah perkalian, maka banyaknya cara membent uk t im t erdiri dari 2 laki-laki dan 2 wanit a
yang diambil dari 8 orang laki-laki dan 6 orang wanit a adalah 28 x 15 = 420 cara.

Cont oh 24 :
Ada berapa banyak cara memilih secara acak 2 bola merah, 3 bola put ih dan 1 bola kuning dari dalam
kot ak yang berisi 5 bola merah, 4 bola put ih, 3 bola kuning dan 4 bola hij au ?

Solusi :
Banyaknya cara memilih 2 bola merah dari 5 bola merah adalah 5C2 = 10.
Banyaknya cara memilih 3 bola put ih dari 4 bola put ih adalah 4C3 = 4.
Banyaknya cara memilih 1 bola kuning dari 3 bola kuning adalah 3C1 = 3.
Maka banyaknya cara memilih 2 bola merah, 3 bola put ih dan 1 bola kuning adalah 10 x 4 x 3 = 120
cara.

Cont oh 25 :
Jika t erdapat 8 siswa dan 7 siswi maka ada berapa cara membent uk panit ia beranggot akan 7 orang
dengan syarat sedikit nya 5 siswi harus masuk dalam kepanit iaan t ersebut ?

Solusi :
Karena sedikit nya 7 orang panit ia t ersebut t erdiri dari 5 siswi, maka akan ada t iga kasus dalam
persoalan ini yait u panit ia t erdiri dari 5 siswi dan 2 siswa at au 6 siswi dan 1 siswa at au semuanya
siswi.
Banyaknya susunan kasus pert ama adalah 7C5 x 8C2 = 588
Banyaknya susunan kasus kedua adalah 7C6 x 8C1 = 56
Banyaknya susunan kasus ket iga adalah 7C7 x 8C0 = 1
Maka banyaknya cara membent uk panit ia adalah 588 + 56 + 1 = 645.

Cont oh 26 :
Seorang siswa harus menj awab 8 soal dari 10 soal yang diuj ikan dengan 5 soal pert ama harus dij awab.
Ada berapa cara siswa t ersebut unt uk memilih soal-soal yang akan dikerj akan ?

Eddy Hermanto, ST 153 Kombinatorik


Pembinaan Olimpiade Matematika

Solusi :
Lima soal pert ama harus dikerj akan yang berart i bukan merupakan pilihan. Maka siswa t ersebut hanya
memilih 3 soal dari 5 soal t ersisa. Banyaknya cara memilih = 5C3 = 10.

Cont oh 27 :
Ada berapa cara memilih 3 huruf berbeda apabila disyarat kan bahwa huruf-huruf t ersebut t erdiri dari
sedikit nya 1 vokal dan sedikit nya sat u konsonan ?

Solusi :
Kemungkinan kat a t ersebut t erdiri dari 2 vokal 1 konsonan at au 1 vokal 2 konsonan.
Kemungkinan pert ama menghasilkan kat a sebanyak 5C2 x 21C1 = 210
Kemungkinan kedua menghasilkan kat a sebanyak 5C1 x 21C2 = 1050
Banyaknya kat a = 210 + 1050 = 1260
Jika huruf-hurufnya t idak harus berbeda maka banyaknya kat a yang dapat t erbent uk = 1260.

Cont oh 28 :

segit iga yang dapat dibuat dengan ket iga t it ik sudut nya dipilih dari 20 t it ik t ersebut adalah ⋅⋅⋅⋅
Jika t erdapat 20 t it ik dengan t idak ada t iga t it ik yang berada pada sat u garis lurus, maka banyaknya

Solusi :
Segit iga dibent uk dari t iga t it ik yang t idak segaris.
Maka banyaknya segit iga yang dapat dibent uk = 20C3 = 1140.

Cont oh 29 :
Berapakah cacah bilangan 8 digit (a1a2a3a4a5a6a7a8) yang t erdiri dari 0 dan 1 (a1 = 1) dan memiliki sifat
a1 + a3 + a5 + a7 = a2 + a4 + a6 + a8 ?

Solusi :
a1 + a3 + a5 + a7 = a2 + a4 + a6 + a8 dengan a1 = 1

• Jika a1 + a3 + a5 + a7 = a2 + a4 + a6 + a8 = 1
Ada 4 Kasus

Maka a3 = a5 = a7 = 0 hanya ada 1 kemungkinan.


Banyaknya t upel (a2, a4, a6, a8) yang memenuhi ada 4 yait u (1,0,0 0), (0,1,0,0), (0,0,1,0) dan

Jadi, banyaknya bilangan yang memenuhi = 1 ⋅ 4 = 4.


(0,0,0,1).

• Jika a1 + a3 + a5 + a7 = a2 + a4 + a6 + a8 = 2
Maka a3 + a5 + a7 = 1. Banyaknya t ripel (a3, a5, a7) yang memenuhi ada 3.
a2 + a4 + a6 + a8 = 2

Jadi, banyaknya bilangan yang memenuhi = 3 ⋅ 6 = 18.


Banyaknya t upel (a2, a4, a6, a8) yang memenuhi = 4C2 = 6.

• Jika a1 + a3 + a5 + a7 = a2 + a4 + a6 + a8 = 3
Maka a3 + a5 + a7 = 2. Banyaknya t ripel (a3, a5, a7) yang memenuhi = 3C2 = 3.
a2 + a4 + a6 + a8 = 3

Jadi, banyaknya bilangan yang memenuhi = 3 ⋅ 4 = 12.


Banyaknya t upel (a2, a4, a6, a8) yang memenuhi = 4C3 = 4.

• Jika a1 + a3 + a5 + a7 = a2 + a4 + a6 + a8 = 4
Yang memenuhi hanya ada 1 yait u 11111111.
Jadi, bilangan yang memenuhi adalah 4 + 18 + 12 + 1 = 35.

Eddy Hermanto, ST 154 Kombinatorik


Pembinaan Olimpiade Matematika

Cont oh 30 :

Bawah. Cara menuj u B dari A dalam 8 langkah at au kurang ada sebanyak ⋅⋅⋅ (A adalah t it ik pada uj ung
(OSK 2010) Diket ahui grid berukuran 4 x 7. Jika langkah yang dimungkinkan Kanan, Kiri, At as, dan

kanan at as pada kot ak paling kiri bawah, sedangkan B adalah t it ik pada uj ung kiri bawah pada kot ak
paling kanan at as)

Solusi :
Jalan t erpendek dari A ke B adalah j ika j alannya hanya Kanan dan At as saj a.
Banyaknya langkah t erpendek adalah 7 sebab banyaknya langkah ke Kanan ada 5 dan ke At as ada 2.
Tidak ada j alan dengan banyaknya langkah t epat 8 sebab j ika berj alan ke Kiri at au ke Bawah sekali,
maka banyaknya langkah t erpendek yang diperlukan adalah 9.
Jadi cukup dihit ung banyaknya j alan dengan banyaknya langkah t epat 7.
Alt ernat if 1 :
Misalkan langkah ke Kanan diberi t anda 1 dan langkah ke At as diberi t anda 2. Maka persoalannya sama
dengan banyaknya susunan angka-angka 1111122, yait u melangkah ke Kanan sebanyak 5 kali dan
melangkah ke At as sebanyak 2 kali.
Banyaknya susunan bilangan 1111122 sama dengan 5!7⋅2! ! = 21.
Maka banyaknya cara melangkah dari A ke B sama dengan 21.

Alt ernat if 2 :
Banyaknya langkah ada 7. Dua di ant aranya adalah ke At as dan 5 ke Kanan. Maka persoalan ini adalah
sama dengan menempat kan 5 obyek ident ik pada 7 t empat berbeda.
Maka banyaknya cara melangkah dari A ke B sama dengan 7C2 = 21.
Jadi, banyaknya cara melangkah dari A ke B sama dengan 21.

Cont oh 31 :
Perhat ikan gambar.

Jika seseorang akan berj alan dari t it ik A ke t it ik B melalui t it ik koordinat yang ada. Ada berapa
banyak cara j alan t erpendek yang dapat dipilihnya ?

Solusi :
Ada 11 langkah yang harus dilakukan bagi orang t ersebut .
Persoalan ini sama saj a dengan mengisi 7 kot ak dari 11 kot ak yang ada dengan obyek ke kanan.
Maka banyaknya cara j alan t erpendek yang dapat dipilih = 11C7 = 330.
Jadi, banyaknya cara j alan t erpendek yang dapat dipilih orang t ersebut adalah 330.

Cont oh 32 :

yang berurut an, j ika bilangan-bilangan t ersebut dipilih dari himpunan {1, 2, 3, ⋅⋅⋅, 9, 10} ?
(OSP 2003) Berapakah banyaknya cara memilih t iga bilangan berbeda sehingga t idak ada dua bilangan

Eddy Hermanto, ST 155 Kombinatorik


Pembinaan Olimpiade Matematika

Misal H = {1, 2, 3, ⋅⋅⋅, 9, 10}


Solusi :

Seandainya t idak ada syarat bahwa t idak ada dua bilangan yang berurut an maka solusi persoalan
t ersebut sangat mudah yait u 10C3.
Adakah cara sehingga persoalan t ersebut merupakan kombinasi t anpa adanya syarat sepert i pada soal
t ersebut ?

yang berurut an sert a a < b < c. Maka syarat yang harus dipenuhi adalah b − a ≥ 2 dan c − b ≥ 2 sert a
Misalkan (a, b, c) adalah 3 bilangan dari H yang memenuhi bahwa t idak ada 2 bilangan di ant aranya

c ≤ 10 dan a ≥ 1.
Seandainya dibuat t iga bilangan (a, b − 1, c − 2) maka t iga bilangan t ersebut haruslah merupakan t iga
bilangan yang berbeda dan merupakan elemen dari himpunan {1, 2, 3, ⋅⋅⋅, 7, 8} sert a t idak memiliki
syarat sepert i pada soal. Maka kit a dapat memodifikasi persoalan menj adi hal yang lebih sederhana.
Persoalan hasil modifikasi adalah menent ukan t iga bilangan asli berbeda yang diambil dari 8 bilangan

Banyaknya cara memilih 3 bilangan dari himpunan {1, 2, 3, ⋅⋅⋅, 7, 8} adalah 8C3 = 56
asli berbeda.

Rumus kombinasi (rumus 1.C.1) t ersebut dapat digunakan unt uk menghit ung banyaknya himpunan

bagian dari suat u himpunan (t ermasuk himpunan kosong, dilambangkan dengan { } at au ∅) yang
bagian yang t erdiri dari beberapa unsur dari suat u himpunan. Unt uk menghit ung banyaknya himpunan

t erdiri dari n unsur at au elemen t elah dipelaj ari sewakt u SLTP yang dirumuskan dengan :

Banyaknya himpunan bagian A = 2n ⋅⋅⋅⋅⋅⋅⋅⋅⋅⋅⋅⋅⋅⋅⋅⋅⋅⋅⋅⋅⋅⋅⋅⋅⋅⋅⋅⋅⋅⋅⋅ (1.C.2)

Cont oh 33 :
Misalkan himpunan A = {a, b, c, d}. Ada berapa banyak himpunan bagian A ? Sebut kan ?

Solusi :
Banyaknya himpunan bagian A = 24 = 16 t erdiri dari :
Himpunan kosong yait u { } ada 1.
Himpunan bagian t erdiri dari 1 elemen yait u {a}, {b}, {c} dan {d} ada 4.
Himpunan bagian t erdiri dari 2 elemen yait u {a, b}, {a, c}, {a, d}, {b, c}, {b, d} dan {c, d} ada 6.
Himpunan bagian t erdiri dari 3 elemen yait u {a, b, c}, {a, b, d}, {a, c, d} dan {b, c, d} ada 4.
Himpunan bagian t erdiri dari 4 elemen yait u {a, b, c, d} ada 1

Bagaimana bila kit a hanya ingin menghit ung banyaknya himpunan bagian yang t erdiri dari r elemen
saj a ? Misalkan A adalah himpunan yang t erdiri dari n elemen. Banyaknya himpunan bagian t erdiri dari
r elemen dirumuskan dengan :

Banyaknya himpunan bagian A yang t erdiri dari r elemen = nCr ⋅⋅⋅⋅⋅⋅⋅⋅⋅⋅⋅⋅⋅⋅⋅⋅⋅⋅⋅⋅⋅⋅⋅⋅⋅ (1.C.3)

Pada cont oh 33 didapat banyaknya himpunan bagian A yang t erdiri dari 3 elemen = 4C3 = 4. Banyaknya
himpunan bagian A yang t erdiri dari 2 elemen = 4C2 = 6.

Cont oh 34 :
Ada berapa banyaknya himpunan bagian dari kat a OLIMPIADE ?

Solusi :
Banyaknya elemen dari kat a OLIMPIADE ada 8 yait u, O, L, I, M, P, A, D dan E karena huruf I muncul
dua kali.
Banyaknya himpunan bagian = 28 = 256

Eddy Hermanto, ST 156 Kombinatorik


Pembinaan Olimpiade Matematika

LATIHAN 1.C

1. Jika nC3 = 2n, maka t ent ukan 2nC7.

2. Dari 8 pemain bulut angkis (5 di ant aranya put era) akan dibent uk t im pasangan ganda campuran.
Maka ada berapa pasangan ganda campuran yang dapat dibent uk ?

3. Dari sekelompok remaj a t erdiri at as 10 orang pria dan 7 wanit a akan dipilih 2 pria dan 3 wanit a,
maka ada berapa banyaknya cara memilih ?

4. Ada berapa banyak segit iga yang dapat dibuat dengan ket iga t it ik sudut segit iga t ersebut adalah
t it ik sudut -t it ik sudut suat u balok ?

5. (OSK 2011) Sekelompok orang akan berj abat t angan. Set iap orang hanya dapat melakukan j abat
t angan sekali. Tidak boleh melakukan j abat t angan dengan dirinya sendiri. Jika dalam sekelompok
orang t erdapat 190 j abat t angan, maka banyaknya orang dalam kelompok t ersebut ada berapa ?

6. Ada berapa banyaknya himpunan bagian dari kat a MATEMATIKA ?

7. Ahmadi berhasil menemukan semua himpunan bagian dari kat a “ BELAJARLAH” . Ada berapa
banyak himpunan bagian yang j umlah anggot anya paling banyak 5 ?

8. Dari 10 orang siswa yang t erdiri 7 orang put era dan 3 orang put eri akan dibent uk t im yang
beranggot akan 5 orang. Jika disyarat kan anggot a t im t ersebut paling banyak 2 orang put eri, maka
ada berapa banyak kemungkinan t im yang dapat dibent uk ?

9. SMA Negeri 5 Bengkulu akan memilih 6 orang wakil sekolah unt uk mengkut i suat u kont es. Kelima
wakil sekolah t ersebut akan dipilih dari 6 siswa put eri dan 5 siswa put era. Jika dipersyarat kan
bahwa j umlah siswa put era minimal 4 orang, ada berapa cara memilih wakil sekolah t ersebut ?

10. Delegasi Indonesia ke suat u pert emuan pemuda int ernasional t erdiri dari 5 orang. Ada 7 orang
pria dan 5 orang wanit a yang mencalonkan diri unt uk menj adi anggot a delegasi. Jika

anggot a delegasi adalah ⋅⋅⋅⋅


dipersyarat kan bahwa paling sedikit seorang anggot a it u harus wanit a, banyaknya cara memilih

11. Hansen memiliki 8 orang sahabat dekat dengan 2 orang di ant aranya berpacaran. Pada suat u hari,
ia ingin mengundang 5 dari 8 sahabat dekat nya t ersebut . Dengan berapa cara ia dapat
mengundang j ika 2 orang sahabat nya yang berpacaran harus diundang keduanya at au t idak
mengundang keduanya ?

12. Sebuah kelas akan mengikut i lomba Fut sal. Dari 12 orang siswa put era akan dipilih 2 orang
berposisi sebagai kiper, 4 orang sebagai pemain belakang dan 3 orang sebagai penyerang dan
t idak ada pemain yang merangkap pada posisi lain. Ada berapa cara memilihnya ?

13. (OSP 2005/ OSK 2008) Ada berapa banyak himpunan X yang memenuhi {1, 2} ⊆ X ⊆ {1, 2, 3, 4, 5} ?

{1, 2, 3, ⋅⋅⋅, 1000} ⊆ X ⊆ {1, 2, 3, ⋅⋅⋅, 2010}


14. (OSK 2010) Banyaknya himpunan X yang memenuhi

adalah ⋅⋅⋅⋅

15. Dalam sebuah grup 15 anak-anak t erdapat 7 pramuka. Dalam berapa cara kit a dapat memilih 12
anak sehingga di dalamnya t erdapat
a. t epat 6 anak pramuka
b. sedikit nya 6 anak pramuka

Eddy Hermanto, ST 157 Kombinatorik


Pembinaan Olimpiade Matematika

16. Liga It alia t erdiri dari 20 t im dan berlaku format kompet isi dengan sist em Home and Away (di
ant ara 2 t im t epat t erj adi dua pert andingan). Ada berapa banyak pert andingan keseluruhan ?
Dalam suat u pert andingan t im yang memenangkan pert andingan mendapat kan nilai 3, yang kalah
0 dan seri mendapat kan nilai 1. Pada akhir kompet isi j umlah nilai seluruh t im adalah 1000. Ada
berapa pert andingan yang berakhir dengan imbang ?

17. (NAHC 1995-1996 First Round) Berapa banyakkah bilangan t erdiri dari 7 digit berbeda yang j ika
dilihat dari kiri ke kanan maka digit nya selalu naik ? Cont oh bilangan t ersebut adalah : 1234567,
1356789, 2345789, 3456789, 1235678. Digit 0 t idak diperbolehkan t erlet ak pada digit pert ama.

18. Semua bilangan enam angka dengan semua angkanya berbeda disusun. Pada masing-masing
bilangan memenuhi set iap angka selain angka awal bersifat kurang dari angka yang ada di
kanannya. Cont oh bilangan t ersebut adalah 123457, 134678, 346789, dan sebagainya. Jika semua

urut an ke-45 adalah ⋅⋅⋅⋅


bilangan t ersebut disusun dari bilangan t erkecil sampai ke t erbesar, maka bilangan yang berada di

19. Banyaknya bilangan asli berbent uk a1a2a3⋅⋅⋅an-1anan-1⋅⋅⋅a3a2a1 dimana 0 < a1 < a2 < a3 < ⋅⋅⋅ < an-1 < an
dan n ≥ 2 ada ⋅⋅⋅⋅⋅⋅⋅⋅⋅⋅⋅⋅

20. (OSP 2010) Bilangan enam digit abcdef dengan a > b > c ≥ d > e > f ada sebanyak ⋅⋅⋅⋅⋅⋅⋅

21. Naek mencoba melet akkan 4 buah pion ident ik pada sebuah papan cat ur 4 x 4. Ada berapa cara ia
melet akkan keempat pion t ersebut ?

22. Lain lagi yang dilakukan Sut an. Ia mencoba melet akkan dua pion put ih ident ik dan 2 pion hit am
ident ik pada papan cat ur 4 x 4. Ada berapa cara ia melet akkan keempat pion t ersebut ?

23. (OSN 2011 SMP/ MTs) Seorang calon dokt er diharuskan magang di rumah sakit selama lima hari
pada bulan Juli 2011. Pimpinan rumah sakit memberikan at uran sebagai berikut :
a. Magang t idak boleh dilakukan dua hari bert urut -t urut
b. Magang hari kelima baru boleh dilakukan set elah empat hari t erhit ung sej ak magang hari
keempat . Misalkan magang hari keempat adalah t anggal 20, maka magang hari kelima baru
boleh dilaksanakan set idaknya pada t anggal 24.
Tent ukan banyak pilihan j adwal yang mungkin bagi calon dokt er t ersebut .

24. (AIME 1983) Ada berapa banyak bilangan 4 angka dengan angka pert ama 1 dan t epat memiliki dua
angka yang sama ? (Cont oh bilangan t ersebut adalah 1447, 1050, 1231 dan sebagainya)

panj ang (t ermasuk persegi) yang dapat dibuat pada persegi panj ang t ersebut adalah ⋅⋅⋅⋅
25. Sebuah persegi panj ang dengan ukuran 6 x 5 dibagi menj adi 30 persegi sat uan. Banyaknya persegi

26. (OSK 2010) Misalkan persegi 4 x 4 akan diberi warna hit am dan put ih pada t iap kot aknya. Cara

put ih sebanyak ⋅⋅⋅⋅⋅⋅ (Pewarnaan dianggap sama j ika didapat dari hasil rot asi yang sama t erhadap
pewarnaan sedemikian sehingga warna hit am hanya diberikan pada 3 kot ak dan sisanya warna

persegi 4 x 4)

27. Di dalam sebuah pest a, 5 orang t eman akan saling memberikan masing-masing sat u hadiah
sehingga set iap orang akan memberi dan menerima hanya sat u (t ent unya t idak akan ada orang
yang akan menerima hadiah dari dirinya). Berapakah j umlah cara yang mungkin dilakukan?

28. (OSP 2009) Misalkan N menyat akan himpunan semua bilangan bulat posit if dan

Eddy Hermanto, ST 158 Kombinatorik


Pembinaan Olimpiade Matematika
⎧ n 2009 + 2 ⎫
S = ⎨n ∈ N ∈ N⎬
⎩ n +1 ⎭
Banyaknya himpunan bagian dari S adalah ⋅⋅⋅⋅⋅⋅⋅⋅⋅

dapat dibuat dengan ket iga t it ik sudut nya dipilih dari 20 t it ik t ersebut adalah ⋅⋅⋅⋅
29. Jika t erdapat 20 t it ik dengan 5 t it ik berada pada sat u garis lurus, maka banyaknya segit iga yang

30. Pada ruas AB dari segit iga ABC diberi 2 t it ik selain AB dan pada ruas BC diberi 3 t it ik selain BC

t it ik sudut nya merupakan t it ik-t it ik yang ada pada segit iga ABC t ersebut adalah ⋅⋅⋅⋅⋅
sert a pada ruas AC diberi 4 t it ik selain AC. Banyaknya segit iga yang dapat dibuat dengan ket iga

31. Pak Eddy mencoba membagi 6 orang siswa menj adi 2 kelompok yang masing-masing beranggot a
t iga orang. Berapa banyakkah cara membent uk kedua kelompok ini ?

32. (OSK 2003) Dari sepuluh orang siswa akan dibent uk 5 kelompok, masing-masing beranggot a dua
orang. Berapa banyaknya cara membent uk kelima kelompok ini ?

33. Sebuah benda akan digerakkan dari t it ik A(0,0) ke t it ik B(6,4) namun benda t ersebut hanya dapat
bergerak ke at as ke kanan melalui t it ik-t it ik koordinat .

a. Ada berapa cara benda t ersebut bergerak dari t it ik A hingga mencapai t it ik B ?


b. Ada berapa cara benda t ersebut bergerak dari t it ik A hingga mencapai t it ik B namun harus
melalui t it ik P(4,2) ?
c. Ada berapa cara benda t ersebut bergerak dari t it ik A hingga mencapai t it ik B namun harus
melalui ruas PQ dengan Q(4,3) ?

34. Seseorang akan melangkah dari A ke B melalui t it ik koordinat pada persegi 4 x 4 sepert i pada

melangkah dari A ke B adalah ⋅⋅⋅⋅


gambar. Jika diinginkan banyaknya langkah adalah minimal maka banyaknya cara orang t ersebut

35. (OSP 2011) Pada ruang Cart esius kit a ingin bergerak dari t it ik (2, 0, 11) ke t it ik (20, 1, 1) selalu

lint asan t erpendek. Cara bergerak yang dimaksud sebanyak ⋅⋅⋅⋅⋅⋅


pada koordinat (x, y, z) dengan paling sedikit dua dari x, y, dan z adalah bilangan bulat , dan

memenuhi persamaan a + b = c + d ada sebanyak ⋅⋅⋅⋅⋅⋅⋅⋅


36. (OSP 2011) Banyaknya kemungkinan bilangan asli berbeda a, b, c, dan d yang kurang dari 10 dan

37. (OSP 2011) Diket ahui segi empat ABCD. Semua t it ik A, B, C, dan D akan diberi nomor 1, 2, 3, 4, 5

Banyaknya cara pemberian nomor dengan cara t ersebut ada sebanyak ⋅⋅⋅⋅⋅⋅
at au 6 sehingga set iap dua t it ik yang t erlet ak dalam sat u sisi empat nomornya berbeda.

Eddy Hermanto, ST 159 Kombinatorik


Pembinaan Olimpiade Matematika

38. (OSP 2011) Sepuluh orang siswa duduk dalam suat u baris. Semua siswa bangkit dan duduk kembali
pada baris t ersebut dengan at uran set iap siswa dapat duduk kembali pada kursi yang sama at au

kembali pada baris t adi ada sebanyak ⋅⋅⋅⋅⋅⋅


pada kursi yang berada di sebelah kursi lamanya. Banyaknya cara semua siswa t ersebut duduk

39. Sebuah komit e mengadakan 40 pert emuan dengan 10 orang anggot a komit e hadir pada masing-
masing pert emuan. Set iap dua orang anggot a komit e menghadiri pert emuan secara bersamaan
paling banyak sat u kali. Tunj ukkan banyaknya anggot a komit e t ersebut lebih dari 60.

D. Persoalan Gabungan
Sebagaimana t elah dij elaskan sebelumnya bahwa sebuah persoalan t erkadang bisa diselesaikan
dengan hanya menggunakan salah sat u cara dari at uran pengisian t empat , permut asi at au kombinasi
saj a. Tet api kadang-kadang sebuah persoalan hanya dapat diselesaikan dengan menggunakan
gabungan dari beberapa cara t ersebut . Berikut beberapa cont oh persoalan.

Cont oh 35 :
Ada berapa banyak cara memilih 2 orang wanit a dari 5 orang wanit a dan 2 orang laki-laki dari 6 orang
laki-laki sebagai ket ua, wakil ket ua dan 2 orang kepala seksi dari suat u organisasi dengan syarat
bahwa ket ua dan wakil ket ua harus laki-laki dan 2 orang kepala seksi harus wanit a ?

Solusi :
2 orang laki-laki dipilih dari 6 orang laki-laki sebagai ket ua dan wakil ket ua yang berart i urut an
diperhat ikan. Maka banyaknya cara memilih ada 6P2 = 30 cara.
2 orang wanit a dipilih dari 5 orang wanit a sebagai kepala seksi yang berart i urut an t idak diperhat ikan.
Maka banyaknya cara memilih ada 5C2 = 10 cara.
Jadi, banyaknya cara memilih ada 30 x 10 = 300 cara.

Cont oh 36 :
Dari lima orang siswa suat u sekolah, akan diambil t iga orang sebagai t im cerdas cermat dengan salah
sat u sebagai j uru bicara dan dua lainnya sebagai pendamping. Ada berapa cara memilihnya ?

Solusi :
Jika kit a menj awab bahwa banyaknya cara dari persoalan di at as adalah 5P3 = 60 cara, maka ada
kesalahan yang kit a buat . Benar j ika dikat akan bahwa t erdapat urut an karena ada yang berperan
sebagai j uru bicara dan ada yang berperan sebagai pendamping t et api dua orang yang berperan
sebagai pendamping t idak diperhat ikan urut annya. Sebagai ilust rasi adalah j ika A sebagai j uru bicara,
B dan C sebagai pendamping maka hal t ersebut sama dengan A sebagai j uru bicara, C dan B sebagai
pendamping yang berart i urut an pada pemilihan pendamping t idak diperhat ikan.
Jika kit a menj awab bahwa banyaknya cara dari persoalan di at as adalah 5C3 = 10 cara, maka
kesalahan yang dibuat adalah bahwa pada pemilihan j uru bicara dan pendamping ada urut an yang
diperhat ikan. Sebagai ilust rasi j ika A sebagai j uru bicara, B dan C sebagai pendamping berbeda
dengan B sebagai j uru bicara, A dan C sebagai pendamping.
Berikut adalah beberapa alt ernat if penyelesaian soal t ersebut dengan beberapa sudut pandang yang
berbeda.

Alt ernat if 1 :
Dari 5 orang siswa kit a pilih salah sat u sat u siswa yang akan berperan sebagai j uru bicara. Karena
hanya ada sat u yang dipilih maka dapat dipandang sebagai permut asi maupun kombinasi. Banyaknya
cara adalah 5C1 at au 5P1 = 5 cara.

Eddy Hermanto, ST 160 Kombinatorik


Pembinaan Olimpiade Matematika

Sisanya adalah memilih dua siswa dari 4 siswa yang akan berperan sebagai pendamping. Karena
urut an t idak diperhat ikan maka banyaknya cara adalah 4C2 = 6 cara.
Dengan kaidah perkalian banyaknya cara memilih adalah 5 x 6 = 30 cara.

Alt ernat if 2 :
Yang dipilih t erlebih dahulu adalah 2 orang siswa dari 5 siswa yang berperan sebagai pendamping.
Karena urut an t idak diperhat ikan maka banyaknya cara adalah 5C2 = 10 cara.
Selanj ut nya adalah memilih 1 siswa dari 3 siswa yang berperan sebagai j uru bicara. Banyaknya cara
ada 3 cara.
Dengan kaidah perkalian banyaknya cara memilih adalah 10 x 3 = 30 cara.

Alt ernat if 3 :
Terlebih dahulu dipilih 3 siswa dari 5 siswa t ersebut t anpa memperhat ikan urut an. Banyaknya cara
adalah 5C3 = 10 cara.
Dari 3 orang t ersebut akan disusun. Pada susunan t ersebut akan t erdapat dua obyek yang sama. Hal
ini sama saj a dengan menyusun huruf A, A, B sebagaimana t elah dij elaskan pada persoalan
sebelumnya. Banyaknya cara menyusuan 3 obyek dengan t erdapat 2 obyek yang sama = 23!! = 3 cara.
Dengan kaidah perkalian banyaknya cara memilih adalah 10 x 3 = 30 cara.

Cont oh 37 :
Dari 4 orang akan dipilih 3 orang yang akan duduk pada 3 kursi yang membent uk lingkaran. Ada
berapa banyak susunan yang dapat dibuat ?

Solusi :

Banyaknya cara menyusun 3 orang yang duduk pada 3 kursi yang membent uk lingkaran adalah (3 − 1)!
Langkah pert ama adalah memilih 3 orang dari 4 orang, banyaknya cara adalah 4C3 = 4 cara.

= 2 cara.
Dengan kaidah perkalian banyaknya cara menyusun 3 orang dari 4 orang yang akan duduk pada 3 kursi
yang membent uk lingkaran adalah 4 x 2 = 8 cara. Dapat kah ada menyebut kan semua kemungkinan
t ersebut j ika keempat orang t ersebut adalah A, B, C dan D.

Cont oh 38 :
Sebuah bangun segienam berat uran dibagi menj adi 6 buah segit iga sama sisi. Keenam segit iga
t ersebut akan diberi warna yang berbeda. Jika t erdapat 2007 bangun segienam berat uran sert a
diinginkan t idak ada corak yang sama di ant ara dua buah bangun segienam, maka ada berapa minimal
warna yang diperlukan ?

Solusi :
Sebuah bangun segienam berat uran j ika dibagi menj adi 6 buah segit iga sama sisi, maka keenam
segit iga t ersebut akan membent uk lingkaran.

corak yang dapat dibent uk adalah (6 − 1)! = 120.


Enam buah warna j ika digunakan unt uk mewarnai sat u buah segienam berat uran maka banyaknya

Maka j ika ada n buah warna maka banyaknya corak yang dapat dibent uk = nC6 ⋅ (6 − 1)! ≥ 2007
Misalkan ada n buah warna. Dari n warna ini akan dipilih 6 buah warna. Banyaknya cara nC6.

n ( n −1)( n − 2 )( n − 3 )( n − 4 )( n − 5 )
≥ 2007
n(n − 1)(n − 2)(n − 3)(n − 4)(n − 5) ≥ 12042
6

Jika n = 7 maka n(n − 1)(n − 2)(n − 3)(n − 4)(n − 5) = 5040 ≤ 12042


Jika n = 8 maka n(n − 1)(n − 2)(n − 3)(n − 4)(n − 5) = 20160 ≥ 12042
Maka banyaknya warna minimal yang diperlukan = 8 warna.

Eddy Hermanto, ST 161 Kombinatorik


Pembinaan Olimpiade Matematika

Cont oh 39 :
Tent ukan banyaknya susunan 3 huruf yang t erdapat pada kat a COMBINATION ?

Solusi :
Terdapat 8 huruf berbeda dari combinat ion yait u C, O, M, B, I, N, A, T.
Ada t erdapat 2 buah huruf O yang sama, 2 buah huruf I yang sama dan 2 buah huruf N yang sama.
Susunan 3 huruf t ersebut dapat berupa ket iga-t iganya berbeda huruf at au dari t iga huruf t ersebut

• Jika ket iga hurufnya berbeda.


t erdapat 2 huruf yang sama dan 1 huruf berbeda t et api t idak mungkin ket iganya hurufnya sama.

Banyaknya susunan adalah sama dengan memilih 3 huruf dari 8 huruf berbeda yang ada.

• Jika t erdapat 2 huruf yang sama.


Banyaknya susunan = 8P3 = 336

2 huruf yang sama t ersebut dapat dipilih dari 3 kemungkinan O, I at au N sedangkan 1 huruf
t erakhir dapat dipilih dari 7 kemungkinan huruf t ersisa.
Susunan 3 huruf dengan 2 huruf yang sama t ersebut merupakan permut asi dengan ada unsur yang
sama.
Banyaknya susunan 3 huruf dengan 2 huruf yang sama pada soal = 23!! x 3 x 7 = 63
Maka banyaknya susunan 3 huruf yang t erdapat pada COMBINATION = 336 + 63 = 399.

LATIHAN 1.D

1. Kelompok A t erdiri dari 3 orang, kelompok B t erdiri dari 5 orang dan kelompok C t erdiri dari 10
orang. Dari anggot a kelompok A dan B masing-masing akan dipilih 1 orang sedangkan dari
kelompok C akan dipilih 2 orang. Keempat orang t ersebut akan dipilih kembali unt uk menj abat
sebagai Ket ua, Wakil Ket ua dan 2 orang Sekret aris. Ada berapa banyak cara memilih dari 18 orang
t ersebut ?

2. (OSN 2003) Balairung sebuah ist ana berbent uk segi-6 berat uran dengan
panj ang sisi 6 met er. Lant ai balairung t ersebut dit ut upi dengan ubin-ubin
keramik berbent uk segit iga samasisi dengan panj ang sisi 50 cm. Set iap ubin
keramik dibagi ke dalam 3 daerah segit iga yang kongruen, lihat gambar.
Set iap daerah segit iga diberi sat u warna t ert ent u sehingga set iap ubin
memiliki t iga warna berbeda. Raj a menginginkan agar t idak ada dua ubin
yang memiliki pola warna sama. Paling sedikit berapa warna yang
diperlukan ?

3. Ada berapa banyak susunan 5 huruf dengan t epat 1 huruf R dan 1 huruf I j ika huruf-huruf t ersebut
diambil dari kat a “ BERANI” ?

4. Lima huruf yang t idak harus berbeda diambil dari kat a SOEHARTO lalu disusun. Ada berapa banyak
susunan kelima huruf t ersebut ?

5. Hansen mencoba menyusun 4 huruf yang huruf-hurufnya diambil dari kat a TERCECER. Ada berapa
susunan yang didapat ?

6. Denny mencoba menyusun 4 huruf yang huruf-hurufnya diambil dari kat a MATEMATIKA. Ada
berapa susunan yang didapat ?

Eddy Hermanto, ST 162 Kombinatorik


Pembinaan Olimpiade Matematika

Misalkan ada n obyek yang akan dilet akkan pada r t empat t anpa urut an dengan r ≤ n. Jika disyarat kan
E. Kombinasi dengan Pengulangan

bahwa sat u t empat hanya bisa menampung paling banyak 1 obyek maka banyaknya cara adalah nCr
yang t elah kit a bahas sebelumnya.
Misalkan t erdapat n obyek ident ik dan disyarat kan bahwa seluruh obyek akan dibagikan ke r buah
t empat dengan masing-masing t empat dapat t idak dit empat i maupun dit empat i sat u at au lebih
obyek. Pert anyaannya adalah ada berapa banyak cara menyusunnya ?

sat u baris. Tambakan r − 1 bat as di ant ara bola-bola t ersebut sehingga kini seolah-olah ada n + r − 1
Karena ident ik maka urut an dalam persoalan ini t idak diperhat ikan. Taruh n obyek t ersebut dalam

’ t empat ’ . Akibat penambahan r − 1 bat as t ersebut maka n bola t ersebut akan t erbagi dalam r bagian,

ke-(r − 1). Masing-masing bagian t ersebut melambangkan banyaknya bola pada masing-masing
yait u di sebelah kiri bat as ke-1, di ant ara bat as ke-1 dan ke-2 sampai dengan di sebelah kanan bat as

t empat . Sehingga persoalannya sekarang adalah memilih (r − 1) t empat dari n + r − 1 t empat yang

⎛ n + r − 1⎞ ⎛ n + r − 1⎞
t ersedia. Banyaknya cara adalah

⎜⎜ ⎟⎟ = ⎜⎜ ⎟⎟
⎝ r −1 ⎠ ⎝ n ⎠

Cont oh 40 :
4 buah bola akan dibagikan seluruhnya ke dalam 3 buah kant ong. Ada berapa banyak cara
menyusunnya ?

Solusi :
Sebagaimana penj elasan sebelumnya, banyaknya cara = 4+3-1C4 = 6C4 = 15 cara. Yang kalau dij abarkan
susunannya adalah (4,0,0), (3,1,0), (3,0,1), (2,2,0), (2,0,2), (2,1,1), (1,3,0), (1,2,1), (1,1,2), (1,0,3),
(0,4,0), (0,3,1), (0,2,2), (0,1,3) dan (0,0,4) dengan (a,b,c) menyat akan kant ong pert ama berisi a
bola, kant ong ke-2 berisi b bola dan kant ong ke-3 berisi c bola.

penyelesaian persamaan linier. Misalkan saj a t erdapat persamaan x1 + x2 + ⋅⋅⋅ + xr = n. Jika xi


Kombinasi dengan pengulangan j uga dapat menyelesaikan persoalan mengenai perhit ungan banyaknya

merupakan bilangan bulat t ak negat if, maka ada berapa banyak penyelesaian yang memenuhi.
Persoalan ini sama saj a dengan membagi n obyek ident ik ke dalam r buah t empat . Banyaknya
penyelesaian adalah n+r-1Cn.

Cont oh 41 :
Tent ukan banyaknya t upel bilangan bulat t ak negat if (x1, x2, x3, x4) yang memenuhi persamaan linier
x1 + x2 + x3 + x4 = 3 ?

Solusi :
Dari penj elasan sebelumnya akan didapat banyaknya t upel bilangan bulat t ak negat if yang memenuhi
adalah 3+4-1C3 = 6C3 = 20. Tupel (x1, x2, x3, x4) yang memenuhi adalah (0,0,0,3), (0,0,1,2), (0,0,2,1),
(0,0,3,0), (0,1,0,2), (0,1,1,1), (0,1,2,0), (0,2,0,1), (0,2,1,0), (0,3,0,0), (1,0,0,2), (1,0,1,1), (1,0,2,0),
(1,1,0,1), (1,1,1,0), (1,2,0,0), (2,0,0,1), (2,0,1,0), (2,1,0,0) dan (3,0,0,0).

Misalkan t erdapat n obyek berbeda yang akan dilet akkan pada r t empat . Jika diperbolehkan ada
pengulangan obyek yang akan dit empat kan sert a urut an diperhat ikan, maka banyaknya cara = nr .
Persoalan ini sudah dibahas sebelumnya. Sebagai cont oh adalah menent ukan banyaknya bilangan 3

Banyaknya bilangan ada 43 = 64, yait u 111, 112, 113, 114, 121, 122, ⋅⋅⋅, 444. Bilangan 112, 121 dan
angka dengan angka-angkanya diambil dari 1, 2, 3, 4 dengan bolehnya ada angka yang berulang.

211 diangap berbeda. Bagaimana persoalannya j ika 112, 121, 211 dianggap sama karena urut annya
t idak diperhat ikan ?

Eddy Hermanto, ST 163 Kombinatorik


Pembinaan Olimpiade Matematika
Pandang n obyek t ersebut sebagai ’ t empat ’ . Persoalannya adalah sepert i menempat kan r ’ obyek’
ident ik pada n ’ t empat ’ . Banyaknya cara adalah r+n-1Cr.

Cont oh 42 :
Dua angka dipilih dari himpunan {1, 2, 3, 4} dengan pengulangan diperbolehkan. Ada berapa cara
memilih dua angka t ersebut ?
Solusi :
Pandang 4 buah kant ong. Dua bola akan dit empat kan pada kant ong-kant ong t ersebut . Jika bola
t ersebut dit empat kan pada kant ong 1 dan 4 maka berart i angka-angka yang dipilih adalah (1, 4). Jika
kedua bola t ersebut dit empat kan pada kant ong 3 maka berart i angka yang dipilih adalah (3, 3).
Banyaknya cara memilih = 2+4-1C2 = 5C2 = 10.
Pasangan angka-angka t ersebut (1,1), (1,2), (1,3), (1,4), (2,2), (2,3), (2,4), (3,3), (3,4) dan (4,4).

LATIHAN 1.E

1. (OSP 2004) Berapakah banyaknya barisan bilangan bulat t ak negat if (x, y, z) yang memenuhi
persamaan x + y + z = 99 ?

2. Sebuah t oko memiliki 10 buah balon merah, 9 buah balon kuning dan 11 buah balon hij au.
Seorang pembeli ingin membeli 8 buah balon. Ada berapa banyak cara pembeli t ersebut membeli
balon ?

3. Tent ukan banyaknya t upel bilangan asli (a, b, c, d) yang memenuhi a + b + c + d = 17.

dengan syarat x ≥ 3 ; y ≥ 4 dan z ≥ 5.


4. Tent ukan banyaknya t ripel bilangan bulat (x, y, z) yang memenuhi persamaan x + y + z = 18

lemparan sehingga j umlah ket iga mat a dadu adalah 8 sebanyak ⋅⋅⋅⋅⋅⋅⋅⋅⋅
5. (OSP 2009) Tiga dadu berwarna hit am, merah, dan put ih dilempar bersama-sama. Macam hasil

syarat 0 ≤ x ≤ 4 ; 0 ≤ y ≤ 5 dan 0 ≤ z ≤ 3.
6. Tent ukan banyaknya t ripel bilangan bulat (x, y, z) yang memenuhi persamaan x + y + z = 9 dengan

7. Tent ukan banyaknya bilangan 5 angka yang j umlah digit nya sama dengan 5.

F. Penjabaran Binom Newton dengan Notasi Kombinasi


Pada saat SMP, siswa t elah diaj arkan menj abarkan bent uk (a + b) n yang unt uk nilai n = 2 dapat
dilakukan dengan perkalian langsung sedangkan unt uk n yang besar dapat dilakukan dengan
menggunakan segit iga pascal unt uk mendapat kan koefisien-koefisien penj abaran.

Unt uk n=1 1 1
Unt uk n=2 1 2 1
Unt uk n=3 1 3 3 1
Unt uk n=4 1 41 6 4

⋅⋅⋅
Unt uk n=5 110 10 5 5 1

Bilangan yang di bawah merupakan penj umlahan dua bilangan di at asnya. Dari segit iga pascal
t ersebut akan didapat

Eddy Hermanto, ST 164 Kombinatorik


Pembinaan Olimpiade Matematika

(a − 2b)5 = (1)(a) 5(−2b)o + (5)(a) 4(−2b) 1 + (10)(a)3(−2b) 2 + (10)(a)2(−2b)3 + (5)(a) 1(−2b)4 + (1)(a) 0(−2b)5
(a − 2b)5 = a5 − 10a4b + 40a3b2 − 80a2b3 + 80ab4 −32b5

Cara lain adalah dengan menggunakan rumus kombinasi.


Jika (a + b) n kit a j abarkan akan didapat rumus sebagai berikut :

(a + b) n = nCo(a) n(b)0 + nC1(a) n-1(b) 1 + nC2(a) n-2(b) 2 + ⋅⋅⋅ + nCn-1(a) 1(b) n-1 + nCn(a) 0(b) n ………….. (1.E.1)

at au dapat j uga dit ulis

(a + b) n = nCo(a) 0(b) n + nC1(a) 1(b) n-1 + nC2(a) 2(b) n-2 + ⋅⋅⋅ + nCn-1(a) n-1(b)1 + nCn(a) n(b)0 …………. (1.E.2)

Cont oh 43 :
Jabarkan (2m + n) 5.

Solusi :
(2m + n)5 = 5Co(2m)5(n) 0 + 5C1(2m)4(n) 1 + 5C2(2m)3(n) 2 + 5C3(2m)2(n) 3 + 5C4(2m)1(n) 4 + 5C5(2m)0(n) 5
(2m + n)5 = (1)(32m5)(1) + (5)(16m4)(n) + (10)(8m3)(n2) + (10)(4m2)(n3) + (5)(2m)(n4) + (1)(1)(n5)
(2m + n)5 = 32m5 + 80m4n + 80m3n2 + 40m2n3 + 10mn4 + n5

Jabarkan bent uk (2x − 3y)3


Cont oh 44 :

(2x − 3y) 3 = 3Co(2x)3(−3y)0 + 3C1(2x)2(−3y) 1 + 3C2(2x)1(−3y) 2 + 3C3(2x)0(−3y)3


Solusi :

(2x − 3y) = (1)(8x3)(1) + (3)(4x2)(−3y) + (3)(2x)(9y2) + (1)(1)(−27y3)


(2x − 3y) 3 = 8x3 − 36x2y + 54xy2 − 27y3

Persoalan t imbul adalah bila variabel yang akan dij abarkan t idak t erdiri dari hanya 2 variabel.
Sebenarnya hal ini t idak t erlalu sulit sebab dengan menggunakan pemisalan maka dari beberapa
variabel dapat diubah menj adi 2 variabel saj a. Misalkan penj abaran (x + y + z) n dapat diubah menj adi
(A + B)n dengan pemisalan A = x dan B = y + z.

Cont oh 45 :
Jabarkan bent uk (a + b + c) 3.

Solusi :
Karena persoalannya t erdiri dari 3 variabel maka dapat kit a pecah seolah-olah menj adi 2 variabel
yait u a dan b + c.
(a + b + c)3 = 3Co(a) 3(b + c) 0 + 3C1(a) 2(b + c) 1 + 3C2(a) 1(b + c)2 + 3C3(a) 0(b + c)3
Dengan menggunakan penj abaran binom sebelumnya dapat diket ahui bahwa :
(b + c) 2 = b2 + 2bc + c2
(b + c) 3 = b3 + 3b2c + 3bc2 + c3
Sehingga didapat :
(a + b + c)3 = a3 + 3a2( b + c) + 3a(b2 + 2bc + c2) + (b + c)3
(a + b + c)3 = a3 + 3a2b + 3a2c + 3ab2 + 6abc + 3ac2 + b3 + 3b2c + 3bc2 + c3
(a + b + c)3 = a3 + b3 + c3 + 3a2b + 3a2c + 3ab2 + 3ac2 + 3b2c + 3bc2 + 6abc
Persoalan berikut nya adalah bagaimana caranya dapat diket ahui koefisien dari suat u variabel t ert ent u
t anpa harus menj abarkan semua suku-sukunya.

Eddy Hermanto, ST 165 Kombinatorik


Pembinaan Olimpiade Matematika

Tent ukan koefisien x6y5 dari penj abaran (2x − 5y)11.


Cont oh 46 :

Solusi :
Karena yang dimint a hanya koefisien x6y5 maka kit a hanya berkonsent rasi pada penj abaran bent uk
(2x)6(5y) 5 saj a.
(2x − 5y) 11 = ⋅⋅⋅ + 11C5 (2x) 6(−5y)5 + ⋅⋅⋅
(2x − 5y) 11 = ⋅⋅⋅ + (462)(64x6)(−3125y5) + ⋅⋅⋅
(2x − 5y) 11 = ⋅⋅⋅ − 92400000 x6y5 + ⋅⋅⋅
Maka koefisien x6y5 dari penj abaran (2x − 5y)11 adalah −92400000.

(x − 1x )10 ?
Cont oh 47 :
Apakah koefisen x6 pada penj abaran

(x − 1x )10 dij abarkan akan didapat :


Solusi :

(x − 1x )10 = L +10 C r (x )10−r (− 1x )r + L


Jika

⎜ x − ⎟ = L + 10 C r (− 1) ( x )
⎛ 1⎞
+L .
10 − 2 r
10

⎝ x⎠
r

Karena yang dit anyakan adalah koefisien x6 maka harus dipenuhi 10 − 2r = 6 sehingga r = 2.
Unt uk r = 2 didapat :

⎜ x − ⎟ = L + 10 C 2 (− 1) ( x ) + L = L + 45 x + L
⎛ 1⎞
10

⎝ ⎠
2 6 6

Maka koefisen x6 pada penj abaran (x − 1x ) adalah 45.


x
10

Selain digunakan dalam penj abaran suku-suku dari suat u binom, met ode yang digunakan dalam
segit iga pascal j uga dapat dit erapkan pada suat u persoalan menarik.

Cont oh 48 :
Tent ukan banyaknya cara menyusun kat a SUKA dari at as ke bawah pada susunan berikut j ika huruf-
huruf yang diambil harus berdekat an.
S
U U
K K K
A A A A

Solusi :
Jika dit uliskan sebagaimana met ode pascal didapat

1
1 1
1 2 1
1 3 3 1

Angka-angka di at as menyat akan banyaknya cara unt uk sampai pada angka t ersebut .
Dari angka-angka t ersebut didapat banyaknya cara unt uk menyusun kat a SUKA = 1 + 3 + 3 + 1 = 8.

Eddy Hermanto, ST 166 Kombinatorik


Pembinaan Olimpiade Matematika
S S S S
U U UU U U U U
K K K K K K K K K K K K
A A A A A A A A A A A A A A A A

S S S S
U U UU U U U U
K K K K K K K K K K K K
A A A A A A A A A A A A A A A A

Jadi, banyaknya cara menyusun kat a SUKA adalah 8.

Cont oh 49 :
Ada berapa banyak cara menyusun kat a MATHEMATICS dimulai dari at as ke bawah j ika huruf-huruf
yang diambil harus berdekat an.

M
A A
T T T
H HH H
EE E E E
M M M M M M
A A A A A
T T T T
I I I
C C
S

Solusi :
Kit a ubah huruf-huruf t ersebut dengan angka-angka sebagai berikut .

1
1 1
1 2 1
1 3 3 1
1 4 1 6 4
1 5
10 10 5 1
6 15 20 15 6
21 35 35 21
56 70 56
126 126
252

Maka banyaknya cara menyusun kat a MATHEMATICS adalah 252.

LATIHAN 1.F

1. Bukt ikan bahwa nCr = n-1Cr-1 + n-1Cr .

2. Jabarkan bent uk (3x − y) 6.

Eddy Hermanto, ST 167 Kombinatorik


Pembinaan Olimpiade Matematika

( )
3. Nur Faj ri berhasil menj abarkan bent uk (2x + 3y) 10. Apakah koefisien x6y4 yang didapat nya ?

4. (OSP 2010) Suku konst an dari x − adalah ⋅⋅⋅⋅⋅⋅⋅


5 8
2
x3

5. Tent ukan koefisien ab2c pada penj abaran (a + 3b − c) 4.

6. Tent ukan koefisien x3y2z4 pada penj abaran (x + y − 2z) 9.

7. Berapakah perbandingan koefisien suku x5 dengan koefisien suku x6 pada penj abaran (2x + 3) 20 ?

8. Jika (3x − 1)7 dij abarkan dalam suku-sukunya akan berbent uk a7x7 + a6x6 + a5x5 + ⋅⋅⋅ + a1x + ao.
Berapakah nilai a1 + a2 + a3 + a4 + a5 + a6 + a7 ?

9. Tent ukan nilai n dalam penj abaran (1 + x) n dengan n > 1, j ika diket ahui
a. koefisien suku x2 sama dengan koefisien suku x3.
b. koefisien suku x3 sama dengan lima kali koefisien suku x5.

10. Berapakah penj umlahan semua koefisien suku-suku pada penj abaran :
a. (x + y) 6
b. (a − 2b)8

11. Tent ukan nilai dari nC0 + nC1 + nC2 + ⋅⋅⋅ + nCn.

⎛ 2009 ⎞ ⎛ 2009 ⎞ ⎛ 2009 ⎞


12. (OSK 2009) Nilai eksak dari ⎜⎜ ⎟⎟ + ⎜⎜ ⎟⎟ + L + ⎜⎜ ⎟⎟ adalah ⋅⋅⋅⋅⋅⋅⋅⋅
⎝ 1 ⎠ ⎝ 2 ⎠ ⎝ 1004 ⎠

⎛ ⎛ n ⎞⎛ ⎛ j ⎞ i ⎞⎞
∑ ⎜⎜ ⎜⎜ j ⎟⎟⎜⎜ ∑ ⎜⎜ i ⎟⎟8 ⎟⎟ ⎟⎟ = ⋅⋅⋅⋅⋅⋅⋅⋅⋅
n j

⎝ ⎝ ⎠⎝ i =0 ⎝ ⎠ ⎠ ⎠
13. (OSP 2010) Nilai
j =0

14. Tent ukan banyaknya cara menyusun kat a OLIMPIADE j ika dimulai dari kiri at as ke kanan bawah
a. O L I M b. O L I M P c. O L I M P I
L I M P L I M P I L I M P I A
I M P I I M P I A I M P I A D
M P I A M P I A D M P I A D E
P I A D P I A D E
I A D E

15. (OSK 2011 Tipe 3) Koefisien x4 dari penj abaran (1 + 2x + 3x2) 10 adalah ⋅⋅⋅⋅⋅

16. (AIME 1983) Tent ukan sisanya j ika 683 + 883 dibagi 49.

17. (AIME 1986) Suku banyak 1 − x + x2 − x3 + ⋅⋅⋅ − x15 + x16 − x17 dapat dit ulis sebagai suku banyak
dalam variabel y dengan y = x + 1. Koefisien dari y2 adalah ⋅⋅⋅⋅⋅

18. (AIME 2001) Tent ukan penj umlahan semua akar-akar persamaan polinomial x2001 + ( 12 − x)2001 = 0.

Eddy Hermanto, ST 168 Kombinatorik


Pembinaan Olimpiade Matematika

2. KEJADIAN DAN PELUANG SUATU KEJADIAN, PENGAMBILAN CONTOH DENGAN DAN TANPA
PENGEMBALIAN

A. Percobaan
Misalkan kit a melempar sekeping uang logam, maka kegiat an ini disebut dengan percobaan. Hasil
percobaan yang didapat biasanya adalah munculnya sisi gambar, G, at au munculnya sisi t ulisan, T.
Sedangkan j ika kit a melempar sebuah dadu, maka hasil percobaan yang didapat adalah mat a dadu 1,
2, 3, 4, 5 at au 6.

B. Ruang Contoh atau Ruang Sampel


Ruang cont oh at au ruang sampel adalah himpunan dari semua hasil percobaan yang mungkin. Ruang
cont oh at au ruang sampel biasanya dilambangkan dengan S yang dalam t eori himpunan disebut
dengan himpunan semest a.
Pada percobaan melempar uang logam, ruang sampelnya adalah {G, T} sedangkan pada percobaan
melempar sat u buah dadu, ruang sampelnya adalah {1, 2, 3, 4, 5, 6}.
Jika {G, T} adalah ruang sampel, maka anggot a-anggot a dari ruang sampel t ersebut disebut t it ik
cont oh. Tit ik cont oh dari {G, T} adalah G dan T. Pada percobaan melempar sat u buah dadu, t it ik

36 buah t it ik cont oh, yait u (1, 1), (1, 2), (1, 3), ⋅⋅⋅, (6, 6).
sampel yang didapat ada 6 yait u 1, 2, 3, 4, 5, 6 sedangkan j ika melempar dua buah dadu akan didapat

C. Kejadian
Kej adian at au perist iwa (event ) adalah himpunan bagian dari ruang cont oh yang dapat berupa
kej adian sederhana maupun kej adian maj emuk. Kej adian sederhana adalah suat u kej adian yang
hanya mempunyai sebuah t it ik cont oh. Jika suat u kej adian memiliki lebih dari sat u t it ik cont oh
disebut dengan kej adian maj emuk.
Kej adian munculnya mat a dadu sat u {1} pada percobaan melempar sebuah dadu adalah cont oh
kej adian sederhana. Cont oh dari kej adian maj emuk adalah munculnya mat a dadu genap pada
percobaan melempar sebuah dadu.

D. Peluang Suatu Kejadian


1) Menghit ung peluang dengan pendekat an frekuensi
Dari suat u percobaan yang dilakukan sebanyak n kali, t ernyat a kej adian A munculnya sebanyak
k kali, maka frekuensi nisbi munculnya kej adian A sama dengan

F ( A) = ⋅⋅⋅⋅⋅⋅⋅⋅⋅⋅⋅⋅⋅⋅⋅⋅⋅⋅⋅⋅⋅⋅⋅⋅⋅⋅⋅⋅⋅⋅⋅
k
(2.D.1)
n
Kalau n semakin besar dan menuj u t ak t erhingga maka nilai F(A) akan cenderung konst an
mendekat i suat u nilai t ert ent u yang disebut dengan peluang munculnya kej adian A.

2) Menghit ung peluang dengan pendekat an definisi peluang klasik


Jika kit a melempar sekeping mat a uang logam secara berulang-ulang, frekuensi nisbi muculnya
sisi gambar maupun sisi t ulisan masing-masing akan mendekat i ½ sehingga dapat dikat akan
bahwa sisi gambar dan sisi t ulisan mempunyai kesempat an yang sama.
Misalkan dalam suat u percobaan menyebabkan dapat munculnya salah sat u dari n hasil yang
mempunyai kesempat an yang sama. Dari hasil n t adi, kej adian A muncul sebanyak k kali maka
peluang munculnya kej adian A sama dengan

P ( A) = ⋅⋅⋅⋅⋅⋅⋅⋅⋅⋅⋅⋅⋅⋅⋅⋅⋅⋅⋅⋅⋅⋅⋅⋅⋅⋅⋅⋅⋅⋅⋅
k
(2.D.2.1)
n
Selain it u, pengert ian peluang dapat j uga dij elaskan sebagai berikut .

Eddy Hermanto, ST 169 Kombinatorik


Pembinaan Olimpiade Matematika
Misalkan S adalah ruang cont oh dari suat u percobaan dengan t iap angot anya S memiliki
kesempat an muncul yang sama.
Jika A adalah suat u kej adian dengan A merupakan himpunan bagian dari S, maka peluang
kej adian A sama dengan :

P ( A) = ⋅⋅⋅⋅⋅⋅⋅⋅⋅⋅⋅⋅⋅⋅⋅⋅⋅⋅⋅⋅⋅⋅⋅⋅⋅⋅⋅⋅⋅⋅⋅
n( A)
(2.D.2.2)
n( S )

n(A) menyat akan banyaknya anggot a dalam himpunan A


n(S) menyat akan banyaknya anggot a dalam himpunan ruang cont oh S.

Dari pendekat an it u semua, j ika peluang suat u kej adian bernilai 0 maka art inya kej adian t ersebut

t erj adi. Peluang suat u kej adian akan berkisar 0 ≤ p(A) ≤ 1.


t idak mungkin t erj adi sedangkan j ika peluang suat u kej adian bernilai 1 art inya kej adian t ersebut past i

Berikut adalah beberapa cont oh persoalan menghit ung peluang suat u kej adian :

Cont oh 50 :
Berapa peluang kej adian munculnya angka ganj il pada percobaan melempar sebuah dadu ?

Solusi :
Pada percobaan melempar sebuah dadu bersisi enam ada 6 hasil yang mungkin muncul dan t iap hasil
mempunyai kesempat an yang sama, maka n = 6.
Kej adian munculnya angka ganj il ada 3 yait u 1, 3 dan 5. Maka k = 3.
Jadi, peluang kej adian = P ( A) = kn = 63 = 0,5.

Cont oh 51 :
Sebuah kot ak berisi 4 bola merah dan 5 bola put ih. Dari kot ak diambil sebuah bola secara acak.
Berapakah peluang bola yang t erambil adalah :
a. berwarna merah
b. berwarna put ih

Solusi :
Jumlah seluruh bola ada 9.
4
a. Banyaknya bola merah ada 4 maka peluang yang t erambil bola merah adalah 9 .
5
b. Banyaknya bola put ih ada 5 maka peluang yang t erambil bola put ih adalah 9 .

Cont oh 52 :
Sebuah kot ak berisi 3 kelereng merah, 4 kelereng hij au dan 5 kelereng biru. Jika dari dalam kot ak
diambil 3 buah kelereng, maka berapa peluang :
a. semua kelereng yang t erambil berwarna merah
b. semuanya biru
c. 2 buah berwarna merah dan 1 berwarna hij au
d. 1 berwarna merah, 1 hij au dan 1 biru

Solusi :
Jumlah seluruh kelereng ada 12.
Banyaknya cara memilih 3 dari 12 kelereng adalah 12C3 = 220.
a. Banyaknya cara memilih 3 kelereng merah dari 3 kelereng merah adalah 3C3 = 1

Eddy Hermanto, ST 170 Kombinatorik


Pembinaan Olimpiade Matematika
1
Peluang 3 kelereng yang t erambil semuanya merah = 220
b. Banyaknya cara memilih 3 kelereng biru dari 5 kelereng biru adalah 5C3 = 10
10 1
Peluang 3 kelereng yang t erambil semuanya biru = 220 = 22
c. Banyaknya cara memilih 2 kelereng merah dari 3 kelereng merah adalah 3C2 = 3
Banyaknya cara memilih 1 kelereng hij au dari 4 kelereng hij au adalah 4C1 = 4
Maka sesuai kaidah perkalian, banyaknya cara memilih 2 buah berwarna merah dan 1 berwarna
hij au = 3 x 4 = 12
12
Peluang 3 kelereng yang t erambil 2 buah berwarna merah dan 1 berwarna hij au = 220
d. Banyaknya cara memilih 1 kelereng merah dari 3 kelereng merah adalah 3C1 = 3
Banyaknya cara memilih 1 kelereng hij au dari 4 kelereng hij au adalah 4C1 = 4
Banyaknya cara memilih 1 kelereng biru dari 5 kelereng biru adalah 5C1 = 5
Maka banyaknya cara memilih 1 buah kelereng merah, 1 hij au dan 1 biru = 3 x 4 x 5 = 60
60
Peluang 3 kelereng yang t erambil 1 berwarna merah, 1 hij au dan 1 biru = 220

Cont oh 53 :
Dua buah dadu dilempar secara bersamaan. Berapakah peluang munculnya j umlah mat a dadu sama
dengan 9 ?

Himpunan semest anya ada 36 kemungkinan yait u (1, 1), (1, 2), (1, 3), ⋅⋅⋅, (6, 6).
Solusi :

Banyaknya kemungkinan j umlah mat a dadu kedua dadu t ersebut sama dengan 9 ada 4 kemungkinan
yait u (3, 6), (4, 5), (5, 4) dan (6, 3).
4
Maka peluang munculnya j umlah mat a dadu sama dengan 9 adalah 36 at au 19 .

Cont oh 54 :
Masing-masing sat u huruf diambil dari kat a “ MUDAH” dan “ BANGET” . Berapakah peluang bahwa kedua
huruf t ersebut t erdiri dari sat u vokal dan sat u konsonan ?

Solusi :
Kemungkinannya adalah sat u vokal dari kat a “ MUDAH” dan sat u konsonan dari kat a “ BANGET” at au

Peluang kasus pert ama = 52 x 64 = 154


sat u konsonan dari kat a “ MUDAH” dan sat u vokal dari kat a “ BANGET”

Peluang kasus kedua = 3


⋅ 62 = 1

+ 15 = 157 .
5 5
4
Maka peluang kej adian t ersebut adalah 15

LATIHAN 2.D

1. Dua buah dadu dilempar secara bersamaan. Berapakah peluang munculnya j umlah mat a dadu
paling t idak 9 ?

2. Alan melempar dua buah dadu bersamaan sat u kali. Berapakah peluang munculnya j umlah mat a
dadu ganj il dan bilangan prima ?

3. Di dalam sebuah kot ak t erdapat 4 bola merah dan 5 bola put ih. Jika Nindya mengambil 3 bola
secara acak maka berapakah peluang t erambilnya :
a. Ket iga-t iganya merah
b. Ket iga-t iganya put ih

Eddy Hermanto, ST 171 Kombinatorik


Pembinaan Olimpiade Matematika
c. 2 bola merah dan 1 bola put ih
d. 1 bola merah dan 2 bola put ih
Berapakah j umlah hasil a, b, c dan d t ersebut ?

secara bersamaan, peluang memperoleh dua bola berwarna sama adalah ⋅⋅⋅⋅⋅⋅
4. (OSK 2006) Dalam sebuah kot ak t erdapat 5 bola merah dan 10 bola put ih. Jika diambil dua bola

5. Sat u huruf diambil secara acak masing-masing dari kat a “ MAKAN” dan “ MANDI” . Berapakah
peluang t erambil dua huruf yang berbeda ?

6. Sebuah kot ak berisi 2006 bola yang t erdiri dari 500 bola merah, 501 bola biru, 502 bola kuning,
502 bola hij au dan 1 bola put ih. Jika dari dalam kot ak diambil bola sat u persat u t anpa
pengembalian, maka t ent ukan peluang bahwa t epat pada pengambilan kelima, bola t ersebut
adalah berwarna put ih.

7. (OSP 2003) Upik melemparkan n dadu. Ia menghit ung peluang t erj adinya j umlah mat a dadu sama
dengan 6. Unt uk n berapakah peluang t ersebut paling besar ?

8. Sepuluh buah bola masing-masing bert uliskan sat u huruf dari kat a MATEMATIKA. Dua bola diambil
secara acak dari sepuluh bola t ersebut . Peluang dua bola yang t erambil bert uliskan huruf yang
berbeda adalah ········

9. Denny berhasil menemukan 2007 kunci dan 1 buah pet i berisi hart a karun dengan 1 buah lubang
kunci. Hanya ada 1 dari 2007 kunci t ersebut yang bisa membuka pet i hart a karun. Ia memberi
t anda pada kunci yang t elah ia gunakan unt uk mencoba membuka pet i hart a karun, sehingga
kunci yang t elah digunakan unt uk mencoba, t idak akan digunakan lagi. Berapakah peluang t epat
pada percobaan ke-7 ia berhasil membuka pet i hart a karun t ersebut ?

10. (OSK 2010) Perempat final Liga Champion 2010 diikut i 8 t eam A, B, C, D, E, F, G dan H yang
bert emu sepert i t ampak dalam undian berikut

1
Set iap t eam mempunyai peluang unt uk melaj u ke babak berikut nya. Peluang kej adian A
bert emu G di final dan pada akhirnya A j uara adalah ⋅⋅⋅⋅⋅⋅
2

11. Sebuah bilangan empat angka berbeda dibent uk dari angka-angka 3, 4, 5 dan 6. Berapakah
peluang bahwa bilangan t ersebut habis dibagi 11.

12. Ahmadi melempar sebuah dadu dilempar 3 kali. Ada berapa cara munculnya j umlah mat a dadu
sama dengan 13 ?

13. (OSK 2010) Misalkan S menyat akan himpunan semua fakt or posit if dari 20102. Sebuah bilangan
diambil secara acak dari S. Peluang bilangan yang t erambil habis dibagi 2010 adalah ⋅⋅⋅⋅⋅

14. Terdapat dua buah kant ong. Kant ong pert ama berisi 5 bola merah dan 3 bola put ih. Kant ong
kedua berisi 4 bola put ih dan 6 bola biru. Sebuah bola diambil dari kant ong pert ama lalu
dimasukkan ke dalam kant ong kedua. Sebuah bola diambil secara acak dari kant ong kedua.
Berapa peluang bola yang t erambil berwarna :

Eddy Hermanto, ST 172 Kombinatorik


Pembinaan Olimpiade Matematika
a. biru
b. merah
c. put ih

15. Lima buah huruf diambil dari huruf-huruf A, B, C, D, E, F, G, H, I. Berapakah peluang yang
t erambil it u t erdiri dari 2 huruf hidup (vokal) dan 3 huruf mat i (konsonan) ?

16. Jika dua buah dadu dilempar bersamaan, berapakah peluang munculnya nilai mut lak selisih dua
dadu t ersebut t idak lebih dari dua ?

17. Dua buah bilangan diambil dari bilangan-bilangan 0, 1, 3, 5, 6, 8, 9. Tent ukan peluang bahwa
selisih kedua buah bilangan t ersebut adalah bilangan ganj il.

18. Terdapat persamaan x2 + ax + b = 0 dengan nilai a dan b diambil dari himpunan {1, 2, 3, 4, 5}.
Diket ahui bahwa a dan b adalah bilangan asli berbeda. Jika a dan b dipilih secara acak maka
peluang kedua akar persamaan x2 + ax + b = 0 merupakan bilangan real adalah ······

2006, 2007, 2008, 2009}. Peluang ab + c genap adalah ⋅⋅⋅⋅⋅


19. Diket ahui a, b, c adalah t iga bilangan berbeda yang angka-angkanya diambil dari himpunan {2005,

20. Tiga buah dadu dilempar sekaligus. Berapakah peluang bahwa hasil kali ket iga mat a dadu
menghasilkan bilangan genap dan penj umlahan ket iga mat a dadu j uga genap ?

21. Eka Yulit a memberi t anda pada sembilan buah kart u dengan angka 1, 2, 3, 4, 5, 6, 7, 8 dan 9.
Secara acak ia mengambil 4 buah kart u dari t umpukan kart u t ersebut sehingga membent uk
sebuah bilangan yang t erdiri dari 4 angka. Berapakah peluang bahwa bilangan t ersebut lebih dari
8000 dan habis dibagi 5 ?

22. Triesna mengambil 2 bilangan dari himpunan bilangan {1, 2, 3, ⋅⋅⋅, n − 1, n}. Peluang bahwa kedua
bilangan yang t erambil merupakan 2 bilangan yang berurut an adalah 20%. Tent ukan n.

23. Dua buah bent eng berbeda dilet akkan secara acak pada pet ak-pet ak papan cat ur 8 x 8. Berapakah
peluang kedua bent eng ini t idak saling memakan ?

24. Hansen memiliki 11 koin perak dan 1 koin emas. Furkan memiliki 12 koin perak. Secara acak 8
koin diambil dari Hansen lalu diberikan ke Furkan. Kemudian dari 20 koin yang dimiliki Furkan
t ersebut diambil 8 koin secara acak lalu diberikan kepada Hansen. Berdasarkan kej adian ini,
berapakah peluang koin emas ada pada Hansen ?

25. Keenam mat a dadu pada umumnya diberi angka 1, 2, 3, 4, 5 dan 6. Jika dua buah dadu dilempar
maka j umlah mat a dadu yang muncul adalah dari 2 sampai 12. Angka-angka pada suat u mat a
dadu digant i menj adi 1, 2, 2, 3, 3 dan 4. Misalkan ini adalah dadu A. Sebuah mat a dadu pada
dadu B j uga akan diberi angka-angka sehingga j ika dadu A dan B dilempar bersamaan akan
menghasilkan j umlah mat a dadu dari 2 sampai 12 j uga sert a peluang munculnya j umlah mat a
dadu set iap angka pada pelemparan kedua dadu t ersebut j uga sama dengan peluang munculnya

harus dit uliskan pada mat a dadu B sehingga hal t ersebut t erpenuhi adalah ⋅⋅⋅⋅⋅
j umlah mat a dadu set iap angka j ika melempar dua buah dadu yang umum. Angka-angka yang

26. Suat u set soal t erdiri dari 2 soal pilihan j awaban Benar (B) at au Salah (S) sert a 2 soal pilihan

peluang ia benar t epat dua soal adalah ⋅⋅⋅⋅⋅⋅⋅


ganda dengan pilihan j awaban A, B at au C. Jika seorang menj awab ke-4 soal secara acak, maka

27. Dari bilangan-bilangan 2006, 2007, 2008, 2009 dan 2010 akan diambil 3 bilangan. Berapakah
peluang j umlah ket iga bilangan t ersebut habis dibagi 3 ?

Eddy Hermanto, ST 173 Kombinatorik


Pembinaan Olimpiade Matematika
28. Pada sebuah dek kart u yang t erdiri dari 20 kart u, kart u pert ama berisi gambar segi-4 berat uran,
kart u kedua berisi gambar segi-5 berat uran, kart u ket iga berisi gambar segi-6 berat uran dan
set erusnya sehingga sehingga kart u ke-20 berisi gambar segi-23 berat uran. Sebuah kart u secara
acak diambil dari 20 t umpukan kart u t ersebut . Misalkan sudut dalam dari segi-n berat uran pada
kart u t ersebut adalah xo, maka berapakah peluang bahwa x adalah bilangan bulat ?

29. (OSN 2008 SMP/ MTs) Bilangan empat angka akan dibent uk dari angka-angka pada 0, 1, 2, 3, 4, 5
dengan syarat angka-angka pada bilangan t ersebut t idak berulang, dan bilangan yang t erbent uk
merupakan kelipat an 3. Berapakah peluang bilangan yang t erbent uk mempunyai nilai kurang dari
3000?

bilangan di ant ara {1, 2, 3, ⋅⋅⋅, 10}. Peluang bahwa bilangan yang dipilih Budi lebih dari j umlah 2
30. Andi secara acak memilih 2 bilangan di ant ara {1, 2, 3, 4, 5} dan Budi secara acak memilih 1

bilangan yang dipilih Andi adalah ⋅⋅⋅⋅

31. Jika sebuah bilangan dipilih secara acak dari bilangan-bilangan yang merupakan fakt or dari 1099
maka peluang bahwa bilangan t ersebut habis dibagi 1088 sama dengan ⋅⋅⋅⋅⋅

32. (OSP 2011) Misalkan A adalah himpunan semua pembagi posit if dari 109. Jika dipilih dua bilangan
sebarang x dan y di A (boleh sama), t ent ukan peluang dari kej adian x membagi y.

A(0, 2), B(4, 0), C(2π + 1, 0), D(2π + 1, 4) dan E(0, 4). Tent ukan peluang bahwa ∠APB t umpul ?
33. Tit ik P dipilih secara acak dan t erlet ak di dalam segilima ABCDE dengan koordinat t it ik sudut nya,

Misalkan ∠PAB = α, ∠PBA = β dan ∠APB = θ. Besarnya peluang θ adalah yang t erbesar di ant ara α,
34. ABCD adalah persegi panj ang dengan AB = 2 dan BC = 1. Tit ik P secara acak t erlet ak pada sisi CD.

β dan θ adalah ⋅⋅⋅⋅⋅⋅

35. (OSP 2006) Win memiliki dua koin. Ia akan melakukan prosedur berikut berulang-ulang selama ia
masih memiliki koin : lempar semua koin yang dimilikinya secara bersamaan; set iap koin yang
muncul dengan sisi angka akan diberikannya kepada Albert . Tent ukan peluang bahwa Win akan
mengulangi prosedur ini lebih dari t iga kali.

36. Tiga t usuk sat e berwarna merah, empat berwarna hij au dan lima berwarna biru dilet akkan dalam
sat u baris secara sebarangan. Tent ukan peluang bahwa t idak ada t usuk biru yang berdampingan ?

37. (OSP 2011) Dua dadu memiliki angka sat u sampai 6 yang dapat dilepas dari dadu. Kedua belas
angka t ersebut dilepaskan dari dadu dan dimasukkan ke dalam suat u kant ong. Secara acak
diambil sat u angka dan dipasangkan ke salah sat u dari kedua dadu t ersebut . Set elah semua angka

sebagai j umlah dari angka pada bagian at as kedua dadu t ersebut adalah ⋅⋅⋅⋅⋅
t erpasangkan, kedua dadu dilemparkan secara bersamaan. Peluang munculnya angka t uj uh

E. Pengambilan Contoh Dengan dan Tanpa Pengembalian


Sebelum menj elaskan t ent ang pengambilan cont oh dengan dan t anpa pengembalian, maka akan
dij elaskan t erlebih dulu mengenai kej adian bersyarat . Kej adian bersyarat adalah kej adian munculnya
B dengan persyarat an t elah munculnya kej adian A.
Rumus peluang munculnya kej adian B dengan syarat kej adian A t elah muncul adalah :

P(B A) =
P( B ∩ A)
dengan P(A) ≠ 0. ⋅⋅⋅⋅⋅⋅⋅⋅⋅⋅⋅⋅⋅⋅⋅⋅⋅⋅⋅⋅⋅⋅⋅⋅⋅⋅⋅⋅⋅⋅⋅ (2.E.1)
P( A)

At au j ika ingin menghit ung P(A∩B)

Eddy Hermanto, ST 174 Kombinatorik


Pembinaan Olimpiade Matematika

P(B∩A) = p(B⏐A) x p(A) ⋅⋅⋅⋅⋅⋅⋅⋅⋅⋅⋅⋅⋅⋅⋅⋅⋅⋅⋅⋅⋅⋅⋅⋅⋅⋅⋅⋅⋅⋅⋅ (2.E.2)

Misalkan kit a akan mengambil dua kart u bridge dari t umpukan 1 set kart u bridge secara berurut an
sat u persat u. Ada 2 cara pengambilan dua kart u t ersebut . Yang pert ama adalah set elah pengambilan
kart u pert ama maka kart u pert ama t ersebut dikembalikan lagi ke dalam t umpukan 1 set kart u bridge
dan kemudian mengambil kart u kedua. Ini dinamakan dengan pengambilan cont oh dengan
pengembalian. Cara kedua adalah set elah pengambilan kart u pert ama maka kart u pert ama t ersebut
t idak dikembalikan lagi ke dalam t umpukan 1 set kart u bridge dan kemudian mengambil kart u kedua.
Ini dinamakan dengan pengambilan cont oh t anpa pengembalian. Pengambilan Cont oh dengan dan
t anpa pengembalian merupakan kej adian bersyarat .

1) Pengambilan Cont oh dengan Pengembalian

Cont oh 55 :
Dari sebuah t umpukan kart u bridge, diambil dua kart u sat u persat u dengan pengembalian.
Berapakah peluang kart u pert ama yang t erambil adalah kart u As dan kart u kedua adalah kart u
berwarna merah ?

Solusi :
Misalkan A adalah kej adian t erambilnya kart u pert ama adalah kart u As.
B adalah kej adian t erambilnya kart u kedua adalah kart u merah.
4
Kart u pert ama yang t erambil adalah kart u As maka peluangnya adalah = p(A) = 52 .
Set elah dikembalikan, j umlah kart u t et ap 52 dengan kart u berwarna merah t et ap 26 buah.
Maka peluang pengambilan kart u kedua adalah kart u berwarna merah set elah pengambilan
26
kart u pert ama adalah kart u As adalah = p(B⏐A) = 52 .

= 261 .
Maka peluang pengambilan pert ama adalah kart u As dan pengambilan kedua adalah kart u
4 26
berwarna merah adalah = P(A∩B) = p(B⏐A) x p(A) = 52 x 52

Cont oh 56 :
Dalam sebuah kot ak t erdapat 3 bola merah dan 4 bola biru. Berapakah peluang j ika diambil dua
bola sat u persat u dengan pengembalian dengan bola pert ama berwarna merah dan bola kedua
berwarna biru ?

Solusi :
Misalkan A adalah kej adian t erambilnya bola pert ama adalah bola merah.
B adalah kej adian t erambilnya bola kedua adalah bola biru.
Bola pert ama yang t erambil adalah bola merah maka peluangnya adalah = p(A) = 73 .
Set elah dikembalikan, j umlah bola t et ap 7 dengan bola biru t et ap 4 buah.
Maka peluang pengambilan bola kedua adalah bola biru set elah pengambilan bola pert ama
adalah merah adalah = p(B⏐A) = 74 .

adalah = P(A∩B) = p(B⏐A) x p(A) = 73 x 74 = 12


Maka peluang pengambilan pert ama adalah bola merah dan pengambilan kedua adalah bola biru
49 .

2) Pengambilan Cont oh Tanpa Pengembalian

Cont oh 57 :
Dari sebuah t umpukan kart u bridge, diambil dua kart u sat u persat u t anpa pengembalian.
Berapakah peluang kart u pert ama yang t erambil adalah kart u As dan kart u kedua adalah kart u
King ?

Eddy Hermanto, ST 175 Kombinatorik


Pembinaan Olimpiade Matematika

Solusi :
Misalkan A adalah kej adian t erambilnya kart u pert ama adalah kart u As.
B adalah kej adian t erambilnya kart u kedua adalah kart u King.
4
Kart u pert ama yang t erambil adalah kart u As maka peluangnya adalah = p(A) = 52 .
Karena kart u t idak dikembalikan maka j umlah kart u t inggal 51 dengan kart u King t et ap 4 buah
j ika kart u pert ama yang t erambil adalah kart u As.
Maka peluang pengambilan kart u kedua adalah kart u berwarna King set elah pengambilan kart u
4
pert ama adalah kart u As adalah = p(B⏐A) = 51 .

x 514 = 663
Maka peluang pengambilan pert ama adalah kart u As dan pengambilan kedua adalah kart u King
4 4
adalah = P(A∩B) = p(B⏐A) x p(A) = 52 .

Cont oh 58 :
Dalam sebuah kot ak t erdapat 3 bola merah dan 4 bola biru. Berapakah peluang j ika diambil dua
bola sat u persat u t anpa pengembalian dengan bola pert ama berwarna biru dan bola kedua
berwarna j uga biru ?

Solusi :
Misalkan A adalah kej adian t erambilnya bola pert ama adalah bola biru.
B adalah kej adian t erambilnya bola kedua adalah bola biru.
Bola pert ama yang t erambil adalah bola biru maka peluangnya adalah = p(A) = 74 .
Karena t anpa pengembalian, j umlah bola t inggal 6 dengan bola biru t inggal 3 buah j ika bola
pert ama yang t erambil adalah bola biru.
Maka peluang pengambilan bola kedua adalah bola biru set elah pengambilan bola pert ama
adalah biru adalah = p(B⏐A) = 63 .

biru adalah = P(A∩B) = p(B⏐A) x p(A) = 74 x 63 = 72 .


Maka peluang pengambilan pert ama adalah bola biru dan pengambilan kedua adalah j uga bola

LATIHAN 2.E

1. Dalam sebuah kant ong t erdapat 6 manik kuning, 4 manik biru dan 3 manik hit am. Jika diambil 2
manik sat u persat u, maka berapakah peluang manik pert ama yang t erambil berwarna kuning dan
kedua hit am apabila :
a. dengan pengembalian
b. t anpa pengembalian

2. Dalam sebuah kot ak t erdapat 5 kelereng merah, 3 kelereng hij au dan 2 kelereng hit am. Jika
diambil 3 kelereng sat u persat u, maka berapakah peluang kelereng pert ama yang t erambil
berwarna merah, kedua hij au dan ket iga merah apabila :
a. dengan pengembalian ?
b. t anpa pengembalian ?

3. Dalam sebuah kot ak t erdapat 6 bola merah dan 4 bola biru. Dari dalam kot ak it u diambil dua bola
sat u demi sat u t anpa pengembalian. Berapa peluang yang t erambil it u :
a. bola merah pada pengambilan pert ama maupun kedua ?
b. bola merah pada pengambilan pert ama dan bola biru pada pengambilan kedua ?
c. bola biru pada pengambilan pert ama dan bola merah pada pengambilan kedua ?
d. bola biru pada pengambilan pert ama maupun kedua ?
Berapakah penj umlahan a, b, c dan d ? Apa kesimpulan Anda.

Eddy Hermanto, ST 176 Kombinatorik


Pembinaan Olimpiade Matematika

4. Dari 1 set kart u bridge diambil dua buah kart u sat u persat u t anpa pengembalian. Berapakah
peluang kart u pert ama yang t erambil adalah kart u As dan kart u kedua adalah kart u hat i ?

5. Dua kart u bridge diambil berurut an secara random dari sat u set kart u bridge. Kart u pert ama
dikembalikan dan kart u diacak kembali, set elah it u kart u kedua diambil. Berapa peluang paling
sedikit sat u dari kedua kart u yang diambil adalah kart u As ?

F. Peluang Komplemen
Misalkan saj a kej adian yang t erj adi adalah huj an at au t idak huj an sert a t idak ada kej adian di ant ara
huj an at au t idak huj an. Seandainya kit a t ahu t ahu bahwa peluang hari ini huj an adalah 60 % maka
t ent unya kit a bisa menghit ung peluang hari ini t idak huj an, yait u 40 %.
Kej adian huj an dan kej adian t idak huj an adalah dua kej adian yang saling komplemen. Jika kej adian
huj an dit ulis dengan A maka kej adian t idak huj an dit ulis dengan A’ at au Ac yang dibaca komplemen
dari A.
Jika A dan A’ adalah dua kej adian yang saling komplemen, maka peluang A’ (dit ulis p(A’ )) dirumuskan
dengan :

p(A’ ) = 1 − p(A) ⋅⋅⋅⋅⋅⋅⋅⋅⋅⋅⋅⋅⋅⋅⋅⋅⋅⋅⋅⋅⋅⋅⋅⋅⋅⋅⋅⋅⋅⋅⋅ (2.F.1)

Cont oh 59 :
Peluang Ahmadi unt uk lulus uj ian adalah 0,8. Berapakah peluang Ahmadi t idak lulus uj ian ?

Peluang Ahmadi unt uk t idak lulus uj ian = 1 − 0,8 = 0,2


Solusi :

Cont oh 60 :
Dalam sebuah kot ak t erdapat 4 manik merah dan 3 manik hij au. Jika diambil 3 buah manik,
berapakah peluang manik yang t erambilnya paling banyak 2 buah berwarna merah ?

Solusi :
Alt ernat if 1 :
Kemungkinan kasus-kasus ini adalah 2 manik merah dan 1 manik hij au at au 1 manik merah dan 2
manik hij au at au ket iga-t iganya manik hij au.
Banyaknya cara memilih adalah 4C2 x 3C1 + 4C1x 3C2+ 4C0x 3C3 = 18 + 12 + 1 = 31 cara.
Tiga manik diambil dari 7 manik, banyaknya cara = 7C3 = 35.
31
Peluang yang t erambil adalah paling banyak 2 manik merah = 35

Alt ernat if 2 :
Komplemen paling banyak 2 manik merah dalam soal ini adalah ket iga manik yang t erambil semuanya
berwarna merah. Banyaknya cara memilih adalah 4C3 x 3C0 = 4 cara.
4
Peluang yang t erambil adalah 3 manik merah = 35 .
Peluang yang t erambil adalah paling banyak 2 manik merah = 1 − 4
35 = 31
35 .

Eddy Hermanto, ST 177 Kombinatorik


Pembinaan Olimpiade Matematika
LATIHAN 2.F

1. Banyaknya bilangan t iga angka yang mempunyai sedikit nya sat u angka genap adalah ⋅⋅⋅⋅⋅

2. Pada papan cat ur 4 x 4 dilet akkan secara acak 4 buah pion ident ik. Berapakah peluang bahwa
keempat pion t ersebut t idak berada pada sat u garis ?

3. Dalam sebuah kot ak t erdapat 7 bola hit am dan 6 bola put ih. Jika diambil 5 buah bola, berapakah
peluang bola yang t erambilnya sedikit nya 2 buah berwarna hit am ?

4. Dalam sebuah kot ak t erdapat 7 bola hit am, 4 bola put ih dan 3 bola merah. Jika diambil 5 buah
bola, berapakah peluang bola yang t erambilnya sedikit nya 2 buah berwarna hit am ?

5. Sebuah kot ak berisi 6 bola merah dan 6 bola put ih. Secara acak Lala mengambil dua bola
sekaligus. Berapakah peluang unt uk mendapat kan dua bola berwarna sama ?

6. Dua kart u bridge diambil berurut an secara random dari sat u set kart u bridge. Kart u pert ama
dikembalikan dan kart u diacak kembali, set elah it u kart u kedua diambil. Berapa peluang paling
sedikit sat u dari kedua kart u yang diambil adalah kart u As ?

yang t erambil ada yang berpasangan adalah ⋅⋅⋅⋅⋅⋅


7. (OSP 2009) Ada empat pasang sepat u akan diambil empat sepat u secara acak. Peluang bahwa

Eddy Hermanto, ST 178 Kombinatorik


Pembinaan Olimpiade Matematika

3. PRINSIP INKLUSI EKSKLUSI, PELUANG KEJADIAN MAJEMUK

A. Prinsip Inklusi Eksklusi


Prinsip Inklusi dan Eksklusi (PIE) adalah bent uk paling umum dari prinsip penambahan pada himpunan.
Perhat ikan gabungan dua himpunan pada diagram venn di bawah.

Himpunan A Himpunan A∩B Himpunan B − A

Misalkan S adalah suat u himpunan t erhingga dengan A dan B sembarang dua himpunan bagian dari S.

banyaknya unsur himpunan A dan himpunan B − A dan kemudian menj umlahkannya. Karena ⏐B − A⏐ =
Maka unt uk mencacah banyaknya unsur di dalam A∪B, kit a dapat melakukannya dengan mencacah

⏐B⏐ − ⏐A∩B⏐ maka :

⏐A∪B⏐ = ⏐A⏐ + ⏐B⏐ − ⏐A∩B⏐ ⋅⋅⋅⋅⋅⋅⋅⋅⋅⋅⋅⋅⋅⋅⋅⋅⋅⋅⋅⋅⋅⋅⋅⋅⋅⋅⋅⋅⋅⋅⋅ (3.A.1)

Cat at an : Not asi ⏐A∪B⏐ dalam buku lain kadang-kadang dit ulis dengan n(A∪B).
Dengan kat a lain, ket ika mencacah unsur-unsur A dan B sendiri-sendiri, unsur-unsur irisan A dan B

pencacahan dari ⏐A⏐ + ⏐B⏐ dengan pada A∩B sekali.


t ercacah dua kali sehingga unt uk mengat asi pencacahan ganda ini, kit a harus mengurangkan hasil

Selain rumus pada persamaan 3.A.1, pada gabungan dua himpunan berlaku persamaan :

⏐(A∪B)’ ⏐ = ⏐S⏐ − ⏐A∪B⏐ ⋅⋅⋅⋅⋅⋅⋅⋅⋅⋅⋅⋅⋅⋅⋅⋅⋅⋅⋅⋅⋅⋅⋅⋅⋅⋅⋅⋅⋅⋅⋅ (3.A.2)

dengan t anda “ ’ ” menyat akan komplemen.


Sesuai hukum de Morgan berlaku :

(A∪B)’ = A’ ∩B’ ⋅⋅⋅⋅⋅⋅⋅⋅⋅⋅⋅⋅⋅⋅⋅⋅⋅⋅⋅⋅⋅⋅⋅⋅⋅⋅⋅⋅⋅⋅⋅ (3.A.3)

(A∩B)’ = A’ ∪B’ ⋅⋅⋅⋅⋅⋅⋅⋅⋅⋅⋅⋅⋅⋅⋅⋅⋅⋅⋅⋅⋅⋅⋅⋅⋅⋅⋅⋅⋅⋅⋅ (3.A.4)

Perhat ikan gabungan t iga himpunan berikut :

Ket ika mencacah unsur-unsur A (a + d + f + g), B (b + d + e + g) dan C (c + e + f + g) sendiri-sendiri,


unsur-unsur irisan t epat A dan B (d), irisan t epat A dan C (f) sert a irisan t epat B dan C (e) t ercacah

ini, kit a harus mengurangkan hasil pencacahan ⏐A⏐ + ⏐B⏐ + ⏐C⏐ masing-masing sekali dengan A∩B,
dua kali sedangkan irisan A, B dan C (g) t ercacah t iga kali. Maka unt uk mengat asi pencacahan ganda

Eddy Hermanto, ST 179 Kombinatorik


Pembinaan Olimpiade Matematika
A∩C dan B∩C. Namun ket ika kit a menghit ung ⏐A∪B∪C⏐ = ⏐A⏐ + ⏐B⏐ +⏐C⏐ − ⏐A∩B⏐ − ⏐A∩C⏐ −
⏐B∩C⏐, irisan A, B dan C (A ∩ B ∩ C) belum t ercacah sama sekali. Unt uk mengat asi hal t ersebut kit a
masih harus menambahkan ⏐A⏐ + ⏐B⏐ +⏐C⏐ − ⏐A∩B⏐ − ⏐A∩C⏐ − ⏐B∩C⏐ dengan ⏐A∩B∩C⏐ sekali.
Maka didapat rumus :

⏐A∪B∪C⏐ = ⏐A⏐ + ⏐B⏐ +⏐C⏐ − ⏐A∩B⏐ − ⏐A∩C⏐ − ⏐B∩C⏐ + ⏐A∩B∩C⏐ ⋅⋅⋅⋅⋅⋅⋅⋅ (3.A.5)

⏐(A∪B∪C)’ ⏐ = ⏐S⏐ − ⏐A∪B∪C⏐ ⋅⋅⋅⋅⋅⋅⋅⋅⋅⋅⋅⋅⋅⋅⋅⋅⋅⋅⋅⋅⋅⋅⋅⋅⋅⋅⋅⋅⋅⋅⋅ (3.A.6)

Jika dikembangkan unt uk gabungan 4 himpunan akan didapat kan :

⏐A∪B∪C∪D⏐ = ⏐A⏐ + ⏐B⏐ +⏐C⏐ + ⏐D⏐ − ⏐A∩B⏐ − ⏐A∩C⏐ − ⏐A∩D⏐− ⏐B∩C⏐ − ⏐B∩D⏐ − ⏐C∩D⏐ +
⏐A∩B∩C⏐ + ⏐A∩B∩D⏐ + ⏐A∩C∩D⏐ + ⏐B∩C∩D⏐ − ⏐A∩B∩C∩D⏐ ⋅⋅⋅⋅⋅⋅⋅⋅⋅⋅⋅⋅⋅⋅⋅⋅⋅ (3.A.7)

⏐(A∪B∪C∪D)’ ⏐ = ⏐S⏐ − ⏐A∪B∪C∪D⏐ ⋅⋅⋅⋅⋅⋅⋅⋅⋅⋅⋅⋅⋅⋅⋅⋅⋅⋅⋅⋅⋅⋅⋅⋅⋅⋅⋅⋅⋅⋅⋅ (3.A.8)

Demikian set erusnya.

Cont oh 61 :
Dari 65 siswa yang disurvei didapat kan dat a bahwa ada 20 siswa yang menyukai Fisika dan 35
menyukai Mat emat ika. Jika t erdapat 25 siswa yang t idak menyukai Mat emat ika maupun Fisika maka
ada berapa siswa yang menyukai Mat emat ika dan Fisika ?

Solusi :
Misalkan F adalah himpunan siswa yang menyukai Fisika dan M adalah himpunan siswa yang menyukai

⏐F⏐ = 20 ; ⏐M⏐ = 35 ; ⏐S⏐ = 65 ; ⏐(F∪M)’ ⏐ = 25


Mat emat ika.

⏐S⏐ = ⏐F∪M⏐ + ⏐(F∪M)’ ⏐ sehingga ⏐F∪M⏐ = 40

⏐F∪M⏐ = ⏐F⏐ + ⏐M⏐ − ⏐F∩M⏐


40 = 20 + 35 − ⏐F∩M⏐ sehingga ⏐F∩M⏐ = 15
Banyaknya siswa yang menyukai Mat emat ika dan Fisika ada 15 orang.

Cont oh 62 :
Sebuah survei dilakukan t erhadap sekumpulan siswa. Dari survei t ersebut didapat bahwa 133 orang
menyukai sedikit nya sat u dari 3 pelaj aran Fisika, Mat emat ika at au Kimia. Sembilan puluh enam di
ant aranya menyukai Mat emat ika, 70 menyukai Fisika dan 66 menyukai Kimia. Dari 96 siswa yang
menyukai Mat emat ika, 40 di ant arnya menyukai Fisika dan 45 di ant aranya menyukai Kimia.
Banyaknya siswa yang menyukai Fisika dan Kimia ada sebanyak 28 orang. Ada berapa banyak siswa
yang menyukai ket iga mat a pelaj aran t ersebut ?

Misalkan banyaknya siswa yang menyukai ket iga pelaj aran = ⏐F∩M∩K⏐ = x
Solusi :

menyukai Mat emat ika dan Fisika t et api t idak menyukai Kimia = 40 − x
Dari dat a didapat yang menyukai Mat emat ika dan Fisika ada sebanyak 40 maka banyaknya siswa yang

menyukai Mat emat ika dan Kimia t et api t idak menyukai Fisika = 45 − x.
Dari dat a didapat yang menyukai Mat emat ika dan Kimia ada sebanyak 45 maka banyaknya siswa yang

menyukai Fisika dan Kimia t et api t idak menyukai Mat emat ika = 28 − x.
Dari dat a didapat yang menyukai Fisika dan Kimia ada sebanyak 28 maka banyaknya siswa yang

Banyaknya siswa yang hanya menyukai Mat emat ika = 96 − (40 − x) − x − (45 − x) = 11 + x
Banyaknya siswa yang hanya menyukai Fisika = 70 − (40 − x) − x − (28 − x) = 2 + x
Banyaknya siswa yang hanya menyukai Kimia = 66 − (28 − x) − x − (45 − x) = x − 7

Eddy Hermanto, ST 180 Kombinatorik


Pembinaan Olimpiade Matematika
Jika digambarkan dalam diagram venn maka :

96 + (2 + x) + (28 − x) + (x − 7) = 133
x = 14
Maka banyaknya siswa yang menyukai ket iga pelaj aran ada sebanyak 14 orang.

Cont oh 63 :
Sebuah sekolah memiliki 760 siswa. Ada 71 siswa yang mengikut i Klub Musik dan 110 siswa yang t idak
mengikut i Klub Sains. Pada Klub Sains, j umlah siswa laki-laki 30 lebih banyak daripada siswa
perempuan. Lima puluh sembilan siswa dengan 35 di ant aranya perempuan mengikut i Klub Musik
maupun Klub Sains. Diket ahui j uga bahwa 86 siswa laki-laki t idak mengikut i Klub Sains maupun Klub
Musik. Set engah dari siswa yang mengikut i Klub Musik t et api t idak mengikut i Klub Sains adalah laki-
laki. Hit unglah :
a. Banyaknya siswa laki-laki dan siswa perempuan di sekolah t ersebut ?
b. Banyaknya siswa perempuan yang t idak mengikut i Klub Musik maupun Klub Sains.

Solusi :
Misalkan banyaknya siswa laki-laki yang mengikut i Klub Musik t et api t idak mengikut i Klub Sains = x.
Maka banyaknya siswa perempuan siswa laki-laki yang mengikut i Klub Musik t et api t idak mengikut i

⏐S⏐ = 760 ; ⏐Musik⏐ = 71 ; ⏐(Sains)’ ⏐ = 110 sehingga ⏐Sains⏐ = 760 − 110 = 650
Klub Sains j uga = x.

Karena ada 59 siswa yang mengikut i Klub Sains maupun Klub Musik dengan 35 di ant aranya perempuan
maka ada 24 siswa laki-laki mengikut i Klub Sains maupun Klub Musik.
Misalkan banyaknya siswa perempuan yang mengikut i Klub Sains t et api t idak mengikut i Klub Musik =
p. Karena pada Klub Sains, j umlah siswa laki-laki 30 lebih banyak daripada siswa perempuan maka
banyaknya siswa laki-laki yang mengikut i Klub Sains t et api t idak mengikut i Klub Musik = p + 41.

Perhat ikan diagram venn berikut .

Pada Klub Sains berlaku :


24 + 35 + (p + 41) + (p) = 650
p = 275

Eddy Hermanto, ST 181 Kombinatorik


Pembinaan Olimpiade Matematika
Pada Klub Musik berlaku :
x + x + 24 + 35 = 71
x=6
Secara keseluruhan :
86 + 650 + 6 + 6 + y = 760
y = 12
a. Banyaknya siswa laki-laki = 86 + 6 + 24 + (275 + 41) = 432
Banyaknya siswa perempuan = 12 + 6 + 35 + (275) = 328
b. Banyaknya siswa perempuan yang t idak mengikut i Klub Musik maupun Klub Sains = y = 12

Cont oh 64 :
Sebanyak n orang pengurus sebuah organisasi akan dibagi ke dalam empat komisi mengikut i ket ent uan
berikut : (i) set iap anggot a t ergabung kedalam t epat dua komisi, dan (ii) set iap dua komisi memiliki
t epat sat u anggot a bersama. Berapakah n ?

Solusi :
(a) set iap anggot a t ergabung ke dalam t epat dua komisi
(b) set iap dua komisi memiliki t epat sat u anggot a bersama

Karena ada 4 komisi maka banyaknya pasangan komisi yang bisa dibuat adalah 4C2 = 6.
Karena banyaknya pasangan komisi ada 6 maka banyaknya anggot a minimal adalah 6 sebab j ika
kurang dari 6 maka akan ada seorang anggot a yang t ergabung dalam lebih dari 2 komisi.
Jika t erdapat lebih dari 6 anggot a maka akan ada seorang anggot a yang masuk dalam sebuah komisi
t et api t idak masuk ke dalam t iga komisi lain. Hal ini bert ent angan dengan (a) bahwa seorang anggot a
t ergabung ke dalam t epat dua komisi. Akibat nya banyaknya anggot a ada 6 orang.

Misalkan komisi t ersebut adalah A, B, C dan D dengan ai menyat akan anggot a ke-i dengan 1 ≤ i ≤ 6.
Cont oh pembagian keenam anggot a ke dalam empat komisi yang memenuhi (a) dan (b) adalah :

Komisi A Komisi B Komisi C Komisi D


a1 a1 a2 a3
a2 a4 a4 a5
a3 a5 a6 a6
Coba kerj akan dengan Prinsip Inklusi Eksklusi.

Cont oh 65 :
Suat u kat a biner yang panj angnya n adalah suat u barisan/ sekuens angka-angka 0 at au 1 yang
panj angnya n. Berapa banyak kat a biner dengan panj ang 10 yang diawali dengan t iga angka 0 at au
diakhiri dengan dua angka 1 ?

Solusi :
Banyaknya kat a biner dengan panj ang 10 yang diawali dengan t iga angka 0 adalah sama dengan
memilih angka 0 at au 1 pada 7 angka t erakhir sebab 3 angka pert ama t idak dapat dipilih. Pilihannya
masing-masing angka hanya 0 at au 1. Banyaknya cara =⏐A⏐= 27.
Banyaknya kat a biner dengan panj ang 10 yang diakhiri dengan dua angka 1 adalah sama dengan

masing-masing angka hanya 0 at au 1. Banyaknya cara = ⏐B⏐ = 28.


memilih angka 0 at au 1 pada 8 angka pert ama sebab 2 angka t erakhir t idak dapat dipilih. Pilihannya

Banyaknya kat a ini = ⏐A∩B⏐ = 25.


Tet api ada kej adian kat a biner t ersebut diawali dengan t iga angka 0 dan diakhiri dengan dua angka 1.

Banyaknya kat a biner yang dapat disusun = ⏐A∪B⏐ = ⏐A⏐ + ⏐B⏐ − ⏐A∩B⏐
Maka banyaknya kat a biner yang dapat disusun = 27 + 28 − 25 = 352

Eddy Hermanto, ST 182 Kombinatorik


Pembinaan Olimpiade Matematika
Cont oh 66 :
Ada berapa banyak bilangan bulat ant ara 1 dan 1000 (t ermasuk 1 dan 1000) yang t idak habis dibagi 2
dan t idak habis dibagi 5 ?

Solusi :
Misalkan A’ adalah himpunan bilangan bulat ant ara 1 dan 1000 (t ermasuk 1 dan 1000) yang t idak habis
dibagi 2 dan B’ adalah himpunan bilangan bulat ant ara 1 dan 1000 (t ermasuk 1 dan 1000) yang t idak
habis dibagi 5 sert a S adalah himpunan bilangan bulat ant ara 1 dan 1000 (t ermasuk 1 dan 1000).
Maka A menyat akan himpunan bilangan bulat ant ara 1 dan 1000 (t ermasuk 1 dan 1000) yang habis
dibagi 2 dan B menyat akan himpunan bilangan bulat ant ara 1 dan 1000 (t ermasuk 1 dan 1000) yang

⏐S⏐ = 1000
habis dibagi 5.

Persoalan yang dit anyakan adalah A’ ∩B’ . Dengan hukum de Morgan maka (A’ ∩B’ ) = (A∪B)’
⏐A⏐ = ⎣1000
2 ⎦ = 500 dan ⏐B⏐ = ⎣ 5 ⎦ = 200
1000

dengan t anda ⎣α⎦ menyat akan bilangan bulat t erbesar kurang dari at au sama dengan α.

⏐A∩B⏐ = ⎣1000
10 ⎦ = 100
Maka A∩B akan menyat akan himpunan bilangan bulat yang habis dibagi 2 dan 5 at au habis dibagi 10.

⏐A∪B⏐ = ⏐A⏐ + ⏐B⏐ − ⏐A∩B⏐ = 600


⏐(A∪B)’ ⏐ = ⏐S⏐ − ⏐(A∪B)⏐ = 400
Maka banyaknya bilangan bulat ant ara 1 dan 1000 (t ermasuk 1 dan 1000) yang t idak habis dibagi 2 dan
t idak habis dibagi 5 = 400 buah.

LATIHAN 3.A

1. Pada suat u klub Musik, 14 orang bermain piano at au git ar, 8 orang adalah pemain piano dan 5
orang memainkan kedua alat t ersebut . Ada berapa orang yang memainkan git ar ?

2. Dari 100 orang siswa t erdapat 30 siswa yang hanya menyukai sepakbola saj a. yang menyukai bola
volly ada 50 siswa. Ada berapa siswa yang t idak menyukai sepak bola maupun bola volly ?

3. Dari 240 siswa kelas 3 suat u SMA, t erdapat 50 orang menyukai sepakbola, 60 orang menyukai
bulut angkis dan 55 menyukai cat ur. Tiga puluh siswa menyukai sepakbola dan bulut angkis
sement ara 10 siswa menyukai bulut angkis dan cat ur t et api t idak menyukai sepakbola. Ada 20
siswa yang menyukai ket iga hobi t ersebut . Jika ada 150 siswa yang t idak menyukai sat u pun di
ant ara ket iga hobi t ersebut , maka ada berapa siswa yang menyukai sepakbola dan cat ur t et api
t idak menyukai bulut angkis ?

4. Ada berapa banyak bilangan bulat ant ara 1 dan 1000 (t ermasuk 1 dan 1000) yang t idak habis
dibagi 2 dan t idak habis dibagi 7 ?

5. Ada berapa banyak bilangan bulat ant ara 1 dan 1000 (t ermasuk 1 dan 1000) yang t idak habis
dibagi 2, 3 dan 7 ?

6. Tent ukan banyaknya bilangan yang t idak habis dibagi 3, 7 dan 11 dan t erlet ak ant ara 79 dan 2120
(t idak t ermasuk 79 dan 2120).

7. Di dalam suat u kelas beberapa orang mempelaj ari Bahasa Inggris sedangkan sisanya mempelaj ari
Bahasa Jerman t et api t idak ada siswa yang mempelaj ari keduanya. Jumlah siswa perempuan yang
mempelaj ari Bahasa Inggris dan siswa laki-laki yang mempelaj ari Bahasa Jerman adalah 16 orang.
Ada 11 siswa yang mempelaj ari Bahasa Inggris dan ada 10 siswa perempuan di kelas. Selain siswa
perempuan yang mempelaj ari Bahasa Inggris, ada 16 orang siswa di kelas. Berapa banyakkah t ot al
siswa di kelas ?

Eddy Hermanto, ST 183 Kombinatorik


Pembinaan Olimpiade Matematika

Jika persamaan-persamaan 3.A.1, 3.A.2, 3.A.5, 3.A.6 kit a bagi dengan ⏐S⏐ dan dengan
B. Peluang Kejadian Majemuk

memperhat ikan pengert ian peluang pada bagian 2.D.2.2 akan didapat rumus-rumus peluang sebagai
berikut :

Unt uk gabungan 2 himpunan :

p(A∪B) = p(A) + p(B) − p(A∩B) ⋅⋅⋅⋅⋅⋅⋅⋅⋅⋅⋅⋅⋅⋅⋅⋅⋅⋅⋅⋅⋅⋅⋅⋅⋅⋅⋅⋅⋅⋅⋅ (3.B.1)

p(A∪B)’ = 1 − p(A∪B) ⋅⋅⋅⋅⋅⋅⋅⋅⋅⋅⋅⋅⋅⋅⋅⋅⋅⋅⋅⋅⋅⋅⋅⋅⋅⋅⋅⋅⋅⋅⋅ (3.B.2)

Unt uk gabungan 3 himpunan :

p(A∪B∪C) = p(A) + p(B) +p(C) − p(A∩B) − p(A∩C) − ⏐B∩C⏐ + ⏐A∩B∩C⏐ ⋅⋅⋅⋅⋅⋅⋅⋅⋅ (3.B.3)

p(A∪B)’ = 1 − p(A∪B) ⋅⋅⋅⋅⋅⋅⋅⋅⋅⋅⋅⋅⋅⋅⋅⋅⋅⋅⋅⋅⋅⋅⋅⋅⋅⋅⋅⋅⋅⋅⋅ (3.B.4)

Khusus unt uk gabungan dua himpunan dikenal adanya dua himpunan saling lepas dan dua himpunan
saling bebas.
Dua himpunan dikat akan saling lepas j ika dua himpunan t ersebut t idak memiliki irisan at au dengan
kat a lain (A∩B) = 0 yang berakibat p(A∩B) = 0. Maka unt uk dua himpunan yang saling lepas berlaku :

p(A∪B) = p(A) + p(B) ⋅⋅⋅⋅⋅⋅⋅⋅⋅⋅⋅⋅⋅⋅⋅⋅⋅⋅⋅⋅⋅⋅⋅⋅⋅⋅⋅⋅⋅⋅⋅ (3.B.5)

Dua himpunan dikat akan saling bebas j ika dua himpunan t ersebut t idak saling mempengaruhi.
Misalkan A dan B adalah dua himpunan saling bebas dan berlaku :

p(A∩B) = p(A) ⋅ p(B) ⋅⋅⋅⋅⋅⋅⋅⋅⋅⋅⋅⋅⋅⋅⋅⋅⋅⋅⋅⋅⋅⋅⋅⋅⋅⋅⋅⋅⋅⋅⋅ (3.B.6)

Pengambilan cont oh dengan pengembalian merupakan cont oh kej adian saling bebas.

Cont oh 67 :
Sebuah dadu dilempar sekali. Berapakah peluang mat a dadu yang muncul adalah bilangan prima at au
bilangan genap ?

Solusi :
Misalkan A adalah munculnya mat a dadu prima dan B adalah munculnya mat a dadu bilangan prima.
Karena bilangan prima yang mungkin ada 3 yait u 2, 3 dan 5 maka p(A) = 12 dan p(B) = 12 . Irisan
1
himpunan A dan B adalah {2}, maka p(A∩B) = .
p(A∪B) = p(A) + p(B) − p(A∩B) = −
6
1 1 1
2 + 2 6
5
Maka peluang mat a dadu yang muncul adalah bilangan prima at au bilangan genap = 6 .

Cont oh 68 :
Peluang Ahmadi lulus uj ian 95% dan peluang Put u lulus uj ian 90 %. Jika dapat dianggap bahwa
kej adian Ahmadi lulus dan Put u lulus merupakan dua kej adian yang saling bebas, berapakah peluang:
a. Ahmadi dan Put u lulus uj ian
b. Ahmadi at au Put u lulus uj ian
c. Ahmadi lulus uj ian dan Put u t idak lulus

Eddy Hermanto, ST 184 Kombinatorik


Pembinaan Olimpiade Matematika
Solusi :
Misalkan A adalah kej adian Ahmadi lulus uj ian dan B adalah kej adian Put u lulus uj ian.

a. p(A∩B) = p(A) ⋅ p (B) = 0,95 ⋅ 0,90 = 0,855


p(A) = 0,95 dan p(B) = 0,90

b. p(A∪B) = p(A) + p(B) − p(A∩B) = 0,95 + 0,90 − 0,855 = 0,995


Peluang Ahmadi dan Put u lulus uj ian = 0,855

c. p(A∩B’ ) = p(A) ⋅ p(B’ ) = 0,95 ⋅ 0,10 = 0,095


Peluang Ahmadi at au Put u lulus uj ian = 0,995

Peluang Ahmadi lulus uj ian dan Put u t idak lulus = 0,095

LATIHAN 3.B

1. Dari hasil penelit ian pada suat u wilayah didapat : 20% penduduk memiliki TV, 40% memiliki radio
sert a 15% memiliki TV dan radio. Berapakah peluang seorang penduduk di wilayah t ersebut yang
dipilih secara acak unt uk memiliki TV at au radio ?

2. Dari 200 orang siswa suat u sekolah yang disurvei diket ahui 100 orang gemar Mat emat ika, 60 orang
gemar Biologi dan 90 orang gemar Fisika, 30 orang gemar Mat emat ika dan Biologi, 25 orang gemar
Mat emat ika dan Fisika, 20 orang gemar Biologi dan Fisika sedangkan 10 orang lagi t idak gemar
ket iga pelaj aran t ersebut . Jika sat u orang diambil dari 200 orang t ersebut secara acak, maka
berapakah peluang yang t erambil menyukai paling sedikit 2 dari 3 pelaj aran t ersebut ?

1
3. Kej adian A dan B adalah kej adian saling bebas t et api t idak saling lepas. Jika p(A) = 3 dan p
3
(A∪B) = 5 , maka t ent ukan p(B).

4. Dua buah dadu bersisi enam dilemparkan serent ak sat u kali. Berapakah peluang munculnya
j umlah mat a dadu sama dengan 7 at au 10 ?

5. Dua buah dadu dilempar bersamaan sat u kali. Berapakah peluang munculnya j umlah mat a dadu
genap at au bilangan prima ?

6. (OSK 2011 Tipe 3) Dua buah dadu dilempar secara bersamaan. Sisi dadu pert ama diberi angka 1,

sisi at as sama dengan 5, 7, at au 9 adalah ⋅⋅⋅⋅⋅⋅⋅


2, 2, 3, 3, dan 4. Sisi dadu kedua diberi angka 1, 3, 4, 5, 6, dan 8. Probabilit as agar j umlah kedua

7. Sebuah kart u diambil secara acak dari 1 set kart u bridge. Berapakah peluang bahwa yang t erambil
it u adalah kart u berwarna hit am at au kart u King ?

8. Peluang Barcelona menang at as Albacet e adalah 85 %sedangkan peluang Real Madrid menang at as
Albacet e adalah 80 %. Berapakah peluang :
a. Barcelona at au Real Madrid menang at as Albacet e
b. Barcelona dan Real Madrid keduanya menang at as Albacet e
c. Barcelona menang at as Albacet e dan Real Madrid t idak menang at as Albacet e
d. Barcelona t idak menang at as Albacet e dan Real Madrid menang at as Albacet e
e. Barcelona dan Real Madrid keduanya t idak menang at as Albacet e
Berapakah penj umlahan hasil b, c, d dan e ?

Eddy Hermanto, ST 185 Kombinatorik


Pembinaan Olimpiade Matematika

4. PIGEON HOLE PRINCIPLE (PRINSIP LUBANG MERPATI)


Pigen Hole Principle (Prinsip Lubang Merpat i) mengat akan bahwa j ika lebih dari n benda dimasukkan ke
dalam n kot ak, maka sedikit nya ada sat u kot ak yang berisi lebih dari sat u benda. Secara umum bahwa
j ika ada lebih dari pn benda dimasukkan ke dalam n kot ak maka sedikit nya ada sat u kot ak berisi lebih
dari p benda.

Bent uk Lain : Jika n bilangan bulat m1, m2, m3, ⋅⋅⋅, mn memiliki rat a-rat a 1 2 n3
m + m + m +L+ m
n
> r − 1 , maka
sedikit nya sat u di ant ara bilangan-bilangan bulat t ersebut lebih besar at au sama dengan r.

Cont oh 69 :
Jika ada 101 surat yang akan dimasukkan ke dalam 50 kot ak pos, bukt ikan bahwa ada sedikit nya sat u
kot ak pos berisi sekurang-kurangnya 3 surat .

Solusi :
Jika seluruh kot ak pos maksimal hanya berisi 2 surat , maka j umlah maksimal surat yang dapat masuk
kot ak pos adalah 100. Tet api j umlah surat yang ada yait u 101. Maka dapat dipast ikan ada sedikit nya sat u
kot ak pos berisi sekurang-kurangnya 3 surat .

Cont oh 70 :
Pada sebuah pest a set iap orang yang hadir diharuskan membawa permen. Jika pada pest a t ersebut
j umlah orang yang hadir ada 10 sedangkan j umlah permen yang ada sebanyak 50 buah, bukt ikan bahwa
ada sekurang-kurangnya 2 orang yang membawa permen dalam j umlah yang sama.

Solusi :

permen yang ada sebanyak 1 + 2 + 3 + ⋅⋅⋅ + 10 = 55 > 50 (t idak memenuhi). Kont radiksi.
Andaikan bahwa seluruh orang membawa permen dalam j umlah yang berbeda maka sedikit nya j umlah

Maka dapat dibukt ikan bahwa ada sekurang-kurangnya 2 orang yang membawa permen dalam j umlah yang
sama.

Cont oh 71 :
Jika t erdapat n2 + 1 t it ik yang t erlet ak di dalam sebuah persegi dengan panj ang sisi n, bukt ikan bahwa
ada sekurang-kurangnya 2 t it ik yang memiliki j arak t idak lebih dari 2 sat uan.
Solusi :
Persegi dengan ukuran n x n dapat dibagi menj adi n2 buah persegi berukuran 1 x 1.
Pada persegi dengan ukuran 1 x 1, j arak t erj auh 2 t it ik adalah j ika keduanya t erlet ak pada t it ik sudut
berlawanan yait u sej auh 2 sat uan.
Karena ada n2 + 1 t it ik dengan ada n2 persegi maka sesuai Pigeon Hole Principle, akan t erdapat sedikit nya
2 t it ik yang t erlet ak pada sat u persegi dengan ukuran 1 x 1 yang sama.
Maka dapat dibukt ikan ada sekurang-kurangnya 2 t it ik yang memiliki j arak t idak lebih dari 2 sat uan.

Cont oh 72 :
Jika t erdapat n2 + 1 t it ik yang t erlet ak di dalam sebuah segit iga sama sisi dengan panj ang sisi n, bukt ikan
bahwa ada sedikit nya 2 t it ik yang j araknya sat u sama lain paling j auh 1.

Solusi :
Bagi segit iga t ersebut menj adi n2 buah segit iga sama sisi yang masing-masing panj ang sisinya 1 sat uan.
Pada segit iga sama sisi dengan panj ang sisi 1 sat uan, j arak t erj auh 2 t it ik adalah j ika keduanya t erlet ak
pada t it ik sudut segit iga yait u sej auh 1 sat uan.

Eddy Hermanto, ST 186 Kombinatorik


Pembinaan Olimpiade Matematika
Karena ada n2 + 1 t it ik dengan ada n2 segit iga sama sisi dengan panj ang sisi 1 sat uan maka sesuai Pigeon
Hole Principle akan t erdapat sedikit nya 2 t it ik yang t erlet ak pada sat u segit iga dengan panj ang sisi 1
sat uan yang sama.
Maka dapat dibukt ikan bahwa ada sedikit nya 2 t it ik yang j araknya sat u sama lain paling j auh 1

Cont oh 73 :
Bukt ikan bahwa di ant ara 7 bilangan bulat , past i ada sekurang-kurangnya sepasang bilangan yang
selisihnya habis dibagi 6.

Solusi :
Kemungkian sisa j ika suat u bilangan bulat dibagi 6 adalah 0, 1, 2, 3, 4, at au 5. Karena ada 6 kemungkinan
dan ada 7 bilangan bulat maka sesuai Pigeon Hole Principle, sedikit nya dua bilangan akan memiliki sisa
yang sama j ika dibagi 6. Misalkan bilangan it u adalah n1 dan n2 dengan sisa j ika dibagi 6 adalah r. Maka

n1 − n2 = 6(k1 + k2) yang merupakan bilangan yang habis dibagi 6 (t erbukt i)


kit a dapat membuat n1 = 6k1 + r dan n2 = 6k2 + r dengan k1 dan k2 bilangan bulat .

Cont oh 74 :
Bukt ikan bahwa di ant ara 5 bilangan bulat past i ada 3 di ant aranya memiliki j umlah habis dibagi 3.

Solusi :
Kemungkian sisa j ika suat u bilangan bulat dibagi 3 adalah 0, 1 at au 2. Misalkan t erdapat t iga bilangan
yang memiliki sisa yang sama j ika dibagi 3. Misalkan j uga sisanya adalah r. Dapat diandaikan bilangan
t ersebut adalah n1 = 3k1 + r, n2 = 3k2 + r dan n3 = 3k3 + r dengan k1, k2 dan k3 bilangan bulat . Jumlah ket iga
bilangan ini akan habis dibagi 3 (t erbukt i).
Andaikan t idak t erdapat 3 bilangan yang memiliki sisa yang sama j ika dibagi 3. Karena kemungkinan sisa
bilangan j ika dibagi 3 ada 3 kemungkinan sedangkan t erdapat 5 bilangan, sesuai Pigeon Hole Principle,
akan ada 3 di ant aranya yang sat u bersisa 0 j ika dibagi 3, salah sat unya bersisa 1 j ika dibagi 3 dan sat unya
lagi bersisa 2 j ika dibagi 3. Misalkan ke-3 bilangan adalah n1 = 3k1, n2 = 3k2 + 1 dan n3 = 3k3 + 2.
n1 + n2 + n3 = 3(k1 + k2 + k3 + 2) yang merupakan bilangan yang habis dibagi 3.

Cont oh 75 :
Tit ik let is pada bidang adalah t it ik yang mempunyai koordinat berupa pasangan bilangan bulat .

Bukt ikan bahwa t erdapat sepasang t it ik (Pi , Pj ), i ≠ j , demikian, sehingga ruas garis Pi Pj memuat sebuah
Misalkan P1, P2, P3, P4, P5 adalah lima t it ik let is berbeda pada bidang.

t it ik let is selain Pi dan Pj .

Solusi :
Misal xij adalah j arak t it ik Pi dan Pj dalam arah sumbu X dan Misal yij adalah j arak t it ik Pi dan Pj dalam arah
sumbu Y.

Jika xij dan yij keduanya genap, maka dapat dipast ikan bahwa sekurang-kurangnya sat u t it ik let is selain
t it ik Pi dan Pj akan t erlet ak pada ruas garis Pi Pj , yait u pada pert engahan ruas garis Pi Pj yang akan berj arak
1 1 1 1
2 xij pada arah sumbu X dan 2 yij pada arah sumbu Y t erhadap t it ik Pi maupun Pj dengan 2 xij dan 2 yij
adalah j uga bilangan bulat .

Bilangan Genap − Bilangan Genap = Bilangan Genap


Sifat penj umlahan berikut j uga akan membant u menj elaskan :

Bilangan Ganj il − Bilangan Ganj il = Bilangan Genap.

Kemungkinan j enis koordinat (dalam bahasa lain disebut parit as) suat u t it ik let is hanya ada 4
kemungkinan yait u (genap, genap), (genap,ganj il), (ganj il, ganj il) dan (ganj il, genap).

Eddy Hermanto, ST 187 Kombinatorik


Pembinaan Olimpiade Matematika
Jika 2 t it ik let is mempunyai parit as yang sama maka sesuai sifat penj umlahan maka dapat dipast ikan
kedua t it ik let is memiliki j arak mendat ar dan j arak vert ikal merupakan bilangan genap yang berart i
koordinat t it ik t engah dari garis yang menghubungkan kedua t it ik let is t ersebut j uga merupakan bilangan
genap.

Karena ada 5 t it ik let is sedangkan hanya ada 4 parit as t it ik let is maka sesuai Pigeon Hole Principle (PHP)
maka dapat dipast ikan sekurang-kurangnya ada dua t it ik let is yang memiliki parit as yang sama.

bidang maka t erdapat sepasang t it ik (Pi , Pj ), i ≠ j , demikian, sehingga ruas garis Pi Pj memuat sebuah t it ik
Dari penj elasan di at as dapat dibukt ikan bahwa j ika P1, P2, P3, P4, P5 adalah lima t it ik let is berbeda pada

let is selain Pi dan Pj .

Cont oh 76 :

dapat menemukan dua di ant aranya, kat akanlah x dan y dengan y > x sedemikian sehingga 1 < ≤ 2.
Tunj ukkan bahwa di ant ara t uj uh bilangan bulat posit if berbeda yang t idak lebih dari 126, kit a selalu
y
x

Karena x dan y berbeda maka x > 1 .


Solusi :
y

• Jika t erdapat salah sat u bilangan t ersebut adalah 1


Alt ernat if 1 :

Maka agar t idak memenuhi syarat maka bilangan t erkecil berikut nya adalah 3. Maka agar hal ini j uga
t idak memenuhi syarat maka 4 bilangan t erkecil berikut nya adalah 7, 15, 31, 63. Tet api bilangan

• Jika 1 t idak t ermasuk ke dalam 7 bilangan t ersebut


maksimal adalah 126 yang mengakibat kan 126 : 63 = 2 (memenuhi syarat pada soal)

Buat bat asan bilangan menj adi enam bagian : 21 ≤ ai ≤ 22 ; 22 ≤ ai ≤ 23 ; 23 ≤ ai ≤ 24 ; 24 ≤ ai ≤ 25 ;


25 ≤ ai ≤ 26 ; 26 ≤ ai ≤ 27.
Karena ada 7 bilangan dan 6 daerah bagian, maka sesuai Pigeon Hole Principle maka akan ada 2
bilangan berada pada sat u daerah yang sama. Pada masing-masing daerah nilai t erkecil adalah 2k dan
t ert ingginya adalah 2k+1 yang menyebabkan rasio bilangan t erbesar dan t erkecil adalah 2.

sedemikian sehingga 1 < ≤ 2.


Maka akan ada dua bilangan kat akanlah x dan y dengan y > x yang berada pada sat u daerah
y
x

Bagi 126 bilangan bulat posit if t ersebut ke dalam 6 himpunan : {1,2}, {3,4,5,6}, {7,8, ⋅⋅⋅,14}, {15,16,⋅⋅⋅,30},
Alt ernat if 2 :

{31,32,⋅⋅⋅,62}, {63,64,⋅⋅⋅,126}.
Karena ada 7 bilangan dan 6 himpunan maka sesuai Pigeon Hole Principle (PHP) maka sedikit nya dua

himpunan yang sama t ersebut . Maka berlaku 1 < ≤ 2 (t erbukt i)


bilangan berada pada sat u himpunan yang sama. Misalkan dua bilangan x dan y berada pada sat u
y
x

Cont oh 77 :
Seorang pemain cat ur memiliki wakt u 11 minggu unt uk menyiapkan diri mengikut i sebuah t urnamen. Ia
memut uskan unt uk berlat ih sedikit nya sat u permainan set iap hari, namun t idak lebih dari 12 permainan
selama seminggu. Perlihat kan bahwa ada beberapa hari bert urut -t urut yang selama it u pecat ur t ersebut
berlat ih t epat 21 permainan.

Misalkan ar menyat akan banyaknya permainan cat ur dalam r hari pert ama dengan 1 ≤ r ≤ 77. Berdasarkan
Solusi :

soal maka kit a akan membukt ikan bahwa t erdapat aj − ai = 21.


Jelas bahwa 1 ≤ a1 < a2 < a3 < ⋅⋅⋅ < a77.
Karena dalam 1 minggu grandmast er memainkan paling banyak 12 permainan maka a77 ≤ 12 ⋅ 11 = 132.
a77 + 21 ≤ 153

Eddy Hermanto, ST 188 Kombinatorik


Pembinaan Olimpiade Matematika
Perhat ikan 154 bilangan a1, a2, a3, ⋅⋅⋅, a77, a1 + 21, a2 + 21, a3 + 21, ⋅⋅⋅, a77 + 21 yang semuanya t erlet ak
ant ara 1 dan 153.

Pigeon Hole Principle maka akan t erdapat dua bilangan yang sama. Karena a1, a2, ⋅⋅⋅, a77 semuanya
Karena banyaknya bilangan 154 sedangkan kemungkinan nilai bilangan hanya 153 maka berdasarkan

aj = ai + 21 sehingga aj − ai = 21
berbeda maka akan t erdapat aj dan ai + 21 yang sama.

Maka akan t erdapat banyaknya t ot al permainan hari ke-(i +1), (i + 2), ⋅⋅⋅, j t epat sama dengan 21.

LATIHAN 4

1. (OSP 2010) Pada suat u bidang t erdapat n t it ik yang berkoordinat pasangan bilangan bulat . Nilai n

adalah ⋅⋅⋅
t erkecil agar t erdapat dua t it ik yang t it ik t engahnya j uga berkoordinat pasangan bilangan bulat

asli S = {1, 2, 3, ⋅⋅⋅, m}, berapa banyak anggot a S harus dipilih agar selalu t erdapat paling sedikit sat u
2. (OSP 2006) Misalkan m bilangan asli yang memenuhi 1003 < m < 2006. Diberikan himpunan bilangan

pasang anggot a t erpilih yang hasil t ambahnya 2006 ?

3. (OSK 2011 Tipe 1) Di lemari hanya ada 2 macam kaos kaki yait u kaos kaki berwarna hit am dan put ih.
Ali, Budi dan Candra berangkat di malam hari saat mat i lampu dan mereka mengambil kaos kaki
secara acak di dalam lemari dalam kegelapan. Berapa kaos kaki minimal harus mereka ambil unt uk
memast ikan bahwa akan ada t iga pasang kaos kaki yang bisa mereka pakai ? (Sepasang kaos kaki harus
memiliki warna yang sama).

4. Tandai sat u buah kart u dengan angka 1, dua buah kart u dengan angka 2, t iga buah kart u dengan
angka sat u hingga lima puluh buah kart u dengan angka 50. Semua kart u t ersebut dimasukkan ke
dalam kot ak. Berapa buah kart u minimal yang harus diambil agar dapat dipast ikan t erdapat sekurang-
kurangnya 10 buah kart u dengan t anda angka yang sama ?

5. Diambil n buah bilangan dari himpunan 2008 bilangan {1, 2, 3, …, 2008}. Tent ukan nilai n minimal
sehingga past i akan didapat dua bilangan asli berbeda di ant aranya yang memenuhi penj umlahan
kedua bilangan t ersebut habis dibagi 8.

6. Dua buah kot ak berisi bola. Jumlah t ot al bola di kedua kot ak t ersebut adalah 65. Ada 4 buah warna
bola : merah, put ih, hit am dan hij au. Selain it u, j ika kit a mengambil 5 buah bola yang berwarna sama
maka sekurang-kurangnya dua di ant aranya memiliki ukuran yang sama. Bukt ikan bahwa salah sat u
kot ak akan berisi sekurang-kurangnya t iga buah bola dengan warna dan ukuran yang sama.

7. Jika diket ahui m buah bilangan bulat a1, a2, a3, ⋅⋅⋅, am, t unj ukkan bahwa ada bilangan bulat k dan s
dengan 0 ≤ k < s ≤ m sedemikian sehingga ak+1 + ak+2 + ⋅⋅⋅ + as habis dibagi m.

8. Di ant ara bilangan-bilangan 1, 2, ⋅⋅⋅, 200, j ika 101 bilangan diambil, maka t unj ukkan bahwa ada dua
bilangan di ant ara yang t erambil sedemikian sehingga yang sat u habis dibagi yang lain.

9. Bukt ikan bahwa j ika 100 bilangan diambil dari himpunan 1, 2, 3, ⋅⋅⋅, 200 sedemikian sehingga
sedikit nya sat u diant aranya lebih kecil dari 15, maka ada dua di ant ara yang t erpilih sehingga yang
sat u habis dibagi yang lain.

10. Misalkan bilangan-bilangan 1 sampai 20 dit empat kan dalam urut an bagaimana pun pada sebuah
lingkaran. Tunj ukkan bahwa :
a. ada t iga bilangan berdekat an yang j umlahnya sedikit nya 32
b. ada empat bilangan berdekat an yang j umlahnya sedikit nya 42

Eddy Hermanto, ST 189 Kombinatorik


Pembinaan Olimpiade Matematika

11. Tit ik let is pada ruang adalah t it ik yang mempunyai koordinat berupa t ripel bilangan bulat (Cont oh :
(3,4,5); (3,−4,6)).

Bukt ikan bahwa t erdapat sepasang t it ik (Pi , Pj ), i ≠ j , demikian, sehingga ruas garis Pi Pj memuat
Misalkan P1, P2, P3, P4, P5, P6, P7, P8, P9 adalah sembilan t it ik let is berbeda pada ruang.

sebuah t it ik let is selain Pi dan Pj .

12. Bukt ikan bahwa j ika dalam sebuah grup 6 orang, set iap 2 orang hanya dapat selalu bersahabat at au
selalu bermusuhan, maka ada sedikit nya 3 orang yang saling bersahabat at au saling bermusuhan sat u
sama lain.

13. Di dalam suat u pest a t erdapat n orang dan mereka saling bersalaman. Jika di ant ara 2 orang t idak ada
yang bersalaman lebih dari 1 kali, bukt ikan bahwa ada sedikit nya 2 orang bersalamaan dalam j umlah
yang sama.

14. Diberikan 7 bilangan real. Bukt ikan bahwa kit a dapat memilih dua di ant aranya kat akan a dan b
tan α − tan β
sehingga 0 ≤ a −b
1+ ab ≤ 1
3
. (Pet unj uk : Rumus yang dapat digunakan adalah tan (α − β) = 1+ tan α tan β )

15. Terdapat 115 bola yang dij aj arkan pada sat u garis lurus dan t erdapat 60 bola merah di ant aranya.
Masing-masing bola diberi nomor berbeda sesuai dengan urut annya yait u nomor 1 sampai 115.
Tunj ukkan bahwa sedikit nya ada 2 bola merah yang t erpisah 4 bola (Misalnya bola merah dengan
nomor 5 dan 9 sert a nomor 36 dan 40 memenuhi syarat ini).

16. Bukt ikan bahwa j ika t erdapat 19 t it ik dalam bidang XY yang memiliki koordinat berupa pasangan
bilangan bulat dengan t idak ada t iga t it ik yang t erlet ak dalam sat u garis lurus, maka dapat dipast ikan
ada t iga di ant aranya memiliki t it ik berat yang j uga merupakan pasangan bilangan bulat .

17. (ME V6N1) Dua puluh delapan bilangan bulat diambil dari himpunan H = {104, 105, 106, 107, ⋅⋅⋅, 208}.
Tunj ukkan bahwa t erdapat dua bilangan yang keduanya mempunyai fakt or persekut uan prima.

Eddy Hermanto, ST 190 Kombinatorik


Pembinaan Olimpiade Matematika

KUNCI JAWABAN LATIHAN

BAB I ALJABAR

1. −4x + 6 1. 2x2 + 9x − 24 = 0
LATIHAN 1 LATIHAN 2 LATIHAN 2 LATIHAN 3 LATIHAN 4 LATIHAN 5A

2. −25 2. p ≤ −2 at au p ≥ 6
1. 3 1. 17 19. 819 1. 2x + 20
2. 3x2 + 3
3. −12 3. p ≥ 4
2. 2 20. 4665 2. 4x + 2
3. 104 3. 21 21. 129 3. x = 4
x +1
4. −136 4. 100 22. 2025 4. f(x) = x 4. −2 4. 7x2 + 4x + 1 = 0

5. 2√10 5. 40300 23. 986 5. 3x − 23 5. 3 5. 2

5x − 1
3
2
6. 7 6. 912 24. 0 at au 5 6. 490x + 7 6. 6. 828
7. {−1, 1, 2} 7. 17 25. 19
4 7. −10 7. 3 7. 13
2 x −1
8. 1
2 + 1
2 √5 8. 101 26. 270 8. 2x 8. a. 27 b. 125 8. 337

− 12 √5 x −5
1 1 11
or 2 9. 93 27. 2419 9. c. 26 d. 76 9. 3
3+ 5 3− 5 11
9. or 10. 0 28. 2949 10. 30 9. 4 10.
10. 6 at au − 19
2 2 5
997501 9 6
10. 4 11. 10 29. 3990006 11. 2 11. 19

11. a=b=c=
3
2 −1 12. 27
32 30. 1002
2003 12. 6 11. 2006 12. −2010

d= 53 2 − 1 13. 2 31. 9 13. 2005 12. −2 + 2√3 13. 2009


4023 1
12. 2 14. 0 < a < 4 32. 2 + 2014 14. 7 13. 23 14. 64
−1+ 161
13. 40 dan 15. 1
8 , 1
64 , 1
512 33. (28,811) 15. 26 14. −15 15. 1
2
−1− 161
16. − 83 34. 898 16. 1
15. 763 16. 3
17. −2
40 3
14. 373 17. 75 35. 618 17. 169 16. 345
1 5
15. 0 at au 144 18. 24 n4 + 12 n3 36. Terbukt i 17. 727 18. Terbukt i
1+ 5 35 25 1999
16. x = 2 + 24 n2 + 12 n 37. 1999 + 2000 18. 1984 19. Terbukt i
17. 20 38. Terbukt i 19. 6 20. 383
18. Terbukt i 20. Terbukt i 21. Terbukt i
19. 31 22. Terbukt i

Eddy Hermanto, ST 191 Kunci Jawaban


Pembinaan Olimpiade Matematika

LATIHAN 5B LATIHAN 5C LATIHAN 5D LATIHAN 5E LATIHAN 6


1
1. 16 1. 12 1. (x−4) 2+(y−3) 2=16 1. t idak ada 1. 3
1
2. 4 2. 2 2. (x−5) 2+(y−6) 2=62 2. 15 2. 9 dan 223
2 2

3 or −3
3. 250 3. 512 3. (x−3) +(y−4) =25 3. 37 3. (−3,−3), (0,3)
4. 5 4. 1 or (x+4) 2+(y+3) 2=25 4. dan (3, 0)
5. {5} 5. 0 < p < 12 4. (1 + 2 3 , 0) 5. 18
5 4. k > √2 or k < −√2

6. 27 6. 4√3 & (1−2 3 , 0) 6. 12 5. 2


7. 4 7. 1/ 5 5. 7x − y = 50 7. −4, 0 or 4 6. Terbukt i
8. 60 6. 0 or 50 8. 10 7. 334

13x −9y = 100


9. 27 7. 2√3 at au 6√3 9. 5101 8. (−5,9),(−2,6)
10. 55 8. 10. 20 (−2,−6), (0,4),
11. 1 or 3x + y = 20 11. 429 (19, 99)
1
12. 9. 3y = 4x + 30 9. (1,1,1), (−2,−2,−2)

( )
3
13. 12 10. 23 10. 158

( )
11. 1 11. (54, 24, 6)
11+ 4 7
12. 140 12. 21 , 22+78 7

11− 4 7
13. 23 21 , 22−78 7

LATIHAN 7 LATIHAN 8

2. −4 < x < 2 2. x = −2 or 9
1. 2 1. 11

3. 112 3. 7
4. Terbukt i 4. 2 : 1
5. 0 5. 81
6. x=y=u=v=1/ 4 da 6. 50
dan z = 3 7. 125
7. (1, 1) 8. 5993
8-10. Terbukt i 9. 45/ 8
11. 2√2 10. 69
12. 2 j ika x = 1/ 2
13-17. Terbukt i
18. 0
19-20. Terbukt i
21. 0
22. 7/ 2
23. 16/ 5
24-27. Terbukt i

Eddy Hermanto, ST 192 Kunci Jawaban


Pembinaan Olimpiade Matematika

BAB II TEORI BILANGAN

LATIHAN 1 LATIHAN 2 LATIHAN 3 LATIHAN 4 LATIHAN 5


1. 30 1. 29 1. 32.571 1. 2259 1. 16
2. 2001 2. 9 2. 33 2. 2521 2. 2006
3. 4022 3. 2.006.005 3. 120 3. 3731 3. 60
4. 2 4. 890 4. a = 5 dan b = 9 4. 1, 3, 16, 33 dan 67 4. 140
5. 83 5. 20 5. 592 5. 3 5. 37
6. Terbukt i 6. 8 6. 76776 6. 2 6. 2
7. Terbukt i 7. 588 7. 0, 4 dan 5 7. 25 7. 432
8. Terbukt i 8. 675 8. 4 8. 35
9. 97 9. 65 9. 378
10. 925 10. (2, 7), (3, 4), (4, 3), 10. 103
11. 30 (6, 9), (7, 2), (9, 6) 11. 224
12. Terbukt i 11. Terbukt i 12. 589
13. Terbukt i 12. Terbukt i 13. 180
14. 4 13. 401 14. ganj il = 12
15. 870 genap = 36
16. Terbukt i
17. 630
18. Terbukt i
19. 41
20. Terbukt i
21. Terbukt i

LATIHAN 6 LATIHAN 7 LATIHAN 8 LATIHAN 9


1. 6 1. 52 21. 1 1. 105 1. 2288
2. 5 2. 108 22. 29 2. 81649 2. 44
3. 13 3. 10 23. 123 3. 4abc899 3. 15
4. 4 4. 200 24. 1 4. Terbukt i 4. 24
5. 7 5. 78 25. 0 5. Tidak ada 5. 6
6. 100 6. 5 26. 3 6. 6 6. 312
7. 3 7. 358 27. Tidak ada 7. 124 7. 27

1<x≤
88
8. 88 8. 84 28. 61 8. Terbukt i 8. 4

n ∈ Bilangan asli
3

2 − √3
9. 9. 145 29. 2 9. 390625 dan 141376 9. 997
10. 144 10. (14, 112) 30. 1999 10. Terbukt i 10.
11. 5 11. 156 31. 5, 13, 17, 97 11. 7 11. 743
12. 16 12. 750 32. 8 12. 38 12. 2013021
7 4
13. 561 13 a = 3 ; b = 2 33. 41 13. Terbukt i 13. 15 dan 15

15. −1
14. 96 14. 400 14. x = 59 dan n = 12 14. 49
15 63 15. 648 15. (4, 2)

12000, ⋅⋅⋅, 9900


16. 375 16. 10000, 11000, 16. 1006
17 9
18. 0 at au 1 17. a. Belum ; b. Past i
19. 192 18. Terbukt i
19. 173
20. 73

Eddy Hermanto, ST 193 Kunci Jawaban


Pembinaan Olimpiade Matematika

BAB III GEOMETRI

LATIHAN 1 LATIHAN 3A LATIHAN 3C LATIHAN 3D


3 + √3
34 o 11
1. 6 21. 9/ 4 1. 36 1. 3 1.

2. Terbukt i 22. − 2009


2. 20 2. 19 : 8 2. 36

− 14
2

−2
5 9 7
3. 23. 4 3. 3 3. 5 3. 24

4. Terbukt i 4. 60o 4.
15
4 − 2
3
4. 7

5. 1
2 √6 5. 3
5 5. 4
3 5. 110
6. Terbukt i LATIHAN 2 6. Terbukt i 6. 4: 9 6. Terbukt i
7. Terbukt i 1. y = −2x + 13 7. 26 7. 1000 7. 4
y = 6x − 22
y=x−2
8. Terbukt i 2. 8. 130 8. 2√10 8. 3
9. Terbukt i 3. 9. (4,5,6) 9. 161/ 3 9. 1/ 12
x+ y
10. 1/ 2 4. 4 10. Terbukt i 10. 3/ 2 10. 2x
11. 150 5. 3 11. Terbukt i 11. Terbukt i
12. 13
4 − √10 6. 49 LATIHAN 3B 12 Terbukt i 12. 3

13. 1 7. 4y = 5x − 15 1. 12
5 13. 8/ 105
7 28
14. 16 3 8. (2√2, 4√2) 2. 5 14. 7
o
15. 22,5 9. 4√2 3. 10 15. 18
16. 1
2 √3 4. 12
5 16. 441

17. 1
√3 5. 2 17. 1 : 3

18. −
2
1
8 6. 5 18. 2√3 − 3
19. 159 7. Terbukt i 19. 27
20. 6 20. PD=PE=PF=1
21. 168/ 295
22. Terbukt i
23. Terbukt i
24. 21

LATIHAN 3E LATIHAN 4 LATIHAN 5 LATIHAN 6 LATIHAN 7


1. 35
6 1. 9: 4 1. √3 : 2 1. 153o 1. Terbukt i

2. 1: 2 2. 10 2. 3
2 √3 2. 2π + 4 2. 25
24
2 3 −3 4x 1
3. 3. 3. 3: 2 3. 18o 3. k2
−4
6 9 9
4. 1: 2 4. 4. 230 4. x=3
5. 2007 5. 70o 5. Terbukt i
LATIHAN 3F 6. 106 6. 3 6. Terbukt i
1. 12 7. 14√3 7. 8 7. Terbukt i
2. 14 8. 13.924 8. 2π 8. Terbukt i
3. 2/ 3 9. 15 9. 63/ 11 9. Terbukt i
4. Terbukt i 10. 50/ 3 10. 10o 10. 130
5. Terbukt i 11. Terbukt i 11. 3 11. Terbukt i
6. Terbukt i 12. Terbukt i
13. Terbukt i
14. Terbukt i
15. Terbukt i

Eddy Hermanto, ST 194 Kunci Jawaban


Pembinaan Olimpiade Matematika

BAB IV KOMBINATORIK

LATIHAN 1.A LATIHAN 1.B LATIHAN 1.D LATIHAN 1F


1. a. 240 ; b. 480 1. 1 11. 26 1. 8100 1. Terbukt i
2. 729x6
−1458x5y
2. 5040 2. 8 12. 277.200 2. 15
3. 12 3. a. 720 13. 8 3. 480
21010 + 1215x4y2
− 540x3y3
4. 240 b. 60 14. 4. 3720
5. 120 c. 360 15. a. 196 ; b. 252 5. 162
+ 135x2y4
− 18xy5
6. 8400 d. 360 16. 380 pert ; 140 seri 6. 758
7. 720 e. 90720 17. 36
8. 90 f. 181440 18. 134.789 + y6

4. −1792
9. 48 g. 151200 19. 502 LATIHAN 1E 3. 1088640

5. −108
10. 94 4. 360 20. 462 1. 5050
11. 499 5. 52 21. 1820 2. 45
12. 72 6. 50 22. 10920 3. 560 6. 20160
13. a. 98 ; b. 60 7. 48 23. 53130 4. 28 7. 5 : 3
14. a. 2559 8. 72 24. 432 5. 21 8. 129
b. 1050 9. 1152 25. 315 6. 10 9. a. 5 ; b. 8
15. 972 10. 267 26. 140 7. 70 10. a. 64 ; b. 1
11. 2n
12. 22008 − 1
16. 9946 27. 44
17. 70 28. 1
18. 108 LATIHAN 1.C 29. 1130 13. 10n
19. 4500 1. 120 30. 186 14. a. 56
20. 112896 2. 15 31. 10 b. 70
21. 32 3. 1575 32. 945 c. 56
4. 56 33. a. 210 ; b.90 ; c.45 15. 8085
5. 20 34. 34 16. 35
6. 64 35. 380.570.190 17. 816
7. 120 36. 34 18. 500
8. 231 37. 630
9. 81 38. 89
10. 771 39. Terbukt i

LATIHAN 2D LATIHAN 2E LATIHAN 3A LATIHAN 4


1. 10/ 36 18. 1/ 2 1. a. 18/ 169 1. 11 1. 5
2. 14/ 36 19. 3/ 10 b. 3/ 26 2. 20 2. 1004
3. a. 1/ 21 ; b. 5/ 42 20. 1/ 2 2. a. 3/ 40 3. 15 3. 7
c. 5/ 14 ; d. 10/ 21 21. 1/ 36 b. 1/ 12 4. 429 4. 415
4. 11/ 21 22. 10 3. a. 1/ 3 5. 286 5. 756
5. 21/ 25 23. 7/ 9 b. 4/ 15 6. 1060 6. Terbukt i
6. 1/ 2006 24. 3/ 5 c. 4/ 15 7. 23 7. Terbukt i
7. 1 25. 1,3,4,5,6,8 d. 2/ 15 8. Terbukt i
8. 8/ 9 26. 13/ 36 4. 1/ 52 9. Terbukt i
9. 1/ 2007 27. 4/ 10 5. 25/ 169 LATIHAN 3B 10. Terbukt i
10. 1/ 32 28. 1/ 2 1. 0,55 11. Terbukt i
11. 1/ 3 29. 36/ 96 LATIHAN 2F 2. 0,225 12. Terbukt i
12. 21 30. 2/ 5 1. 775 3. 2/ 5 13. Terbukt i
13. 16/ 81 31. 9/ 625 2. 181/ 182 4. 1/ 4 14. Terbukt i
14. a. 6/ 11 ; b. 5/ 88 32. 121/ 202 3. 392/ 429 5. 8/ 9 15. Terbukt i

√3 − 1
c. 35/ 88 33. 5/ 16 4. 124/ 143 6. 14/ 36 16. Terbukt i
15. 10/ 21 34. 5. 5/ 11 7. 7/ 13 17. Terbukt i
16. 2/ 3 35. 15/ 64 6. 25/ 169 8. a. 0,97 ; b. 0,68
17. 4/ 7 36. 7/ 99 7. 27/ 35 c. 0,17 ; d. 0,12
37. 1/ 11 e. 0,03

Eddy Hermanto, ST 195 Kunci Jawaban


DAFTAR PUSTAKA

1. Anderson, Ian. 2002. A First Course in Discret e Mat hemat ics. Springer-Verlag, London.

2. Budhi, Wono Set ya. 2006. Langkah Awal Menuj u ke Olimpiade Mat emat ika. Ricardo, Jakart a.

3. Clark, W. Edwin. 2003. Element ary Number Theory. Depart ment of Mat hemat ics Universit y of
Sout h Florida.

4. Engel, Art hur. 1998. Problem-Solving St rat egies. Springer-Verlag, New York.

5. Haese, R. C. dan Haese, S. H. 1981. Compet it ion Mat hemat ics. Haese Publicat ions.

6. Manfrino RB, dkk. 2005. Inequalit ies. Inst it ut o de Mat emat icas Universidad Nacional Mexico.

7. Posament ier, Alfred S dan Salkind, Charles. 1988. Challenging Probl ems in Geomet ry. Dover
Publicat ions Inc, Newyork.

8. Susant o H., Sisworo, dan As’ ari, AR. 2006. Napak Tilas Ol impiade Sains Nasional : Mat emat ika
SMP. Universit as Negeri Malang Press.

9. Susiant o, Bambang. 2006, Olimpiade Mat emat ika dengan Proses Berpikir : Alj abar dan Bilangan.
Grasindo, Jakart a.

10. Wirodikromo, Sart ono. 1995. Mat emat ika unt uk SMU Kelas 1 Cat ur Wulan 3. Erlangga, Jakart a.

11. Wirodikromo, Sart ono. 2000. Mat emat ika unt uk SMU Kelas 3 Cat ur Wulan 1. Erlangga, Jakart a.

12. Zeit h, Paul. 2007. The Art and Craf t of Probl em Solving. Jon Wiley and Son.
RIWAYAT HIDUP PENULIS

Eddy Hermanto lahir di Desa Bunut Tinggi, Kecamatan Talo,


Kabupaten Bengkulu Selatan (sekarang Kabupaten Seluma) pada
tanggal 9 September 1979. Pendidikan SD dan SLTP
diselesaikannya di Lampung, yaitu SD di SD Negeri 2 Bandar Jaya,
Lampung Tengah dan SLTP di SMP Negeri Bandar Jaya, Lampung
Tengah. Sedangkan pendidikan SLTA dilaluinya di SMU Negeri 5
Bengkulu. Penulis yang juga merupakan putera asli Bengkulu ini
kemudian melanjutkan pendidikan S1 ke Jurusan Teknik Sipil
Fakultas Teknik Universitas Gadjah Mada Yogyakarta pada tahun 1997 yang
diselesaikannya pada bulan Februari 2002 dengan predikat Cum Laude.

Saat ini Penulis bekerja sebagai PNS di Pemerintah Kota Bengkulu pada Bagian
Administrasi Pembangunan Setda Kota Bengkulu (dulu bernama Bagian Penyusunan
Program Setda Kota Bengkulu) yang telah digeluti sejak Desember 2002. Selain bekerja
di Pemerintah Kota Bengkulu, Penulis juga aktif membina siswa-siswa di SMA N 5
Bengkulu baik dalam persiapan menghadapi Ujian Masuk Universitas Gadjah Mada
(UM-UGM), Seleksi Penerimaan Mahasiswa baru (SPMB) maupun ketika SMA N 5
Bengkulu akan menghadapi perlombaan-perlombaan baik tingkat kota, provinsi
maupun nasional. Penulis juga pernah beberapa kali membina siswa-siswa dari Provinsi
Bengkulu yang akan mengikuti Olimpiade Sains Nasional Bidang Matematika.

Anda mungkin juga menyukai